0% found this document useful (1 vote)
6K views

NBME 12 - Answers

A 57-year-old man presents with a 1-year history of low back pain radiating to the buttocks with walking that is relieved by rest. He also has a 1-year history of erectile dysfunction and decreased pulses. Physical exam reveals decreased pulses. The most likely diagnosis is aortoiliac occlusion, also known as Leriche syndrome, a type of peripheral artery disease presenting with claudication, erectile dysfunction, and decreased pulses caused by narrowing of the iliac arteries. Differential diagnoses such as herniated disc, discitis, pyelonephritis, and spinal stenosis were excluded based on presentation.

Uploaded by

1140271
Copyright
© © All Rights Reserved
We take content rights seriously. If you suspect this is your content, claim it here.
Available Formats
Download as PDF, TXT or read online on Scribd
0% found this document useful (1 vote)
6K views

NBME 12 - Answers

A 57-year-old man presents with a 1-year history of low back pain radiating to the buttocks with walking that is relieved by rest. He also has a 1-year history of erectile dysfunction and decreased pulses. Physical exam reveals decreased pulses. The most likely diagnosis is aortoiliac occlusion, also known as Leriche syndrome, a type of peripheral artery disease presenting with claudication, erectile dysfunction, and decreased pulses caused by narrowing of the iliac arteries. Differential diagnoses such as herniated disc, discitis, pyelonephritis, and spinal stenosis were excluded based on presentation.

Uploaded by

1140271
Copyright
© © All Rights Reserved
We take content rights seriously. If you suspect this is your content, claim it here.
Available Formats
Download as PDF, TXT or read online on Scribd
You are on page 1/ 286

Exam Section 1: ltem 1 of 50 National Board of Medical Examiners

Comprehensive Clinical Science Self-Assessment

"I 1. A57-year-old man comes to the physician because of a 1-year history of low back pain that radiates to the buttocks when he walks. He has not had leg pain or pain at rest. The pain is relieved within 1
minute when he stops walking. He has a 1-year history of erectile dysfunction. He has smoked one pack of cigarettes daily for 30 years. His pulse is 85/min, and blood pressure is 160/100 mm Hg.
Flexion of either hip with the knee extended does not cause pain. Pedal pulses are decreased. Sensation is intact. His hematocrit is 40%, erythrocyte sedimentation rate is 20 mm/h, and serum glucose
concentration is 180 mg/dL. Which of the following is the most likely diagnosis?

A) Aortoiliac occlusion
B) Dissecting aortic aneurysm
C) Herniated nucleus pulposus
D) Lumbar discitis
E ) Pyelonephritis
F) Spinal stenosis
Correct Answer: A.

Aortoiliac occlusion, also known as iliac occlusive disease (Leriche syndrome), is a specific subtype of peripheral arterial disease (PAD) that presents with claudication, erectile dysfunction, and decreased
lower extremity arterial pulses. Claudication refers to muscular pain that develops with exercise and improves with rest caused by the impaired ability of the narrowed arteries to provide adequate blood flow
to muscles during exercise. PAD can progress lo cause chronic extremity ischemia, rest pain, ulceration, and gangrene. Diagnosis is typically made by evaluation of ankle brachia! systolic pressure indices,
with a score of less than 0.90 being indicative of PAD, or vascular imaging. Risk factors for development include obesity, hypertension, hyperlipidemia, tobacco use, diabetes mellitus, male sex, older age,
and concomitant arterial disease elsewhere. Treatment is supportive, although severe occlusive disease may be treated with surgical revascularization. This patient presents with low back and buttock
claudication, erectile dysfunction, and diminished peripheral pulses, making aortoiliac occlusion the most likely diagnosis.

Incorrect Answers: B, C, D, E, and F.

Dissecting aortic aneurysm (Choice B) commonly presents with the acute onset of chest or abdominal pain that radiates to the back associated with a blood pressure differential in the extremities. Other
symptoms and complications are dependent on the extent and location of dissection and can include stroke, end-stage kidney disease, mesenteric ischemia, heart murmur or bruit, hypotension, and
syncope. X-rays may show a widened mediastinum. Dissecting aortic aneurysms are surgical emergencies associated with high morbidity and mortality and would not likely cause ongoing lower extremity
symptoms for more than a year.

Herniated nucleus pulposus (Choice C) can cause radicular back pain as a result of nerve root compression. It is commonly associated with acute onset back pain that radiates down the leg, often after
some degree of straining while lifting or bending. On physical examination, a straight leg raise will reproduce the symptoms. This patient's straight leg raise was normal, making a herniated nucleus pulposus
unlikely.

Lumbar discitis (Choice D) is an infection or inflammation of an intervertebral disc in the lumbar region. It would be expected to present with fever, localized back pain without radiation, limited range of
motion as a result of pain, leukocytosis, and increased inflammatory markers.

Pyelonephritis (Choice E) is an infection of the kidney, most commonly secondary to an ascending urinary tract infection. It presents with fever, dysuria, hematuria, urinary frequency and urgency, and flank
pain. It can also cause sepsis with associated hvpotension and shock. This patient has no sians or symptoms of infection.

https://t.me/USMLENBME2CK ti
Next Score Report Lab Values Calculator Help Pause
Exam Section 1: ltem 1 of 50 National Board of Medical Examiners
Comprehensive Clinical Science Self-Assessment

Lumbar discitis
E ) Pyelonephritis
F) Spinal stenosis
Correct Answer: A.

Aortoiliac occlusion, also known as iliac occlusive disease (Leriche syndrome), is a specific subtype of peripheral arterial disease (PAD) that presents with claudication, erectile dysfunction, and decreased
lower extremity arterial pulses. Claudication refers to muscular pain that develops with exercise and improves with rest caused by the impaired ability of the narrowed arteries to provide adequate blood flow
to muscles during exercise. PAD can progress to cause chronic extremity ischemia, rest pain, ulceration, and gangrene. Diagnosis is typically made by evaluation of ankle brachia! systolic pressure indices,
with a score of less than 0.90 being indicative of PAD, or vascular imaging. Risk factors for development include obesity, hypertension, hyperlipidemia, tobacco use, diabetes mellitus, male sex, older age,
and concomitant arterial disease elsewhere. Treatment is supportive, although severe occlusive disease may be treated with surgical revascularization. This patient presents with low back and buttock
claudication, erectile dysfunction, and diminished peripheral pulses, making aortoiliac occlusion the most likely diagnosis.

Incorrect Answers: B, C, D, E, and F.

Dissecting aortic aneurysm (Choice B) commonly presents with the acute onset of chest or abdominal pain that radiates to the back associated with a blood pressure differential in the extremities. Other
symptoms and complications are dependent on the extent and location of dissection and can include stroke, end-stage kidney disease, mesenteric ischemia, heart murmur or bruit, hypotension, and
syncope. X-rays may show a widened mediastinum. Dissecting aortic aneurysms are surgical emergencies associated with high morbidity and mortality and would not likely cause ongoing lower extremity
symptoms for more than a year.

Herniated nucleus pulposus (Choice C) can cause radicular back pain as a result of nerve root compression. It is commonly associated with acute onset back pain that radiates down the leg, often after
some degree of straining while lifting or bending. On physical examination, a straight leg raise will reproduce the symptoms. This patient's straight leg raise was normal, making a herniated nucleus pulposus
unlikely.

Lumbar discitis (Choice D) is an infection or inflammation of an intervertebral disc in the lumbar region. It would be expected to present with fever, localized back pain without radiation, limited range of
motion as a result of pain, leukocytosis, and increased inflammatory markers.

Pyelonephritis (Choice E) is an infection of the kidney, most commonly secondary to an ascending urinary tract infection. It presents with fever, dysuria, hematuria, urinary frequency and urgency, and flank
pain. It can also cause sepsis with associated hypotension and shock. This patient has no signs or symptoms of infection.

Spinal stenosis (Choice F) presents with neurogenic, rather than vascular, claudication. This claudication, or pain, worsens with exercise, standing, or certain positions but is relieved by a change in position
to sitting or lying. The pain does not improve solely with rest.

Educational Objective: Aortoiliac occlusion, also known as iliac occlusive disease (Leriche syndrome), is a specific peripheral arterial disease that presents with low back and buttocks claudication, erectile
dysfunction, and decreased or absent lower extremity arterial pulses. Vascular claudication refers to muscular pain that occurs with exercise and improves with rest caused by the inability of the narrowed
vessels to provide adequate blood flow to the exercising muscles. Risk factors for development include obesity, hypertension, hyperlipidemia, tobacco use, diabetes mellitus, male sex, older age, and
concomitant arterial disease elsewhere.

https://t.me/USMLENBME2CK ti
Next Score Report Lab Values Calculator Help Pause
Exam Section 1: Item 2 of 50 National Board of Medical Examiners
Comprehensive Clinical Science Self-Assessment

"I 2. A 60-year-old female store clerk comes to the office for an annual health maintenance examination. Her medical history is unremarkable except for gastroesophageal reflux disease, for which she takes
ranitidine at bedtime with good relief. Her last menstrual period was 6 years ago. Mammogram 5 years ago and Pap smear 2 years ago were normal. She does not exercise regularly. She eats a regular
diet. She has smoked one pack of cigarettes daily for 40 years and drinks one to two glasses of wine weekly. Her family history is significant only for diet-controlled diabetes mellitus in her mother. The
patient is 157 cm (5 ft 2 in) tall and weighs 68 kg (150 lb); BMI is 27 kg/m?. Her waist-to-hip ratio is 1.05. Cardiopulmonary, abdominal, and genitourinary examinations are normal for her age and sex
assigned at birth. This patient's greatest risk of mortality at this time is from which of the following?

A) Breast cancer
B) Cardiovascular disease
C) Cervical cancer
D) Hip fracture
E) Lung cancer
Correct Answer: B.

Cardiovascular disease is the leading cause of mortality in the United States and the risk significantly increases for women following the menopausal transition. Female hormones have cardioprotective
associations, and the changes in sex hormones that occur with menopause reduce this protection. In the absence of other significant medical comorbidities or specific risk factors, this patient's greatest risk
of mortality is from cardiovascular disease. Other factors that contribute to increased cardiovascular disease risk include smoking, sedentary lifestyle, hypertension, obesity, hypercholesterolemia, age, and
family history of cardiovascular disease. In addition to routine screening, this patient would benefit from smoking cessation, healthy diet and regular exercise, and monitoring of blood pressure, hemoglobin
ye. and lipid concentrations.
Incorrect Answers: A, C, D, and E.

Breast cancer (Choice A) is a common cause of morbidity and mortality in women. Breast cancer risk is associated with endogenous estrogen concentrations, which decrease in postmenopausal women
compared to premenopausal women with similar background characteristics. Exposure to exogenous hormone therapy increases breast cancer risk in postmenopausal women.

Cervical cancer (Choice C) is more common in women who have human papillomavirus (HPV) infection, particularly the strains HPV-16 and HPV-18. A history of multiple sexual partners, tobacco use, or
immunosuppression also places patients at increased risk.

Hip fracture (Choice 0) risk increases as patients age and does increase significantly in postmenopausal women, as decreased estrogen concentrations increase the rate of bone resorption over bone
formation. This leads to decreased bone mineral density (osteopenia and osteoporosis), which increases the risk for hip fracture. Hip fracture is associated with increased mortality in patients over age 60
years, though cardiovascular disease comprises a significantly higher burden.

Lung cancer (Choice E) is one of the top causes of mortality behind cardiovascular disease worldwide. Some studies have shown an association between hormone therapy for menopause and an increased
risk for developing lung cancer.

Educational Objective: Cardiovascular disease is the leading cause of mortality in the United States, and the risk significantly increases for postmenopausal women as a result of the loss of cardioprotective
benefits from estrogens. Modifiable risk factors include diet, activity levels, management of comorbidities, and smoking cessation.

https://t.me/USMLENBME2CK ts e t
Previous Next Score Report Lab Values Calculator Help pause
Exam Section 1: Item 2 of 50 National Board of Medical Examiners
Comprehensive Clinical Science Self-Assessment
%% te_

diet. She has smoked one pack of cigarettes daily for 40 years and drinks one to two glasses of wine weekly. Her family history is significant only for diet-controlled diabetes mellitus in her mother. The
patient is 157 cm (5 ft 2 in) tall and weighs 68 kg (150 lb); BMI is 27 kg/m?. Her waist-to-hip ratio is 1.05. Cardiopulmonary, abdominal, and genitourinary examinations are normal for her age and sex
assigned at birth. This patient's greatest risk of mortality at this time is from which of the following?

A) Breast cancer
B) Cardiovascular disease
C) Cervical cancer
D) Hip fracture
E) Lung cancer
Correct Answer: B.

Cardiovascular disease is the leading cause of mortality in the United States and the risk significantly increases for women following the menopausal transition. Female hormones have cardioprotective
associations, and the changes in sex hormones that occur with menopause reduce this protection. In the absence of other significant medical comorbidities or specific risk factors, this patient's greatest risk
of mortality is from cardiovascular disease. Other factors that contribute to increased cardiovascular disease risk include smoking, sedentary lifestyle, hypertension, obesity, hypercholesterolemia, age, and
family history of cardiovascular disease. In addition to routine screening, this patient would benefit from smoking cessation, healthy diet and regular exercise, and monitoring of blood pressure, hemoglobin
Aye. and lipid concentrations.

Incorrect Answers: A, C, D, and E.

Breast cancer (Choice A) is a common cause of morbidity and mortality in women. Breast cancer risk is associated with endogenous estrogen concentrations, which decrease in postmenopausal women
compared to premenopausal women with similar background characteristics. Exposure to exogenous hormone therapy increases breast cancer risk in postmenopausal women.

Cervical cancer (Choice C) is more common in women who have human papillomavirus (HPV) infection, particularly the strains HPV-16 and HPV-18. A history of multiple sexual partners, tobacco use, or
immunosuppression also places patients at increased risk.

Hip fracture (Choice 0) risk increases as patients age and does increase significantly in postmenopausal women, as decreased estrogen concentrations increase the rate of bone resorption over bone
formation. This leads to decreased bone mineral density (osteopenia and osteoporosis), which increases the risk for hip fracture. Hip fracture is associated with increased mortality in patients over age 60
years, though cardiovascular disease comprises a significantly higher burden.

Lung cancer (Choice E) is one of the top causes of mortality behind cardiovascular disease worldwide. Some studies have shown an association between hormone therapy for menopause and an increased
risk for developing lung cancer.

Educational Objective: Cardiovascular disease is the leading cause of mortality in the United States, and the risk significantly increases for postmenopausal women as a result of the loss of cardioprotective
benefits from estrogens. Modifiable risk factors include diet, activity levels, management of comorbidities, and smoking cessation.

https://t.me/USMLENBME2CK ti e
Previous Next Score Report Lab Values Calculator Help pause
Exam Section 1: ltem 3 of 50 National Board of Medical Examiners
Comprehensive Clinical Science Self-Assessment

3. A 36-year-old man is brought to the emergency department after he developed chest pain and dyspnea while attempting a 10K run. He has had mild asthma for several years but he takes no
medications for it. He says that he has developed a nonproductive cough recently. He has not had fever, but he has noticed a decrease in his appetite and a 2.5-kg (5-lb) weight loss during the past 6
weeks. Vital signs are pulse 100/min, respirations 26/min, and blood pressure 100/72 mm Hg. Pulse oximetry on room air shows an oxygen saturation of 92%. Breath sounds are diminished on the
right and there is increased resonance on percussion. Chest x-ray is shown. Which of the following is the most appropriate treatment at this time?

A) Endotracheal intubation
B) Nebulized albuterol
C) Observation and administration of nasal oxygen
f)) Teraootemw

https://t.me/USMLENBME2CK ti
Previous Next Score Report Lab Values Calculator Help pause
Exam Section 1: ltem 3 of 50 National Board of Medical Examiners
Comprehensive Clinical Science Self-Assessment

A 36-year-old man is brought to the emergency department after he developed chest pain and dyspnea while attempting a 10K run. He has had mild asthma for several years but he takes no
medications for it. He says that he has developed a nonproductive cough recently. He has not had fever, but he has noticed a decrease in his appetite and a 2.5-kg (5-lb) weight loss during the past 6
weeks. Vital signs are pulse 100/min, respirations 26/min, and blood pressure 100/72 mm Hg. Pulse oximetry on room air shows an oxygen saturation of 92%. Breath sounds are diminished on the
right and there is increased resonance on percussion. Chest x-ray is shown. Which of the following is the most appropriate treatment at this time?

A) Endotracheal intubation
B) Nebulized albuterol
C) Observation and administration of nasal oxygen
D) Thoracotomy
E) Tube thoracostomy
Correct Answer: E.

Pneumothorax results from air entering the intrapleural space from either an external (eg, stab wound) or internal (eg, ruptured bleb or bronchus, bronchopleural fistula) source. As air accumulates within
the space, the lung parenchyma becomes compressed, resulting in reduced breath sounds and respiratory distress. In severe cases, mediastinal structures may be stretched or compressed, compromising
venous return, resulting in obstructive shock (known as a tension pneumothorax). On physical examination, a pneumothorax presents as unilateral diminished breath sounds with hyperresonance to
percussion. Pneumothorax can appear on chest x-ray as radiolucent air and the absence of interstitial lung markings in the pleural space adjacent to the lung parenchyma. If a pneumothorax is large, tube
thoracostomy should be performed to evacuate the air in the pleural space to aid in lung re-expansion. Once a chest tube is placed and healing of the inciting injury occurs, leakage of air into the pleural
space should stop. If a persistent air leak is noted, the source of the leak should be investigated. A ruptured bronchus requires extended time to heal and is a potential source of air leak. If the leak persists,
surgical correction of the rupture may be required to prevent chronic pneumothorax from a resultant bronchopleural fistula.

Incorrect Answers: A, B, C, and D.

Endotracheal intubation (Choice A) is indicated in patients who are apneic, hypoxic, or hypercapnic; not responding to less invasive means; or are unable to protect their airway (eg, lethargic, no gag or
cough reflex). However, in patients with pneumothorax, intubation can worsen intrapleural air and lead to tension pneumothorax. If a pneumothorax is known, tube thoracostomy should be performed prior
to intubation if the patient requires intubation.

Nebulized albuterol (Choice 8) would be indicated to treat asthma or chronic obstructive pulmonary disease exacerbation. In either exacerbation, diffuse expiratory wheezes would be heard on auscultation.
This patient's clinical presentation and x-ray are more suggestive of a pneumothorax.

Observation and administration of nasal oxygen (Choice C) can be lrialed in patients with small or moderate pneumothoraces, which can sometimes self-resolve. Large pneumothoraces are unlikely to self-
resolve and usually require tube thoracostomy. Generally, oxygen is supplied via a nonrebreather mask to promote lung healing and blood flow regardless of oxygen saturation, not via nasal cannula.

Thoracotomy (Choice D) is an invasive procedure that allows for control of thoracic hemorrhage, decompression of cardiac tamponade, or management of aortic or pulmonary hemorrhage. Thoracotomy
may be indicated in cases of penetrating trauma with hemodynamic instability or in cases of cardiac arrest of less than 15 minutes duration with witnessed loss of signs of life. This patient is
hemodynamically stable and should be managed by tube thoracostomy, not thoracotomy.

Educational Objective: Pneumothorax results from air entering the intrapleural space from either an external (eg, stab wound) or internal (eg, ruptured bleb or bronchus, bronchopleural fistula) source. Tube
thoracostomy should be performed in large pneumothoraces to evacuate the air in the pleural space to aid in lung re-expansion.

https://t.me/USMLENBME2CK ti e
Previous Next Score Report Lab Values Calculator Help pause
Exam Section 1: ltem 4 of 50 National Board of Medical Examiners
Comprehensive Clinical Science Self-Assessment

"I 4. A 15-year-old girl is brought lo the emergency department by her mother because of a 2-week history of heavy vaginal bleeding. Menarche was at the age of 14 years. Menses occur at irregular 21- to
35-day intervals and last 10 days with moderate flow. She has never been sexually active. She is not in distress. She is at the 55th percentile for height, 70th percentile for weight, and 60th percentile for
BMI. Her temperature is 38C (100.4F), and blood pressure is 90/55 mm Hg. Examination shows pallor. Laboratory studies show a hemoglobin concentration of 9 g/dl, leukocyte count of 5500/mm3,
and platelet count of 15,000/mm3. Which of the following is the most likely diagnosis?

A) Cervical ectropion
B) Endometrial polyp
C) Immune thrombocytopenic purpura
D) Submucous myoma
E) Thrombotic thrombocytopenic purpura
Correct Answer: C.

Immune thrombocytopenic purpura (ITP) occurs secondary to an autoimmune process leading to the destruction of circulating platelets followed by the subsequent development of thrombocytopenia. Signs
and symptoms include petechiae, purpura, mucosa! bleeding, menorrhagia, and prolonged bleeding from minor injuries. The underlying cause is incompletely understood but involves formation of antiplatelet
lgG antibodies with subsequent platelet opsonization and phagocytosis by splenic macrophages. The diagnosis is one of exclusion after other potential causes of thrombocytopenia, such as viral infection,
hematologic malignancy, and adverse effects of medication have been ruled out. Bone marrow biopsy will most likely show an increased number of megakaryocytes indicating adequate platelet production
but increased peripheral destruction of platelets. Platelets appear normal on peripheral blood smear. Coagulation studies are also normal, although bleeding lime is prolonged. ITP often occurs following an
infection, frequently with Helicobacter pylori, Epstein-Barr virus, or HIV, and is generally self-limited. Chronic ITP is first treated with steroids, intravenous immunoglobulin, or immunomodulatory drugs.
Refractory ITP is treated with splenectomy, which results in remission of thrombocytopenia in most cases.

Incorrect Answers: A, B, D, and E.

Cervical ectropion (Choice A) occurs when there is slight eversion of the cervix, exposing a portion of the endocervix and its columnar mucosa to the vaginal canal. In young women, it is a normal variant and
does not warrant treatment, although it does increase the risk for acquiring sexually transmitted infections such as Chlamydia trachomatis, Neisseria gonorrhoeae, and HIV. While it may lead to abnormal
vaginal bleeding, it does not manifest with thrombocytopenia.

Endometrial polyps (Choice B) may cause uterine bleeding that is irregular, unpredictable, or not associated with usual menstrual signs and symptoms (eg, bloating, breast fullness, uterine cramps). It does
not cause lhrombocytopenia.

Submucous myoma (Choice 0), also known as leiomyomata uteri or uterine fibroids, are benign neoplasms of the uterine myometrium. They are a potential cause of abnormal uterine bleeding and pelvic
pain, but do not cause thrombocytopenia.

Thrombotic thrombocytopenic purpura (Choice E) presents with fever, microangiopathic hemolytic anemia, thrombocytopenia, renal dysfunction, and neurologic abnormalities, although not all these features
may be present. This patient lacks all these features with the exception of thrombocytopenia.

Educational Objective: ITP results from the immune-mediated destruction of circulating platelets. It presents with thrombocytopenia, mucosal bleeding, and petechiae or purpura from cutaneous hemorrhage.
4ooo4 4 ,4 et 44. .u+

https://t.me/USMLENBME2CK ts e t
Previous Next Score Report Lab Values Calculator Help pause
Exam Section 1: ltem 4 of 50 National Board of Medical Examiners
Comprehensive Clinical Science Self-Assessment

' 44i . • • • • • • • • • • • • • •• • • • ••
.

A) Cervical ectropion
B) Endometrial polyp
C) Immune thrombocytopenic purpura
D) Submucous myoma
E) Thrombotic thrombocytopenic purpura
Correct Answer: C.

Immune thrombocytopenic purpura (ITP) occurs secondary to an autoimmune process leading to the destruction of circulating platelets followed by the subsequent development of thrombocytopenia. Signs
and symptoms include petechiae, purpura, mucosal bleeding, menorrhagia, and prolonged bleeding from minor injuries. The underlying cause is incompletely understood but involves formation of antiplatelet
lgG antibodies with subsequent platelet opsonization and phagocytosis by splenic macrophages. The diagnosis is one of exclusion after other potential causes of thrombocytopenia, such as viral infection,
hematologic malignancy, and adverse effects of medication have been ruled out. Bone marrow biopsy will most likely show an increased number of megakaryocytes indicating adequate platelet production
but increased peripheral destruction of platelets. Platelets appear normal on peripheral blood smear. Coagulation studies are also normal, although bleeding lime is prolonged. ITP often occurs following an
infection, frequently with Helicobacter pylori, Epstein-Barr virus, or HIV, and is generally self-limited. Chronic ITP is first treated with steroids, intravenous immunoglobulin, or immunomodulatory drugs.
Refractory tTP is treated with splenectomy, which results in remission of thrombocytopenia in most cases.

Incorrect Answers: A, B, D, and E.

Cervical ectropion (Choice A) occurs when there is slight eversion of the cervix, exposing a portion of the endocervix and its columnar mucosa to the vaginal canal. In young women, it is a normal variant and
does not warrant treatment, although it does increase the risk for acquiring sexually transmitted infections such as Chlamydia trachomatis, Neisseria gonorrhoeae, and HIV. While it may lead to abnormal
vaginal bleeding, it does not manifest with thrombocytopenia.

Endometrial polyps (Choice B) may cause uterine bleeding that is irregular, unpredictable, or not associated with usual menstrual signs and symptoms (eg, bloating, breast fullness, uterine cramps). It does
not cause thrombocytopenia.

Submucous myoma (Choice 0), also known as leiomyomata uteri or uterine fibroids, are benign neoplasms of the uterine myometrium. They are a potential cause of abnormal uterine bleeding and pelvic
pain, but do not cause thrombocytopenia.

Thrombotic thrombocytopenic purpura (Choice E) presents with fever, microangiopathic hemolytic anemia, thrombocytopenia, renal dysfunction, and neurologic abnormalities, although not all these features
may be present. This patient lacks all these features with the exception of thrombocytopenia.

Educational Objective: ITP results from the immune-mediated destruction of circulating platelets. It presents with thrombocytopenia, mucosal bleeding, and petechiae or purpura from cutaneous hemorrhage.
Platelets appear normal on peripheral blood smear. Coagulation studies are normal, although bleeding time will be prolonged. Treatment includes steroids, intravenous immunoglobulin, or splenectomy, if
refractory.

https://t.me/USMLENBME2CK ti
Previous Next Score Report Lab Values Calculator Help pause
Exam Section 1: Item 5 of 50 National Board of Medical Examiners
Comprehensive Clinical Science Self-Assessment

"I 5. A49-year-old man comes to the office because of a 2-week history of dark urine. He has had a 5-kg (11-lb) weight loss during the past month despite no change in appetite. He has no history of serious
illness and takes no medications. His temperature is 37.5°C (99.5F), pulse is 80/min, respirations are 12/min, and blood pressure is 132/86 mm Hg. Examination shows conjunctival icterus. A 5-cm,
nontender mass is palpated in the right upper quadrant of the abdomen. Serum studies show a total bilirubin concentration of 6 mg/dl, with a direct component of 5.2 mg/dl, and a lactate
dehydrogenase activity of 480 U/l. Abdominal ultrasonography shows an enlarged gallbladder with no calculi, a 2-cm-wide common bile duct with no calculi that tapers at the distal end, and a 5-cm
mass in the head of the pancreas. A CT scan of the abdomen shows a 5-cm mass in the head of the pancreas compressing the superior mesenteric vein and partially encasing the superior mesenteric
artery. Results of a fine-needle biopsy show hyperchromatic, pleomorphic cells with a high nucleus-to-cytoplasm ratio. Further diagnostic studies show no evidence of metastatic disease. Which of the
following factors is most predictive of a poor prognosis in this patient?

A) Bilirubin concentration
B ) Patient's age
C) Patient's sex assigned at birth
D) Tumor encasement of the superior mesenteric vessels
E) Tumor location in the head of the pancreas rather than the tail
Correct Answer: D.

Pancreatic cancer is associated with a high mortality rate, as it commonly presents after becoming locally invasive or metastatic. It classically presents with fatigue, unexplained weight loss, and painless
jaundice. Physical examination may disclose hepatomegaly, cachexia, and a right upper quadrant or epigastric mass on palpation of the abdomen (distended, nontender gallbladder). Results of laboratory
studies frequently show conjugated hyperbitirubinemia, indicative of cholestasis secondary to compression by a pancreatic head carcinoma, as seen in this patient. For tumors arising in the body or tail of the
pancreas, migratory superficial thrombophlebitis, as well as arterial and venous emboli, can occur. Occasionally, pancreatic cancer is discovered as an incidental finding on an imaging study used in the
evaluation of another pathology. Risk factors for its development include smoking, obesity, and a family history or genetic predisposition, among others. Treatment with surgical resection is the best curative
option. A mass isolated to the pancreas is more likely to be resectable and thus associated with an improved prognosis. One contraindication to surgical resection is the encasement of the superior
mesenteric artery, which is seen in this patient. As such, this patient cannot undergo surgical resection of the tumor and has increased mortality given local advancement and involvement of vital major
vessels.

Incorrect Answers: A, B, C, and E.

Bilirubin concentration (Choice A) is not directly related to pancreatic cancer prognosis. Increased bilirubin concentration is caused by compression of the common bile duct by the tumor, causing conjugated
hyperbilirubinemia. The bilirubin concentration is dependent on mass size and the level of compression, but it gives no information about the local spread of disease.

Patient's age (Choice 8) by itself does not put him at increased risk of mortality as a result of his pancreatic cancer. He is relatively young and would likely tolerate a major operation required to surgically
resect the mass if it was not contraindicated. If he were elderly, his age may worsen his prognosis, but to a lesser degree than the local spread of his cancer.

Patient's sex assigned al birth (Choice C) is not associated with a worse prognosis. The factor most associated with prognosis is the cancer stage, with metastases to distant locations conferring the worst
prognosis. Local invasion of the tumor, especially when it prevents surgical resection, is associated with worsened prognosis.

Tumor location in the head of the pancreas rather than the tail (Choice E) is associated with improved prognosis. This is most commonly a result of pancreatic body and tail tumors presenting al a later stage,
i 4

https://t.me/USMLENBME2CK ti
Previous Next Score Report Lab Values Calculator Help pause
Exam Section 1: Item 5 of 50 National Board of Medical Examiners
Comprehensive Clinical Science Self-Assessment

following factors is most predictive of a poor prognosis in this patient?

A) Bilirubin concentration
B ) Patient's age
C) Patient's sex assigned at birth
D) Tumor encasement of the superior mesenteric vessels
E) Tumor location in the head of the pancreas rather than the tail
Correct Answer: D.

Pancreatic cancer is associated with a high mortality rate, as it commonly presents after becoming locally invasive or metastatic. It classically presents with fatigue, unexplained weight loss, and painless
jaundice. Physical examination may disclose hepatomegaly, cachexia, and a right upper quadrant or epigastric mass on palpation of the abdomen (distended, nontender gallbladder). Results of laboratory
studies frequently show conjugated hyperbilirubinemia, indicative of cholestasis secondary to compression by a pancreatic head carcinoma, as seen in this patient. For tumors arising in the body or tail of the
pancreas, migratory superficial thrombophlebitis, as well as arterial and venous emboli, can occur. Occasionally, pancreatic cancer is discovered as an incidental finding on an imaging study used in the
evaluation of another pathology. Risk factors for its development include smoking, obesity, and a family history or genetic predisposition, among others. Treatment with surgical resection is the best curative
option. A mass isolated to the pancreas is more likely to be resectable and thus associated with an improved prognosis. One contraindication to surgical resection is the encasement of the superior
mesenteric artery, which is seen in this patient. As such, this patient cannot undergo surgical resection of the tumor and has increased mortality given local advancement and involvement of vital major
vessels.

Incorrect Answers: A, B, C, and E.

Bilirubin concentration (Choice A) is not directly related to pancreatic cancer prognosis. Increased bilirubin concentration is caused by compression of the common bile duct by the tumor, causing conjugated
hyperbilirubinemia. The bilirubin concentration is dependent on mass size and the level of compression, but it gives no information about the local spread of disease.

Patient's age (Choice B) by itself does not put him at increased risk of mortality as a result of his pancreatic cancer. He is relatively young and would likely tolerate a major operation required to surgically
resect the mass if it was not contraindicated. If he were elderly, his age may worsen his prognosis, but to a lesser degree than the local spread of his cancer.

Patient's sex assigned at birth (Choice C) is not associated with a worse prognosis. The factor most associated with prognosis is the cancer stage, with metastases to distant locations conferring the worst
prognosis. Local invasion of the tumor, especially when it prevents surgical resection, is associated with worsened prognosis.

Tumor location in the head of the pancreas rather than the tail (Choice E) is associated with improved prognosis. This is most commonly a result of pancreatic body and tail tumors presenting at a later stage,
often after they have already metastasized.

Educational Objective: Pancreatic cancer is associated with significant morbidity and mortality, as it often presents at an advanced stage. The most common presentation is fatigue, unexplained weight loss,
and painless jaundice. The curative treatment is surgical resection, with contraindications including metastases and vascular encasement. The inability to surgically resect pancreatic cancer is associated
with a poor prognosis.

https://t.me/USMLENBME2CK ti
Previous Next Score Report Lab Values Calculator Help pause
Exam Section 1: Item 6 of 50 National Board of Medical Examiners
Comprehensive Clinical Science Self-Assessment

"I 6. A 60-year-old man comes to the emergency department 6 hours after the onset of shortness of breath associated with a nonproductive cough. One year ago, he was diagnosed with mild hypertension.
His pulse is 100/min, respirations are 26/min, and blood pressure is 140/105 mm Hg. Ophtha!moscopic examination shows arteriolar narrowing but no papilledema. There is no peripheral edema. An S:i,
S and crackles at the bases of both lungs are heard, and jugular veins are distended. Which of the following is the most likely diagnosis?
A) Cardiac amyloidosis
B) Chronic aortic dissection
C) Congestive heart failure
D) Constrictive pericarditis
E) Renal artery stenosis
Correct Answer: C.

This patient's history, signs, symptoms, and examination findings are consistent with acute congestive heart failure. On physical examination, there is an S4 gallop suggestive of a stiff and noncompliant left
ventricle and an Sg suggestive of reduced emptying of the ventricular chambers. Jugular venous distension and pulmonary crackles are suggestive of impaired forward flow of blood. Retinal arteriolar
narrowing is associated with chronic hypertension. Hypertension is a common cause of congestive heart failure and is considered a modifiable risk factor. Chronic hypertension may result in left ventricular
hypertrophy as a result of constant work against an increased afterload. The thickened myocardium results in impaired left ventricular relaxation and increased diastolic filling pressures that lead to left atrial
enlargement and right-sided heart failure over lime.

Incorrect Answers: A, B, D, and E.

Cardiac amyloidosis (Choice A) is an infiltrative cardiomyopathy caused by the deposition of amyloid protein throughout the extracellular space of the heart. Advanced disease results in restrictive
cardiomyopathy and development of heart failure. It is less common than congestive heart failure caused by chronic hypertension and ischemic heart disease.

Chronic aortic dissection (Choice B) refers to dissection that has been present for greater than 90 days and has undergone vascular remodeling. Complications include acute rupture, which presents with
tearing pain radiating to the back, and malperfusion, with symptoms dependent on the affected vascular beds. This patient's physical examination findings are not consistent with chronic aortic dissection.

Constrictive pericarditis (Choice 0) typically presents with sharp chest pain that is aggravated by inspiration and alleviated by sitting and leaning forward. A coarse, sandpaper-like friction rub may be heard
on cardiac auscultation. Physical examination findings also include fever, hypotension, tachycardia, pulsus paradoxus, and the Kussmaul sign (an increase in jugular venous distension during inspiration).

Renal artery stenosis (Choice E) is a cause of secondary hypertension as a result of excessive production of renin and angiotensin. It can result from atherosclerosis in older patients and may be a cause of
acute congestive heart failure. Hypertension in renal artery stenosis tends to be resistant lo treatment, and multiple antihypertensive agents are needed.

Educational Objective: Congestive heart failure is a commonly encountered clinical syndrome associated with chronic hypertension and coronary artery disease. Physical examination findings can include
elevated jugular venous pressure, peripheral edema, ascites, hepatojugular reflux, pulmonary crackles, and an Sa or S, gallop.

https://t.me/USMLENBME2CK ti
Previous Next Score Report Lab Values Calculator Help pause
Exam Section 1: Item 7 of 50 National Board of Medical Examiners
Comprehensive Clinical Science Self-Assessment

✓ 7. Immediately after delivery to a 17-year-old primigravid patient, a 500-g (1-lb 2-oz) male newborn fails to breathe spontaneously and has a pulse of 30/min. The mother had no prenatal care and says
that her last menstrual period was 2 or 3 months ago. Examination of the newborn shows edematous hands and feet and dark red skin without lanugo. The eyelids are fused. He is hypotonic. All four
extremities are in extension. The nipples are barely visible, and the testes are undescended. After several minutes of bag-valve-mask ventilation with 100% oxygen and chest compressions, the pulse
falls below 30/min, and there are still no spontaneous respirations. Which of the following is the most appropriate next step in management?

A) Continued bag-valve-mask ventilation and chest compressions until the pulse is absent
B) Administration of surfactant
C) Endotracheal intubation and continued bag-valve ventilation with 100% oxygen
D) Placement of an umbilical venous catheter for infusion of epinephrine
E) Discontinuation of resuscitative efforts
Correct Answer: E.

This newborn patient has undergone premature delivery, likely during the first or early second trimester of pregnancy, as evidenced by the newborn's size, incompletely developed eyes, nipples, and
extremities, and lack of lanugo (body hair that typically develops in the 4th or 5th month of gestation). Premature delivery in early pregnancy results in stillbirth caused by incomplete development of the
neonatal lungs and brain, which do not complete their maturation until the end of the second trimester. The diagnosis of death in newborns requires a careful examination, which may be complicated by the
fact that many reflexes are not fully developed, and ocular examination (as well as examination of corneal, vestibular, and oculocephalic reflexes) may be impossible. Apnea testing is a reliable method for
diagnosing death and involves providing adequate oxygenation via ventilator, followed by observation of a rise in carbon dioxide levels caused by lack of spontaneous respirations as the ventilator is
weaned. EEG is not usually necessary, but, if performed, demonstrates electrocerebral silence, or lack of electrical waveforms. In the setting of death in a nonviable, premature newborn, it is appropriate to
discontinue resuscitative efforts. Generally, any fetus of less than gestational age 22 to 23 weeks (depending on the particular center caring for the child) will be nonviable, and resuscitation efforts would be
inappropriate.

Incorrect Answers: A, B, C, and D.

Continued bag-valve-mask ventilation and chest compressions until the pulse is absent (Choice A) is incorrect. This newborn demonstrates a lack of spontaneous respirations as a result of prematurity and
incomplete lung and brainstem development. Maintaining ventilation is not possible within this setting, and it is not necessary to continue ventilation and chest compressions until the pulse is absent.

Administration of surfactant (Choice B) is unlikely to be successful. Apnea in this setting is caused not only by incomplete lung maturation, but also incomplete brainstem development. Endotracheal
intubation and continued bag-valve ventilation with 100% oxygen (Choice C) is unlikely to be successful for similar reasons.

Placement of an umbilical venous catheter for infusion of epinephrine (Choice 0) is incorrect. Umbilical venous catheters can be useful for obtaining vascular access in newborns. However, it is
inappropriate to administer epinephrine as part of continued resuscitative efforts given that this newborn demonstrates clear evidence of death.

Educational Objective: Diagnosing death in newborns requires a careful examination of cranial reflexes, apnea, lack of responsiveness, and may be supported by EEG. In the setting of death in a
nonviable, premature newborn, it is appropriate to discontinue resuscitative efforts.

https://t.me/USMLENBME2CK ti e
Previous Next Score Report Lab Values Calculator Help pause
Exam Section 1: Item 8 of 50 National Board of Medical Examiners
Comprehensive Clinical Science Self-Assessment

Fifteen days after sustaining third-degree burns of the abdomen and lower extremities, a hospitalized 67-year-old man has fever. He is intubated and mechanically ventilated. He is receiving cefazolin,
lorazepam, and atenolol. His temperature is 38.9°C (102F), pulse is 90/min, and blood pressure is 110/70 mm Hg. A small amount of cloudy fluid is draining around the nasogastric tube. The burn
wounds are clean. The lungs are clear to auscultation. The abdomen is soft. Rectal examination shows no abnormalities. Femoral pulses are intact. Laboratory studies show a hematocrit of 35% and
leukocyte count of 15,000/mm3. A chest x-ray shows no abnormalities; endotracheal and nasogastric tubes are in proper position. Which of the following is the most appropriate next step in diagnosis?

A) CT angiography of the chest


B) CT scan of the abdomen and pelvis
C) CT scan of the sinuses
D) MRI of the anorectal region
E) MRI of the brain
Correct Answer: C.

Acute, nosocomial sinusitis can develop in patients in the intensive care unit. Both endotracheal tubes and nasogastric tubes can decrease sinus drainage, promoting the accumulation of secretions and
growth/introduction of bacterial pathogens. Bacterial sinusitis commonly presents with fever and purulent nasal drainage, as well as cough and headache. CT scan of the sinuses is used to make the
diagnosis in critically ill patients, with opacified sinuses indicative of sinusitis. Management includes antibiotic therapy targeted toward the causative organism based on sinus fluid cultures, as well as
removing endotracheal and nasogastric tubes, if possible, to allow drainage. Operative sinus drainage may be necessary. This patient has a fever, leukocytosis, and purulent nasal drainage in the setting of
an endotracheal and nasogastric tube with no other signs of infection. He should therefore undergo a CT scan of the sinuses.

Incorrect Answers: A, B, D, and E.

CT angiography of the chest (Choice A) is used to evaluate for pulmonary emboli or vascular malformations in the chest. While pulmonary emboli can cause fever, they commonly cause tachycardia,
hypoxia, and pleuritic chest pain. Based on physical examination, the more likely cause of fever in this patient is acute nosocomial sinusitis.

CT scan of the abdomen and pelvis (Choice B) can be used to evaluate for intra-abdominal sources of infection, such as abscesses, pyelonephritis, or cholecystitis. This patient has symptoms of a systemic
infection with fever and leukocytosis but has had no intra-abdominal interventions and no other signs of intra-abdominal infection, such as tenderness to palpation or hematuria. Therefore, with the presence
of purulent nasal discharge, acute sinusitis is a more likely cause of his fever.

MRI of the anorectal region (Choice D) can be used to evaluate the soft tissues of the anus and rectum. It can be used in the evaluation of anal or rectal cancer, fistulas, pilonidal lesions, and anal fissures.
This patient has no abnormalities on rectal examination and is unlikely to have developed a cutaneous fistula or pilonidal lesion during his hospital stay, making an MRI of the anorectal region inappropriate.

MRI of the brain (Choice E) is commonly used in the evaluation of neurologic malignancies, ischemic and hemorrhagic strokes, and congenital malformations, although it can also be used to evaluate for
intracerebral infections, such as brain abscesses. This patient has no focal neurologic deficits to suggest an intracranial lesion. Given his nasogastric tube placement and purulent nasal discharge, acute
sinusitis is a more likely cause of his fever.

Educational Objectives: Both endotracheal tubes and nasogastric tubes can cause acute sinusitis in critically ill patients as a result of impaired sinus drainage. Symptoms include fever, cough, headache,
and purulent nasal drainage. Diagnosis is made with CT scan of the sinuses, which will show opacification of the sinuses, and culture of the sinus fluid. Management is with antibiotic therapy and removal of
the obstruction, although operative sinus drainage may also be required.

https://t.me/USMLENBME2CK ts e t
Previous Next Score Report Lab Values Calculator Help pause
Exam Section 1: Item 9 of 50 National Board of Medical Examiners
Comprehensive Clinical Science Self-Assessment

"I 9. A 13-month-old boy is brought to the physician for a follow-up examination. He completed a 10-day course of amoxicillin for an episode of pneumonia 10 days ago. He has had four episodes of
pneumonia since birth and had severe chickenpox at the age of 8 months. He is below the 5th percentile for length and weight. His temperature is 37.4°C (99.3°F), pulse is 120/min, respirations are
30/min, and blood pressure is 94/50 mm Hg. Examination shows no abnormalities except for oral thrush. Laboratory studies show a leukocyte count of 1500/mm3 (50% segmented neutrophils, 8%
eosinophils, 1% basophils, 35% lymphocytes, and 6% monocytes), a C04+ T-lymphocyte count of 225/mm3 (Norma1>500), and a low lymphocyte proliferative response. Which of the following is the
most likely diagnosis?

A) Combined immunodeficiency
B) Cystic fibrosis
C) Hypogammaglobulinemia
0) Thymic-parathyroid dysplasia (OiGeorge syndrome)
E) X-linked agammaglobulinemia
Correct Answer: A.

Combined immunodeficiency describes a syndrome of failed 8- and T-lymphocyte activity leading to impaired cellular and humeral immunity. There are a variety of causes of combined immunodeficiency.
Adenosine deaminase deficiency is a frequent underlying cause. Patients present with numerous infections in early life, and characteristically manifest a combination of viral, bacterial, and fungal infections.
Common infections include otitis media, oral thrush, pneumonia, gastroenteritis, and cutaneous fungal infections. Infants may demonstrate failure to thrive or developmental delay. The diagnosis is
established through immune cell counts, mitogen assay, and genetic testing. Chest x-ray may also show the absence of a thymic shadow. Treatment is aimed toward appropriately preventing and treating
infections. Currently, the only definitive treatment is bone marrow transplantation, although genetic therapies are under development in clinical trials.

Incorrect Answers: B, C, 0, and E.

Cystic fibrosis (Choice B) is a syndrome resulting from defects in the CFTR gene, leading to a deficiency in a chloride channel that secretes chloride in the lungs and gastrointestinal tract, and reabsorbs
chloride in sweat glands. Characteristic findings include bronchiectasis, recurrent pulmonary infections, pancreatic insufficiency, diarrhea, malnutrition, and weight loss. Patients with cystic fibrosis are not
particularly vulnerable to extrapulmonary infections.

Hypogammaglobulinemia (Choice C) is often a result of failed 8 lymphocyte development and is seen in conditions such as X-linked agammaglobulinemia. X-linked agammaglobulinemia (Choice E) occurs
as a result of mutations in the BTK gene that encodes a tyrosine kinase essential for all stages of B lymphocyte development and proliferation. Patients with X-linked agammaglobulinemia lack functioning B
lymphocytes and therefore have recurrent infections caused by impaired humeral immunity. They do not typically demonstrate susceptibility to viral or fungal infections, which are typically combated by T
lymphocytes.

Thymic-parathyroid dysplasia (OiGeorge syndrome) (Choice 0) presents with thymic hypoplasia or aplasia and consequent impaired immunity, conotruncal cardiac defects, hypocalcemia caused by
parathyroid hypoplasia, and craniofacial abnormalities.

Educational Objective: Combined immunodeficiency describes a syndrome of failed B- and T-lymphocyte activity leading to impaired cellular and humeral immunity. Patients present with numerous infections
in early life, and characteristically manifest a combination of viral, bacterial, and fungal infections.

https://t.me/USMLENBME2CK ti e
Previous Next Score Report Lab Values Calculator Help pause
Exam Section 1: ltem 10 of 50 National Board of Medical Examiners
Comprehensive Clinical Science Self-Assessment

"I 10. Twenty-eight hours after undergoing uncomplicated elective right total knee replacement, a 76-year-old woman is alert and fully oriented but has nausea and pain in her right hip. Eight hours later, she
is found unresponsive. She has hypertension. Her preadmission medication is lisinopril. Promethazine, hydromorphone, and enoxaparin were added to the regimen postoperatively. Her temperature is
37°C (98.6F), pulse is 90/min, respirations are 6/min, and blood pressure is 90/50 mm Hg. Physical examination shows a clean surgical site with no drainage or bleeding. On neurologic examination,
she does not respond to auditory stimuli. She responds to painful stimuli with a faint grimace. The eyes are closed; when the physician manually opens the patient's eyes, the pupils are 3 mm. There
are no spontaneous movements of the extremities. The extremities do not move with painful stimuli. Which of the following is most likely to show the cause of this patient's change in level of
consciousness?

A) Arterial blood gas analysis


B) Complete blood count
C) Complete metabolic profile
D) CT scan of the head
E) Pulmonary CT angiography
Correct Answer: A.

Opioid medications, such as hydromorphone, are useful in the treatment of postoperative pain. However, they are associated with several adverse effects and thus administration needs to be carefully
titrated and monitored. One adverse effect associated with significant morbidity and mortality is respiratory depression, which occurs with the overdose of opioid pain medications. Respiratory depression can
cause hypercarbia, which can lead to altered mental status, coma, and seizures. Opioid overdose can also lead to respiratory arrest and death. Given this patient's respiratory rate of 6 breaths per minute,
hydromorphone overdose leading to hypoventilation and hypercarbia is the most likely cause of her change in level of consciousness. This is further supported by miosis on examination, a common sign of
opioid intoxication. Hypercarbia secondary to hydromorphone use would most easily be identified on an arterial blood gas analysis, making this the best next step in determining the underlying cause of this
patient's unresponsiveness.

Incorrect Answers: B, C, D, and E.

Complete blood count (Choice B) may indicate anemia if hemoglobin concentrations are decreased, or infection if leukocytosis is present. Severe anemia may result in unresponsiveness, but this would
commonly present with profound hypotension and tachycardia, which this patient does not have. Similarly, severe infection, especially of the central nervous system, could produce unresponsiveness but
would be expected to present with a fever. Thus, hydromorphone overdose leading to respiratory depression is the more likely cause.

Complete metabolic profile (Choice C) may show metabolic abnormalities, such as hypernatremia or hyponatremia, that may lead to altered mental status or unresponsiveness. However, these are more
commonly seen with volume depletion or the syndrome of inappropriate antidiuretic hormone secretion, of which this patient has no other signs. Additionally, electrolyte abnormalities would be unlikely to
explain her respiratory rate of 6 breaths per minute.

CT scan of the head (Choice 0) can show intracranial processes that may lead to altered levels of consciousness, such as hemorrhage. This patient's only risk factor for stroke is hypertension. Additionally,
she has no focal neurologic signs to suggest stroke. Thus, a CT scan of the head would be unlikely to reveal the cause of her unresponsiveness.

Pulmonary CT angiography (Choice E) is used in the diagnosis of pulmonary emboli. These most commonly present with dyspnea, tachypnea, tachycardia, and hypoxia. Hypotension can occur if they are
large. This patient has bradypnea, making a pulmonary embolism unlikely.

https://t.me/USMLENBME2CK ti
Previous Next Score Report Lab Values Calculator Help pause
Exam Section 1: ltem 10 of 50 National Board of Medical Examiners
Comprehensive Clinical Science Self-Assessment

. .
consciousness?

A) Arterial blood gas analysis


B) Complete blood count
C) Complete metabolic profile
D) CT scan of the head
E) Pulmonary CT angiography
Correct Answer: A.

Opioid medications, such as hydromorphone, are useful in the treatment of postoperative pain. However, they are associated with several adverse effects and thus administration needs to be carefully
titrated and monitored. One adverse effect associated with significant morbidity and mortality is respiratory depression, which occurs with the overdose of opioid pain medications. Respiratory depression can
cause hypercarbia, which can lead to altered mental status, coma, and seizures. Opioid overdose can also lead to respiratory arrest and death. Given this patient's respiratory rate of 6 breaths per minute,
hydromorphone overdose leading to hypoventilation and hypercarbia is the most likely cause of her change in level of consciousness. This is further supported by miosis on examination, a common sign of
opioid intoxication. Hypercarbia secondary to hydromorphone use would most easily be identified on an arterial blood gas analysis, making this the best next step in determining the underlying cause of this
patient's unresponsiveness.

Incorrect Answers: B, C, D, and E.

Complete blood count (Choice B) may indicate anemia if hemoglobin concentrations are decreased, or infection if leukocytosis is present. Severe anemia may result in unresponsiveness, but this would
commonly present with profound hypotension and tachycardia, which this patient does not have. Similarly, severe infection, especially of the central nervous system, could produce unresponsiveness but
would be expected to present with a fever. Thus, hydromorphone overdose leading to respiratory depression is the more likely cause.

Complete metabolic profile (Choice C) may show metabolic abnormalities, such as hypernatremia or hyponatremia, that may lead to altered mental status or unresponsiveness. However, these are more
commonly seen with volume depletion or the syndrome of inappropriate anlidiuretic hormone secretion, of which this patient has no other signs. Additionally, electrolyte abnormalities would be unlikely to
explain her respiratory rate of 6 breaths per minute.

CT scan of the head (Choice 0) can show intracranial processes that may lead to altered levels of consciousness, such as hemorrhage. This patient's only risk factor for stroke is hypertension. Additionally,
she has no focal neurologic signs lo suggest stroke. Thus, a CT scan of the head would be unlikely to reveal the cause of her unresponsiveness.

Pulmonary CT angiography (Choice E) is used in the diagnosis of pulmonary emboli. These most commonly present with dyspnea, tachypnea, tachycardia, and hypoxia. Hypotension can occur if they are
large. This patient has bradypnea, making a pulmonary embolism unlikely.

Educational Objective: Opioid medications can lead to respiratory depression if not carefully titrated and monitored. This respiratory depression causes hypercarbia, which can result in altered mental status
and unresponsiveness. If untreated, it can lead to respiratory arrest and death. Hypercarbia is most easily diagnosed with arterial blood gas analysis.

https://t.me/USMLENBME2CK ti
Previous Next Score Report Lab Values Calculator Help pause
Exam Section 1: Item 11 of 50 National Board of Medical Examiners
Comprehensive Clinical Science Self-Assessment

"I 11. A 78-year-old woman comes to the physician because of a 2-week history of mild right knee pain, especially when she walks up stairs. During this time, she also has had stiffness in her knee in the
morning. She enjoys walking around her neighborhood but has been unable to walk because of the pain. She has gastroesophageal reflux disease and takes omeprazole occasionally. Vital signs are
within normal limits. On examination, active and passive range of motion of the right knee is full; there is moderate crepitus and no effusion. Which of the following is the most appropriate
pharmacotherapy?

A) Acetaminophen
B) Prednisone
C) Colchicine
D) Ibuprofen
E) Ciprofloxacin
Correct Answer: A.

Acetaminophen is the most appropriate medication to administer to this patient who most likely has osteoarthritis (OA). Older patients commonly develop OA in the fingers and in weight-bearing joints, such
as the knees and hips. The characteristics of the pain are important in differentiating OA from inflammatory arthritis, such as rheumatoid arthritis, which can also present in older patients. OA pain tends to
worsen throughout the day with continued use, whereas inflammatory pain tends to be worse in the morning and improve after about 1 hour of continual use. Brief stiffness in the morning may occur with OA
but typically resolves within about 1 hour. The diagnosis is clinical, although plain film x-rays may be helpful and often show Joint space narrowing, osteophyte formation, and subchondral sclerosis. First-line
treatment of OA includes acetaminophen. Other medications include nonsteroidal anti-inflammatory drugs (NSAIDs), although these should be avoided in patients with a history of gastrointestinal bleeding or
chronic kidney disease. Topical NSAID preparations are another viable alternative that provides relief to some patients without the systemic effects of oral NSAIDs.

Incorrect Answers: B, C, D, and E.

Prednisone (Choice B) is a viable treatment for acute gout attacks in patients who are unable to take NSAIDs or colchicine. It has little to no effect on OA and is associated with numerous adverse effects.

Colchicine (Choice C) is used to treat inflammatory arthritis and is classically used in the treatment of gout. It inhibits microtubule formation, thereby preventing white blood cell translocation into the Joint,
which limits joint space inflammation and associated arthropathy. It plays no role in the management of OA.

Ibuprofen (Choice D) is a reasonable alternative therapy for OA in patients who do not respond to acetaminophen. Acetaminophen is preferable for first-line treatment given the adverse effect profile of
NSAIDs, which includes peptic ulcers and nephrotoxicity.

Ciprofloxacin (Choice E) is incorrect. Antibiotics may be warranted for the treatment of septic arthritis, which typically presents with fever, Joint swelling, erythema, calor, and effusion. First-line treatment is
primarily against the most frequent causative organisms, gram-positive cocci, and includes intravenous ceftriaxone or vancomycin, rather than ciprofloxacin.

Educational Objective: Osteoarthritis (OA) is a common condition leading to joint pain in older patients. OA pain tends to worsen throughout the day with continued use, whereas inflammatory pain tends to
be worse in the morning and improve after about 1 hour of continual use. First-line therapy is acetaminophen.

https://t.me/USMLENBME2CK ti e
Previous Next Score Report Lab Values Calculator Help pause
Exam Section 1: Item 12 0f 50 National Board of Medical Examiners°
Comprehensive Clinical Science Self-Assessment

✓ 12. A meta-analysis of nine small, randomized-controlled trials is performed to evaluate the efficacy of inhaled corticosteroid therapy in patients with a persistent cough
after an upper respiratory tract infection. Results of one of the trials show a statistically significant benefit with an average reduction in cough of 5 days in the treated
group as compared with the control group. Two other studies show p-values for benefit of 0.07. The results of the studies are shown. Which of the following is the
strongest rationale for performing a meta-analysis?

A) Decrease confounding
B) Decrease selection bias
C) Decrease type I error
D) Increase the confidence interval
E) Increase the power

20 10 0 -10
Reduction of cough
(Days)
Correct Answer: E.

While randomized controlled trials form the gold standard of medical research and generate more powerful evidence than other forms of observational or descriptive studies, they remain susceptible to
error. A common form of error results from limitations in the number of study participants, which in turn limits statistical power, or the ability of the study to detect a difference between groups if one exists.
Increasing the power of a study can generally be completed through a larger sample size and improving precision in measurements. When the power of a study is increased, the incidence of a type II error
decreases. A type II error occurs when a study demonstrates no difference between the null and alternative hypotheses, when in fact, a difference exists. The type II error rate is calculated by subtracting
the power from 100%. A meta-analysis is a form of study in which statistical analysis is performed lo combine the results of multiple studies addressing the same question. Meta-analysis is useful for
overcoming limitations of the constituent studies within the meta-analysis. By pooling the results of multiple studies, meta-analyses reduce the risk for type II error caused by the smaller sample sizes within
each constituent randomized controlled trial. However, meta-analyses are susceptible to their own errors, primarily related to inclusion criteria, statistical methods, and the errors of constituent studies.

Incorrect Answers: A, 8, C, and D.

Decrease confounding (Choice A) is incorrect. Confounding variables influence both the independent and dependent variables and can lead to false associations if the confounding variable is not evenly
distributed in both study groups. Confounding is an inherent error within the structure of a constituent study and is not overcome by incorporation within a meta-analysis.

Decrease selection bias (Choice B) is incorrect. Selection bias occurs when there is a nonrandom or biased selection of individuals for inclusion in a particular study group, or for data analysis. Similar to
confounding, it represents an inherent flaw within an individual study that is not overcome by inclusion within a meta-analysis.

Decrease type I error (Choice C) is incorrect. Type I error represents the likelihood of erroneously rejecting the null hypothesis (concluding that the alternative hypothesis is true). The acceptable rate of a
type I error is a percentage called the alpha level and is set by the investigators. When the alpha level is converted to a decimal, ii can be compared with the p-value of the experiment. An alpha level of 5%
or 0.05 indicates that if the null hypothesis is true, there is a 5% chance that the investigators will erroneously reject it based on the study results. Inclusion within a meta-analysis does not inherently affect
the risk for tvoe I error.
« https://t.me/USMLENBME2CK ts e t
Previous Next Score Report Lab Values Calculator Help pause
Exam Section 1: Item 12 of 50 National Board of Medical Examiners"
Comprehensive Clinical Science Self-Assessment

Increase the confidence interval


E) Increase the power

20 10
• 0 -10
Reduction of cough
(Days)
Correct Answer: E.

While randomized controlled trials form the gold standard of medical research and generate more powerful evidence than other forms of observational or descriptive studies, they remain susceptible to
error. A common form of error results from limitations in the number of study participants, which in turn limits statistical power, or the ability of the study to detect a difference between groups if one exists.
Increasing the power of a study can generally be completed through a larger sample size and improving precision in measurements. When the power of a study is increased, the incidence of a type II error
decreases. A type II error occurs when a study demonstrates no difference between the null and alternative hypotheses, when in fact, a difference exists. The type II error rate is calculated by subtracting
the power from 100%. A meta-analysis is a form of study in which statistical analysis is performed to combine the results of multiple studies addressing the same question. Meta-analysis is useful for
overcoming limitations of the constituent studies within the meta-analysis. By pooling the results of multiple studies, meta-analyses reduce the risk for type II error caused by the smaller sample sizes within
each constituent randomized controlled trial. However, meta-analyses are susceptible to their own errors, primarily related to inclusion criteria, statistical methods, and the errors of constituent studies.

Incorrect Answers: A, 8, C, and D.

Decrease confounding (Choice A) is incorrect. Confounding variables influence both the independent and dependent variables and can lead lo false associations if the confounding variable is not evenly
distributed in both study groups. Confounding is an inherent error within the structure of a constituent study and is not overcome by incorporation within a meta-analysis.

Decrease selection bias (Choice 8) is incorrect. Selection bias occurs when there is a nonrandom or biased selection of individuals for inclusion in a particular study group, or for data analysis. Similar to
confounding, it represents an inherent flaw within an individual study that is not overcome by inclusion within a meta-analysis.

Decrease type I error (Choice C) is incorrect. Type I error represents the likelihood of erroneously rejecting the null hypothesis (concluding that the alternative hypothesis is true). The acceptable rate of a
type I error is a percentage called the alpha level and is set by the investigators. When the alpha level is converted to a decimal, it can be compared with the p-value of the experiment. An alpha level of 5%
or 0.05 indicates that if the null hypothesis is true, there is a 5% chance that the investigators will erroneously reject it based on the study results. Inclusion within a meta-analysis does not inherently affect
the risk for type I error.

Increase the confidence interval (Choice D) is incorrect. Confidence intervals are a measure of the precision of a statistical estimate. A 95% confidence interval indicates the range of outcomes expected on
repeated trials. Confidence intervals are direct results of the sample size and outcomes of a constituent study and are not altered by inclusion within a meta-analysis. In general, well-conducted meta-
analyses have narrower confidence intervals than their constituent studies, representing a greater degree of precision of statistical estimation.

Educational Objective: A meta-analysis is a form of study in which statistical analysis is performed to combine the results of multiple studies addressing the same question. Meta-analysis is useful for
overcoming limitations of the constituent studies within the meta-analysis. By pooling the results of multiple studies, meta-analyses reduce the risk for type II error caused by the smaller sample sizes within
each constituent randomized controlled trial.

« https://t.me/USMLENBME2CK ts e t
Previous Next Score Report Lab Values Calculator Help pause
Exam Section 1: Item 13 of 50 National Board of Medical Examiners
Comprehensive Clinical Science Self-Assessment

"I 13. A 63-year-old woman comes to the office for an examination before undergoing ventral hernia repair. She has a 3-month history of increasingly severe abdominal cramps and low back pain, which are
worse at night, and heartburn, which typically occurs after she pushes her grandson on a swing. She avoids most physical activity because of these symptoms. She has type 2 diabetes mellitus treated
with metformin. Fifteen years ago, she underwent total abdominal hysterectomy for dysfunctional uterine bleeding. She smoked one pack of cigarettes daily for 8 years but quit 35 years ago. She is
160 cm (5 fl 3 in) tall and weighs 72 kg (160 1b); BMI is 28 kg/m2. Vital signs are within normal limits. Cardiopulmonary examination shows no abnormalities. There is truncal obesity. A10x 10-cm,
easily reducible mass is palpable under the midline scar. Which of the following is the most appropriate next step in management?

A) Cardiac stress scintigraphy


B) Pulmonary function testing
C) Recommendation for a low-sodium, low-fat diet
D) Recommendation to wear an abdominal binder
E) Switching from metformin to insulin
Correct Answer: A.

This patient has risk factors for coronary artery disease (age, diabetes mellitus, obesity, prior tobacco use), and the association of her symptoms with exertion warrants further workup. Cardiac stress testing
is the next diagnostic step in management. There are multiple ways the stress test can be performed, and professional society guidelines provide various algorithms on selecting the appropriate test for each
patient. The most important clinical factor to consider is whether the patient can tolerate exercise. Other factors to consider include underlying conduction abnormalities that may make an exercise stress test
dangerous or difficult to interpret. For these patients, a pharmacologic stress test with cardiac imaging is the preferred test. Cardiac stress scintigraphy assesses myocardial perfusion at rest and under
stress, which may be induced by exercise or vasodilators. It is the most sensitive noninvasive test for assessing coronary artery perfusion.

Incorrect Answers: B, C, D, and E.

Pulmonary function testing (Choice B) consists of spirometry, total body plethysmography, and diffusion capacity testing, and is useful in the workup and monitoring of respiratory disorders. This patient's
symptoms are more concerning for a cardiac source.

Recommendation for a low-sodium, low-fat diet (Choice C) alone is not appropriate for this patient. The presence of symptoms with exertion and cardiovascular risk factors warrants further cardiac workup.

Recommendation to wear an abdominal binder (Choice 0) will not address the likely cause of this patient's symptoms. Her abdominal pain and heartburn are likely presenting symptoms of coronary artery
disease, and an abdominal binder will not provide relief.

Switching from metformin to insulin (Choice E) is not indicated, though this patient's hemoglobin A1c should be checked as part of the workup to evaluate glycemic control. Appropriate management of
diabetes mellitus reduces the risk for future cardiovascular disease.

Educational Objective: Patients with symptoms that are consistent with angina pectoris or with nonspecific symptoms exacerbated by exertion should undergo further evaluation with a stress test. Those who
are capable of exercising should undergo either an exercise nuclear stress test or an exercise treadmill stress test.

https://t.me/USMLENBME2CK ts e t
Previous Next Score Report Lab Values Calculator Help pause
Exam Section 1: Item 14 of 50 National Board of Medical Examiners
Comprehensive Clinical Science Self-Assessment

44 Physician A is approaching the end of his shift at the outpatient clinic when he hears the sound of glass breaking in Physician B's office. When Physician A enters Physician B's office to ensure that she is
OK, he sees her cleaning up pieces of a wine glass. Physician B says she bought a bottle of wine earlier today and initially intended to bring it home, but she decided to open it and drink "a little" because
she was stressed following a meeting with the department chair. Physician B says she was just about to go see her last patient of the day. Which of the following is the most appropriate immediate action
by Physician A?

A) Explain the situation to the waiting patient


B) Inform Physician B that another staff physician will see the waiting patient
C) Notify the clinic administrator
D) Report Physician B to the state medical board
E) Report Physician B's behavior to the clinic's Medical Director
Correct Answer: B.

The most appropriate immediate action for physicians who witness that a colleague is intoxicated is to take steps to ensure that their colleague does not continue to see patients while impaired. The witnessing
physician should then report the impairment to the appropriate authorities. Most state medical boards legally require this reporting. States typically provide physician health programs to evaluate, treat, and
monitor physicians with substance use disorders or other impairing mental or physical conditions. The physician health program will assess whether the physician poses a danger to patients and may mandate
that the physician temporarily withdraw from practice. Impaired physicians may present with mood changes, worsening personal hygiene, increased mistakes, decreased productivity, and/or decreased
functioning in social interactions.

Incorrect Answers: A, C, D, and E.

Explaining the situation to the waiting patient (Choice A) would violate your colleague's privacy and is not necessary. It is more appropriate to tell the waiting patient that they will be seen by a different
physician.

Reporting the impairment to the appropriate authorities, such as the state medical board (Choice D) or the clinic's Medical Director (Choice E), is appropriate but must be performed after ensuring that the
impaired physician is no longer seeing patients in the clinic. Physicians should consult their local and state reporting guidelines, which may differ in their reporting requirements. In general, the clinic
administrator (Choice C) is not the appropriate channel for reporting physician impairment.

Educational Objective: The most appropriate immediate action for physicians who witness that a colleague is intoxicated is to take steps to ensure that their colleague does not continue to see patients while
impaired. Physicians should then report the impairment to the appropriate authorities.

https://t.me/USMLENBME2CK ts e t
Previous Next Score Report Lab Values Calculator Help pause
Exam Section 1: Item 15 of 50 National Board of Medical Examiners
Comprehensive Clinical Science Self-Assessment

"I 15. A 65-year-old man comes to the office for follow-up 5 days after discharge from the hospital, where he received treatment for an exacerbation of chronic obstructive pulmonary disease. Medical history
otherwise is unremarkable. Medications are fluticasone-salmeterol, tiotropium, albuterol as necessary, and a 5-day prednisone taper; he also has 4 days remaining of a 7-day course of doxycycline.
Since discharge, he has been taking his medications as directed and has noticed improvement in his breathing and decreased coughing. He smoked one pack of cigarettes daily for 50 years but quit
before his hospital admission. He does not drink alcoholic beverages. Vital signs are pulse 90/min, respirations 20/min, and blood pressure 120/80 mm Hg. Pulse oximetry on room air shows an
oxygen saturation of 94%. Auscultation of the lungs discloses decreased breath sounds with occasional wheezing and no crackles. Cardiac examination discloses a soft S,and S_ There is no
clubbing of the digits or edema of the extremities. In considering this patient's risk for readmission, which of the following is the most appropriate management?

A) Incentive spirometry
B) Long-term prednisone therapy
C) Oxygen therapy
D) Pulmonary rehabilitation
E) Suppressive azithromycin therapy three limes weekly
Correct Answer: D.

Pulmonary rehabilitation is recommended for patients with Global Initiative for Chronic Obstructive Lung Disease (GOLD) group B, C, or D chronic obstructive pulmonary disease (COPD) as it has been
shown to improve symptoms and quality of life as well as reduce hospital readmissions for COPD exacerbations. These are patients who experience moderate to severe COPD symptoms or are at high risk
for exacerbations and hospitalization. Pulmonary rehabilitation is primarily exercise training to improve conditioning and strengthen respiratory muscles, but many programs also offer lifestyle and behavioral
counseling as well as psychological and social support. Many patients with COPD also benefit from breathing retraining to improve ventilation by avoiding rapid, shallow breathing. This reduces air trapping
and dead space ventilation.

Incorrect Answers: A, B, C, and E.

Incentive spirometry (Choice A) is useful to prevent pulmonary complications in postoperative patients and improve ventilation in patients with atelectasis. It does not have a significant role in routine
management of COPD.

Long-term prednisone therapy (Choice B) is not recommended for management of COPD because of serious adverse effects with chronic use and increased risk for morbidity and mortality. Short-term use of
systemic glucocorticoids is standard in treatment of acute exacerbations.

Oxygen therapy (Choice C) is strongly recommended for COPD patients with chronic, exertional, or nocturnal hypoxemia, as it has been shown to improve overall survival in patients with peripheral oxygen
saturations of 88% or less. This patient demonstrates adequate oxygenation, and oxygen therapy is unlikely to provide benefit at this time.

Suppressive azilhromycin therapy three times weekly (Choice E) is considered for patients with severe COPD who continue to have exacerbations despite maximal bronchodilator therapy. Azithromycin
provides both antimicrobial and anti-inflammatory benefits, though it is reserved for specific clinical situations because of the risks of antibiotic resistance and long-term cardiovascular effects.

Educational Objective: Pulmonary rehabilitation is recommended for patients with Global Initiative for Chronic Obstructive Lung Disease (GOLD) group 8, C, or D chronic obstructive pulmonary disease
(COPD). These are patients who experience moderate to severe COPD symptoms or are at high risk for exacerbations and hospitalization. It has been shown to improve symptoms and quality of life as well
as reduce hospital readmissions and durations of stay for exacerbations.

https://t.me/USMLENBME2CK ts e t
Previous Next Score Report Lab Values Calculator Help pause
Exam Section 1: ltem 16 of 50 National Board of Medical Examiners
Comprehensive Clinical Science Self-Assessment

✓ 16. A 32-year-old primigravid woman at 16 weeks' gestation comes to the physician for a prenatal visit. She has bipolar disorder. Her medications are lithium carbonate and a prenatal vitamin. Physical
examination shows a uterus consistent in size with a 16-week gestation. On mental status examination, she is cooperative and fully oriented. She has a euthymic mood and congruent affect. Thought
processes are linear. She reports reading about the possible adverse effects of mood stabilizers on the developing fetus and asks if she should stop taking lithium carbonate. Which of the following is
the most appropriate recommendation regarding this patient's medication regimen during pregnancy?

A) Discontinue mood stabilizer pharmacotherapy


B) Switch from lithium carbonate to carbamazepine
C) Switch from lithium carbonate to clozapine
0) Switch from lithium carbonate to valproic acid
E) Continue the current regimen
Correct Answer: E.

Lithium is a mood stabilizing medication utilized to treat acute mania and as maintenance treatment in bipolar disorder to prevent the recurrence of manic or depressive episodes. Lithium readily crosses
the blood-placental barrier. There is mixed evidence that lithium is associated with an increase in the rate of cardiac malformations if used during the first trimester. In particular, there may be an
approximately 1- to 2-fold increase in the rate of Ebstein anomaly, in which the tricuspid valve leaflets are displaced toward the right ventricle, with subsequent right ventricular atrialization and increased
risk for supraventricular tachycardia or Wolff-Parkinson-White syndrome. However, the overall rate of Ebstein anomaly remains low even when lithium is continued during the first trimester, and the risk for
potential fetal abnormalities must be weighed against the risks to both the mother and fetus associated with acute manic or depressive episodes, which may include death. Data on long-term developmental
outcomes following fetal lithium exposure is scarce. This patient presents after the first trimester, al which time, organogenesis has already occurred. In this setting, lithium should be continued to minimize
the risk for exacerbation of the patient's bipolar disorder. If continued during pregnancy, careful dose monitoring should be performed, as dose adjustments are often necessary because of lithium's narrow
therapeutic window and changes in body weight and metabolism associated with pregnancy. Fetal cardiac ultrasonography is recommended if lithium is continued during pregnancy.

Incorrect Answers: A, B, C, and D.

Discontinue mood stabilizer pharmacotherapy (Choice A) is not recommended and is associated with a substantial increase in the risk for exacerbations of bipolar disorder and suicide especially in the
setting of pregnancy.

Switch from lithium carbonate to carbamazepine (Choice B) is not recommended. Carbamazepine is a voltage-gated sodium channel blocker and can be used as a mood stabilizer in bipolar disorder or as
an antiepileptic. It is associated with neural tube defects. In general, switching from one mood stabilizer to another is associated with an increased risk for illness exacerbation. This patient has done well on
lithium, is past the first trimester, and should continue lithium treatment.

Switch from lithium carbonate to clozapine (Choice C) is incorrect. Clozapine is an antipsychotic used to treat refractory schizophrenia. Its major toxicities include agranulocytosis, which can be severe and
life threatening in the setting of infection.

Switch from lithium carbonate to valproic acid (Choice D) is incorrect. Valproic acid is an antiepileptic drug and mood stabilizer that is occasionally used to treat patients with schizoaffective or bipolar
disorder. It is associated with cardiac, craniofacial, and developmental fetal abnormalities, and should be avoided during pregnancy.
•----•· - ' ' ·- _, ,. %
_,_ - __ .,. __ ._ --•-----·- l .let f 4

https://t.me/USMLENBME2CK ti
Previous Next Score Report Lab Values Calculator Help pause
Exam Section 1: ltem 16 of 50 National Board of Medical Examiners
Comprehensive Clinical Science Self-Assessment

A) Discontinue mood stabilizer pharmacotherapy


B) Switch from lithium carbonate to carbamazepine
C) Switch from lithium carbonate to clozapine
0) Switch from lithium carbonate to valproic acid
E) Continue the current regimen
Correct Answer: E.

Lithium is a mood stabilizing medication utilized lo treat acute mania and as maintenance treatment in bipolar disorder to prevent the recurrence of manic or depressive episodes. Lithium readily crosses
the blood-placental barrier. There is mixed evidence that lithium is associated with an increase in the rate of cardiac malformations if used during the first trimester. In particular, there may be an
approximately 1- to 2-fold increase in the rate of Ebstein anomaly, in which the tricuspid valve leaflets are displaced toward the right ventricle, with subsequent right ventricular atrialization and increased
risk for supraventricular tachycardia or Wolff-Parkinson-White syndrome. However, the overall rate of Ebstein anomaly remains low even when lithium is continued during the first trimester, and the risk for
potential fetal abnormalities must be weighed against the risks lo both the mother and fetus associated with acute manic or depressive episodes, which may include death. Data on long-term developmental
outcomes following fetal lithium exposure is scarce. This patient presents after the first trimester, at which time, organogenesis has already occurred. In this setting, lithium should be continued to minimize
the risk for exacerbation of the patient's bipolar disorder. If continued during pregnancy, careful dose monitoring should be performed, as dose adjustments are often necessary because of lithium's narrow
therapeutic window and changes in body weight and metabolism associated with pregnancy. Fetal cardiac ultrasonography is recommended if lithium is continued during pregnancy.

Incorrect Answers: A, B, C, and D.

Discontinue mood stabilizer pharmacotherapy (Choice A) is not recommended and is associated with a substantial increase in the risk for exacerbations of bipolar disorder and suicide especially in the
setting of pregnancy.

Switch from lithium carbonate to carbamazepine (Choice B) is not recommended. Carbamazepine is a voltage-gated sodium channel blocker and can be used as a mood stabilizer in bipolar disorder or as
an antiepileptic. It is associated with neural tube defects. In general, switching from one mood stabilizer to another is associated with an increased risk for illness exacerbation. This patient has done well on
lithium, is past the first trimester, and should continue lithium treatment.

Switch from lithium carbonate to clozapine (Choice C) is incorrect. Clozapine is an antipsychotic used to treat refractory schizophrenia. Its major toxicities include agranulocytosis, which can be severe and
life threatening in the setting of infection.

Switch from lithium carbonate to valproic acid (Choice D) is incorrect. Valproic acid is an antiepileptic drug and mood stabilizer that is occasionally used to treat patients with schizoaffective or bipolar
disorder. It is associated with cardiac, craniofacial, and developmental fetal abnormalities, and should be avoided during pregnancy.

Educational Objective: Lithium may be continued during pregnancy, including the first trimester, under specific circumstances in women with bipolar disorder. The risk for potential fetal abnormalities, in
particular Ebstein anomaly, must be discussed with the patient and weighed against the potentially severe risks to both the mother and fetus associated with exacerbations of the psychiatric disorder. Most
data suggest that lithium is safe during pregnancy after the first trimester. Lithium levels should be monitored carefully during pregnancy and fetal cardiac ultrasonography should be performed.

https://t.me/USMLENBME2CK ti
Previous Next Score Report Lab Values Calculator Help pause
Exam Section 1: Item 17 of 50 National Board of Medical Examiners
Comprehensive Clinical Science Self-Assessment

"I 17. A 30-year-old primigravid woman at 10 weeks gestation comes lo the physician for her first prenatal visit. She has polycystic ovarian syndrome. She conceived following clomiphene therapy, and her
only current medication is a prenatal vitamin. She is 165 cm (5 ft 5 in) tall and weighs 82 kg (180 lb); BMI is 30 kg/m2. Examination shows a uterus consistent in size with a 10-week gestation. Which of
the following is the most appropriate recommendation for this patient during her pregnancy?

A) Amniocentesis
B) Cesarean delivery
C) Chorionic villus sampling
D) Limiting pregnancy weight gain
E) Measurement of serum fasting insulin concentration
Correct Answer: D.

The initial prenatal visit is imperative to a healthy pregnancy. It allows the practitioner to assess the patient's current health status, intervene to prevent complications, and educate the patient. During the visit,
the obstetrician can complete a comprehensive history and physical examination, as well as order appropriate laboratory testing and imaging. This visit should specifically include an assessment of
immunization status, substance use, medical problems, family history of heritable diseases, height, weight, and baseline vital signs, such as blood pressure. Recommendations for weight gain during
pregnancy, which can be established at this initial prenatal visit, are based on a patient's prepregnancy BMI. Those who are underweight are recommended to gain 28 to 40 pounds, while those with obesity
are recommended to gain 11 to 20 pounds. This is because patients who are underweight are al increased risk for having newborns who are small for gestational age, whereas patients with obesity are at
increased risk for having newborns who are large for gestational age. Additional weight gain in patients with baseline obesity can also increase the risk for gestational hypertension and preeclampsia,
gestational diabetes, and cesarean delivery. Thus, the most appropriate recommendation for this patient is to limit pregnancy weight gain to the recommended 11 to 20 pounds to prevent pregnancy
complications. This is additionally important given her history of polycystic ovarian syndrome, which increases her risk for diabetes.

Incorrect Answers: A, B, C, and E.

Amniocentesis (Choice A) is used to diagnose a congenital genetic abnormality in the fetus, such as trisomy 18. It may also be used in the evaluation of hemolytic anemia or neural tube defects and can be
used therapeutically for polyhydramnios. As ii is an invasive procedure and associated with risk to the fetus, ii is reserved for diagnostic confirmation after a noninvasive test, such as cell-free DNA or
ultrasonography. This patient currently has no need to undergo amniocentesis.

Cesarean delivery (Choice B) is not indicated at this lime. Cesarean deliveries are the preferred form of delivery for breech presentations or placental abnormalities, though they are also indicated following
arrest of labor. While this patient may require a cesarean delivery, ii is not yet possible to predict the likelihood of her having one.

Chorionic villus sampling (Choice C) is an invasive test used to identify genetic abnormalities of the fetus. As ii carries risk to the fetus, ii is often reserved for further evaluation of abnormalities on aneuploidy
screening or ultrasonography. There is no indication of a genetic abnormality in this patient, making chorionic villus sampling an inappropriate recommendation at this lime.

Measurement of serum fasting insulin concentration (Choice E) is not frequently done in pregnant patients. More commonly, patients undergo oral glucose testing to evaluate for gestational diabetes, which
can be associated with a variety of maternal and fetal complications. This occurs at approximately 24 to 28 weeks of gestation.

,_, - -•-- •--•·-- ,._ --·- o 44ti 4sf44 4lo444 4..


Educational Objective: The initial prenatal visit is imperative to a healthy pregnancy as ii allows obstetricians to complete a comprehensive history and physical examination, as well as order appropriate
±

https://t.me/USMLENBME2CK ts e t
Previous Next Score Report Lab Values Calculator Help pause
Exam Section 1: Item 17 of 50 National Board of Medical Examiners
Comprehensive Clinical Science Self-Assessment

A) Amniocentesis
B) Cesarean delivery
C) Chorionic villus sampling
D) Limiting pregnancy weight gain
E) Measurement of serum fasting insulin concentration
Correct Answer: D.

The initial prenatal visit is imperative to a healthy pregnancy. It allows the practitioner to assess the patient's current health status, intervene to prevent complications, and educate the patient. During the visit,
the obstetrician can complete a comprehensive history and physical examination, as well as order appropriate laboratory testing and imaging. This visit should specifically include an assessment of
immunization status, substance use, medical problems, family history of heritable diseases, height, weight, and baseline vital signs, such as blood pressure. Recommendations for weight gain during
pregnancy, which can be established at this initial prenatal visit, are based on a patient's prepregnancy BMI. Those who are underweight are recommended to gain 28 to 40 pounds, while those with obesity
are recommended to gain 11 to 20 pounds. This is because patients who are underweight are at increased risk for having newborns who are small for gestational age, whereas patients with obesity are at
increased risk for having newborns who are large for gestational age. Additional weight gain in patients with baseline obesity can also increase the risk for gestational hypertension and preeclampsia,
gestational diabetes, and cesarean delivery. Thus, the most appropriate recommendation for this patient is to limit pregnancy weight gain to the recommended 11 to 20 pounds to prevent pregnancy
complications. This is additionally important given her history of polycystic ovarian syndrome, which increases her risk for diabetes.

Incorrect Answers: A, B, C, and E.

Amniocentesis (Choice A) is used to diagnose a congenital genetic abnormality in the fetus, such as trisomy 18. It may also be used in the evaluation of hemolytic anemia or neural tube defects and can be
used therapeutically for polyhydramnios. As it is an invasive procedure and associated with risk to the fetus, it is reserved for diagnostic confirmation after a noninvasive test, such as cell-free DNA or
ultrasonography. This patient currently has no need to undergo amniocentesis.

Cesarean delivery (Choice B) is not indicated at this time. Cesarean deliveries are the preferred form of delivery for breech presentations or placental abnormalities, though they are also indicated following
arrest of labor. While this patient may require a cesarean delivery, it is not yet possible to predict the likelihood of her having one.

Chorionic villus sampling (Choice C) is an invasive test used to identify genetic abnormalities of the fetus. As it carries risk to the fetus, it is often reserved for further evaluation of abnormalities on aneuploidy
screening or ultrasonography. There is no indication of a genetic abnormality in this patient, making chorionic villus sampling an inappropriate recommendation at this time.

Measurement of serum fasting insulin concentration (Choice E) is not frequently done in pregnant patients. More commonly, patients undergo oral glucose testing to evaluate for gestational diabetes, which
can be associated with a variety of maternal and fetal complications. This occurs at approximately 24 to 28 weeks of gestation.

Educational Objective: The initial prenatal visit is imperative to a healthy pregnancy as it allows obstetricians to complete a comprehensive history and physical examination, as well as order appropriate
laboratory testing and imaging. Recommendations for weight gain during pregnancy, which can be established at this initial prenatal visit, are based on a patient's prepregnancy BMI. Patients with obesity
are encouraged to limit pregnancy weight gain, as excessive weight gain is associated with newborns who are large for gestational age, gestational hypertension and preeclampsia, gestational diabetes, and
cesarean delivery.

https://t.me/USMLENBME2CK ts e t
Previous Next Score Report Lab Values Calculator Help pause
Exam Section 1: Item 18 of 50 National Board of Medical Examiners
Comprehensive Clinical Science Self-Assessment

✓ 18. A previously healthy 57-year-old man comes to the emergency department 1 hour after the sudden onset of severe occipital headache. His temperature is 37.2C(99F), pulse is 92/min, respirations
are 18/min, and blood pressure is 180/98 mm Hg. He is mildly lethargic but oriented to person, place, and time. Neurologic examination shows no other abnormalities. A CT scan of the head shows
no abnormalities. Which of the following is the most appropriate next step in management?

A) Measurement of erythrocyte sedimentation rate


B) Measurement of intraocular pressure
C) Carotid duplex ultrasonography
D) MRI of the brain with contrast
E) Intravenous prochlorperazine therapy
F) Cerebrospinal fluid analysis
Correct Answer: F.

Noncontrast CT scan of the head is a highly sensitive, but not perfectly sensitive, method for the detection of subarachnoid hemorrhage (SAH), which must be excluded in the setting of a severe headache
presenting with hypertension. CT scan is most sensitive to detect SAH when performed within 6 hours from the onset of symptoms and may reveal hyperdense blood products in the subarachnoid space or
ventricles. The sensitivity of a CT scan for SAH declines over time from the onset of bleeding or with smaller volume hemorrhages. Patients with a presenting history concerning for SAH who have a
negative CT scan should undergo emergent lumbar puncture for cerebrospinal fluid analysis. Lumbar puncture is useful for identifying findings of SAH, including xanthochromia, increased opening
pressure, and/or an increased cerebrospinal fluid concentration of red blood cells that does not clear across several tubes of collected fluid.

Incorrect Answers: A, B, C, D, and E.

Measurement of erythrocyte sedimentation rate (Choice A) is useful for the diagnosis of temporal arteritis. Temporal arteritis is rare in patients younger than age 50 years. It typically presents with unilateral
or bilateral temporal headaches of subacute onset that are often accompanied by jaw claudication, amaurosis fugax, or sudden vision loss. This patient's history of severe, acute headache is more
consistent with SAH.

Measurement of intraocular pressure (Choice 8) is useful for the diagnosis of acute or chronic glaucoma. Glaucoma occurs as a result of increases in pressure within the anterior chamber of the eye, and
may present as acute, painful or chronic, painless vision loss.

Carotid duplex ultrasonography (Choice C) is useful for the diagnosis of carotid stenosis but has little role in the diagnosis of SAH.

MRI of the brain with contrast (Choice D) has high sensitivity for the detection of SAH, tumor, or abscess, but is a lime-consuming study and will result in a delay in treatment. Lumbar puncture allows for
the more immediate detection of SAH.

Intravenous prochlorperazine therapy (Choice E) would be useful in the treatment of a primary headache such as migraine. However, this patient's history is concerning for SAH, and further diagnostic
evaluation is warranted.

Educational Obiective: Noncontrast CT scan of the head has a hiah but not oerfect rate of sensitivitv for the detection of subarachnoid hemorrhaoe (SAH). Patients with a presentina historv concernina for

https://t.me/USMLENBME2CK ts e t
Previous Next Score Report Lab Values Calculator Help pause
Exam Section 1: Item 18 of 50 National Board of Medical Examiners
Comprehensive Clinical Science Self-Assessment

A) Measurement of erythrocyte sedimentation rate


B) Measurement of intraocular pressure
C) Carotid duplex ultrasonography
D) MRI of the brain with contrast
E) Intravenous prochlorperazine therapy
F) Cerebrospinal fluid analysis
Correct Answer: F.

Noncontrast CT scan of the head is a highly sensitive, but not perfectly sensitive, method for the detection of subarachnoid hemorrhage (SAH), which must be excluded in the setting of a severe headache
presenting with hypertension. CT scan is most sensitive to detect SAH when performed within 6 hours from the onset of symptoms and may reveal hyperdense blood products in the subarachnoid space or
ventricles. The sensitivity of a CT scan for SAH declines over time from the onset of bleeding or with smaller volume hemorrhages. Patients with a presenting history concerning for SAH who have a
negative CT scan should undergo emergent lumbar puncture for cerebrospinal fluid analysis. Lumbar puncture is useful for identifying findings of SAH, including xanthochromia, increased opening
pressure, and/or an increased cerebrospinal fluid concentration of red blood cells that does not clear across several tubes of collected fluid.

Incorrect Answers: A, B, C, D, and E.

Measurement of erythrocyte sedimentation rate (Choice A) is useful for the diagnosis of temporal arteritis. Temporal arteritis is rare in patients younger than age 50 years. It typically presents with unilateral
or bilateral temporal headaches of subacute onset that are often accompanied by jaw claudicalion, amaurosis fugax, or sudden vision loss. This patient's history of severe, acute headache is more
consistent with SAH.

Measurement of intraocular pressure (Choice 8) is useful for the diagnosis of acute or chronic glaucoma. Glaucoma occurs as a result of increases in pressure within the anterior chamber of the eye, and
may present as acute, painful or chronic, painless vision loss.

Carotid duplex ultrasonography (Choice C) is useful for the diagnosis of carotid stenosis but has little role in the diagnosis of SAH.

MRI of the brain with contrast (Choice D) has high sensitivity for the detection of SAH, tumor, or abscess, but is a lime-consuming study and will result in a delay in treatment. Lumbar puncture allows for
the more immediate detection of SAH.

Intravenous prochlorperazine therapy (Choice E) would be useful in the treatment of a primary headache such as migraine. However, this patient's history is concerning for SAH, and further diagnostic
evaluation is warranted.

Educational Objective: Noncontrast CT scan of the head has a high but not perfect rate of sensitivity for the detection of subarachnoid hemorrhage (SAH). Patients with a presenting history concerning for
SAH who have a negative CT scan should undergo emergent lumbar puncture for cerebrospinal fluid analysis, which may reveal xanthochromia, increased opening pressure, and/or a persistently
increased cerebrospinal fluid concentration of red blood cells.

https://t.me/USMLENBME2CK ts e t
Previous Next Score Report Lab Values Calculator Help pause
Exam Section 1: Item 19 of 50 National Board of Medical Examiners
Comprehensive Clinical Science Self-Assessment

"I 19. A45-year-old woman comes to the clinic because of a 2-month history of increasingly severe epigastric pain, bloating, and intermittent nausea and vomiting. During this time, she has had decreased
appetite resulting in a 3.6-kg (8-lb) weight loss. She has hypertension and type 2 diabetes mellitus. The conditions have been poorly controlled during the past 2 years, because she has not been
adherent to her medication regimen. The patient has no history of operative procedures. Medications are metformin, insulin glargine, atorvastatin, lisinopril, and daily aspirin. She appears well. She is
157 cm (5 ft 2 in) tall and weighs 91 kg (200 lb); BMI is 37 kg/m2. Temperature is 37.0°C (98.6°F), pulse is 78/min, respirations are 12/min, and blood pressure is 124/68 mm Hg. The abdomen is
protuberant; there is no organomegaly or tenderness to palpation. Bowel sounds are decreased. Results of a complete blood count, metabolic profile, and determination of serum lipase activity are
within the reference ranges. Which of the following is the most appropriate next step in diagnosis?

A) Endoscopic retrograde cholangiopancreatography


B) Esophageal manometry
C) Gastric emptying study
D) MRI of the abdomen
E) Ultrasonography of the abdomen
Correct Answer: C.

Diabetic neuropathy is a microvascular complication of long-standing diabetes mellitus that typically presents in a "stocking and glove" distribution of the hands and feet. If associated neuropathy of the
autonomic nervous system occurs, this can result in complications such as diabetic gastroparesis or dysautonomia. Gastroparesis typically presents with slowly progressive early satiety, nausea, vomiting of
undigested food, and abdominal bloating with a succussion splash heard during physical examination. The diagnosis can be confirmed with a nuclear medicine gastric emptying study. A gastric emptying
scan is considered the gold standard diagnostic test and is performed by the ingestion of a radiolabeled tracer, commonly technetium 99m, which evaluates the rate of emptying of contents from the
stomach. Gastroparesis may be treated with agents that increase gastrointestinal contraction and motility, such as the D2 receptor antagonist metoclopramide. Erythromycin is also used as it is an agonist of
the hormone motilin.

Incorrect Answers: A, B, D, and E.

Endoscopic retrograde cholangiopancreatography (Choice A) is utilized for the evaluation and treatment of various biliary pathologies. It does not play a role in the evaluation of diabetic gastroparesis.

Esophageal manometry (Choice B) is useful for the diagnosis of achalasia and diffuse esophageal spasm, but ii is not useful for the diagnosis of gastroparesis.

MRI of the abdomen (Choice D) is useful for the diagnosis of certain abdominal malignancies, such as hepatocellular carcinoma, renal carcinoma, or metastases to the liver.

Ultrasonography of the abdomen (Choice E) may demonstrate gastric distension but is not useful for assessing gastric motility.

Educational Objective: Diabetic autonomic neuropathy may cause gastroparesis, which presents with early satiety, nausea, vomiting of undigested food, bloating, and a succussion splash. The diagnosis can
be confirmed with a nuclear medicine gastric emptying study.

https://t.me/USMLENBME2CK ts e t
Previous Next Score Report Lab Values Calculator Help pause
Exam Section 1: Item 20 of 50 National Board of Medical Examiners
Comprehensive Clinical Science Self-Assessment

"I 20. A 54-year-old man comes to the office because of a 3-week history of a dull ache along the back of his left ankle that is worse in the morning when he gets out of bed. The pain also worsens when he
plays tennis on weekends. Acetaminophen once daily has provided moderate relief of the pain. Medical history is unremarkable, and the patient takes no other medications. He is 178 0m (5 ft 10 in) tall
and weighs 99 kg (219 lb); 8MI is 31 kg/m2. Vital signs are within normal limits. The patient appears well. There is mild swelling and tenderness to palpation at the proximal aspect of the Achilles
tendon. The remainder of the examination discloses no abnormalities. Thompson test is negative. In addition to recommending rest, which of the following is the most appropriate next step in
management?

A) Add oxycodone as needed for pain


8) Increase the frequency of acetaminophen to four limes daily
C) Inject cortisone into the calcaneal insertion site of the Achilles tendon
D) Recommend physical therapy
E) Schedule surgical intervention for the Achilles tendon
Correct Answer: D.

Achilles tendinopathy is a common overexertion injury that presents as posterior heel pain. Most heel pain is caused by mechanical musculoskeletal pathology, though it is important to evaluate for other
causes including inflammatory conditions (eg, gout, rheumatoid arthritis), infections (especially in patients with diabetes mellitus or arterial insufficiency), and neurologic disorders (eg, neuroma, lumbosacral
radiculopathy, peripheral nerve entrapment). The location of the pain is the best indicator of the underlying pathology. The Thompson test is used to assess the integrity of the Achilles tendon. A positive test
indicates significant or complete rupture of the Achilles tendon and requires more aggressive management. Mild tendinopathy associated with activity can be treated conservatively with rest, icing,
compression, and over-the-counter nonsteroidal anti-inflammatory drugs. The physician should also recommend physical therapy directed at gentle stretching and strengthening of the affected tendon. The
goal of therapy is a return to activity.

Incorrect Answers: A, 8, C, and E.

Add oxycodone as needed for pain (Choice A) is not indicated for Achilles tendinopathy. Ice, gentle stretching, physical rehabilitation, and nonsteroidal anti-inflammatory drugs are recommended.

Increase the frequency of acetaminophen to four times daily (Choice 8) is not likely to provide as much benefit as physical therapy but may be considered as an adjunct. Nonsteroidal anti-inflammatory drugs
are recommended for short-term relief for patients who do not have a contraindication for their use.

Inject cortisone into the calcaneal insertion site of the Achilles tendon (Choice C), and other injection therapies, may be considered if the patient continues to have symptoms despite appropriate treatment
and engagement in physical therapy, though the evidence for their efficacy is limited. Some studies show an increased risk for tendon rupture after injection with glucocorticoids.

Schedule surgical intervention for the Achilles tendon (Choice E) is not indicated for this degree of injury. Surgery may be considered for cases of complete tendon rupture, though the benefits and risks are
comparable to nonoperative management strategies.

Educational Objective: Heel pain is a common presenting complaint, and the clinician must be careful to distinguish a musculoskeletal overexertion injury from other pathology. Achilles tendinopathy can be
treated with rest, icing, compression, nonsteroidal anti-inflammatory drugs, and physical therapy.

https://t.me/USMLENBME2CK ts e t
Previous Next Score Report Lab Values Calculator Help pause
Exam Section 1: Item 21 of 50 National Board of Medical Examiners
Comprehensive Clinical Science Self-Assessment

"I 21. A 10-year cohort study is conducted to investigate the effect of cigarette smoking on the incidence of head and neck cancers in adults. A total of 10,000 adult participants with no history of any type of
cancer are enrolled in the study; 2000 participants are current cigarette smokers and 8000 are current nonsmokers and have never smoked cigarettes. During the 10-year study period, 20 smokers
and eight nonsmokers develop head and neck cancer. Based on these data, which of the following best represents the attributable risk from smoking for head and neck cancer among current cigarette
smokers?

A) 0.001
8) 0.009
C) 0.01
D) 0.09
E)0.1
Correct Answer: B.

Attributable (excess) risk (AR) describes the risk for developing disease or the outcome under study (in this case, development of head and neck cancers) that can be attributed to the exposure as compared
with the risk that exists without the exposure. AR is calculated as the difference between disease or outcome incidence among those with the exposure and those without the exposure. In biostatistics, the
lowercase letters a, b, c, and dare frequently used to represent numbers of patients exposed to a risk, receiving an intervention, or having a disease. The number of persons having the exposure or
receiving the intervention who have the disease or condition in question is represented by 'a'. The number of persons having the exposure or receiving the intervention who do not have the disease or
condition in question is represented by 'b'. The number of persons who do not have the exposure or who do not receive the intervention in question, and who do have the disease or condition, is represented
by 'c'. The number of persons who do not have the exposure or who do not receive the intervention in question, and who do not have the disease, is represented by 'd'. By this convention, the incidence of
those with the exposure is computed by determining the number of patients with the exposure and who developed disease (a) and dividing it by the total number of patients who were exposed (a+ b). The
incidence of those without the exposure who developed disease is computed by determining the number of patients without the exposure who developed disease (c) and dividing it by the total number of
patients who were not exposed (c + d). AR is defined as AR= (a/ (a+ b)) - (c/(c + d)). In this scenario, the AR can be determined by (20 /(20 + 1980))-(8 /(8 + 7992)) = 20/2000 - 8/8000 = 0.010--0.001
= 0.009.

Incorrect Answers: A, C, D, and E.

0.001 (Choice A) represents an incorrect mathematical assumption involving the calculation of (c/( c + d)), which represents the incidence of those without the exposure who developed disease.

0.01 (Choice C) represents the number of persons with the exposure who developed disease but does not factor out the number who developed it without the exposure.

0.09 (Choice D) involves an incorrect mathematical assumption involving (a/ (a+ b)) - (c/(c + d)) multiplied by the duration of the study in years.

0.1 (Choice E) involves an incorrect mathematical assumption involving (a/ (a + b)) multiplied by the duration of the study in years.

Educational Objective: Attributable risk describes the risk for developing disease or the outcome that can be attributed to the exposure as compared with the risk that exists without the exposure. It is
calculated as the difference between disease or outcome incidence among those with the exposure and those without the exposure.

https://t.me/USMLENBME2CK ti
Previous Next Score Report Lab Values Calculator Help pause
Exam Section 1: Item 22 0f 50 National Board of Medical Examiners°
Comprehensive Clinical Science Self-Assessment

"I 22. A 15-year-old girl is brought to the office for a well-child examination. She has no history of serious illness and takes no medications. Her
temperature is 36.9C(98.4F), pulse is 86/min, respirations are 16/min, and blood pressure is 100/60 mm Hg. A photograph of the back is
shown. The remainder of the examination shows no abnormalities. In addition to routine annual growth and development assessments, which of
the following is the most appropriate recommendation regarding routine screening in this patient?

A) Yearly CT scan of the chest and abdomen


8) Yearly MRI of the brain
C) Yearly nerve conduction studies
D) Yearly ophthalmologic examination
E) No additional screening is indicated

Correct Answer: D.

Neurofibromatosis type 1 is an autosomal dominant neurocutaneous disorder that typically results from an inherited mutation of the NF1 tumor suppressor gene on chromosome 17. Neurofibromatosis type 1
is inherited with complete penetrance and variable expressivity. Rarely, the mutation in the NF1 gene occurs sporadically. The disorder presents with cutaneous neurofibromas (ie, benign neoplasms derived
from neural crest cells), cafe-au-lait spots, axillary or inguinal freckling, pigmented iris hamartomas (eg, Lisch nodules), optic pathway gliomas (OPGs), and/or pheochromocytomas. According to guidelines
from the American Academy of Pediatrics (AAP), patients with neurofibromatosis type 1 require annual, multidisciplinary screening, including ophthalmic, neurologic, and dermatologic examinations, blood
pressure monitoring, and growth and bone health monitoring. Ophthalmic examination is useful for the early detection and treatment of OPGs, which tend to occur before age 6 years. Treatment includes
surgical debulking of tumors, management of seizures and neurologic complications, and genetic counseling.

Incorrect Answers: A, B, C, and E.

The use of routine diagnostic imaging in asymptomatic children with neurofibromatosis type 1, such as with CT scan of the chest and abdomen (Choice A) or MRI of the brain (Choice B) remains
controversial. According to the most recent AAP 2019 guidelines, diagnostic imaging studies should be performed as needed. Routine ophthalmic examination and visual field testing is often sufficiently
sensitive to detect the presence of OPGs.

« https://t.me/USMLENBME2CK ts e t
Previous Next Score Report Lab Values Calculator Help pause
Exam Section 1: Item 22 of 50 National Board of Medical Examiners"
Comprehensive Clinical Science Self-Assessment

D) Yearly ophthalmologic examination


E) No additional screening is indicated

Correct Answer: D.

Neurofibromatosis type 1 is an autosomal dominant neurocutaneous disorder that typically results from an inherited mutation of the NF1 tumor suppressor gene on chromosome 17. Neurofibromatosis type 1
is inherited with complete penetrance and variable expressivity. Rarely, the mutation in the NF1 gene occurs sporadically. The disorder presents with cutaneous neurofibromas (ie, benign neoplasms derived
from neural crest cells), cafe-au-lait spots, axillary or inguinal freckling, pigmented iris hamartomas (eg, Lisch nodules), optic pathway gliomas (OPGs), and/or pheochromocytomas. According to guidelines
from the American Academy of Pediatrics (AAP), patients with neurofibromatosis type 1 require annual, multidisciplinary screening, including ophthalmic, neurologic, and dermatologic examinations, blood
pressure monitoring, and growth and bone health monitoring. Ophthalmic examination is useful for the early detection and treatment of OPGs, which tend to occur before age 6 years. Treatment includes
surgical debulking of tumors, management of seizures and neurologic complications, and genetic counseling.

Incorrect Answers: A, B, C, and E.

The use of routine diagnostic imaging in asymptomatic children with neurofibromatosis type 1, such as with CT scan of the chest and abdomen (Choice A) or MRI of the brain (Choice B) remains
controversial. According to the most recent AAP 2019 guidelines, diagnostic imaging studies should be performed as needed. Routine ophthalmic examination and visual field testing is often sufficiently
sensitive to detect the presence of OPGs.

Yearly nerve conduction studies (Choice C) are not necessary for the detection of new neurofibromas. If a new clinically relevant neurofibroma is suspected based on neurologic symptoms, dedicated
neuroimaging with MRI is the preferred diagnostic modality.

No additional screening (Choice E) is incorrect. Neurofibromatosis type 1 can lead to significant medical complications related to the presence of OPGs and pheochromocytoma if appropriate screening is
not performed.

Educational Objective: Neurofibromatosis type 1 is an autosomal dominant neurocutaneous disorder that presents with cutaneous neurofibromas, cafe-au-lait spots, axillary or inguinal freckling, pigmented
iris hamartomas (eg, Lisch nodules), optic pathway gliomas (OPGs), and/or pheochromocytomas. Annual screening is multidisciplinary and includes ophthalmic screening to detect OPGs.

« https://t.me/USMLENBME2CK ts e t
Previous Next Score Report Lab Values Calculator Help pause
Exam Section 1: Item 23 of 50 National Board of Medical Examiners
Comprehensive Clinical Science Self-Assessment

23. A 7-year-old girl is brought to the physician by her mother because she is concerned that her daughter has had two episodes of vaginal bleeding during the past 3 months. She also has had breast
development during this time. Development had been appropriate for age. She is at the 48th percentile for height and 47th percentile for weight and BMI. Breast development is sexual maturity rating
(SMR) stage 2, and axillary and pubic hair development is SMR stage 3. Examination shows normal-appearing external genitalia. Pelvic examination is deferred. An MRI of the pelvis shows a normal
vagina and cervix. The uterus is 4 cm long, and a 76-0m left ovarian mass is seen. Which of the following is the most appropriate next step in management?

A) Chromosomal analysis
B) Measurement of serum CA 125 concentration
C) CT scan of the pituitary gland
D) Fine-needle aspiration of the mass
E) Left oophorectomy
Correct Answer: E.

Puberty refers to the normal hormonal and physiologic changes that occur as children age. The most common age range for the onset of puberty in females is age 9 to 12 years, although this can vary
markedly among patients based on factors such as body weight and ethnicity. Precocious puberty refers to the onset of secondary sexual characteristics before the age of 8 years in females or 9 years in
males. It can be idiopathic or have a variety of causes, such as central nervous system lesions, ovarian or germ cell tumors, adrenal disorders, or genetic defects such as McCune-Albright syndrome.
Precocious puberty can be further defined as central or peripheral. Central precocious puberty is caused by an early activation of the hypothalamic-pituitary-gonadal axis, whereas peripheral precocious
puberty is caused by the excessive release of estrogens or androgens. Ovarian tumors, such as sex cord-stromal tumors, can present with precocious puberty in children, which is the likely cause in this
patient given the MRI findings. As they may be malignant, the most appropriate next step in management of this patient's condition is left oophorectomy to remove the ovarian mass.

Incorrect Answers: A, B, C, and D.

Chromosomal analysis (Choice A) is not necessary. While some genetic mutations can cause precocious puberty, they are extraordinarily rare and would not be evident on chromosomal analysis.
Additionally, this patient has a large ovarian mass, making this a more likely cause of her precocious puberty.

Measurement of serum CA 125 concentration (Choice 8) is occasionally used in the diagnosis of ovarian cancer. However, ii is nonspecific and would not confirm the diagnosis in this patient. Surgical
resection would allow histologic analysis of the mass and would also likely improve this patient's symptoms.

CT scan of the pituitary gland (Choice C) can be considered in the evaluation of precocious puberty as central nervous system lesions can lead to overactivation of the hypothalamic-pituitary-gonadal axis.
However, this patient has a large mass of the left ovary and would be unlikely to also have an intracranial lesion in the absence of other symptoms.

Fine-needle aspiration of the mass (Choice D) would be a less appropriate option than surgical resection of the mass. In addition to the risk for seeding the abdomen if the mass is malignant, delaying the
time to operative removal of the mass will delay the time until this patient's symptoms resolve. Similarly, if the mass is malignant, a delay in surgical resection increases the risk for metastasis.

Educational Objective: Ovarian tumors, specifically sex cord-stromal tumors, can cause precocious puberty in children. Precocious puberty refers to the onset of secondary sexual characteristics before the
age of 8 years in females or 9 years in males. II can be idiopathic or have a variety of other causes, such as central nervous system lesions, adrenal disorders, or genetic defects such as McCune-Albright
syndrome. When caused by an ovarian tumor, treatment with surgical resection is required.

https://t.me/USMLENBME2CK ts e t
Previous Next Score Report Lab Values Calculator Help pause
Exam Section 1: Item 24 0f 50 National Board of Medical Examiners
Comprehensive Clinical Science Self-Assessment

"I 24. A 3-year-old boy is brought to the physician because of a scaling lesion on the scalp for 7 days. The family has no pets; the day-care center that the child attends has a pet turtle. The other children at the
day-care center are asymptomatic. Examination of the scalp shows a thin plaque with alopecia and scaling. Which of the following is the most appropriate pharmacotherapy to prevent spread to the
children in the day-care center?

A) Oral griseofulvin for the patient only


B) Oral griseofulvin for the patient and his classmates
C) Topical hydrocortisone for the patient only
D) Topical hydrocortisone for the patient and his classmates
E) Topical selenium sulfide for the patient only
F) Topical selenium sulfide for the patient and his classmates
Correct Answer: A.

Tinea capitis is a fungal infection of the scalp that is common in children. It presents as scaly patches with alopecia, patches of alopecia with black dots that represent broken hairs, and cervical
lymphadenopathy. The most common causal organism is Trichophyton tonsurans, although infections can also be caused by other Trichophyton, Microsporum, and Epidermophyton species. A potassium
hydroxide preparation of a skin scraping will show fungal hyphae or spores. It is treated with oral antifungals, such as griseofulvin or terbinafine.

Incorrect Answers: B, C, D, E, and F.

Treatment of classmates (Choices 8, D, and F) is not typically necessary. While tinea capitis may be transmitted through close contacts in schools or day-care centers, or zoonotically, commonly through dogs,
cats, or livestock, it is not necessary to treat close contacts who are otherwise asymptomatic.

Topical hydrocortisone for the patient only (Choice C) is useful for the treatment of atopic dermatitis. It is not useful for the treatment of, and may worsen, tinea capitis.

Topical selenium sulfide for the patient only (Choice E) is incorrect. Selenium sulfide is an antifungal that is used topically as a lotion or shampoo in the treatment of pityriasis versicolor, dandruff, or seborrheic
dermatitis. It is not effective against tinea capitis.

Educational Objective: Tinea capitis presents commonly in children as scaly patches, hair loss, and cervical lymphadenopathy. A frequent causative organism is Trichophyton tonsurans. It is treated with oral
antifungals, such as griseofulvin or terbinafine.

https://t.me/USMLENBME2CK ti
Previous Next Score Report Lab Values Calculator Help pause
Exam Section 1: Item 25 of 50 National Board of Medical Examiners°
Comprehensive Clinical Science Self-Assessment

"I 25. A 5-year-old boy is brought to the physician by his parents as a new patient because they are concerned that he may have an
immunodeficiency. The family recently moved to the area. The parents say their son's previous physician had been examining him for the
condition. The patient has had six episodes of pneumonia and 10 episodes of sinusitis since birth. He underwent uncomplicated surgical
drainage of the sinuses at the ages of 3 and 4 years. He had his first episode of otitis media at the age of 2 months and underwent
uncomplicated placement of tympanostomy tubes al the age of 7 months. His umbilical stump fell off at the age of 2 weeks. The patient's
previous leukocyte, lymphocyte, and platelet counts; serum immunoglobulin and CH50 concentrations; and diphtheria, tetanus,
pneumococcal, and Haemophifus influenzae type b titers are within the reference ranges. Six previous chest x-rays showed lower lobe
consolidations in inconsistent locations; subsequent x-rays showed clearing of the infiltrates. Today, vital signs are within normal limits.
Examination shows no abnormalities. A CT scan of the chest is shown. Which of the following is the most likely diagnosis?

A) Common variable immunodeficiency


B) Complement deficiency
C) Leukocyte adhesion defect
D) Primary ciliary dyskinesia
E) Wiskott-Aldrich syndrome

Correct Answer: D.

Primary ciliary dyskinesia (PCD), also referred to as immotile-cilia syndrome, is a rare congenital disorder characterized by defective cilia and an inability to clear mucus and debris from the airways. There
are numerous mutations that can cause the syndrome, resulting either in impaired ciliary function or absent cilia. Clinical manifestations vary, though most patients will experience recurrent upper and lower
respiratory infections. Other manifestations include chronic rhinosinusitis, otitis media, and infertility. Notably, approximately half of patients with PCD will have situs inversus. Kartagener syndrome refers to
the subset of patients with PCD who have the clinical triad of situs inversus, chronic sinusitis, and bronchiectasis.

Incorrect Answers: A, B, C, and E.

Common variable immunodeficiency (Choice A) classically presents in the second to fourth decades of life as multiple, recurrent sinus or pulmonary infections, including pneumonia, bronchitis, and sinusitis.
Testing will show low immunoglobulin concentrations.

Complement deficiency (Choice 8), particularly of the terminal complement proteins, prevents the formation of the membrane attack complex, which is a critical component in the complement cascade,
typically leading to the formation of a pore in the membrane of pathogenic organisms. Deficiency predisposes patients to recurrent and invasive infections with Neisseria species.

Leukocyte adhesion defect (LAD) (Choice C) disorders are characterized by impaired attachment of leukocytes to the vascular endothelium, and recruitment and migration of leukocytes to sites of
extravascular inflammation or infection is reduced. Laboratory studies in patients with LAD will show increased absolute leukocyte counts in the blood, as the leukocytes are unable to maintain their normal
marginated position outside of the blood stream against the vessel walls.

« https://t.me/USMLENBME2CK ts e t
Previous Next Score Report Lab Values Calculator Help pause
Exam Section 1: Item 25 of 50 National Board of Medical Examiners"
Comprehensive Clinical Science Self-Assessment
«
pneumococcal, and Haemophifus influenzae type b titers are within the reference ranges. Six previous chest x-rays showed lower lobe
consolidations in inconsistent locations; subsequent x-rays showed clearing of the infiltrates. Today, vital signs are within normal limits.
''.\
- ¥ •
, ,·
Examination shows no abnormalities. A CT scan of the chest is shown. Which of the following is the most likely diagnosis? ;
-- •

°':'-•,..
A) Common variable immunodeficiency fl-s
8) Complement deficiency / I • \ -
C) Leukocyte adhesion defect
D) Primary ciliary dyskinesia
E) Wiskott-Aldrich syndrome

Correct Answer: D.

Primary ciliary dyskinesia (PCD), also referred to as immotile-cilia syndrome, is a rare congenital disorder characterized by defective cilia and an inability to clear mucus and debris from the airways. There
are numerous mutations that can cause the syndrome, resulting either in impaired ciliary function or absent cilia. Clinical manifestations vary, though most patients will experience recurrent upper and lower
respiratory infections. Other manifestations include chronic rhinosinusitis, otitis media, and infertility. Notably, approximately half of patients with PCD will have situs inversus. Kartagener syndrome refers to
the subset of patients with PCD who have the clinical triad of situs inversus, chronic sinusitis, and bronchiectasis.

Incorrect Answers: A, B, C, and E.

Common variable immunodeficiency (Choice A) classically presents in the second to fourth decades of life as multiple, recurrent sinus or pulmonary infections, including pneumonia, bronchitis, and sinusitis.
Testing will show low immunoglobulin concentrations.

Complement deficiency (Choice B), particularly of the terminal complement proteins, prevents the formation of the membrane attack complex, which is a critical component in the complement cascade,
typically leading to the formation of a pore in the membrane of pathogenic organisms. Deficiency predisposes patients to recurrent and invasive infections with Neisseria species.

Leukocyte adhesion defect (LAD) (Choice C) disorders are characterized by impaired attachment of leukocytes to the vascular endothelium, and recruitment and migration of leukocytes to sites of
extravascular inflammation or infection is reduced. laboratory studies in patients with LAD will show increased absolute leukocyte counts in the blood, as the leukocytes are unable to maintain their normal
marginated position outside of the blood stream against the vessel walls.

Wiskott-Aldrich syndrome (Choice E) is caused by a defective protein involved in interactions between T lymphocytes, antigen-presenting cells, and B lymphocytes, leading to an impaired innate and
adaptive immune system. Patients present with eczema, thrombocytopenia, and infections with encapsulated bacteria and opportunistic pathogens.

Educational Objective: Primary ciliary dyskinesia is a rare congenital disorder characterized by defective cilia and an inability to clear mucus and debris from the airways. Patients typically present with
recurrent respiratory infections. Kartagener syndrome is a type of ciliary dyskinesia with the clinical triad of situs inversus, chronic sinusitis, and bronchiectasis.

« https://t.me/USMLENBME2CK ts e t
Previous Next Score Report Lab Values Calculator Help pause
Exam Section 1: Item 26 of 50 National Board of Medical Examiners
Comprehensive Clinical Science Self-Assessment

"I 26. A physician, who is covering for a colleague who is out of town, receives a telephone call at the office from a dentist regarding whether antibiotic prophylaxis is needed for a scaling procedure for the
colleague's patient. The patient is a50-year-old woman with milral valve prolapse diagnosed 15 years ago prior to her transferring lo the colleague's care. She has never had endocarditis. Further review
of the patient's medical record indicates no documentation of echocardiography. She also has hypertension, type 2 diabetes mellitus, and an allergy to penicillin. Medications are lisinopril and metformin.
Cardiac examination from the patient's last office visit 6 months ago showed a normal pulse and no murmurs. Which of the following is the most appropriate next step in management of the colleague's
patient?

A) Advise the dentist to proceed with the procedure without antibiotic prophylaxis
B) Ask the dentist to postpone the procedure, and then schedule outpatient echocardiography
C) Ask the dentist to postpone the procedure, and then schedule an examination in the office
D) Prescribe amoxicillin to be taken 1 hour before the procedure
E) Prescribe clindamycin to be taken 1 hour before the procedure
Correct Answer: A.

Cardiology societies in the United States and Europe have guidelines recommending the use of antimicrobial prophylaxis during invasive dental procedures for the prevention of bacterial endocarditis in certain
situations. Dental procedures that disrupt the mucosa I lining of the mouth are known to cause transient bacteremia; however, the risk for subsequent infective endocarditis is less established. The
recommendations for antimicrobial prophylaxis are based on the expected risk from a patient's underlying endocardial condition and the degree of potential harm if infective endocardilis does develop. These
conditions include the presence of a prosthetic valve, a prior history of infective endocarditis, congenital heart disease, and cardiac transplantation patients with valvulopathy. For an otherwise healthy patient
with mitral valve prolapse, the physician should advise the dentist to proceed with the procedure without antibiotic prophylaxis.

Incorrect Answers: B, C, D, and E.

Ask the dentist to postpone the procedure, and then schedule outpatient echocardiography (Choice B) or ask the dentist to postpone the procedure, and then schedule an examination in the office (Choice C)
are not necessary for this patient who was seen in the past 6 months and has not developed any new cardiac symptoms.

Prescribe amoxicillin to be taken 1 hour before the procedure (Choice D) is appropriate for patients at highest risk for complications from endocarditis. It is not needed for this patient with stable, asymptomatic
mitral valve prolapse and no intracardiac medical devices.

Prescribe clindamycin to be taken 1 hour before the procedure (Choice E) is not recommended for any patient undergoing dental procedures because of the frequency of adverse events and limited proof of
benefit.

Educational Objective: Patients undergoing dental procedures that disrupt the mucosa I lining of the oral cavity are at increased theoretical risk for developing infective endocarditis. Antibiotic prophylaxis with
amoxicillin is recommended for patients with a prosthetic valve, a prior history of infective endocarditis, congenital heart disease, or a cardiac transplantation complicated by valvulopathy.

https://t.me/USMLENBME2CK ti e
Previous Next Score Report Lab Values Calculator Help pause
Exam Section 1: Item 27 of 50 National Board of Medical Examiners
Comprehensive Clinical Science Self-Assessment

"I 27. A 52-year-old woman with amyotrophic lateral sclerosis has been receiving mechanical ventilation in the hospital for the past 4 weeks. She is taking no medications. Attempts at discontinuing
mechanical ventilation have been unsuccessful. Her parents visit her often and participate in her health care decisions. The physician tells her and her parents that she could require mechanical
ventilation for several more years and recommends transfer to a facility for ventilator-dependent patients. Although her parents approve of this recommendation, the patient refuses placement and asks
to have mechanical ventilation and all other life-sustaining treatments withheld. She says, "I would rather die now than spend the rest of my life in one of those places." Which of the following is the
most appropriate next step in management?

A) Ask the parents privately about the patient's prior beliefs about long-term care
B) Assess the patient's mental capacity to make her own health care decisions
C) Continue current treatment and start additional treatment as needed
D) Continue current treatment but do not start any new life-sustaining treatment
E) Seek a court order to appoint a guardian to make her health care decisions
Correct Answer: B.

To meet the criteria for decisional capacity, patients must express a consistent choice, demonstrate an understanding of the risks and benefits of the choice, demonstrate an appreciation of the significance
of the choice for their personal circumstances, and illustrate an ability to reason through the options. Capacity assessments are based on a discussion with the patient in order to assess these four factors of
understanding, appreciation, reasoning, and choice, and can be conducted by any physician. Capacity is not a global assessment but rather pertains to a specific decision, and therefore the assessment
should be based on the individual patient, their condition at the lime, and the particular medical decision being made. The assessment must be performed rigorously in settings where the consequences of
the decision are particularly serious, as in this case. This patient with amyotrophic lateral sclerosis has a poor long-term prognosis and is facing end-of-life decision making. Some patients may value quality
of life or comfort rather than prolongation of life, and patients have a right to express these values in their decision making. This patient is alert and appears to have decisional capacity. A further assessment
of her capacity should be conducted to explore her reasoning and to exclude other factors, such as depression, that may limit her capacity.

Incorrect Answers: A, C, D, and E.

Asking the parents privately about the patient's prior beliefs about long-term care (Choice A) may be helpful for understanding her reasoning but does not supersede her current decision-making capacity.

Continuing current treatment with or without starting additional treatment (Choices C and 0) is potentially unethical given that this patient on mechanical ventilation has expressed a desire to discontinue life-
sustaining treatments. A capacity assessment should first be performed before further decisions to continue or discontinue treatment are made.

Seeking a court order to appoint a guardian to make her health care decisions (Choice E) is inappropriate and neglects the patient's autonomy. Appointment of a legal guardian can be pursued when a
patient is thought to lack decision-making capacity across all domains, which this patient has not demonstrated.

Educational Objective: To meet the criteria for decisional capacity, patients must express a consistent choice, demonstrate an understanding of the risks and benefits of the choice, demonstrate an
appreciation of the significance of the choice for their personal circumstances, and illustrate an ability to reason through the options. For patients making end-of-life decisions, a capacity assessment should
be performed to explore the patient's reasoning and to exclude other factors, such as depression, that may limit decisional capacity.

https://t.me/USMLENBME2CK ti
Previous Next Score Report Lab Values Calculator Help pause
Exam Section 1: Item 28 of 50 National Board of Medical Examiners
Comprehensive Clinical Science Self-Assessment

"I 28. A 62-year-old man comes to the physician because of a lesion on his right temple for 3 months. He works as a farmer. Examination shows a 1.5-cm ulcerated lesion over the right temple area; the lesion
has a raised margin and a waxy border. If left untreated, which of the following is the most likely sequela of the lesion?

A) Local spread
B) Lymphatic invasion
C) Regression
D) Systemic metastases
Correct Answer: A.

Basal cell carcinoma is the most common form of skin cancer and is derived from the basal cells of the epidermis. The primary risk factor for developing basal cell carcinoma is ultraviolet light exposure, though
age, fair skin, and family history are also contributory. Basal cell carcinomas typically present as pink, pearly papules or nodules with rolled borders and central ulceration commonly on sun-exposed areas of
the head and neck. Subtypes of basal cell carcinoma include those that are less invasive, such as superficial and nodular, and those that are more invasive, such as infiltrative and morpheaform. In all
subtypes, the risk for metastasis is low, although the tumors can invade locally and cause significant tissue damage. Biopsy may be performed with a variety of techniques, including excisional, shave, or punch
biopsy. Treatment of basal cell carcinoma requires complete surgical removal, either via excision or Mohs micrographic surgery. In the case of incomplete excision, re-excision with negative biopsy margins is
indicated to reduce the risk for further local spread.

Incorrect Answers: B, C, and D.

Lymphatic invasion (Choice B) occurs in particular malignancies, such as cutaneous and oropharyngeal squamous cell carcinoma, but it does not occur in cutaneous basal cell carcinoma.

Regression (Choice C) may occur spontaneously in a variety of cutaneous neoplasms, particularly keratoacanthoma. Keratoacanthoma is characterized by a dome-shaped proliferation of keratinocytes with a
central, keratin-filled crater and also typically occurs on sun-exposed areas. While most keratoacanthomas are characterized by a benign course, often undergoing spontaneous involution, some lesions
progress to squamous cell carcinoma.

Systemic metastases (Choice 0) are typical of cutaneous melanoma, which commonly metastasizes to the liver, lungs, and brain. Basal cell carcinoma does not typically result in systemic metastases.

Educational Objective: Basal cell carcinomas typically present as pink, pearly papules or nodules with rolled borders and central ulceration commonly on sun-exposed areas of the head and neck. Basal cell
carcinoma does not tend to metastasize but can spread locally. Treatment of basal cell carcinoma requires complete surgical removal, either via excision or Mohs micrographic surgery.

https://t.me/USMLENBME2CK ts e t
Previous Next Score Report Lab Values Calculator Help pause
Exam Section 1: Item 29 of 50 National Board of Medical Examiners
Comprehensive Clinical Science Self-Assessment

"I 29. A57-year-old man comes to the emergency department because of a 6-hour history of fatigue and mild shortness of breath. He has type 2 diabetes mellitus, hypertension, and anuric renal failure
requiring hemodialysis. He reports having missed his regularly scheduled dialysis session 2 days ago. His current medications are enalapril, amlodipine, insulin, calcitriol, and calcium supplementation.
His temperature is 37°C (98.6°F), pulse is 110/min, respirations are 24/min, and blood pressure is 160/100 mm Hg. Examination shows no jugular venous distention. Crackles are heard at both lung
bases. Heart sounds are normal. Serum studies show a potassium concentration of 6.8 mEq/L, urea nitrogen concentration of 120 mg/dl, and creatinine concentration of 9.9 mg/dl. An ECG shows
sinus tachycardia at 115/min, with widening of the PR and ORS intervals and peaked T waves. Intravenous administration of which of the following is the most appropriate next step in treatment?

A) Atenolol
B) Calcium chloride
C) Furosemide
D) Lidocaine
E) Magnesium sulfate
F) 0.9% Saline
Correct Answer: B.

This patient's presentation is consistent with hyperkalemia in the setting of end-stage kidney disease, with resultant cardiac membrane irritability as evidenced by ECG findings consistent with hyperkalemia.
The patient requires immediate intervention to reduce the risk for a fatal arrhythmia. ECG evidence of hyperkalemia includes flattening of the P-waves, lengthening of the PR interval, widening of the ORS
complex, and the presence of tall, peaked T waves. Acute management in order of priority involves stabilization of the myocardium with calcium gluconate or calcium chloride, shifting serum potassium into
the intracellular compartment (using insulin and dextrose, sodium bicarbonate, beta-2 adrenergic agonists), and removal of potassium from the body (using diuretics, oral potassium binders, hemodialysis).
After the patient is stabilized, evaluation should be directed to identifying the cause of the hyperkalemia. In this patient, the most likely explanation of hyperkalemia is missing dialysis with underlying end-
stage kidney disease. Next steps include laboratory studies, urgent hemodialysis, and admission for further management.

Incorrect Answers: A, C, D, E, and F.

Atenolol (Choice A) is a 31-selective antagonist with greater activity at 31 receptors than 32 receptors. It is used in the treatment of hypertension, arrhythmias, and heart failure. This patient's hypertension
and pulmonary edema is caused by volume overload secondary to missed dialysis, less likely caused by heart failure or hypertensive emergency.

Furosemide (Choice C) may be trialed in this patient in combination with intravenous fluids to decrease the total body potassium concentration by blocking absorption of potassium in the loop of Henle.
Calcium chloride or gluconate and attempts to shift potassium into the intracellular compartment should be initiated first. The presence of anuria may limit any application of furosemide in this case, and
acute hemodialysis will likely be required.

Lidocaine (Choice 0) is a class lb antiarrhythmic and local anesthetic that inhibits sodium channels. It can be used intravenously to treat ventricular tachyarrhythmias or subcutaneously to provide local
anesthetic for minor surgical procedures. It does not have a role in treating hyperkalemia.

Magnesium sulfate (Choice E) is used intravenously in the management of torsades de pointes. It inhibits calcium influx into myocardial cells, shortens the QT interval, and decreases the frequency of early
afterdepolarizations that propagate the arrhythmia. This patient's ECG changes are secondary to hyperkalemia, not torsades de pointes.

https://t.me/USMLENBME2CK ts e t
Previous Next Score Report Lab Values Calculator Help pause
Exam Section 1: Item 29 of 50 National Board of Medical Examiners
Comprehensive Clinical Science Self-Assessment

B) Calcium chloride
C) Furosemide
D) Lidocaine
E) Magnesium sulfate
F) 0.9% Saline
Correct Answer: B.

This patient's presentation is consistent with hyperkalemia in the setting of end-stage kidney disease, with resultant cardiac membrane irritability as evidenced by ECG findings consistent with hyperkalemia.
The patient requires immediate intervention to reduce the risk for a fatal arrhythmia. ECG evidence of hyperkalemia includes flattening of the P-waves, lengthening of the PR interval, widening of the ORS
complex, and the presence of tall, peaked T waves. Acute management in order of priority involves stabilization of the myocardium with calcium gluconate or calcium chloride, shifting serum potassium into
the intracellular compartment (using insulin and dextrose, sodium bicarbonate, beta-2 adrenergic agonists), and removal of potassium from the body (using diuretics, oral potassium binders, hemodialysis).
After the patient is stabilized, evaluation should be directed to identifying the cause of the hyperkalemia. In this patient, the most likely explanation of hyperkalemia is missing dialysis with underlying end-
stage kidney disease. Next steps include laboratory studies, urgent hemodialysis, and admission for further management.

Incorrect Answers: A, C, D, E, and F.

Atenolol (Choice A) is a 31-selective antagonist with greater activity at 31 receptors than 32 receptors. It is used in the treatment of hypertension, arrhythmias, and heart failure. This patient's hypertension
and pulmonary edema is caused by volume overload secondary to missed dialysis, less likely caused by heart failure or hypertensive emergency.

Furosemide (Choice C) may be trialed in this patient in combination with intravenous fluids to decrease the total body potassium concentration by blocking absorption of potassium in the loop of Henle.
Calcium chloride or gluconate and attempts to shift potassium into the intracellular compartment should be initiated first. The presence of anuria may limit any application of furosemide in this case, and
acute hemodialysis will likely be required.

Lidocaine (Choice 0) is a class lb antiarrhythmic and local anesthetic that inhibits sodium channels. It can be used intravenously to treat ventricular tachyarrhythmias or subcutaneously to provide local
anesthetic for minor surgical procedures. It does not have a role in treating hyperkalemia.

Magnesium sulfate (Choice E) is used intravenously in the management of torsades de pointes. It inhibits calcium influx into myocardial cells, shortens the QT interval, and decreases the frequency of early
afterdepolarizations that propagate the arrhythmia. This patient's ECG changes are secondary to hyperkalemia, not torsades de pointes.

0.9% saline (Choice F) is unlikely to have a beneficial role in an anuric patient with end-stage kidney disease and signs of hypervolemia. It would not take the place of immediate stabilization of the cardiac
membrane with calcium.

Educational Objective: Hyperkalemia presents a risk for fatal arrhythmias, and immediate treatment is required. The initial step is stabilization of the myocardium using intravenous calcium formulations (eg,
calcium gluconate, calcium chloride), followed by attempts to shift potassium into the intracellular compartment, remove potassium from the body, and address the underlying cause.

https://t.me/USMLENBME2CK ts e t
Previous Next Score Report Lab Values Calculator Help pause
Exam Section 1: Item 30 of 50 National Board of Medical Examiners°
Comprehensive Clinical Science Self-Assessment

"I 30. A 13-year-old boy is brought to the physician because of intermittent pain in his right thigh during the past 6 weeks. The symptoms are
exacerbated by running, and his mother reports that he limps for 30 minutes after running. He has no history of serious illness and takes no
medications. He recalls no recent trauma to either leg. He has practiced with the school football team three times weekly for the past 2 months.
He is at the 75th percentile for height and 95th percentile for weight and BMI. Examination shows a normal gait. There is no tenderness of the
right hip or thigh. Internal rotation of the right hip is difficult, especially when the knee is flexed. Genital and pubic hair development are sexual
maturity rating (SMR) stage 3. An x-ray of the hips is shown. Which of the following is the most appropriate treatment?

A) Physical therapy
B) Nonsteroidal anti-inflammatory drug therapy
C) Casi immobilization of the right hip and thigh
D) Surgical stabilization of the femoral head
E) No treatment is indicated

Correct Answer: D.

Slipped capital femoral epiphysis (SCFE) most commonly occurs in children between the ages of 10 to 15 years who are overweight. SCFE is a displacement of the femoral epiphysis relative to the femoral
neck secondary to anterosuperior movement of the metaphysis. It is a Salter-Harris type 1 fracture of the growth plate. SCFE typically presents with a painful limp and, if severe, may result in restricted range
of motion of the hip joint (eg, limited abduction and internal rotation of the hip as in this case) and inability to bear weight on the affected lower extremity. Pain often refers to the knee and thigh. X-ray
evaluation of both hips in frontal and frog-leg views permits side-by-side comparison to diagnose the condition. If untreated, complications include avascular necrosis of the femoral head, limited range of
motion and gait impairment, and premature osteoarthritis. To prevent these complications, treatment requires surgical stabilization of the femoral head.

Incorrect Answers: A, B, C, and E.

Physical therapy (Choice A) and nonsteroidal anti-inflammatory drug therapy (Choice 8) would be appropriate for conditions such as osteoarthritis, a noninflammatory arthropathy that occurs secondary to
deterioration of articular cartilage. It typically occurs in older patients and may affect any joint.

Cast immobilization of the right hip and thigh (Choice C) would be indicated in nonsurgical fractures, which are uncommon. A hip spica cast may be appropriate in cases of hip dysplasia or certain fractures
of the hip or pelvis. It is not a commonly accepted modality in the management of SCFE.

No treatment (Choice E) would be inappropriate, as SCFE requires surgical fixation.

feluoatienal fiaoties· lino oanital famoral aninheie nracante uith hir thieh nnel lonoo nvain uith lirnital ranee of motion anel an antaleio it anarlhw in hilel or taonaoear hie ooruaieht lt ies

« https://t.me/USMLENBME2CK ts e t
Previous Next Score Report Lab Values Calculator Help pause
Exam Section 1: Item 30 of 50 National Board of Medical Examiners"
Comprehensive Clinical Science Self-Assessment

He is at the 75th percentile for height and 95th percentile for weight and BMI. Examination shows a normal gait. There is no tenderness of the
right hip or thigh. Internal rotation of the right hip is difficult, especially when the knee is flexed. Genital and pubic hair development are sexual
maturity rating (SMR) stage 3. An x-ray of the hips is shown. Which of the following is the most appropriate treatment?

A) Physical therapy
B) Nonsteroidal anti-inflammatory drug therapy
C) Cast immobilization of the right hip and thigh
D) Surgical stabilization of the femoral head
E) No treatment is indicated

Correct Answer: D.

Slipped capital femoral epiphysis (SCFE) most commonly occurs in children between the ages of 10 to 15 years who are overweight. SCFE is a displacement of the femoral epiphysis relative lo the femoral
neck secondary to anterosuperior movement of the metaphysis. It is a Salter-Harris type 1 fracture of the growth plate. SCFE typically presents with a painful limp and, if severe, may result in restricted range
of motion of the hip joint (eg, limited abduction and internal rotation of the hip as in this case) and inability to bear weight on the affected lower extremity. Pain often refers to the knee and thigh. X-ray
evaluation of both hips in frontal and frog-leg views permits side-by-side comparison to diagnose the condition. If untreated, complications include avascular necrosis of the femoral head, limited range of
motion and gait impairment, and premature osteoarthritis. To prevent these complications, treatment requires surgical stabilization of the femoral head.

Incorrect Answers: A, B, C, and E.

Physical therapy (Choice A) and nonsteroidal anti-inflammatory drug therapy (Choice B) would be appropriate for conditions such as osteoarthritis, a noninflammatory arthropathy that occurs secondary to
deterioration of articular cartilage. It typically occurs in older patients and may affect any joint.

Cast immobilization of the right hip and thigh (Choice C) would be indicated in nonsurgical fractures, which are uncommon. A hip spica cast may be appropriate in cases of hip dysplasia or certain fractures
of the hip or pelvis. It is not a commonly accepted modality in the management of SCFE.

No treatment (Choice E) would be inappropriate, as SCFE requires surgical fixation.

Educational Objective: Slipped capital femoral epiphysis presents with hip, thigh, and knee pain with limited range of motion and an antalgic gait, generally in a child or teenager who is overweight. It is a
Salter-Harris type 1 fracture of the physis, and treatment requires surgical stabilization of the femoral head.

« https://t.me/USMLENBME2CK ts e t
Previous Next Score Report Lab Values Calculator Help pause
Exam Section 1: ltem 31 of 50 National Board of Medical Examiners
Comprehensive Clinical Science Self-Assessment

✓ 31. An 87-year-old woman with type 2 diabetes mellitus comes to the physician for evaluation of a nonhealing foot ulcer that has increasingly worse tissue necrosis and erythema. She has a2-year
history of mild to moderate dementia, Alzheimer type, with no recent changes in cognitive status. The patient lives alone with the help of various neighbors. Her only family is an out-of-town sister,
with whom she last spoke 1 year ago. Physical examination shows a 2-cm ulcer that exposes the head of the first metatarsal. There is 1 cm of necrosis and 2 cm of erythema around the edge of the
ulcer. On mental status examination, the patient is alert and oriented to person and place but thinks that the year is 1999. She recalls one of three objects after 5 minutes. A
below-the-knee amputation is recommended. The risks and benefits of the operation are discussed with the patient. She understands and repeats the discussion correctly. Attempts to contact her
sister are unsuccessful. Which of the following is the most appropriate next step in management?

A) Consult with the hospital attorney


B) Delay the amputation until her sister can be contacted
C) Obtain a psychiatric assessment of this patient's mental capacity
0) Schedule a follow-up examination after 1 month of antibiotic therapy
E) Proceed with the amputation
Correct Answer: E.

To meet the criteria for decisional capacity, patients must express a consistent choice, demonstrate an understanding of the risks and benefits of the choice, demonstrate an appreciation of the significance
of the choice for their personal circumstances, and illustrate an ability to reason through the options. Although she displays evidence of mild to moderate dementia, this patient appears to meet the criteria
of decisional capacity. Capacity assessments should be based on the individual patient and the particular medical decision being made. While patients with dementia may have impairment of their cognitive
abilities, it should not be assumed that such patients lack capacity, as many are still able to appreciate risks, conduct reasoning, and express choice.

Incorrect Answers: A, B, C, and D.

Consulting with the hospital attorney (Choice A) is not necessary, as this patient possesses capacity to make this particular decision about her medical care. Similarly, delaying the amputation until her sister
can be contacted (Choice B) would be correct if the patient lacked capacity; however, in this case, ii appears she is able to comprehend and reason through the decision to have the surgery. In cases of
advanced dementia, though, it may be appropriate to identify a surrogate decision maker.

Obtaining a psychiatric assessment of this patient's mental capacity (Choice C) is unnecessary. Mental capacity differs from decision-making capacity. Decision-making capacity assessments are based on
a discussion with the patient in order to assess understanding, appreciation, reasoning, and choice, and can be conducted by any physician.

Scheduling a follow-up examination after 1 month of antibiotic therapy (Choice 0) is inappropriate. While it is appropriate to attempt to restore capacity in situations where the patient is thought to
temporarily lack capacity, such as in the setting of delirium, it is not appropriate to delay necessary care for prolonged periods of time.

Educational Objective: Capacity assessments should be based on the individual patient and the particular medical decision being made. They are based on a discussion with the patient in order to assess
understanding, appreciation, reasoning, and choice. While patients with dementia may have impairment of their cognitive abilities, it should not be assumed that such patients lack capacity.

https://t.me/USMLENBME2CK ts e t
Previous Next Score Report Lab Values Calculator Help pause
Exam Section 1: ltem 32 of 50 National Board of Medical Examiners
Comprehensive Clinical Science Self-Assessment

"I 32. A47-year-old woman comes to the physician because of a 2-month history of substernal chest lightness and shortness of breath associated with exercise and relieved by rest. She previously walked
2 miles daily, but now she is limited to walking one-half block because of chest pain, dyspnea, and light-headedness. She has never smoked and drinks one to two beers daily. Her temperature is
36.8°C (98.2°F), pulse is 98/min, respirations are 20/min, and blood pressure is 110/82 mm Hg. Jugular venous pressure is 8 cm H0. Crackles are heard at both lung bases. The point of maximal
impulse is increased but not displaced; S,is normal, but S is decreased. There is a grade 3/6, harsh systolic murmur at the cardiac base. Serum studies show a total cholesterol concentration of
210 mg/dl, H DL-cholesterol concentration of 95 mg/dl, and LDL-cholesterol concentration of 100 mg/dL. Which of the following is the most likely cause of this patient's heart failure?

A) Alcohol use disorder


B ) Amyloidosis
C) Anemia
D) Aortic stenosis
E) Atrial fibrillation
F) Car pulmonale
G) Hemochromatosis
H) lschemic heart disease
I ) Mitral regurgitation
Correct Answer: D.

The presence of a harsh, systolic murmur and a decreased S heart sound suggests that aortic stenosis is the cause of this patient's symptoms. Many people will develop some degree of valve stenosis in
their lifetime as a result of chronic inflammation with resultant calcification and fibrosis over time, with the aortic valve most commonly affected. Early onset aortic stenosis (ages 40 to 60 years), as in this
patient, can occur in the setting of a congenital bicuspid valve or chronic rheumatic heart disease. Patients may complain of fatigue, shortness of breath, cough, diminished exercise tolerance, chest pain, or
syncope with exertion. Examination findings include a crescendo-decrescendo systolic murmur best heard at the upper right sternal border. Pulsus parvus et tardus (weak and delayed) may be noted on
examination of peripheral pulses. A diminished S heart sound results from weakened closing of the valve. Severe aortic stenosis can lead to left ventricular hypertrophy and heart failure over time.

Incorrect Answers: A, B, C, E, F,G, H, and I.

Alcohol use disorder (Choice A) causes hypertension, which can contribute lo heart failure, and some patients may develop dilated cardiomyopathy from direct toxic effects to cardiomyocytes. These
complications are associated with heavier alcohol use patterns than this patient has reported, and her physical examination findings are more consistent with aortic stenosis.

Amyloidosis (Choice B), in which proteins deposit in tissues and cause organ dysfunction, can cause restrictive cardiomyopathy. Amyloidosis is commonly associated with plasma cell dyscrasias such as
multiple myeloma, and chronic inflammatory conditions such as rheumatoid arthritis, systemic lupus erythematosus, and inflammatory bowel disease. It is uncommon compared with aortic stenosis and is
unlikely in this patient.

Anemia (Choice C), if severe, can cause a loud systolic flow murmur best heard at the apex. Other conditions that cause a similar murmur include fever, pregnancy, and hyperthyroidism. Tachycardia would
be expected.

https://t.me/USMLENBME2CK ts e t
Previous Next Score Report Lab Values Calculator Help pause
Exam Section 1: ltem 32 of 50 National Board of Medical Examiners
Comprehensive Clinical Science Self-Assessment

Correct Answer: D.

The presence of a harsh, systolic murmur and a decreased S heart sound suggests that aortic stenosis is the cause of this patient's symptoms. Many people will develop some degree of valve stenosis in
their lifetime as a result of chronic inflammation with resultant calcification and fibrosis over time, with the aortic valve most commonly affected. Early onset aortic stenosis (ages 40 to 60 years), as in this
patient, can occur in the setting of a congenital bicuspid valve or chronic rheumatic heart disease. Patients may complain of fatigue, shortness of breath, cough, diminished exercise tolerance, chest pain, or
syncope with exertion. Examination findings include a crescendo-decrescendo systolic murmur best heard at the upper right sternal border. Pulsus parvus et tardus (weak and delayed) may be noted on
examination of peripheral pulses. A diminished S heart sound results from weakened closing of the valve. Severe aortic stenosis can lead to left ventricular hypertrophy and heart failure over time.

Incorrect Answers: A, B, C, E, F,G, H, and I.

Alcohol use disorder (Choice A) causes hypertension, which can contribute lo heart failure, and some patients may develop dilated cardiomyopathy from direct toxic effects to cardiomyocytes. These
complications are associated with heavier alcohol use patterns than this patient has reported, and her physical examination findings are more consistent with aortic stenosis.

Amyloidosis (Choice B), in which proteins deposit in tissues and cause organ dysfunction, can cause restrictive cardiomyopathy. Amyloidosis is commonly associated with plasma cell dyscrasias such as
multiple myeloma, and chronic inflammatory conditions such as rheumatoid arthritis, systemic lupus erythematosus, and inflammatory bowel disease. It is uncommon compared with aortic stenosis and is
unlikely in this patient.

Anemia (Choice C), if severe, can cause a loud systolic flow murmur best heard at the apex. Other conditions that cause a similar murmur include fever, pregnancy, and hyperthyroidism. Tachycardia would
be expected.

Atrial fibrillation (Choice E) with rapid ventricular response presents with episodic tachycardia, heart palpitations, shortness of breath, dizziness, and anxiety. Patients will have an irregularly irregular pulse,
and, typically, heart rates greater than 100 beats per minute.

Car pulmonale (Choice F) describes right-sided heart failure secondary to chronic lung pathology. Chronic lung disease is associated with pulmonary hypertension, and, similar to systemic hypertension
causing left-sided heart failure, over time the right side of the heart fails as a result of the increased demands of pumping against elevated pulmonary arterial resistance. This patient has no history of
pulmonary disease.

Hemochromatosis (Choice G) presents with liver failure, diabetes mellitus, arthritis, heart failure, darkening of the skin, and gonadal atrophy secondary to excess total body iron. It is typically diagnosed in
men over the age of 40.

lschemic heart disease (Choice H) would be unexpected in a young, premenopausal patient with no family history of cardiac disease.

Mitra! regurgitation (Choice I) presents with a holosystolic murmur best heard in the fourth or fifth left intercostal space along the midclavicular line and radiates to the left axilla. It is commonly associated
with milral valve prolapse, acute rheumatic fever, endocarditis, and prior myocardial infarction.

Educational Objective: Aortic stenosis is common and typically presents with a crescendo-decrescendo systolic murmur best heard at the upper right sternal border. It classically occurs secondary to age-
related fibrotic and calcific changes of the valve but can occur earlier in life in cases of bicuspid aortic valve or chronic rheumatic heart disease.

https://t.me/USMLENBME2CK ts e t
Previous Next Score Report Lab Values Calculator Help pause
Exam Section 1: ltem 33 of 50 National Board of Medical Examiners
Comprehensive Clinical Science Self-Assessment

"I 33. A 30-year-old man who is in the US Army is brought to the emergency department of a military hospital 40 minutes after being hit by a motor vehicle traveling 25 mph while he was walking. On arrival,
he is unconscious. Temperature is 36.0"C (96.8°F), pulse is 113/min, respirations are 22/min, and blood pressure is 80/55 mm Hg. Pulse oximetry on room air shows an oxygen saturation of 90%. On
examination, the abdomen is rigid. He is resuscitated with 8 units of packed red blood cells, 6 units of fresh frozen plasma, and 2 units of platelets. Emergency laparotomy shows transection of the
small bowel 5 cm proximal to the cecum. Laboratory studies done 45 minutes after arriving in the surgical intensive care unit show an activated partial thromboplastin lime of 42 seconds and a
prothrombin time of 25 seconds (INR=3.2). Which of the following is the most likely cause of the coagulation abnormalities in this patient?

A) Cirrhosis
B) Disseminated intravascular coagulation
C) Factor VII deficiency
D) Hyperfibrinogenemia
E) Thrombocytopenia
Correct Answer: B.

Disseminated intravascular coagulation {DIC) is a systemic disorder characterized by the widespread activation of the clotting cascade resulting in microthrombi formation, thrombocytopenia, and bleeding
complications caused by consumption of platelets and clotting factors. Causes of DIC are many, including trauma, sepsis, malignancy, pancreatitis, and exposure to toxins. Regardless of underlying cause,
the clinical effects include stigmata of bleeding {petechiae, purpura, oozing at surgical and venipuncture sites, clinical evidence of bleeding), and laboratory abnormalities reflecting disseminated activation of
the clotting cascade {increased activated partial thromboplastin time, prothrombin time, and INR) and consumption of platelets (thrombocytopenia). Additional laboratory abnormalities include increased D-
dimer, caused by the presence of fibrin split products, and decreased fibrinogen concentration, along with increased markers of hemolysis (detectable free hemoglobin, decreased haptoglobin, increased
lactate dehydrogenase). lntravascular fibrin strands and microthrombi cause shearing of erythrocytes, leading to microangiopathic hemolytic anemia. Treatment includes correcting the underlying cause and
resulting coagulopathy and thrombocytopenia.

Incorrect Answers: A, C, D, and E.

Cirrhosis (Choice A) can cause bleeding and impaired coagulation. It also presents with stigmata of liver failure including ascites, increased bilirubin, jaundice, and signs of portal hypertension, such as
esophageal varices, splenomegaly, caput medusae, and rectal varices. It typically occurs in patients with preceding conditions such as alcohol abuse or chronic hepatitis.

Factor VII deficiency (Choice C) is a rare bleeding disorder with a spectrum of clinical severity. Patients who are most affected present with heavy menstrual bleeding or bleeding following invasive
procedures. Treatment is with factor replacement, prothrombin complex concentrate, or frozen plasma.

Hyperfibrinogenemia (Choice D) promotes thrombosis by enhancing fibrin formation and stability. It leads to increased coagulation. DIC leads to decreased fibrinogen concentrations, whereas
hyperfibrinogenemia presents with increased fibrinogen. Unless inadvertently identified in the setting of traumatic injury, DIC is a more likely cause given the temporal correlation with trauma and
intraabdominal injury, and consistent laboratory studies.

Thrombocytopenia (Choice E) refers to abnormally decreased platelet count and may present with bleeding as a result of an insufficient quantity of platelets. Bleeding, if it occurs, tends to be
mucocutaneous. Symptoms can include abnormal bleeding or bruising, often presenting with petechiae, ecchymoses, and mucosal bleeding (eg, menorrhagia, epistaxis), along with persistent oozing at
puncture and operative sites. The prothrombin lime and partial thromboplastin time would be normal.

https://t.me/USMLENBME2CK ti
Previous Next Score Report Lab Values Calculator Help pause
Exam Section 1: ltem 33 of 50 National Board of Medical Examiners
Comprehensive Clinical Science Self-Assessment

A) Cirrhosis
B) Disseminated intravascular coagulation
C) Factor VII deficiency
D) Hyperfibrinogenemia
E) Thrombocytopenia
Correct Answer: B.

Disseminated intravascular coagulation {DIC) is a systemic disorder characterized by the widespread activation of the clotting cascade resulting in microthrombi formation, thrombocytopenia, and bleeding
complications caused by consumption of platelets and clotting factors. Causes of DIC are many, including trauma, sepsis, malignancy, pancreatitis, and exposure to toxins. Regardless of underlying cause,
the clinical effects include stigmata of bleeding {petechiae, purpura, oozing at surgical and venipuncture sites, clinical evidence of bleeding), and laboratory abnormalities reflecting disseminated activation of
the clotting cascade (increased activated partial thromboplastin time, prothrombin time, and INR) and consumption of platelets (thrombocytopenia). Additional laboratory abnormalities include increased D-
dimer, caused by the presence of fibrin split products, and decreased fibrinogen concentration, along with increased markers of hemolysis (detectable free hemoglobin, decreased haptoglobin, increased
lactate dehydrogenase). Intravascular fibrin strands and microthrombi cause shearing of erythrocytes, leading to microangiopathic hemolytic anemia. Treatment includes correcting the underlying cause and
resulting coagulopathy and thrombocytopenia.

Incorrect Answers: A, C, D, and E.

Cirrhosis (Choice A) can cause bleeding and impaired coagulation. It also presents with stigmata of liver failure including ascites, increased bilirubin, jaundice, and signs of portal hypertension, such as
esophageal varices, splenomegaly, caput medusae, and rectal varices. It typically occurs in patients with preceding conditions such as alcohol abuse or chronic hepatitis.

Factor VII deficiency (Choice C) is a rare bleeding disorder with a spectrum of clinical severity. Patients who are most affected present with heavy menstrual bleeding or bleeding following invasive
procedures. Treatment is with factor replacement, prothrombin complex concentrate, or frozen plasma.

Hyperfibrinogenemia (Choice D) promotes thrombosis by enhancing fibrin formation and stability. It leads to increased coagulation. DIC leads to decreased fibrinogen concentrations, whereas
hyperfibrinogenemia presents with increased fibrinogen. Unless inadvertently identified in the setting of traumatic injury, DIC is a more likely cause given the temporal correlation with trauma and
intraabdominal injury, and consistent laboratory studies.

Thrombocytopenia (Choice E) refers to abnormally decreased platelet count and may present with bleeding as a result of an insufficient quantity of platelets. Bleeding, if it occurs, tends to be
mucocutaneous. Symptoms can include abnormal bleeding or bruising, often presenting with petechiae, ecchymoses, and mucosal bleeding (eg, menorrhagia, epistaxis), along with persistent oozing at
puncture and operative sites. The prothrombin time and partial thromboplastin time would be normal.

Educational Objective: Disseminated intravascular coagulation can be provoked by numerous causes, including trauma, sepsis, malignancy, pancreatitis, and exposure to toxins. It is characterized by
depletion of coagulation factors, microangiopathic hemolytic anemia, thrombocytopenia, and bleeding.

https://t.me/USMLENBME2CK ti
Previous Next Score Report Lab Values Calculator Help pause
Exam Section 1: Item 34 of 50 National Board of Medical Examiners
Comprehensive Clinical Science Self-Assessment

✓ 34. A 25-year-old woman, gravida 1, para 1, comes to the office because of a 3-month history of chronic moderate perianal pain and bleeding that began 3 days after she gave birth via cesarean delivery.
Before the onset of her symptoms, she had constipation associated with the use of postdelivery oxycodone therapy. The constipation resolved after she discontinued the medication, but her bowel
movements have been painful and she has noticed blood on the toilet tissue. One week ago, the patient's obstetrician prescribed topical hydrocortisone and dilliazem, which provide temporary relief but
do not resolve her symptoms. The patient's medical history otherwise is unremarkable and she takes no other medications. Family history is unremarkable. Vital signs are temperature 37.2°C (99.0°F),
pulse 80/min, respirations 20/min, and blood pressure 130/80 mm Hg. Examination of the anus shows a deep wound in the posterior midline with exposure of the internal sphincter muscle and an
associated 1.5-cm skin tag. Which of the following is the most appropriate next step in management?

A) Biopsy of the wound


B) Colonoscopy
C) Metronidazole therapy
D) Surgical repair
Correct Answer: D.

Anal fissures commonly present with rectal pain, particularly with defecation, and bleeding caused by a tear in anal tissue, most commonly at the posterior midline. This tear in the mucosa may expose the
internal anal sphincter and may be associated with local skin tags. Pain can occur from irritation of the tear itself or a spasm of the internal sphincter, the latter of which can cause ischemia and prevent
healing. Causes of primary anal fissures include constipation, diarrhea, and vaginal delivery, although secondary fissures can occur in the setting of systemic illnesses such as sarcoidosis, inflammatory bowel
disease, or HIV infection. Initial management of anal fissures includes the prevention or treatment of constipation, topical analgesics, and topical vasodilators. However, these methods are typically ineffective
for chronic anal fissures, which are present for longer than 8 weeks. Chronic fissures, or those that are unresponsive to conservative management, should undergo surgical repair.

Incorrect Answers: A, B, and C.

Biopsy of the wound (Choice A) is not necessary and may exacerbate this patient's pain. She has the classic signs and symptoms of an anal fissure, with the fissure being evident on physical examination. As
her symptoms have not responded to conservative medical management, she should undergo surgical repair.

Colonoscopy (Choice 8) is used to evaluate the rectum and colon and is commonly used for cancer screening or inflammatory bowel disease diagnosis. This patient's symptoms are related to an external
anal injury. Anal fissures are not frequently related to internal disorders in the absence of other symptoms, so colonoscopy is not necessary.

Metronidazole therapy (Choice C) is used in the treatment of anaerobic infections, such as intra-abdominal abscesses. It is occasionally used in the treatment of sexually transmitted infections. This patient
has clear evidence of an anal fissure on examination and no clear signs of infection that would warrant metronidazole therapy.

Educational Objective: Anal fissures commonly present with rectal pain and bleeding with defecation caused by local trauma that causes a tear in the anal tissue, most commonly in the posterior midline.
Significant pain occurs as a result of irritation to the tear itself or a spasm of the internal sphincter. Management includes the prevention or treatment of constipation, topical analgesics, and topical
vasodilators. Chronic fissures, or those that are unresponsive to conservative management, require surgical repair.

https://t.me/USMLENBME2CK ts e t
Previous Next Score Report Lab Values Calculator Help pause
Exam Section 1: ltem 35 of 50 National Board of Medical Examiners
Comprehensive Clinical Science Self-Assessment

"I 35. A 42-year-old man is brought to the emergency department by paramedics 1 hour after his girlfriend found him confused and agitated in his home. The girlfriend says that when she spoke with him on
the telephone last night, she noted no problems, but she became worried when he did not answer his telephone 3 hours ago. He has a3-year history of chronic back pain for which he takes
oxycodone. He also has major depressive disorder and seasonal allergies treated with citalopram and diphenhydramine, respectively. On arrival, he is confused, screaming loudly, and actively pulling
against wrist restraints. His temperature is 38°C (100.4°F), pulse is 163/min, respirations are 24/min, and blood pressure is 215/105 mm Hg. Physical examination shows diaphoresis and
tremulousness. The pupils are dilated. Which of the following is the most likely cause of this patient's confusion?

A) Acetaminophen overdose
B) Cannabis withdrawal
C) Cocaine intoxication
D) Diphenhydramine toxicity
E) Opioid withdrawal
Correct Answer: C.

Cocaine blocks the presynaptic reuptake of dopamine, serotonin, and norepinephrine, which increases the concentration of these substances in the synaptic cleft. Cocaine is an addictive recreational drug
that typically causes euphoria, restlessness, and increased sympathetic lone (ie, tachycardia, hypertension, pupillary dilation). Increased synaptic dopamine leads to addiction (and in severe cases,
hallucinations and paranoia). Increased synaptic serotonin leads to euphoria, while increased synaptic norepinephrine can cause tachycardia and hypertension through activation of sympathetic
adrenoreceptors leading to increased cardiac chronotropy and peripheral vasoconstriction. Further, cocaine can act as a local anesthetic by decreasing sodium channel permeability and thereby slowing or
blocking pain signal conduction.

Incorrect Answers: A, B, D, and E.

Acetaminophen overdose (Choice A) presents with nonspecific symptoms of fatigue, malaise, and abdominal pain followed by signs and symptoms of liver failure, including Jaundice, altered mental status,
and coagulopathy.

Cannabis withdrawal (Choice B) is characterized by irritability, anxiety, depression, insomnia, restlessness, and decreased appetite.

Diphenhydramine toxicity (Choice D) can lead lo central nervous system H1-receptor blockade, anlicholinergic, and anti-a-adrenergic adverse effects. This results in a constellation of symptoms such as dry
mouth, urinary retention, constipation, hallucinations, delirium, ataxia, flushed skin, visual disturbances (eg, cycloplegia), and sedation.

Opioid withdrawal (Choice E) causes restlessness, tachycardia, hypertension, tremors, nausea, and vomiting, as well as lacrimation, rhinorrhea, piloerection, and yawning.

Educational Objective: Cocaine is a central nervous system stimulant that causes euphoria and increases sympathetic tone, leading to tachycardia, hypertension, pupillary dilation, and restlessness. As a
result of increased synaptic dopamine, hallucinations and paranoia can occur.

https://t.me/USMLENBME2CK ti
Previous Next Score Report Lab Values Calculator Help pause
Exam Section 1: Item 36 of 50 National Board of Medical Examiners
Comprehensive Clinical Science Self-Assessment

A 21-year-old woman, gravida 2, para 1, at 36 weeks' gestation is admitted to the hospital in labor. She received no prenatal care. She has no history of operative procedures. She reports that she
occasionally used cocaine before she knew she was pregnant but has abstained from illicit drug use during the past 8 months. On admission, the cervix is 5 cm dilated and 100% effaced; the vertex
is at O station. There is a moderate amount of blood-tinged mucus. The membranes are artificially ruptured, yielding clear fluid. Her blood group is A, Rh-negative; a serum anti-D antibody titer is
positive. During the next 30 minutes, fetal heart monitoring shows regular, sinusoidal wave forms. Which of the following is the most likely cause of this fetal heart rate pattern?

A) Abruptio placentae
B) Fetal heart defect
C) Maternal cocaine use
D) Placenta previa
E) Rh isoimmunization
Correct Answer: E.

Rh(D)-negative mothers are at a high risk for developing anti-Rh{D) antibodies, referred to as Rh isoimmunization, if exposed to blood from a Rh{D)-positive fetus. While this commonly occurs at delivery, it
can also occur prior to delivery with vaginal bleeding or procedures such as amniocentesis. If Rh{D) antigens from the fetal circulation enter the maternal circulation, it is possible that the mother's immune
system will form antibodies against the Rh(D) antigen. This can result in hemolytic anemia of the fetus {erythroblastosis fetalis), especially with subsequent pregnancies, as the mother's antibodies may
cross the placenta and cause hemolysis of fetal red blood cells. Rh(D) immune globulin {RhoGAM) acts by binding to Rh{D)-positive cells that enter the maternal circulation, preventing the immune system
from developing antibodies to the Rh{D) antigen. It is commonly given to Rh(D)-negative mothers at 28 weeks' gestation. The sinusoidal pattern on fetal heart rate monitoring is indicative of anemia of the
fetus, likely secondary to hemolysis in the setting of Rh isoimmunization.

Incorrect Answers: A, B, C, and D.

Abruptio placentae (Choice A) is the premature separation of the placenta from the uterus. It commonly presents with vaginal bleeding, severe uterine pain, and tetanic contractions, typically in the third
trimester. While cocaine toxicity is a risk factor for placental abruption, this patient has not used cocaine in 8 months. Given her blood group and lack of prenatal care, Rh isoimmunization is a more likely
cause of the sinusoidal pattern on fetal heart rate monitoring.

Fetal heart defect (Choice 8) can include a variety of pathologies, such as ventricular septal defects and Tetralogy of Fallot. They are often diagnosed on prenatal ultrasonography. While these defects may
be associated with cardiac arrhythmias, they are unlikely lo cause the sinusoidal pattern seen in this fetus.

Maternal cocaine use {Choice C) can predispose patients to placental abruption which, if severe, can lead to fetal exsanguination and the sinusoidal pattern seen in this fetus. However, placental abruption
is more likely to occur during or shortly after use of cocaine or in the setting of trauma. It is associated with severe abdominal pain and vaginal bleeding, which this patient does not show.

Placenta previa (Choice D) occurs when the placenta overlies the cervical os, which presents with painless vaginal bleeding. It is a contraindication to digital cervical examination and vaginal delivery, as
these events can cause fetomaternal hemorrhage. This patient has had minimal vaginal bleeding, making placenta previa unlikely.

Educational Objective: A sinusoidal pattern on fetal heart rate monitoring is indicative of anemia of the fetus. One cause of this anemia is Rh isoimmunization of the mother during a prior exposure to the
Rh{D) antigen, which can occur during delivery, spontaneous abortion, procedures such as amniocentesis, and other causes of fetomaternal bleeding. Rh{D) immune globulin (RhoGAM) acts by binding to
Rh{D)-positive cells that enter the maternal circulation, preventing the immune system from developing antibodies to the Rh(D) antigen. It is commonly given to Rh{D)-negative mothers at 28 weeks'
gestation.

https://t.me/USMLENBME2CK ts e t
Previous Next Score Report Lab Values Calculator Help pause
Exam Section 1: ltem 37 of 50 National Board of Medical Examiners
Comprehensive Clinical Science Self-Assessment

A 22-year-old college student comes to the physician because of a3-year history of falling asleep during her classes; she worries that she will not be able to continue with her education because of
this. She says that she always has had difficulty staying awake during class, even in high school. She also occasionally has trouble staying awake during movies. She says she has no problems
getting to sleep or waking up. She goes to bed between midnight and 1 AM and gets up at 8 AM. She has tried sleeping for longer durations, even up to 8½ hours, but she still has been unable to stay
awake during the day. She has the greatest difficulty staying awake during her classes after lunch. She thinks that she sleeps very soundly and moves little during sleep because her bedsheets are
neat when she awakens in the morning. She does not remember having dreams. She reports that no one has ever told her that she snores. She is 170 cm (5 ft 5 in) tall and weighs 62 kg (136 lb);
BMI is 21 kg/m?. Physical examination shows no abnormalities. On mental status examination, she has a concerned mood and a reactive affect. Which of the following is the most appropriate next
step in diagnosis?

A) Minnesota Multiphasic Personality Inventory


B) Overnight pulse oximetry
C) Pulmonary function tests
D) MRI of the brain
E) Multiple sleep latency tests
Correct Answer: E.

Narcolepsy is the most likely diagnosis in this patient with daytime somnolence despite restorative sleep and no other features pathognomonic of other sleep disorders. The pathogenesis of narcolepsy
involves the loss of orexin-A and orexin-B, which are neurotransmitters found primarily in the hypothalamus that promote wakefulness and inhibit REM sleep. Loss of these neurotransmitters permits
abnormal transition from wakefulness to REM sleep. Patients may experience hallucinations, cataplexy, and sleep paralysis. The diagnosis can be made in patients with daytime somnolence and cataplexy,
or in cases where cataplexy is not present, with multiple sleep latency tests that show a shorter time from wakefulness to the initiation of REM sleep. Occasionally, patients undergo lumbar puncture to
directly measure levels of orexin-A in the cerebrospinal fluid.

Incorrect Answers: A, B, C, and D.

Minnesota Multiphasic Personality Inventory (Choice A) is a test used to evaluate for psychopathology. It may be considered if there is concern for factitious or somatization disorder, but workup for organic
sleep disorders should be performed first.

Overnight pulse oximetry (Choice B) is useful in the initial evaluation of sleep apnea. Obstructive sleep apnea is commonly seen in patients with obesity, pharyngeal muscle weakness, or upper airway
deformities and often presents with snoring and nighttime apneic episodes. Both obstructive and central sleep apnea present with daytime somnolence. Multiple sleep latency tests will evaluate for these
disorders as well as narcolepsy.

Pulmonary function tests (Choice C) are unlikely to provide useful information in this context. Her symptoms are not suggestive of a respiratory disorder.

MRI of the brain (Choice D) may be considered if the diagnostic workup for narcolepsy and other sleep disorders is unrevealing to evaluate for central nervous system lesions that may be causing her
symptoms. It is not indicated at this time.

Educational Objective: Narcolepsy should be considered in patients who present with daytime somnolence despite restorative sleep. The disorder is characterized by rapid, abnormal transitions from
wakefulness to REM sleep. The diagnosis can be made in patients with daytime somnolence and cataplexy, or in cases where cataplexy is not present, with multiple sleep latency tests that show a shorter
time from wakefulness to the initiation of REM sleep.

https://t.me/USMLENBME2CK ts e t
Previous Next Score Report Lab Values Calculator Help pause
Exam Section 1: ltem 38 of 50 National Board of Medical Examiners
Comprehensive Clinical Science Self-Assessment

"I 38. A 38-year-old woman comes to the office because of a 6-monlh history of progressive fatigue and muscle weakness. During this lime, she has not been able to concentrate at work and has noticed
that her voice has become hoarse. During the past 3 months, she has had a 5.4-kg (12-lb) weight gain despite limiting her food intake. She has bipolar disorder, type 2 diabetes mellitus, and
hypercholesterolemia. Her medications are metformin, lisinopril, lithium, and pravastatin. She does not smoke cigarettes, drink alcohol, or use illicit drugs. Her temperature is 37°C (98.6°F), pulse is
50/min, respirations are 14/min, and blood pressure is 130/90 mm Hg. Examination shows cold skin and coarse hair over the scalp. There is a firm, nontender, symmetrically enlarged thyroid gland.
Deep tendon reflexes are 1+, Her serum thyroid-stimulating hormone concentration is 25 µU/ml. Results of serum electrolyte concentrations and renal function tests are within the reference ranges.
Antithyroid peroxidase antibody testing is negative. Which of the following is the most likely cause of these findings?

A) Autoimmune thyroiditis
B) Granulomatous infiltration
C) Iodine deficiency
D) Medication adverse effect
E) Pituitary dysfunction
Correct Answer: D.

lithium is a mood-stabilizing medication used to treat acute mania and as maintenance treatment in bipolar disorder. Al therapeutic doses, lithium can cause a variety of adverse effects, including tremor,
nystagmus, nephrogenic diabetes insipidus, and thyroid dysfunction. lithium-induced thyroid dysfunction occurs in as many as 50% of patients being treated with lithium, usually within 1 to 2 years of
beginning treatment. The most common manifestations of lithium-induced thyroid dysfunction are hypothyroidism and goiter, although hyperthyroidism and autoimmune thyroiditis may also occur. The goiter
that occurs in lithium-induced thyroid dysfunction is usually diffuse and nonnodular. Patients beginning lithium treatment should undergo baseline thyroid function evaluation. Treatment does not differ from
that of thyroid disorders of other causes. It is usually not necessary to discontinue lithium treatment in the setting of thyroid dysfunction.

Incorrect Answers: A, B, C, and E.

Autoimmune thyroiditis (Choice A), also known as chronic lymphocytic (Hashimoto) thyroiditis, is the most common form of thyroiditis and is characterized by the presence of antithyroid peroxidase and
anlithyroglobulin antibodies. Patients with chronic disease present with signs and symptoms of hypothyroidism, including fatigue, cold intolerance, weight gain, hyporefiexia, myxedema, and dry, cool skin.
This patient with negative antithyroid peroxidase antibody testing is more likely to have lithium-induced hypothyroidism.

Granulomatous infiltration (Choice B) is characteristic of subacute granulomatous thyroiditis, also known as subacute (de Quervain) thyroiditis. Subacute granulomatous thyroiditis is a self-limited condition
that often follows an acute viral illness and may present with symptoms of hyperthyroidism. A painful, tender thyroid is highly suggestive of this diagnosis.

Iodine deficiency (Choice C) is the cause of endemic goiter. It is rare in developed countries because of the routine fortification of common food staples with iodine, such as salt.

Pituitary dysfunction (Choice E) caused by mass lesions or ischemia may lead to secondary hypothyroidism as a result of inadequate TSH production. This patient has an increased TSH concentration,
which is not consistent with secondary hypothyroidism.

Educational Objective: lithium-induced thyroid dysfunction occurs in as many as 50% of patients being treated with lithium, usually within 1 to 2 years of beginning treatment. The most common
manifestations of lithium-induced thyroid dysfunction are hypothyroidism and goiter, although hyperthyroidism and autoimmune lhyroiditis may also occur.

https://t.me/USMLENBME2CK ts e t
Previous Next Score Report Lab Values Calculator Help pause
Exam Section 1: ltem 39 of 50 National Board of Medical Examiners
Comprehensive Clinical Science Self-Assessment

"I 39. A27-year-old man comes to the emergency department because of a progressive rash and wheezing. Yesterday, the patient received the diagnosis of cellulitis of his right leg, and cephalexin therapy
was begun. He has a history of mild, intermittent asthma and eczema. His only other medication is inhaled albuterol as needed. He had a mild allergic reaction to ampicillin as a child. He is in
moderate distress. His temperature is 37°C (98.6°F), pulse is 118/min, respirations are 22/min, and blood pressure is 90/60 mm Hg. Examination shows a 3 x 3-cm, erythematous, warm, tender rash
on the right leg that is unchanged from yesterday. There are new, raised, 1x 2-cm pruritic lesions on the upper and lower extremities and trunk. Diffuse, scattered inspiratory and expiratory wheezes
are heard bilaterally. Which of the following is the most likely diagnosis?

A) Anaphylactic reaction
B) Rocky Mountain spotted fever
C) Staphylococcal scalded skin syndrome
D) Toxic shock syndrome
E) Urticarial vasculitis
Correct Answer: A.

An anaphylactic reaction is a hypersensitivity reaction to an exogenous factor, most commonly medications, foods, or insect stings. It is mediated by activation of lgE, which leads to the release of histamine
and other cytokines from mast cells and basophils. This subsequently leads to acute inflammation and profound vasodilation in multiple organ systems, generally the cardiovascular, respiratory,
gastrointestinal, and integumentary systems. Symptoms include the rapid onset of bronchospasm with dyspnea or wheezing, urticaria, flushing, nausea and emesis, edematous lips and tongue, and
hypotension with tachycardia. When severe and untreated, ii can progress to circulatory collapse with cardiovascular arrest or death caused by asphyxiation and airway obstruction. Treatment includes the
administration of epinephrine, antihistamines, bronchodilators, and glucocorticoids, as well as intravenous fluids. Severe cases may require endotracheal intubation and the administration of vasopressors.
Patients with a history of atopy, such as asthma and eczema, and a history of medication allergies are at increased risk for anaphylaxis. This patient has a history of ampicillin allergy, asthma, and eczema,
and is presenting with symptoms of urticaria, wheezing, tachycardia, and hypotension after initiating a new medication, making an anaphylactic reaction the most likely diagnosis.

Incorrect Answers: B, C, D, and E.

Rocky Mountain spotted fever (Choice B) is a tick-borne illness that is most common in the Southeastern and South Central United States during the spring and summer months. Symptoms occur 2 to 14
days after infection and include fever, headache, malaise, myalgias, arthralgias, as well as an erythematous maculopapular rash that progresses to petechiae. This patient has urticaria and wheezing without
a prior tick bile, making anaphylaxis a more likely diagnosis.

Staphylococcal scalded skin syndrome (Choice C), or toxic epidermal necrolysis, occurs as a result of the release of exotoxin from a Staphylococcus aureus infection. It most commonly presents in children
and immunosuppressed adults with a diffuse erythematous rash that progresses to desquamation, associated with fever and malaise. While this patient's cellulitis may be caused by a staphylococcal skin
infection, his symptoms are otherwise not characteristic for scalded skin syndrome.

Toxic shock syndrome (Choice 0) occurs as a result of the release of exotoxin from a Staphylococcus aureus infection, which leads to the release of cytokines throughout the body. It commonly presents with
fever, diffuse macular erythroderma that progresses to desquamation, and hypotension. It is most often associated with women who leave menstrual tampons in place for an extended period of time. It does
not cause urticaria or wheezing, and therefore, anaphylaxis is a more likely diagnosis in this patient.

'- - - -• 4 .o
Urticaria I vasculitis (Choice E) is a rare vasculitis of the small cutaneous vessels that presents with urticaria, as well as a variety of systemic symptoms, such as arthralgias, abdominal pain, proteinuria,
' -- 4"-4,44 Ali

https://t.me/USMLENBME2CK ts e t
Previous Next Score Report Lab Values Calculator Help pause
Exam Section 1: ltem 39 of 50 National Board of Medical Examiners
Comprehensive Clinical Science Self-Assessment

A) Anaphylactic reaction
B) Rocky Mountain spotted fever
C) Staphylococcal scalded skin syndrome
D) Toxic shock syndrome
E) Urticarial vasculitis
Correct Answer: A.

An anaphylactic reaction is a hypersensitivity reaction to an exogenous factor, most commonly medications, foods, or insect stings. It is mediated by activation of lgE, which leads to the release of histamine
and other cytokines from mast cells and basophils. This subsequently leads to acute inflammation and profound vasodilation in multiple organ systems, generally the cardiovascular, respiratory,
gastrointestinal, and integumentary systems. Symptoms include the rapid onset of bronchospasm with dyspnea or wheezing, urticaria, flushing, nausea and emesis, edematous lips and tongue, and
hypotension with tachycardia. When severe and untreated, it can progress to circulatory collapse with cardiovascular arrest or death caused by asphyxiation and airway obstruction. Treatment includes the
administration of epinephrine, antihistamines, bronchodilators, and glucocorticoids, as well as intravenous fluids. Severe cases may require endotracheal intubation and the administration of vasopressors.
Patients with a history of atopy, such as asthma and eczema, and a history of medication allergies are at increased risk for anaphylaxis. This patient has a history of ampicillin allergy, asthma, and eczema,
and is presenting with symptoms of urticaria, wheezing, tachycardia, and hypotension after initiating a new medication, making an anaphylactic reaction the most likely diagnosis.

Incorrect Answers: B, C, D, and E.

Rocky Mountain spotted fever (Choice B) is a tick-borne illness that is most common in the Southeastern and South Central United States during the spring and summer months. Symptoms occur 2 to 14
days after infection and include fever, headache, malaise, myalgias, arthralgias, as well as an erythematous maculopapular rash that progresses to petechiae. This patient has urticaria and wheezing without
a prior tick bite, making anaphylaxis a more likely diagnosis.

Staphylococcal scalded skin syndrome (Choice C), or toxic epidermal necrolysis, occurs as a result of the release of exotoxin from a Staphylococcus aureus infection. It most commonly presents in children
and immunosuppressed adults with a diffuse erythematous rash that progresses to desquamation, associated with fever and malaise. While this patient's cellulitis may be caused by a staphylococcal skin
infection, his symptoms are otherwise not characteristic for scalded skin syndrome.

Toxic shock syndrome (Choice 0) occurs as a result of the release of exotoxin from a Staphylococcus aureus infection, which leads to the release of cytokines throughout the body. It commonly presents with
fever, diffuse macular erythroderma that progresses to desquamation, and hypotension. It is most often associated with women who leave menstrual tampons in place for an extended period of lime. It does
not cause urticaria or wheezing, and therefore, anaphylaxis is a more likely diagnosis in this patient.

Urticaria I vasculitis (Choice E) is a rare vasculitis of the small cutaneous vessels that presents with urticaria, as well as a variety of systemic symptoms, such as arthralgias, abdominal pain, proteinuria,
hematuria, cough, hemoptysis, and uveitis. It is more prevalent in women and commonly occurs later in life. It is less likely the cause of this patient's urticaria than anaphylaxis.

Educational Objective: Anaphylaxis is an lgE-mediated hypersensitivity reaction that leads to extensive degranulation of mast cells and basophils, which results in acute multisystem inflammation and
vasodilation. Symptoms include bronchospasm with associated dyspnea or wheezing, urticaria, flushing, nausea and emesis, edematous lips and tongue, and hypotension with tachycardia. Treatment
includes epinephrine, antihistamines, bronchodilators, glucocorticoids, and fluids.

https://t.me/USMLENBME2CK ts e t
Previous Next Score Report Lab Values Calculator Help pause
Exam Section 1: Item 40 of 50 National Board of Medical Examiners
Comprehensive Clinical Science Self-Assessment

✓ 40. A 37-year-old man comes to the physician because he has had two bowel movements containing bright red blood during the past 6 hours. He has a 3-day history of intermittent severe abdominal
cramps and diarrhea. He has had five loose stools daily during this period. He has no history of serious illness and takes no medications. His last visit to a physician was 5 years ago. His most recent
travel experience was a trip to Canada 6 months ago. His temperature is 37°C (98.6°F), pulse is 98/min, and blood pressure is 126/78 mm Hg. Abdominal examination shows diffuse severe tenderness
with no peritoneal signs. Bowel sounds are increased. Rectal examination shows no abnormalities. There is no stool in the rectal vault. His hemoglobin concentration is 13 g/dL, leukocyte count is
12,300/mm 3, and platelet count is 302,000/mm3. Which of the following is the most likely causal organism?

A) Clostridium difficile
B) Clostridium perfringens
C) Cryptosporidium parvum
D) Enterotoxigenic Escherichia coli
E) Eschenchia coli 0157:H?
Correct Answer: E.

This patient is presenting with generalized abdominal cramping and diarrhea, likely caused by a viral, bacterial, or parasitic enteritis. Escherichia coli 0157:H? is a bacterium that often causes hemorrhagic
diarrhea. It is differentiated from other strains of E. coli because it produces a Shiga toxin that damages the intestinal wall, which causes hemorrhagic enteritis manifesting as bloody diarrhea. It is also known
as enterohemorrhagic E. coli infection. Hemolytic uremic syndrome is a complication often triggered by infection with Shiga toxin-producing E. colt 0157:H?. It typically presents with the triad of hemolytic
anemia, thrombocytopenia, and acute kidney injury. Treatment for uncomplicated cases is supportive with oral or intravenous hydration. Antibiotics may be required in severe cases.

Incorrect Answers: A, B, C, and D.

Clostridium difficile (Choice A) infection is an opportunistic and often hospital-acquired infection that commonly occurs after the use of antibiotics caused by the loss of native colonic microbiota that allows
competition-free proliferation of C. difficile. Patients typically present with watery diarrhea and abdominal pain.

Clostridium perfringens (Choice 8) can result in myonecrosis (gas gangrene) following traumatic injury. Its spores can also survive in undercooked food, and, if ingested, can result in food poisoning. It would
not typically cause bloody diarrhea.

Cryptosporidium parvum (Choice C) is a parasitic infection that causes cryptosporidiosis, which is characterized by watery diarrhea. It does not typically cause bloody diarrhea and more commonly affects
immunocompromised hosts.

Enterotoxigenic Escherichia coli (Choice D) is a strain of E. coli that causes watery, not bloody, diarrhea.

Educational Objective: Escherichia coli 0157:H? is a bacterium that produces a Shiga toxin, which damages the intestinal wall, leading to bloody diarrhea. Complications include hemolytic uremic syndrome.

https://t.me/USMLENBME2CK ti
Previous Next Score Report Lab Values Calculator Help pause
Exam Section 1: Item 41 of 50 National Board of Medical Examiners
Comprehensive Clinical Science Self-Assessment

"I 41. A 50-year-old man with a 30-year history of paranoid schizophrenia is admitted to a psychiatric facility because of an exacerbation of his condition. His history includes multiple admissions to the
hospital for his condition. The physician prescribes oral antipsychotic therapy and the patient agrees to take the medication while in the facility, but he says that he does not need it and he will stop
taking the medication as soon as he is discharged. He states, "I don't trust any of you. You're all out to get me. But I'll play along until my time here is done." Which of the following is the most
appropriate next step in management?

A) Administer the antipsychotic therapy until the patient's paranoia completely resolves
B) Change the oral antipsychotic therapy to a long-acting injectable formulation
C) Commit the patient to the facility against his will
D) Discharge the patient with support from the local community mental health center, telling him that he can always return to the facility if he needs help in the future
E) Maximize the dose of the anti psychotic therapy until the patient gains insight into his illness
Correct Answer: B.

Nonadherence is a common challenge in patients with schizophrenia. Most antipsychotic agents are dosed with daily oral dosing, which creates a risk for undertreatmenl, relapse, and potential
hospitalization or death in patients who self-discontinue therapy. Long-acting injectable (LAI) antipsychotics provide a useful method of sustaining treatment and reducing relapse and hospitalization in
patients with high risk for or known history of nonadherence. A variety of dosing regimens may be used. Available LAI antipsychotics include both first-generation antipsychotics, such as haloperidol, and
atypical antipsychotics, such as aripiprazole, olanzapine, paliperidone, or risperidone. The adverse effect profiles of LAI antipsycholics are typically similar to their oral equivalents. Selection of an LAI
antipsychotic is based on history of prior treatment response and medication adverse effect profiles.

Incorrect Answers: A, C, D, and E.

Administering the antipsychotic therapy until the patient's paranoia completely resolves (Choice A) is incorrect. This patient has a known history of multiple hospitalizations and remains at high risk for relapse
following discharge from the hospital even if his symptoms are well-controlled at that time.

Committing the patient to the facility against his will (Choice C) is incorrect. The patient has currently consented to treatment but has stated an intention not to adhere following discharge. Indefinite
commitment to the facility is not an effective solution to nonadherence.

Discharging the patient with support from the local community mental health center, telling him that he can always return to the facility if he needs help in the future (Choice D) is incorrect. Many patients with
psychotic disorders experience a cycle of hospitalization and relapse that is exacerbated by nonadherence, unstable living conditions, and frequent concomitant drug abuse. LAI antipsychotics are a better
option to help this patient break the cycle of hospitalization that he has previously experienced.

Maximizing the dose of the anti psychotic therapy until the patient gains insight into his illness (Choice E) is incorrect. Many anti psychotics have adverse effects, including extra pyramidal effects, tardive
dyskinesia, and metabolic acidosis. Higher oral doses may exacerbate adverse effects and contribute lo medication nonadherence.

Educational Objective: Long-acting injectable (LAI) antipsychotics provide a useful method of sustaining treatment and reducing relapse and hospitalization in patients with a high risk or known history of
nonadherence. Both first-generation and atypical anlipsychotics are available in LAI formulations.

https://t.me/USMLENBME2CK ts e t
Previous Next Score Report Lab Values Calculator Help pause
Exam Section 1: ltem 42 of 50 National Board of Medical Examiners
Comprehensive Clinical Science Self-Assessment

"I 42. An 8-year-old boy is brought to the emergency department because of a 2-day history of urinary urgency and frequency and right-sided abdominal cramps. The cramps are intermittent, and he cannot
get comfortable in any position when they occur. He has not had fever, diarrhea, or constipation, but he has vomited twice. He has no history of similar episodes or serious illness. He takes no
medications. His paternal uncle has a history of renal calculi, and his cousin has Crohn disease. The patient has no dietary restrictions. His temperature is 37.5€ (99.5°F), The right flank is tender to
palpation. An abdominal x-ray shows a 3-mm ovoid, opaque mass in the right pelvis. Laboratory studies show:
Serum
Urea nitrogen 20 mg/dl
Creatinine 0.1 mg/dL
Urine
pH 6.5
RBC 30-50/hpf
WBC 3-5/hpf

In addition to administration of morphine, which of the following is the most appropriate next step in management?

A) Oral administration of allopurinol


B) Oral administration of fluids
C) Oral administration of hydrochlorothiazide
D) Oral administration of potassium citrate
E) Oral administration of pyridoxine
F) Lithotripsy
Correct Answer: B.

Nephrolithiasis, or ureterolithiasis, describes the formation of calculi within the urinary collecting system, otherwise known as renal calculi or ureteral stones. The typical presentation of renal calculi that have
become lodged in the ureter (ureterolithiasis) includes severe, intermittent, sharp flank pain with either gross or microscopic hematuria, leukocytosis, and, occasionally, evidence of a concomitant urinary tract
infection. Calcium-containing stones are the most common varieties, although renal calculi may also be composed of cystine, struvite, and uric acid. While abdominal plain x-rays were traditionally used for
diagnosis, urate stones are radiolucent, and given the availability and sensitivity of advanced imaging, a noncontrast CT scan of the abdomen and pelvis is now the preferred imaging modality for definitive
diagnosis. One exception to this is in pediatrics, where imaging modalities that involve less radiation, such as ultrasonography and x-ray, are often used before modalities such as CT. Acute management
depends on the size and location of the calculi in addition to complicating factors such as abnormal anatomy or the concurrent presence of a urinary tract infection. Calculi larger than 6 mm often require
intervention as they are unlikely to pass on their own. For smaller calculi in clinically stable patients, treatment with oral hydration and analgesia is appropriate.

Incorrect Answers: A, C, D, E, and F.

Oral administration of allopurinol (Choice A) may be useful in the management of renal calculi composed of uric acid. Uric acid stones are typically radiolucent. This patient's stone is radiopaque, suggesting
a stone composed of either calcium oxalate, calcium phosphate, or ammonium magnesium phosphate (struvite). Oral administration of potassium citrate (Choice 0) is useful for urine alkalinization, which
increases the solubility of uric acid, thereby preventing uric acid stone formation or aiding in the dissolution of existing uric acid stones.

https://t.me/USMLENBME2CK ti
Previous Next Score Report Lab Values Calculator Help pause
Exam Section 1: ltem 42 of 50 National Board of Medical Examiners
Comprehensive Clinical Science Self-Assessment

A) Oral administration of allopurinol


B) Oral administration of fluids
C) Oral administration of hydrochlorothiazide
D) Oral administration of potassium citrate
E) Oral administration of pyridoxine
F) Lithotripsy
Correct Answer: B.

Nephrolithiasis, or ureterolithiasis, describes the formation of calculi within the urinary collecting system, otherwise known as renal calculi or ureteral stones. The typical presentation of renal calculi that have
become lodged in the ureter (ureterolithiasis) includes severe, intermittent, sharp flank pain with either gross or microscopic hematuria, leukocytosis, and, occasionally, evidence of a concomitant urinary tract
infection. Calcium-containing stones are the most common varieties, although renal calculi may also be composed of cystine, struvite, and uric acid. While abdominal plain x-rays were traditionally used for
diagnosis, urate stones are radiolucent, and given the availability and sensitivity of advanced imaging, a noncontrast CT scan of the abdomen and pelvis is now the preferred imaging modality for definitive
diagnosis. One exception to this is in pediatrics, where imaging modalities that involve less radiation, such as ultrasonography and x-ray, are often used before modalities such as CT. Acute management
depends on the size and location of the calculi in addition to complicating factors such as abnormal anatomy or the concurrent presence of a urinary tract infection. Calculi larger than 6 mm often require
intervention as they are unlikely to pass on their own. For smaller calculi in clinically stable patients, treatment with oral hydration and analgesia is appropriate.

Incorrect Answers: A, C, D, E, and F.

Oral administration of allopurinol (Choice A) may be useful in the management of renal calculi composed of uric acid. Uric acid stones are typically radiolucent. This patient's stone is radiopaque, suggesting
a stone composed of either calcium oxalate, calcium phosphate, or ammonium magnesium phosphate (struvite). Oral administration of potassium citrate (Choice 0) is useful for urine alkalinization, which
increases the solubility of uric acid, thereby preventing uric acid stone formation or aiding in the dissolution of existing uric acid stones.

Oral administration of hydrochlorothiazide (Choice C) reduces renal excretion of calcium and is useful for preventing formation of calcium-based renal calculi. Thiazides are typically reserved for the
prevention of recurrent calcium stones after dietary modification has failed.

Oral administration of pyridoxine (Choice E) is not recommended. Pyridoxine (vitamin 86) has been proposed to reduce renal oxalate excretion, thereby lowering the risk for formation of calcium oxalate
stones. However, recent, large trials have not found pyridoxine to have a significantly beneficial effect in preventing the formation of renal calculi.

Lithotripsy (Choice F) is a method of using extracorporeal shock waves to fragment renal calculi, aiding in their passage. Most calculi less than 6 mm are able to pass spontaneously and do not require
lithotripsy.

Educational Objective: Acute management of nephrolithiasis depends on the size and location of the calculi in addition to complicating factors such as abnormal anatomy or the concurrent presence of a
urinary tract infection. Calculi larger than 6 mm often require intervention as they are unlikely to pass on their own. For smaller calculi in clinically stable patients, treatment with oral hydration and analgesia
is appropriate.

https://t.me/USMLENBME2CK ti
Previous Next Score Report Lab Values Calculator Help pause
Exam Section 1: Item 43 of 50 National Board of Medical Examiners
Comprehensive Clinical Science Self-Assessment

"I 43. A previously healthy 57-year-old woman comes to the physician because of a 6-month history of muscle weakness and fatigue. The weakness began in the muscles of her face and has progressed to
all of the muscles in her body. Examination shows bilateral ptosis and weakness of the left abducens muscle. An x-ray of the chest shows a mediaslinal mass. Which of the following is the most
appropriate next step in diagnosis of this patient's neurologic findings?

A) Nerve conduction studies with repetitive stimulation


B) MRI of the cervical spine
C) Somatosensory evoked potentials
D) Biopsy of a peripheral nerve
E) Biopsy of a skeletal muscle
Correct Answer: A.

Myasthenia gravis is an autoimmune disorder of neuromuscular transmission that has a bimodal age of onset, peaking in both young adults (teens to 20s) and older adults (50s to 60s). Patients with
myasthenia gravis possess antibodies that block or destroy nicotinic acetylcholine receptors (nAChRs), which interferes with neuromuscular Junction signaling. Many patients with myasthenia gravis have
thymic hyperplasia or thymoma. In myasthenia gravis, the thymus, which is the site of T-lymphocyte maturation, may show abnormalities in the presentation of the nAChR as an antigen to helper T
lymphocytes. This abnormal antigen presentation is postulated to result in autoimmune attack of the nAChR on skeletal muscle cells. Patients present with progressively worsening skeletal muscle weakness
at rest and fatigability on activation in focal or generalized muscle groups. Thus, nerve conduction studies with repetitive stimulation are used to confirm the diagnosis. The ocular, bulbar, and neck muscles
are commonly affected. Most concerningly, respiratory muscle weakness can lead to respiratory insufficiency, and in an acute flare, these patients may require mechanical ventilation. Management includes
symptomatic treatments such as acetylcholinesterase inhibitors along with immune modulating therapies. In patients with or without thymoma, complete thymectomy may be curative.

Incorrect Answers: B, C, D, and E.

MRI of the cervical spine (Choice B) would be helpful in diagnosing central cord syndrome, which presents with decreased sensation and strength of the upper extremities. However, bulbar weakness would
not be present.

Somatosensory evoked potentials (Choice C) are used to evaluate electrical activity of the brain resulting from stimulation of peripheral sensory fibers to assess somatosensory functioning. Myasthenia
gravis is a disorder of neuromuscular, not somatosensory transmission.

Biopsy of a peripheral nerve (Choice 0) can be helpful in diagnosing causes of peripheral neuropathy such as mononeuritis multiplex. This patient presents with symptoms of bulbar weakness, which are
muscles supplied by cranial nerves. Biopsy may be helpful to exclude alternative diagnoses, however nerve conduction studies would be performed first.

Biopsy of a skeletal muscle (Choice E) would be helpful in diagnosing inflammatory disorders of the muscle such as dermatomyositis, polymyositis, or inclusion body myositis. It is characterized by infiltration
of inflammatory cells (generally lymphocytes and macrophages) on histology along with fibrosis.

Educational Objective: Myasthenia gravis is an autoimmune disorder of neuromuscular transmission with progressively worsening skeletal muscle weakness at rest and fatigability on activation in focal or
generalized muscle groups. Nerve conduction studies with repetitive stimulation are used to confirm the diagnosis.

https://t.me/USMLENBME2CK ts e t
Previous Next Score Report Lab Values Calculator Help pause
Exam Section 1: Item 44 of 50 National Board of Medical Examiners
Comprehensive Clinical Science Self-Assessment

"I 44. A 29-year-old woman, gravida 1, para 1, comes to the physician 1 week after noticing a mildly tender lump in her left breast. Five months ago, she delivered a healthy newborn at term following an
uncomplicated pregnancy and spontaneous vaginal delivery. She is breast-feeding and says her infant feeds equally from both breasts. The patient has no history of serious illness, and her only
medication is a prenatal vitamin. Her mother was diagnosed with breast cancer at the age of 61 years. The patient's vital signs are within normal limits. Examination shows no axillary or supraclavicular
lymphadenopathy. There is a 3-cm, smooth, mobile, cystic, tender mass in the upper outer quadrant of the left breast. Which of the following is the most appropriate next step in management?

A) Bromocriptine therapy
B) Dicloxacillin therapy
C) Discontinuing breast-feeding
D) Fine-needle aspiration of the mass
E) Increasing breast-feeding on the left breast
Correct Answer: D.

A galactocele is a benign cyst of the mammary glands, caused by an obstruction to lactiferous duct outflow. Given its location, it is filled with milk rather than clear fluid. They occur in lactating women, or
women who recently discontinued lactating, and present with a firm, fluctuant mass that is painless or mildly tender, commonly located in the subareolar area. Vital signs will be within normal limits. Diagnosis is
typically clinical but can also be based on fine-needle aspiration, which would be the most appropriate next step in management for this patient. If obtained, ultrasonography would show a cystic mass.
Galactoceles generally resolve on their own but can be aspirated or excised if they persist and are symptomatic. Complications are uncommon but include superimposed infection and abscess formation, as
well as rupture and subsequent inflammation.

Incorrect Answers: A, B, C, and E.

Bromocriptine therapy (Choice A) is often used in the treatment of prolactinomas or hyperprolactinemia, which can present with galactorrhea. It does not play a role in the treatment of galactocele.

Dicloxacillin therapy (Choice B) is used to treat skin and soft tissue infections, such as those caused by Staphylococcal and Streptococcal species. It can be used in the treatment of mastitis or breast abscess,
which present with erythema, induration, and pain of the breast. This patient has a fluctuant mass with no systemic symptoms such as fever, which is more representative of a galactocele.

Discontinuing breast-feeding (Choice C) or increasing breast-feeding on the left breast (Choice E) would not affect this patient's galactocele. While commonly occurring in lactating women, changes in breast-
feeding have not consistently been associated with an improvement in galactocele symptoms. Rather, this patient should undergo fine-needle aspiration to confirm the diagnosis and improve her symptoms.

Educational Objective: A galactocele is a benign cyst of the mammary glands, caused by an obstruction to lactiferous duct outflow. They commonly present with a firm, fluctuant mass that is painless or mildly
tender in lactating women. Diagnosis is based on physical examination and the results of a fine-needle aspiration, which will show milky fluid.

https://t.me/USMLENBME2CK ti
Previous Next Score Report Lab Values Calculator Help pause
Exam Section 1: Item 45 of 50 National Board of Medical Examiners°
Comprehensive Clinical Science Self-Assessment

✓ Inspiration Expiration

45. An 18-year-old man is brought to the physician 1 hour after he felt light-headed while playing basketball. He has not had chest pain, palpitations, or shortness of breath. He has no history of serious
illness and takes no medications. His father died in a motor vehicle collision at the age of 35 years. There is no family history of serious illness. The patient's pulse is 80/min, and blood pressure is
115/85 mm Hg. There is no jugular venous distention. Pulmonary examination shows no abnormalities. A diagram of cardiac auscultation is shown. The murmur does not change with inspiration,
position, or Valsalva maneuver. An ECG shows no abnormalities. Which of the following is the most appropriate next step in management?

A) Aortic valve repair


B) Implantation of a cardiac defibrillator
C) Recommendation not to play competitive sports
D) Warfarin therapy
E) No management is indicated at this time
Correct Answer: E.

No management is indicated at this time for this otherwise healthy 18-year-old patient with no apparent risk factors for cardiac disease. Benign murmurs are common in childhood and adolescence and do
not require treatment. They are also referred to as innocent murmurs and result from normal blood flow. However, the clinician should consider the possibility of serious cardiac disease, as a murmur may
be the single presenting feature. Risk factors include a family history of cardiac disease or sudden cardiac death, exposure to teratogens in utero, or a prior history of mucocutaneous lymph node syndrome
(Kawasaki disease) or rheumatic fever. On physical examination, a holosystolic or diastolic murmur should prompt concern and further evaluation. Additionally, since benign murmurs arise from normal
blood flow, they should decrease in intensity with maneuvers that decrease blood return to the heart, such as the Valsalva maneuver, though this is not always apparent in the clinical setting. Further
workup is indicated if the patient presents with vital sign abnormalities and/or significant symptoms, especially chest pain, syncope, palpitations, shortness of breath, poor exercise tolerance, or failure to
thrive.

Incorrect Answers: A, B, C, and D.

Aortic valve repair (Choice A) would be indicated if this patient's symptoms were caused by a bicuspid aortic valve or other congenital valve defect. The cardiac auscultation characteristics in a stenosed
valve may include a mid-systolic ejection murmur, an abnormally quiet S caused by impaired closing of the valve, and paradoxical splitting of the So.

Implantation of a cardiac defibrillator (Choice B) would be recommended if the patient was at risk for torsades de pointes from congenital long QT syndrome or if there was concern for hypertrophic,
obstructive cardiomyopathy. Additionally, there would be a strong recommendation not to play competitive sports (Choice C) given risk for sudden cardiac death. However, this patient has no family history
of cardiac conduction abnormalities nor concerning features on his ECG.

Warfarin therapy (Chis D)) is indicated for natients with atrial sental defects or ventricular sental defects who are at risk for paradoxical thromhaemholism Atrial sental defects are associated with a fixed

« https://t.me/USMLENBME2CK ts e t
Previous Next Score Report Lab Values Calculator Help pause
Exam Section 1: Item 45 of 50 National Board of Medical Examiners
Comprehensive Clinical Science Self-Assessment

45. An 18-year-old man is brought to the physician 1 hour after he felt light-headed while playing basketball. He has not had chest pain, palpitations, or shortness of breath. He has no history of serious
illness and takes no medications. His father died in a motor vehicle collision at the age of 35 years. There is no family history of serious illness. The patient's pulse is 80/min, and blood pressure is
115/85 mm Hg. There is no jugular venous distention. Pulmonary examination shows no abnormalities. A diagram of cardiac auscultation is shown. The murmur does not change with inspiration,
position, or Valsalva maneuver. An ECG shows no abnormalities. Which of the following is the most appropriate next step in management?

A) Aortic valve repair


B) Implantation of a cardiac defibrillator
C) Recommendation not to play competitive sports
D) Warfarin therapy
E) No management is indicated at this time
Correct Answer: E.

No management is indicated at this time for this otherwise healthy 18-year-old patient with no apparent risk factors for cardiac disease. Benign murmurs are common in childhood and adolescence and do
not require treatment. They are also referred to as innocent murmurs and result from normal blood flow. However, the clinician should consider the possibility of serious cardiac disease, as a murmur may
be the single presenting feature. Risk factors include a family history of cardiac disease or sudden cardiac death, exposure to teratogens in utero, or a prior history of mucocutaneous lymph node syndrome
(Kawasaki disease) or rheumatic fever. On physical examination, a holosystolic or diastolic murmur should prompt concern and further evaluation. Additionally, since benign murmurs arise from normal
blood flow, they should decrease in intensity with maneuvers that decrease blood return to the heart, such as the Valsalva maneuver, though this is not always apparent in the clinical setting. Further
workup is indicated if the patient presents with vital sign abnormalities and/or significant symptoms, especially chest pain, syncope, palpitations, shortness of breath, poor exercise tolerance, or failure to
thrive.

Incorrect Answers: A, B, C, and D.

Aortic valve repair (Choice A) would be indicated if this patient's symptoms were caused by a bicuspid aortic valve or other congenital valve defect. The cardiac auscultation characteristics in a stenosed
valve may include a mid-systolic ejection murmur, an abnormally quiet S caused by impaired closing of the valve, and paradoxical splitting of the So.

Implantation of a cardiac defibrillator (Choice B) would be recommended if the patient was at risk for torsades de pointes from congenital long QT syndrome or if there was concern for hypertrophic,
obstructive cardiomyopathy. Additionally, there would be a strong recommendation not to play competitive sports (Choice C) given risk for sudden cardiac death. However, this patient has no family history
of cardiac conduction abnormalities nor concerning features on his ECG.

Warfarin therapy (Choice D) is indicated for patients with atrial septal defects or ventricular septa! defects who are at risk for paradoxical thromboembolism. Atrial septal defects are associated with a fixed
split S, and ventricular septal defects tend to have a holosystolic murmur, neither of which is present on the cardiac auscultation tracing.

Educational Objective: Benign murmurs are common in childhood and adolescence, and most cases can be managed with observation. Patients who present with severe cardiopulmonary symptoms, have
a family history of cardiac disease, or are at risk for congenital or acquired heart defects from prior exposures should receive further workup.

https://t.me/USMLENBME2CK ti e
Previous Next Score Report Lab Values Calculator Help pause
Exam Section 1: Item 46 of 50 National Board of Medical Examiners
Comprehensive Clinical Science Self-Assessment

✓ 46. A 37-year-old woman is brought to the emergency department 30 minutes after an episode of syncope at a shopping mall. She says she had shortness of breath and was sweating before she passed
out. She has a childhood history of asthma. She currently has panic disorder and gastroesophageal reflux disease. Current medications include sertraline, omeprazole, and an oral contraceptive. On
arrival, she is alert and oriented to person, place, and time. Her temperature is 37.4°C (99.3°F), pulse is 110/min, and blood pressure is 90/60 mm Hg. Pulse oximetry on room air shows an oxygen
saturation of 90%. Examination, including neurologic examination, shows no abnormalities. Laboratory studies show:
Hemoglobin 12 g/dL
Hematocrit 36%
Leukocyte count 9000/mm

Arterial blood gas analysis on room air:


pH 7.46
Pco, 30 mm Hg
Po, 62mm Hg

Which of the following is the most likely explanation for these findings?

A) Asthma
B) Congestive heart failure
C) Panic episode
D) Pneumonia
E) Pulmonary embolism
Correct Answer: E.

This patient's presentation and findings are concerning for acute pulmonary embolism (PE). It classically presents with acute chest pain, shortness of breath, and hypoxemia. Some patients may also
experience unexplained syncope. It is often difficult to distinguish PE from other acute respiratory disorders, and the differential includes asthma exacerbation, vocal cord dysfunction, acute coronary
syndrome, panic episode, and vasovagal syncope. The clinician must maintain a high index of suspicion, and the use of clinical decision aids such as Wells score is recommended. Risk factors for acute
PE include immobility, obesity, recent surgery, malignancy, pregnancy, use of oral contraceptives, trauma, fracture of long bones, and inherited thrombophilia. ECG often displays normal sinus rhythm or
sinus tachycardia. Arterial blood gas analysis most typically shows acute respiratory alkalosis, hypoxemia, and an increased A-a gradient caused by ventilation-perfusion mismatch. CT pulmonary
angiography is the preferred method to confirm the diagnosis. This patient may be experiencing a massive PE given tachycardia and borderline hypotension, and she should be treated with anticoagulants
and also potentially thrombolytics.

Incorrect Answers: A, B, C, and D.

Asthma (Choice A) exacerbation typically presents with wheezing, dry cough, and dyspnea. Syncope is less common.

fonaoctiua hoart failure (heino [ ie inlikol niuan the aheanoo ef peeniater] nhweial ayamination finelinre Thaeo incl\o inalar vanone rlietan«inn norinhoral olama aeoitae honatoinolar reflux

https://t.me/USMLENBME2CK ti
Previous Next Score Report Lab Values Calculator Help pause
Exam Section 1: Item 46 of 50 National Board of Medical Examiners
Comprehensive Clinical Science Self-Assessment

Which of the following is the most likely explanation for these findings?

A) Asthma
B) Congestive heart failure
C) Panic episode
D) Pneumonia
E) Pulmonary embolism
Correct Answer: E.

This patient's presentation and findings are concerning for acute pulmonary embolism (PE). It classically presents with acute chest pain, shortness of breath, and hypoxemia. Some patients may also
experience unexplained syncope. It is often difficult to distinguish PE from other acute respiratory disorders, and the differential includes asthma exacerbation, vocal cord dysfunction, acute coronary
syndrome, panic episode, and vasovagal syncope. The clinician must maintain a high index of suspicion, and the use of clinical decision aids such as Wells score is recommended. Risk factors for acute
PE include immobility, obesity, recent surgery, malignancy, pregnancy, use of oral contraceptives, trauma, fracture of long bones, and inherited thrombophilia. ECG often displays normal sinus rhythm or
sinus tachycardia. Arterial blood gas analysis most typically shows acute respiratory alkalosis, hypoxemia, and an increased A-a gradient caused by ventilation-perfusion mismatch. CT pulmonary
angiography is the preferred method to confirm the diagnosis. This patient may be experiencing a massive PE given tachycardia and borderline hypotension, and she should be treated with anticoagulants
and also potentially thrombolytics.

Incorrect Answers: A, B, C, and D.

Asthma (Choice A) exacerbation typically presents with wheezing, dry cough, and dyspnea. Syncope is less common.

Congestive heart failure (Choice B) is unlikely, given the absence of associated physical examination findings. These include jugular venous distension, peripheral edema, ascites, hepatojugular reflux,
pulmonary crackles, and an Sg or S4 gallop.

Panic episode (Choice C) may be considered, but other reversible causes of this patient's symptoms should be assessed first. Patients in acute panic episode typically hyperventilate, which does cause
respiratory alkalosis on arterial blood gas analysis; however, a normal A-a gradient and normal oxygenation would be expected.

Pneumonia (Choice D) would be unlikely, as it typically presents with dyspnea, productive cough, and signs of systemic inflammation such as fever and malaise. Leukocytosis would also support infection
as the cause of the patient's symptoms.

Educational Objective: Acute pulmonary embolism (PE) classically presents with acute chest pain, shortness of breath, and hypoxemia; however, vague and nonspecific presentations are common.
Unexplained syncope should prompt consideration for PE, especially in the setting of risk factors such as immobility, obesity, recent surgery, malignancy, pregnancy, use of oral contraceptives, trauma,
fracture of long bones, and inherited thrombophilia.

https://t.me/USMLENBME2CK ti
Previous Next Score Report Lab Values Calculator Help pause
Exam Section 1: Item 47 of 50 National Board of Medical Examiners
Comprehensive Clinical Science Self-Assessment

• 4n. A 62-year-old woman comes to the physician because of a 2-month history of generalized fatigue and mild low back pain. She has not had
any other symptoms. She has no history of serious illness and takes no medications. Her last health maintenance examination was 10
months ago, and a complete blood count at that time was within the reference range. Examination today shows no abnormalities.
I
Laboratory studies show:
Hematocrit 30%
Mean corpuscular volume 103 m3
Leukocyte count 8200/mm3
Segmented neutrophils 67%
Bands 4% •
Lymphocytes 29% 0
Platelet count 165,000/mm° (
Serum
Ca?+ 10.9 mg/dL
Creatinine 1.8 mg/dL

A blood smear is shown. Which of the following is the most appropriate next step to confirm the diagnosis?

A) Liver function tests


B) Measurement of serum vitamin B, (cobalamin) concentration
C) Serum iron studies
D) Colonoscopy
E ) Bone marrow biopsy

Correct Answer: E.

This patient's presenting findings of bone pain, anemia, hypercalcemia, and renal dysfunction are consistent with a diagnosis of multiple myeloma. Multiple myeloma is a malignancy caused by the
neoplastic proliferation of plasma cells. Neoplastic plasma cells overproduce monoclonal immunoglobulin light or heavy chains, which may result in acute renal failure. Patients commonly present with
constitutional symptoms of fatigue and weight loss, lytic bone lesions, hepatosplenomegaly, symptoms of hypercalcemia (eg, abdominal cramping, kidney stones, psychiatric disturbance), symptoms of
anemia (eg, pallor, lightheadedness, dyspnea on exertion), or with opportunistic infections secondary to immune dysfunction. Further workup is recommended with a complete blood count, complete
metabolic panel, urinalysis, urine and serum protein electrophoresis, and imaging to determine the extent of malignancy. Peripheral blood smear classically discloses red blood cell aggregation in a
rouleaux formation. The diagnosis is confirmed with bone marrow biopsy.

Incorrect Answers: A, 8, C, and D.

Liver function tests (Choice A) frequently disclose hepatic dysfunction in patients with multiple myeloma, often caused by infiltration of the liver by plasma cells. However, abnormal liver function testing is
nonspecific and not helpful in confirming the diagnosis.

https://t.me/USMLENBME2CK ts e t
Previous Next Score Report Lab Values Calculator Help pause
Exam Section 1: Item 47 of 50 National Board of Medical Examiners
Comprehensive Clinical Science Self-Assessment

165,000/mm°
Serum
Ca?+ 10.9 mg/dL
Creatinine 1.8 mg/dL

A blood smear is shown. Which of the following is the most appropriate next step to confirm the diagnosis?

A) liver function tests


B) Measurement of serum vitamin B,(cobalamin) concentration
C) Serum iron studies
D) Colonoscopy
E ) Bone marrow biopsy
Correct Answer: E.

This patient's presenting findings of bone pain, anemia, hypercalcemia, and renal dysfunction are consistent with a diagnosis of multiple myeloma. Multiple myeloma is a malignancy caused by the
neoplastic proliferation of plasma cells. Neoplastic plasma cells overproduce monoclonal immunoglobulin light or heavy chains, which may result in acute renal failure. Patients commonly present with
constitutional symptoms of fatigue and weight loss, lytic bone lesions, hepatosplenomegaly, symptoms of hypercalcemia (eg, abdominal cramping, kidney stones, psychiatric disturbance), symptoms of
anemia (eg, pallor, lightheadedness, dyspnea on exertion), or with opportunistic infections secondary to immune dysfunction. Further workup is recommended with a complete blood count, complete
metabolic panel, urinalysis, urine and serum protein electrophoresis, and imaging to determine the extent of malignancy. Peripheral blood smear classically discloses red blood cell aggregation in a
rouleaux formation. The diagnosis is confirmed with bone marrow biopsy.

Incorrect Answers: A, B, C, and D.

Liver function tests (Choice A) frequently disclose hepatic dysfunction in patients with multiple myeloma, often caused by infiltration of the liver by plasma cells. However, abnormal liver function testing is
nonspecific and not helpful in confirming the diagnosis.

Measurement of serum vitamin B, (cobalamin) concentration (Choice 8) is not helpful for confirming the diagnosis of multiple myeloma. Some patients with multiple myeloma may display megaloblastic
anemia, which is typically caused by the underlying excessive utilization of folate during the neoplastic production of plasma cells.

Serum iron studies (Choice C) and colonoscopy (Choice D) are useful for the evaluation of iron deficiency anemia, which often occurs secondary to dietary iron deficiency, menstrual abnormalities, or
chronic gastrointestinal bleeding. However, iron deficiency anemia classically presents with microcytosis, rather than macrocytosis.

Educational Objective: Multiple myeloma is a malignant plasma cell neoplasm that typically presents with fatigue, weight loss, bone pain, hepatosplenomegaly, immune deficiency, anemia, hypercalcemia,
renal dysfunction, hypergammaglobulinemia, and lytic bone lesions. Peripheral blood smear discloses red blood cell aggregation in a rouleaux formation. The diagnosis is confirmed by bone marrow biopsy.

https://t.me/USMLENBME2CK ts e t
Previous Next Score Report Lab Values Calculator Help pause
Exam Section 1: Item 48 of 50 National Board of Medical Examiners
Comprehensive Clinical Science Self-Assessment

"I 48. A 37-year-old man comes to the physician for a routine health maintenance examination. He feels well. He has no history of serious illness and takes no medications. He received the hepatitis 8
vaccine at the age of 28 years. He does not smoke or drink alcohol. During the past 6 months, he has been sexually active with one male partner, and they do not use condoms. The patient works as
an accountant and has not traveled outside the USA. Examination shows no abnormalities. A blood sample is obtained for serum lipid studies and an HIV antibody test. In addition to counseling this
patient about risk factors for sexually transmitted diseases, which of the following is the most appropriate next step in management?

A) PPD skin test


8 ) Haemophilus influenzae type b vaccine
C) Hepatitis A vaccine
D) Meningococcal vaccine
E) Pneumococcal polysaccharide vaccine, 23-valent
Correct Answer: C.

Hepatitis A virus (HAV) is a nonenveloped, single-stranded RNA virus that is spread through fecal-oral transmission. II is a common, yet preventable cause of liver disease. It is commonly acquired through
the ingestion of poorly cooked, improperly handled, or raw foods. Presenting signs and symptoms include fever, nausea, vomiting, poor appetite, abdominal pain, Jaundice, and hepatomegaly, although many
patients are asymptomatic. Laboratory findings include transaminitis, increased alkaline phosphatase, hyperbilirubinemia, and bilirubinuria. Liver biopsy is not routinely required but may show ballooning
hepatocyte degeneration and Councilman bodies. Outbreaks of HAV are common among men who have sex with men (MSM). The Centers for Disease Control Advisory Committee on Immunization
Practices (CDC ACIP) recommends routine vaccination against HAV among MSM patients, as well as other high-risk populations, including international travelers to high-risk areas, HIV-positive patients,
patients experiencing homelessness, and users of injection drugs. The HAV vaccine is an inactivated vaccine and is given via intramuscular injection in a series of two injections.

Incorrect Answers: A, B, D, and E.

PPD skin test (Choice A) is useful for the identification of tuberculosis and is indicated in health care personnel and other high-risk populations, such as patients with HIV or patients who use injection drugs.
PPD testing is not routinely recommended for MSM patients.

Haemophi/us influenzae type b vaccine (Choice 8) is a conjugate vaccine for the prevention of infections caused by H. influenzae type b. H. influenzae is a gram-negative bacterium that can result in various
mucosal infections such as conjunctivitis, otitis media, bronchitis, and pneumonia. The vaccine is given in a four-dose series beginning at age 2 months.

Meningococcal vaccine (Choice D) is indicated for all children, with an additional serogroup B vaccine indicated for all children over age 10 years al increased risk for meningococcal disease (eg, dormitory,
military living).

Pneumococcal polysaccharide vaccine, 23-valent (Choice E) is indicated for all adults over age 65 years and for patients with high-risk conditions such as chronic heart, renal, and liver disease or chronic
smokers.

Educational Objective: Hepatitis A virus is a common but preventable cause of liver disease. Outbreaks are common among men who have sex with men. The Centers for Disease Control Advisory
Committee on Immunization Practices recommends routine vaccination among men having sex with men, as well as other high-risk populations.

https://t.me/USMLENBME2CK ts e t
Previous Next Score Report Lab Values Calculator Help pause
Exam Section 1: Item 49 of 50 National Board of Medical Examiners
Comprehensive Clinical Science Self-Assessment

"I 49. A27-year-old man comes to the physician because of a 1-month history of sudden episodes of a "scary choking feeling." He has had three episodes during this period; the episodes are accompanied
by nausea, sweating, and "heart pounding" sensations. The symptoms peak within 5 to 10 minutes and disappear about 15 minutes later. The first two episodes "came out of the blue" while he was at
work; the last episode occurred when he was stuck in traffic. He is fearful that he will have another episode and that "someone will notice and think I'm going crazy." He has no history of serious
medical illness and takes no medications. He is a veteran of the U.S. Marine Corps and experienced some intense fighting 5 years ago. He has occasional nightmares involving combat and
intermittent sadness when he thinks about military comrades who were killed. He has occasional difficulty falling asleep. He has had no changes in energy level or appetite. He is 175 cm (5 ft 9 in) tall
and weighs 82 kg (180 lb); 8MI is 27 kg/m2. Vital signs are within normal limits. Physical examination shows no abnormalities. On mental status examination, he is anxious but relates well with the
physician. His serum thyroid-stimulating hormone concentration is within the reference range. A 12-lead ECG shows a normal sinus rhythm. Which of the following is the most appropriate
pharmacotherapy?

A) 8upropion
B) 8uspirone
C) Haloperidol
0) Lithium carbonate
E) Paroxetine
F I Risperdal
Correct Answer: E.

Panic disorder features panic attacks that occur unexpectedly and are associated with changes in behavior to avoid panic attack triggers. Panic attacks feature acute fear or anxiety that peaks within minutes
and is associated with four additional physical symptoms or associated mental states. These additional symptoms may include heart palpitations or tachycardia, shortness of breath, chest pain, dizziness,
the sensation of choking, gastrointestinal distress, paresthesias, sweating, chills, trembling, derealization, fear of dying, and/or fear of losing control. In patients with an initial episode of panic symptoms, a
thorough history and physical examination along with a basic laboratory workup and a potential ECG should be performed. Panic disorder can be distinguished from post-traumatic stress disorder by the
absence of several characteristic features, such as re-experiencing the initial traumatic event, symptoms being triggered by reminders of the traumatic event, hyperarousal, such as increased vigilance or
exaggerated startle response, and negative symptoms, such as depressed mood, loss of interest, or difficulty concentrating. Treatment of panic disorder typically includes psychotherapy, selective serotonin
reuptake inhibitors, such as paroxetine, and short-term benzodiazepines. Other classes of medications that may be used for treating panic disorder include monoamine oxidase inhibitors and tricyclic
antidepressants.

Incorrect Answers: A, 8, C, D, and F.

8upropion (Choice A) is a dopamine/norepinephrine reuptake inhibitor, which is used as an antidepressant and smoking cessation aid. It is not known to be effective in the treatment of panic disorder.

8uspirone (Choice 8) is a serotonin modulator utilized to treat generalized anxiety disorder. Its use is not well-supported in the treatment of panic disorder.

Haloperidol (Choice C) is an antipsychotic medication that reversibly antagonizes D receptors, blocking dopamine's downstream effects nonspecifically across different brain regions. It has no role in the
treatment of panic disorder.

lithium oarhonate (feioe fie a meal ctahilior marlioatinn filiarl in hinalar lienrlor anl eohinhroni

https://t.me/USMLENBME2CK ts e t
Previous Next Score Report Lab Values Calculator Help pause
Exam Section 1: Item 49 of 50 National Board of Medical Examiners
Comprehensive Clinical Science Self-Assessment

A) Bupropion
B) Buspirone
C) Haloperidol
0) Lithium carbonate
E) Paroxetine
F I Risperdal
Correct Answer: E.

Panic disorder features panic attacks that occur unexpectedly and are associated with changes in behavior to avoid panic attack triggers. Panic attacks feature acute fear or anxiety that peaks within minutes
and is associated with four additional physical symptoms or associated mental states. These additional symptoms may include heart palpitations or tachycardia, shortness of breath, chest pain, dizziness,
the sensation of choking, gastrointestinal distress, paresthesias, sweating, chills, trembling, derealization, fear of dying, and/or fear of losing control. In patients with an initial episode of panic symptoms, a
thorough history and physical examination along with a basic laboratory workup and a potential ECG should be performed. Panic disorder can be distinguished from post-traumatic stress disorder by the
absence of several characteristic features, such as re-experiencing the initial traumatic event, symptoms being triggered by reminders of the traumatic event, hyperarousal, such as increased vigilance or
exaggerated startle response, and negative symptoms, such as depressed mood, loss of interest, or difficulty concentrating. Treatment of panic disorder typically includes psychotherapy, selective serotonin
reuptake inhibitors, such as paroxetine, and short-term benzodiazepines. Other classes of medications that may be used for treating panic disorder include monoamine oxidase inhibitors and tricyclic
antidepressants.

Incorrect Answers: A, B, C, D, and F.

Bu prop ion (Choice A) is a dopamine/norepinephrine reuptake inhibitor, which is used as an antidepressant and smoking cessation aid. It is not known to be effective in the treatment of panic disorder.

Buspirone (Choice B) is a serotonin modulator utilized to treat generalized anxiety disorder. Its use is not well-supported in the treatment of panic disorder.

Haloperidol (Choice C) is an antipsychotic medication that reversibly antagonizes D receptors, blocking dopamine's downstream effects nonspecifically across different brain regions. It has no role in the
treatment of panic disorder.

Lithium carbonate (Choice D) is a mood stabilizer medication utilized in bipolar disorder and schizophrenia.

Risperdal (Choice F) is an atypical antipsychotic that is useful for the treatment of bipolar disorder and schizophrenia.

Educational Objective: Panic attacks feature acute fear or anxiety that peaks within minutes and is associated with symptoms of sympathetic hyperactivity such as tachycardia, heart palpitations, sweating,
and trembling, along with shortness of breath, chest pain, dizziness, gastrointestinal distress, and paresthesias. Panic disorder is characterized by recurrent panic attacks that are unexpected and associated
with worry about future panic attacks or avoidance of panic attack triggers. The most appropriate pharmacotherapy is selective serotonin reuptake inhibitors, such as paroxetine.

https://t.me/USMLENBME2CK ti
Previous Next Score Report Lab Values Calculator Help pause
Exam Section 1: Item 50 of 50 National Board of Medical Examiners
Comprehensive Clinical Science Self-Assessment

"I 50. A previously healthy 25-year-old man is brought lo the emergency department 2 hours after being involved in a motor vehicle collision. He was the restrained driver. On arrival, he is alert and oriented
lo person, place, and time. He says he has severe lower abdominal pain. His temperature is 37.5°C (99.5°F), pulse is 105/min, respirations are 18/min, and blood pressure is 120/70 mm Hg.
Abdominal examination shows ecchymoses over the lower quadrants and an abrasion where the seat belt was fastened. There is suprapubic tenderness. Palpation of the iliac crests produces pain.
There is blood at the urethral meatus. His hematocrit is 30%. X-rays of the pelvis show a fracture of the superior and inferior right pubic rami and the right sacroiliac Joint. Which of the following is the
most appropriate next step in management?

A) Cystography
B) Retrograde urethrography
C) Placement of a supra pubic catheter
D) Intravenous pyelography
E ) Bladder irrigation
Correct Answer: B.

Blood at the urethral meatus following trauma raises concern for a urethral injury. Urethral injuries are characterized by difficulty voiding, blood at the urethral meatus, perinea! ecchymosis, and may be
associated with a pelvic fracture. Straddle injuries can cause trauma to the anterior urethra, while pelvic fractures may injure the posterior urethra. Urethral injuries are evaluated with retrograde
urethrography, which involves the injection of contrast into the urethra to evaluate for the extravasation of contrast, indicating urethral discontinuity. Depending on the location and severity of urethral injury,
patients may require surgical repair or the placement of a urethral catheter to help promote healing and prevent complications of urinary extravasation, infection, urethrocutaneous fistula, or urethral stricture.

Incorrect Answers: A, C, D, and E.

Cystography (Choice A) is indicated for evaluation of the urinary bladder when a bladder injury is suspected. This allows for characterization of an intraperitoneal versus extra peritoneal bladder rupture,
which dictates the next step in management.

Placement of a suprapubic catheter (Choice C) may be necessary, but first, delineation of injury location and pattern should occur. An indwelling urethral catheter is less invasive than a suprapubic catheter
and is preferred if insertion would not exacerbate trauma.

Intravenous pyelography (Choice 0) involves the intravenous injection of radiopaque contrast to subsequently visualize the kidney collecting system, ureters, and bladder under fluoroscopy. This allows for
the investigation of suspected urinary tract anatomic abnormalities, malignancies, and strictures.

Bladder irrigation (Choice E) would be helpful in the management of hematuria but would not be indicated in urethral injury. In addition, bladder irrigation is typically done with a urinary catheter, and insertion
of a urinary catheter is contraindicated in urethral injuries until the location and pattern of such injury is known.

Educational Objective: Urethral injuries can occur from straddle injuries (anterior urethra) or pelvic fractures (posterior urethra). Retrograde urethrography allows evaluation of the urethra as a source of
hematuria.

https://t.me/USMLENBME2CK ti
Previous Next Score Report Lab Values Calculator Help pause
Exam Section 2: ltem 1 of 50 National Board of Medical Examiners
Comprehensive Clinical Science Self-Assessment

"I 1. A 47-year-old woman comes to the physician because of a 3-week history of moderate pain in her arms and legs. She states that her "bones ache" in all Joints both al rest and with movement. She has
hypertension, hyperlipidemia, and type 2 diabetes mellitus well controlled with diet. Current medications include hydrochlorothiazide and simvastatin. Her sister has Sjogren syndrome, and her mother
has coronary artery disease. The patient does not smoke and drinks one glass of wine daily. Her blood pressure is 144/80 mm Hg. Cardiopulmonary examination shows no abnormalities. There is
tenderness to palpation over the wrist, metacarpophalangeal, knee, and ankle joints. Clubbing of the fingers and toes is noted. A chest x-ray shows a left upper lobe mass with perihilar
lymphadenopathy. X-rays of the tibia and fibula show periostosis. Which of the following is the most likely diagnosis?

A) Bone metastases
B) Hypertrophic pulmonary osteoarthropathy
C) Neuropathic arthropathy (Charcot joint)
D) Sarcoidosis
E) Systemic sclerosis (scleroderma)
Correct Answer: B.

Hypertrophic osteoarthropathy (HOA) is a syndrome caused by abnormal fibrovascular proliferation and is characterized by the triad of digital clubbing, periostosis (inflammation of the periosteum,
particularly of the long bones), and severe arthralgia. It may be caused by rare congenital conditions, but most cases are secondary to a noncongenital condition. Of these, pulmonary pathology is the most
common cause. The pathophysiology is associated with hypoxemia-related changes in systemic growth factor regulation. Other causes include primary cardiac, gastrointestinal, or hepatic disease, as well as
metastatic malignancy. Patients present with debilitating arthralgia which is often initially diagnosed as inflammatory arthritis. However, for this patient, the finding of a lung mass with regional
lymphadenopathy makes secondary HOA as a result of pulmonary malignancy the most likely diagnosis.

Incorrect Answers: A, C, D, and E.

Bone metastases (Choice A) typically appear as destructive lesions on plain x-rays, with patchy areas of decreased bone density, disruptions in the normal bone architecture, or lesions with a sclerotic lining.
A mix of lytic and sclerotic features may be present depending on the primary malignancy.

Neuropathic arthropathy (Charcot joint) (Choice C) occurs secondary to repetitive minor injuries that are not noticed by the patient as a result of underlying afferent sensory nerve dysfunction (eg, in diabetic
neuropathy). This results in progressive osteolysis with destruction, fragmentation, erosion, and misalignment of bony articulations.

Sarcoidosis (Choice 0) is a noncaseating granulomatous disease that often involves the lung, and bilateral perihilar lymphadenopathy is a characteristic imaging finding. Bone involvement usually manifests
with chronic arthritis and cysts on imaging caused by areas of periosteal resorption. The presence of a lung mass and periostosis on imaging make lung malignancy with HOA a more likely diagnosis.

Systemic sclerosis (scleroderma) (Choice E) is an autoimmune disorder characterized by collagen deposition and progressive fibrosis of the skin, soft tissues, and internal organs as well as noninflammatory
vasculopathy. The presence of a lung mass and periostosis on imaging make lung malignancy with HOA a more likely diagnosis.

Educational Objective: Hypertrophic osteoarthropathy is a syndrome caused by abnormal fibrovascular proliferation and is characterized by the triad of digital clubbing, periostosis, and severe arthralgia. It is
frequently associated with chronic pulmonary pathology and malignancy.

https://t.me/USMLENBME2CK ti e
Previous Next Score Report Lab Values Calculator Help pause
Exam Section 2: Item 2 of 50 National Board of Medical Examiners
Comprehensive Clinical Science Self-Assessment

"I 2. A healthy 16-year-old girl comes lo the physician for a school physical examination. She is sexually active with one partner and does not use contraception. She has smoked one pack of cigarettes daily
for 2 years. She is 163 cm (5 ft 4 in) tall and weighs 70 kg (156 lb); BMI is 27 kg/m2. Physical examination shows no abnormalities. Which of the following behavioral modifications is most likely to have
the greatest impact on mortality in this patient during the next 10 years?

A) Condom use
B) Monthly breast self-examination
C) Seat belt use
D) Smoking cessation
E) Weight loss
Correct Answer: C.

The most common cause of death in teens and young adults is accidental death, frequently in relation to fatal motor vehicle collisions. This is followed by homicide, suicide, malignancy, and heart disease.
This patient has several factors that are likely to lead to adverse health outcomes, such as being overweight, using tobacco products, and having unprotected intercourse. Being overweight increases the
likelihood of diabetes mellitus, hypertension, hyperlipidemia, and heart disease, while tobacco use can increase these things as well as the likelihood of cancer after years of use. Unprotected intercourse
can lead to sexually transmitted infections and unintended pregnancy. While complicated pregnancy can lead to death, it is much less common than accidental death in young adults. Thus, the behavioral
modification that is most likely to have the greatest impact on this patient's mortality during the next 10 years is increasing her seat belt use.

Incorrect Answers: A, B, D, and E.

Condom use (Choice A) would decrease the risk for sexually transmitted infections and unintended pregnancy. While sexually transmitted infections can be associated with long-term complications and
complicated pregnancy can lead to death, these outcomes are much less likely than accidental death caused by a motor vehicle collision.

Monthly breast self-examination (Choice B) is used to evaluate for the development of lumps that may signify breast cancer to promote early detection. Breast cancer is rare in young patients, unless the
patient carries a BRCA 1 or BRCA2 mutation, which may present with a positive family history. Therefore, this is unlikely to decrease the risk of mortality lo this patient during the next 10 years in comparison
to avoiding traumatic injury.

Smoking cessation (Choice 0) would be of benefit in this case, as long-term tobacco use can lead to chronic obstructive pulmonary disease, heart disease, and lung cancer, among other problems. However,
tobacco use is unlikely to lead lo death in the next 10 years.

Weight loss (Choice E) would not improve this patient's mortality during the next ten years. She is only mildly overweight with a BMI of 27 kg/m, and additional weight loss would be unlikely to lead to
improved mortality during the next 10 years. However, if she remained overweight long-term or her BM I increased into the obesity range, she would be at increased risk for the development of diabetes,
hypertension, and heart disease, which carry an increased risk of mortality.

Educational Objective: The most common cause of death in teens and young adults is accidental death, frequently caused by motor vehicle collisions. All teens and young adults should therefore be
encouraged to wear seat belts in vehicles to decrease their risk. Other health behaviors that should be encouraged in this patient population include exercise, maintaining a healthy weight, abstaining from
tobacco use, and avoiding excessive alcohol use. ....

https://t.me/USMLENBME2CK ts e t
Previous Next Score Report Lab Values Calculator Help pause
Exam Section 2: ltem 3 of 50 National Board of Medical Examiners
Comprehensive Clinical Science Self-Assessment

"I 3. An 82-year-old woman who resides in a skilled nursing care facility is transferred to the hospital because of a 2-day history of fever and cellulitis of the left side of her face. She is hemiparetic, aphasic,
and unable to swallow. She is fed through a gastrostomy tube. She has congestive heart failure, type 2 diabetes mellitus, hypertension, hyperlipidemia, major depressive disorder, and osteoporosis. Her
medications are furosemide, metoprolol, digoxin, glyburide, lisinopril, carvedilol, atorvastatin, amitriptyline, alendronate, and 81-mg aspirin. She appears ill and moans frequently. Her temperature is
39.1C (102.4F), pulse is 84/min, respirations are 24/min, and blood pressure is 112/72 mm Hg. A firm, exquisitely tender mass is palpated from the left cheek to the angle of the mandible; this area is
also erythematous and edematous. The left pinna appears normal, and the otic canal is not inflamed. The mouth is dry. Bilateral basilar crackles are heard. Cardiac examination shows no abnormalities.
The abdomen is soft and nontender. The gastrostomy tube is in place with no inflammation. Her leukocyte count is 28,000/mm3 (78% segmented neutrophils, 18% bands, and 4% lymphocytes), and
serum glucose concentration is 168 mg/dL. A chest x-ray shows cardiomegaly; there are bilateral basilar increased markings and atelectasis in the left lung base but no focal infiltrates. Which of the
following is the most likely cause of this patient's fever?

A) Acute suppurative parotitis


B) Aspiration pneumonia
C) Mandibular osteomyelitis
D) Osteonecrosis of the jaw
E) Urinary tract infection
Correct Answer: A.

Facial swelling, erythema, and edema at the angle of the mandible accompanied by fever raises suspicion for acute suppurative parotitis. Acute parotitis is most often caused by mixed oral flora and
Staphylococcus aureus. Risk factors for the development of parotitis include being elderly, dehydration, poor dentition, lack of salivary flow (in setting of poor or absent chewing or oral intake), or debilitation.
In addition, the parotid duct can be obstructed by calculi or tumor, which can predispose patients to infection. The diagnosis is clinical, and imaging with an ultrasound or CT scan can further evaluate for the
presence of complications, such as abscess formations or a tumor. Imaging can also delineate involved structures when the diagnosis is unclear, or differential remains mixed. Management of acute
suppurative parotitis involves administration of intravenous antibiotics and monitoring to evaluate for clinical improvement.

Incorrect Answers: B, C, D, and E.

Aspiration pneumonia (Choice B) presents with cough, shortness of breath, pleuritic chest pain, fever, and abnormal breath sounds. Chest x-ray will show patchy infiltrates, most commonly in the right lower
lobe if the patient was upright while the aspiration occurred, or the posterior, gravity-dependent portions of the lungs if the patient was supine.

Mandibular osteomyelitis (Choice C) can cause Jaw pain but is typically nonsuppurative and does not usually cause erythema and swelling unless severe or rapidly progressing. The diagnosis cannot be
excluded on the clinical picture alone in this case, but is less likely.

Osteonecrosis of the jaw (Choice 0) can occur in the setting of radiation cancer treatments, steroid use, or use of bisphosphonates. It does not typically cause fever, erythema, or swelling as seen in
parotitis.

Urinary tract infection (Choice E) is a common cause of fever and sepsis in skilled nursing facilities. However, in the setting of facial cellulitis and an edematous tender mass on the left cheek, acute parotitis
is more likely.
44 out 4o.lieu4 4.

https://t.me/USMLENBME2CK ts e t
Previous Next Score Report Lab Values Calculator Help pause
Exam Section 2: ltem 3 of 50 National Board of Medical Examiners
Comprehensive Clinical Science Self-Assessment

C (102.4 F), pulse is 84/min, respirations are 24/min, and blood pressure is 112/72 mm Hg. A firm, exquisitely tender mass is palpated from the left cheek to the angle of the mandible; this area is
also erythematous and edematous. The left pinna appears normal, and the otic canal is not inflamed. The mouth is dry. Bilateral basilar crackles are heard. Cardiac examination shows no abnormalities.
The abdomen is soft and nontender. The gastrostomy tube is in place with no inflammation. Her leukocyte count is 28,000/mm3 (78% segmented neutrophils, 18% bands, and 4% lymphocytes), and
serum glucose concentration is 168 mg/dL. A chest x-ray shows cardiomegaly; there are bilateral basilar increased markings and atelectasis in the left lung base but no focal infiltrates. Which of the
following is the most likely cause of this patient's fever?

A) Acute suppurative parotitis


B) Aspiration pneumonia
C) Mandibular osteomyelitis
D) Osteonecrosis of the jaw
E) Urinary tract infection
Correct Answer: A.

Facial swelling, erythema, and edema at the angle of the mandible accompanied by fever raises suspicion for acute suppurative parotitis. Acute parotitis is most often caused by mixed oral flora and
Staphylococcus aureus. Risk factors for the development of parotitis include being elderly, dehydration, poor dentition, lack of salivary flow (in setting of poor or absent chewing or oral intake), or debilitation.
In addition, the parotid duct can be obstructed by calculi or tumor, which can predispose patients to infection. The diagnosis is clinical, and imaging with an ultrasound or CT scan can further evaluate for the
presence of complications, such as abscess formations or a tumor. Imaging can also delineate involved structures when the diagnosis is unclear, or differential remains mixed. Management of acute
suppurative parotitis involves administration of intravenous antibiotics and monitoring to evaluate for clinical improvement.

Incorrect Answers: B, C, D, and E.

Aspiration pneumonia (Choice B) presents with cough, shortness of breath, pleuritic chest pain, fever, and abnormal breath sounds. Chest x-ray will show patchy infiltrates, most commonly in the right lower
lobe if the patient was upright while the aspiration occurred, or the posterior, gravity-dependent portions of the lungs if the patient was supine.

Mandibular osteomyelitis (Choice C) can cause Jaw pain but is typically nonsuppurative and does not usually cause erythema and swelling unless severe or rapidly progressing. The diagnosis cannot be
excluded on the clinical picture alone in this case, but is less likely.

Osteonecrosis of the jaw (Choice 0) can occur in the setting of radiation cancer treatments, steroid use, or use of bisphosphonates. It does not typically cause fever, erythema, or swelling as seen in
parotitis.

Urinary tract infection (Choice E) is a common cause of fever and sepsis in skilled nursing facilities. However, in the setting of facial cellulitis and an edematous tender mass on the left cheek, acute parotitis
is more likely.

Educational Objective: Acute suppurative parotitis clinically presents with fever, unilateral erythema, tenderness, and swelling over the parotid gland. Management involves administration of intravenous
antibiotics and monitoring to evaluate for clinical improvement.

https://t.me/USMLENBME2CK ts e t
Previous Next Score Report Lab Values Calculator Help pause
Exam Section 2: ltem 4 of 50 National Board of Medical Examiners
Comprehensive Clinical Science Self-Assessment

"I 4. A hospitalized 42-year-old man has chills and a temperature of 38.3°C (101F) 1 hour after transfusion of packed red blood cells for treatment of anemia secondary to a bleeding duodenal ulcer. He has
never received a blood transfusion in the past. He has no known allergies. He is pale and flushed. He is 178 cm (5 ft 10 in) tall and weighs 79 kg (175 lb); BMI is 25 kg/m2. His pulse is 98/min and regular,
and blood pressure is 128/82 mm Hg. Examination shows no abnormalities. The most appropriate next step in management is administration of which of the following?

A) Oral acetaminophen
B) Oral ibuprofen
C) Subcutaneous epinephrine
D) Intravenous diphenhydramine
E) Intravenous methylprednisolone
Correct Answer: A.

Febrile nonhemolytic transfusion reactions are caused by the release of cytokines from leukocytes during blood product storage and are a common transfusion reaction. It presents during or shortly after
transfusion with isolated fever and possible rigors without other systemic symptoms. Laboratory and radiographic evaluations are unremarkable. Risk factors include the administration of platelets or red blood
cells, especially in children, and long storage time of the blood products. Prestorage leukoreduction, or the removal of leukocytes from the blood products, decreases the risk for development by decreasing the
extent of cytokine release. Treatment includes stopping the transfusion, treating the fever, and evaluating for other causes of fever. Therefore, in this patient with isolated fever following a blood transfusion,
antipyretic treatment with oral acetaminophen is the most appropriate next step in management.

Incorrect Answers: B, C, D, and E.

Oral ibuprofen (Choice B) is a nonsteroidal anti-inflammatory drug used to treat inflammatory conditions, pain, and fever. While ii may treat the fever associated with this patient's febrile nonhemolytic
transfusion reaction, nonsteroidal anti-inflammatory drugs can cause gastritis, as well as gastric and duodenal ulcers. As this patient is already anemic from a bleeding duodenal ulcer, oral acetaminophen is a
better choice than ibuprofen for treating fever.

Subcutaneous epinephrine (Choice C), intravenous diphenhydramine (Choice 0), and intravenous methylprednisolone (Choice E) are used in the treatment of anaphylaxis, which can occur in response to
blood product transfusion, most commonly in those with lgA deficiency. Anaphylaxis presents with bronchospasm with associated dyspnea or wheezing, urticaria, flushing, nausea and emesis, edematous lips
and tongue, and hypotension with tachycardia. Intravenous diphenhydramine can also be used to treat isolated hives or urticaria associated with blood product transfusion. This patient is presenting with
isolated fever, indicative of a febrile nonhemolytic reaction, making oral acetaminophen the most appropriate next step in management.

Educational Objective: Febrile nonhemolytic transfusion reactions are a common type of reaction caused by the release of cytokines from leukocytes in stored blood products, especially platelets and red blood
cells. Symptoms include isolated fever with possible rigors and without other systemic symptoms. Prestorage leukoreduction decreases the risk for development. Treatment includes stopping the transfusion
and antipyretic administration, as well as evaluating for other causes of fever.

https://t.me/USMLENBME2CK ts e t
Previous Next Score Report Lab Values Calculator Help pause
Exam Section 2: Item 5 of 50 National Board of Medical Examiners
Comprehensive Clinical Science Self-Assessment

"I 5. A 77-year-old woman comes to the office because of a 1-year history of frequent urinary incontinence during the day and at night. The incontinence is not associated with coughing or sneezing, and she
does not have urinary urgency before the episodes. She has not had pain with urination or vaginal discharge. She has a 10-year history of type 2 diabetes mellitus well controlled with metformin. Her
other medications are atorvastalin and lisinopri1. Examination shows some wetting of an absorbent pad that she is wearing. Results of laboratory studies are shown:
Hemoglobin A,, 6.2%
Serum
Na 138 mEq/L
K+ 3.7 mEq/L
CI- 98 mEq/L
HCO- 3 26 mEq/L
Urea nitrogen 12 mg/dL
Glucose 180 mg/dL
Creatinine 1.1 mg/dL
Urine
pH normal
Glucose 1+
Protein 2+
RBC none
WBC 3/hpf
Casts none

Which of the following is the most likely cause of this patient's urinary incontinence?

A) Hyperglycemia
B) Interstitial cystitis
C) Medication adverse effect
D) Neurogenic bladder
E) Stress incontinence
Correct Answer: D.

Autonomic neuropathy is common in patients with both type 1 and type 2 diabetes mellitus. Autonomic neuropathy may occur independently of or accompanying peripheral sensory neuropathy. Autonomic
neuropathy may present with cardiovascular manifestations of exercise intolerance or orthostasis; gastrointestinal manifestations such as gastroesophageal reflux, gastroparesis, or diarrhea; or genitourinary
manifestations such as neurogenic bladder or erectile dysfunction. Neurogenic bladder results in impaired autonomic sensation of bladder filling with subsequent overflow or other neurologic patterns of
incontinence. Overflow incontinence results in urinary retention, which is evidenced by an increased postvoid residual bladder volume. When the intravesical hydrostatic pressure eventually overwhelms the
internal and external urinary sphincter closing pressures, the patient will demonstrate passive leakage of urine. Patients present with incomplete bladder emptying and a weak urinary stream.

https://t.me/USMLENBME2CK ts e t
Previous Next Score Report Lab Values Calculator Help pause
Exam Section 2: Item 5 of 50 National Board of Medical Examiners
Comprehensive Clinical Science Self-Assessment

Protein 2+
RBC none
WBC 3/hpf
Casts none

Which of the following is the most likely cause of this patient's urinary incontinence?

A) Hyperglycemia
B) Interstitial cystitis
C) Medication adverse effect
D) Neurogenic bladder
E) Stress incontinence
Correct Answer: D.

Autonomic neuropathy is common in patients with both type 1 and type 2 diabetes mellitus. Autonomic neuropathy may occur independently of or accompanying peripheral sensory neuropathy. Autonomic
neuropathy may present with cardiovascular manifestations of exercise intolerance or orthostasis; gastrointestinal manifestations such as gastroesophageal reflux, gastroparesis, or diarrhea; or genitourinary
manifestations such as neurogenic bladder or erectile dysfunction. Neurogenic bladder results in impaired autonomic sensation of bladder filling with subsequent overflow or other neurologic patterns of
incontinence. Overflow incontinence results in urinary retention, which is evidenced by an increased postvoid residual bladder volume. When the intravesical hydrostatic pressure eventually overwhelms the
internal and external urinary sphincter closing pressures, the patient will demonstrate passive leakage of urine. Patients present with incomplete bladder emptying and a weak urinary stream.

Incorrect Answers: A, B, C, and E.

Hyperglycemia (Choice A) may lead to polyuria in the setting of diabetes mellitus. While polyuria commonly causes urinary frequency and urgency, it does not usually cause incontinence.

Interstitial cystitis (Choice 8) is an idiopathic, inflammatory disorder of the bladder that presents with urinary frequency, urgency, chronic suprapubic pain, and dyspareunia. It does not present with overflow
incontinence.

Medication adverse effects (Choice C) do not usually cause incontinence. Use of diuretics may lead to polyuria with urinary frequency and urgency.

Stress incontinence (Choice E) refers to incontinence that is triggered by an increase in intra-abdominal pressure (eg, lifting, sneezing, Valsalva maneuver). Stress incontinence results from urethral
hypermobility caused by pelvic floor weakness or prolapse, or intrinsic sphincter deficiency. Risk factors include multiple vaginal deliveries, obesity, and prostate surgery.

Educational Objective: Patients with diabetes mellitus frequently present with signs and symptoms of autonomic neuropathy, including overflow incontinence secondary to neurogenic bladder. Neurogenic
bladder results in impaired autonomic sensation of bladder filling, with subsequent overflow incontinence, infrequent urination with incomplete bladder emptying, and a weakened urinary stream.

https://t.me/USMLENBME2CK ts e t
Previous Next Score Report Lab Values Calculator Help pause
Exam Section 2: Item 6 of 50 National Board of Medical Examiners°
Comprehensive Clinical Science Self-Assessment

6. A71-year-old man is brought to the emergency department because of a 1-week history of increasingly severe pain in his left third toe. During the past 36 hours, he has not slept because of the pain.
He has coronary artery disease, hypertension, and hyperlipidemia. Six years ago, he had a myocardial infarction. He has never undergone an operation. His medications are ramipril, labetalol, and
aspirin. Acetaminophen and ibuprofen have provided no pain relief. He smoked one pack of cigarettes daily for 40 years but quit 6 years ago. He appears uncomfortable. His pulse is 84/min, and blood
pressure is 170/90 mm Hg. A 4-cm, pulsalile, nontender mass is palpated in the left inguinal region. The left third toe is necrotic and exquisitely tender to palpation; there is no erythema or drainage.
Pedal pulses are 2+ on the left. The remainder of the examination shows no abnormalities. A CT scan of the abdomen and pelvis is shown. In addition to intravenous analgesic therapy, which of the
following is the most appropriate next step in management?

A) Piperacillin-tazobactam prescription
B) Placement of an Unna boot
C) Placement of a vena cava filter
D) Rivaroxaban therapy
E) Surgical arterial repair
Correct Answer: E.

Femoral artery aneurysms often involve the common femoral artery and are frequently associated with peripheral arterial disease. Risk factors include male sex, older age, tobacco use, hypertension,
hyperlipidemia, and coronary artery disease, all of which are seen in this patient. Symptoms can vary, with some patients being asymptomatic. When present, however, symptoms include a pulsatile groin
mass and vascular claudication. Femoral artery aneurysms can be complicated by acute limb ischemia, rupture, or distal embolism. Diagnosis often includes duplex ultrasonography of the groin or CT
angiography. Symptomatic patients or those with large (2 3 cm) aneurysms should undergo surgical arterial repair. This patient with numerous cardiovascular risk factors presents with third toe ischemia
most likely caused by lhromboembolism in the setting of a pulsatile groin mass felt on physical examination. This pulsatile mass can be more clearly seen as an enlarged femoral artery with intraluminal

« https://t.me/USMLENBME2CK ts e t
Previous Next Score Report Lab Values Calculator Help pause
Exam Section 2: Item 6 of 50 National Board of Medical Examiners
Comprehensive Clinical Science Self-Assessment

pressure is 170/90 mm Hg. A 4-cm, pulsatile, nontender mass is palpated in the left inguinal region. The left third toe is necrotic and exquisitely tender to palpation; there is no erythema or drainage.
Pedal pulses are 2+ on the left. The remainder of the examination shows no abnormalities. A CT scan of the abdomen and pelvis is shown. In addition to intravenous analgesic therapy, which of the
following is the most appropriate next step in management?

A) Piperacillin-tazobactam prescription
B) Placement of an Unna boot
C) Placement of a vena cava filter
D) Rivaroxaban therapy
E) Surgical arterial repair
Correct Answer: E.

Femoral artery aneurysms often involve the common femoral artery and are frequently associated with peripheral arterial disease. Risk factors include male sex, older age, tobacco use, hypertension,
hyperlipidemia, and coronary artery disease, all of which are seen in this patient. Symptoms can vary, with some patients being asymptomatic. When present, however, symptoms include a pulsatile groin
mass and vascular claudication. Femoral artery aneurysms can be complicated by acute limb ischemia, rupture, or distal embolism. Diagnosis often includes duplex ultrasonography of the groin or CT
angiography. Symptomatic patients or those with large (2 3 cm) aneurysms should undergo surgical arterial repair. This patient with numerous cardiovascular risk factors presents with third toe ischemia
most likely caused by thromboembolism in the setting of a pulsatile groin mass felt on physical examination. This pulsatile mass can be more clearly seen as an enlarged femoral artery with intraluminal
thrombus on CT scan. Thus, the most appropriate next step in management of this patient is surgical arterial repair.

Incorrect Answers: A, B, C, and D.

Piperacillin-tazobactam prescription (Choice A) is used in the treatment of a variety of infections, such as pneumonia or intra-abdominal infections. While bacteremia can cause aneurysms, this is
uncommon and more likely to lead to an infected pseudoaneurysm. This patient has no other signs of infection, making antibiotic therapy unnecessary.

Placement of an Unna boot (Choice B) is often used in the treatment of venous stasis ulcers. It is a compressive dressing that contains zinc oxide, which expedites healing. This would be inappropriate for
this patient, as it would not treat the underlying cause of his necrotic toe.

Placement of a vena cava filter (Choice C) is used to prevent the embolization of deep venous thrombosis to the pulmonary vasculature in patients who have a contraindication to anticoagulation. This
patient has no evidence of a deep venous thrombosis on physical examination. Rather, he has toe ischemia caused by a thromboembolism.

Rivaroxaban therapy (Choice D) is an anticoagulant used in the treatment of deep venous thrombosis, pulmonary embolism, and atrial fibrillation. While anticoagulation may be warranted in this patient,
initiation should be delayed until after surgical arterial repair as it can increase surgical bleeding.

Educational Objective: Femoral artery aneurysms often present with vascular claudication and a pulsatile groin mass. Complications include acute limb ischemia, rupture, or distal embolism. Risk factors
include male sex, older age, tobacco use, hypertension, hyperlipidemia, and coronary artery disease. Diagnosis is most frequently made with duplex ultrasonography or CT angiography. Treatment of
symptomatic or large (2 3 cm) aneurysms consists of surgical arterial repair.

https://t.me/USMLENBME2CK ti
Previous Next Score Report Lab Values Calculator Help pause
Exam Section 2: Item 7 of 50 National Board of Medical Examiners
Comprehensive Clinical Science Self-Assessment

"I 7. A 37-year-old woman comes to the physician requesting assistance in applying for disability income. She fell at home 12 months ago and left her job at a day-care center because of subsequent low
back pain. She received the diagnosis of major depressive disorder 8 months ago; her sleep, appetite, and mood have improved with citalopram therapy. Her only other medication is ibuprofen. She
asks the physician to document a permanent and total disability on an application form for disability income so that she will have the financial resources to start college education on a full-time basis.
She explains that she will become depressed again if she resumes her stressful Job as a child-care provider. She does not appear to be in distress. She is 157 cm (5 ft 2 in) tall and weighs 68 kg
(150 lb); BMI is 27 kg/m?. Her temperature is 37°C (98.6°F), pulse is 64/min, and blood pressure is 108/60 mm Hg. Examination of the back shows no tenderness; range of motion of the lumbar spine is
full. Neurologic examination shows no abnormalities. The physician will not fill out the form. In addition to informing the patient, which of the following is the most appropriate next step in management?

A) Reassure the patient that she will be all right


B) Confront the patient about requesting participation in fraud
C) Discuss options for the patient to work while pursuing an education
D) Explain that the patient is unlikely to have another depressive episode if she follows her medication regimen
E) Obtain an x-ray of the lumbosacral spine
Correct Answer: C.

Lying is viewed negatively in most philosophical systems because it exerts a morally negative effect on the person telling the lie, violates the autonomy of the person who is lied to, and undermines trust in
society. In addition to its basic ethical value, truth-telling is a core value that underlies the physician's role as a healer and patient advocate. This patient is asking the physician to falsely certify a disability
form. Falsifying a disability form is both unethical and is viewed as fraud by medical licensing authorities. This physician should decline to complete the form. However, the physician remains obligated to
assist the patient to the greatest of their ability within the scope of ethical conduct. This physician should assist the patient by discussing options for her to work while pursuing an education.

Incorrect Answers: A, B, D, and E.

Reassuring the patient that she will be all right (Choice A) does not directly address the patient's desire to pursue an education.

Confronting the patient about requesting participation in fraud (Choice B) is likely lo harm the patient-physician relationship and is not in the patient's best interest.

Explaining that the patient is unlikely to have another depressive episode if she follows her medication regimen (Choice D), while helpful for encouraging compliance with her medication, may not be
accurate and may set a false expectation for the patient's course of treatment.

Obtaining an x-ray of the lumbosacral spine (Choice E) is unnecessary given her normal physical exam.

Educational Objective: Truth-telling is a core value that underlies the physician's role as a healer and patient advocate. Falsifying a disability form is both unethical and is viewed as fraud by medical licensing
authorities. However, the physician remains obligated to assist the patient to the greatest of their ability within the scope of ethical conduct. This physician should assist the patient by discussing options for
her to work while pursuing an education.

https://t.me/USMLENBME2CK ti e
Previous Next Score Report Lab Values Calculator Help pause
Exam Section 2: Item 8 of 50 National Board of Medical Examiners
Comprehensive Clinical Science Self-Assessment

✓ 8. A67-year-old woman comes to the physician because of a 1-year history of a nonproductive cough and progressive shortness of breath with exertion. Her symptoms were not a problem at first, but
she now has trouble walking her dog because of shortness of breath after walking one-half block. She has not had fever or any other symptoms. She has no history of serious illness and takes no
medications. She does not smoke. She retired from her position as an executive secretary 5 years ago. Vital signs are within normal limits. Pulse oximetry on room air shows an oxygen saturation of
92%. Examination shows no cyanosis. Fine, dry crackles are heard at both lung bases. Examination of the hands shows clubbing. There is no peripheral edema. A chest x-ray shows fine linear
opacities in the lower third of the lungs bilaterally. A CT scan of the chest confirms an interstitial pattern. Pulmonary function tests show an FVC of 48% of predicted and a diffusion capacity of the lung
for carbon monoxide of 55% of predicted. Which of the following is the most appropriate next step in diagnosis?

A) Measurement of serum ACE activity


B) Measurement of serum a +-antitrypsin concentration
C) Serum antinuclear antibody assay
D) Bronchoalveolar lavage
E) Lung biopsy
Correct Answer: E.

This patient's presentation and diagnostic studies are consistent with a diagnosis of interstitial lung disease (ILD). A lung biopsy is the most appropriate next step given the severity of her symptoms and her
significantly impaired pulmonary function, as well as a lack of apparent risk factors to direct the clinician toward an underlying cause. ILD encompasses a heterogeneous group of disorders that result in
pulmonary fibrosis and progressively impaired lung function. Causes include various infections, occupational and environmental exposures, drug-induced pulmonary toxicity, radiation-induced lung injury,
connective tissue disorders, hypersensitivity pneumonitis, and idiopathic disease. Patients classically present with progressive dyspnea and chronic nonproductive cough. lnspiratory crackles and
hypertrophic osteoarthropathy (clubbing) may be present on physical examination. Chest imaging tends to show interstitial opacification in the periphery of the lungs, though radiologic patterns are
dependent on the specific underlying cause.

Incorrect Answers: A, B, C, and D.

Measurement of serum ACE activity (Choice A) may support a diagnosis of sarcoidosis, which can cause ILD with progressive fibrosis. The test is not specific nor sensitive enough for diagnosis, and a lung
biopsy is a more appropriate diagnostic test for this patient.

Measurement of serum a,-antitrypsin concentration (Choice B) is not likely to aid in diagnosis in this case. Alpha-1 antitrypsin deficiency in the lungs causes chronic obstructive pulmonary disease with
emphysema and bronchiectasis apparent on imaging.

Serum antinuclear antibody assay (Choice C) may be included as part of the diagnostic workup to evaluate for connective tissue disease as an underlying cause but is not specific nor sensitive enough to
forego performing a lung biopsy.

Bronchoalveolar lavage (Choice D) is commonly performed in the workup of ILD, especially in patients presenting with rapid decompensation, hemoptysis, or a clinical picture suggestive of certain causes
(particularly infections, sarcoidosis, hypersensitivity pneumonitis, or pulmonary Langerhans cell histiocytosis) as these conditions have specific findings on fluid analysis. In this case, the patient should
undergo lung biopsy for more definitive diagnosis.

https://t.me/USMLENBME2CK ts e t
Previous Next Score Report Lab Values Calculator Help pause
Exam Section 2: Item 8 of 50 National Board of Medical Examiners

. . .
• • '
Comprehensive Clinical Science Self-Assessment
Li[-I1[=1$e)1llf±-10l(-I61ff1st
for carbon monoxide of 55% of predicted. Which of the following is the most appropriate next step in diagnosis?

A) Measurement of serum ACE activity


B) Measurement of serum a +-antitrypsin concentration
C) Serum antinuclear antibody assay
D) Bronchoalveolar lavage
E) Lung biopsy
Correct Answer: E.

This patient's presentation and diagnostic studies are consistent with a diagnosis of interstitial lung disease (ILD). A lung biopsy is the most appropriate next step given the severity of her symptoms and her
significantly impaired pulmonary function, as well as a lack of apparent risk factors to direct the clinician toward an underlying cause. ILD encompasses a heterogeneous group of disorders that result in
pulmonary fibrosis and progressively impaired lung function. Causes include various infections, occupational and environmental exposures, drug-induced pulmonary toxicity, radiation-induced lung injury,
connective tissue disorders, hypersensitivity pneumonitis, and idiopathic disease. Patients classically present with progressive dyspnea and chronic nonproductive cough. lnspiratory crackles and
hypertrophic osteoarthropathy (clubbing) may be present on physical examination. Chest imaging tends to show interstitial opacification in the periphery of the lungs, though radiologic patterns are
dependent on the specific underlying cause.

Incorrect Answers: A, B, C, and D.

Measurement of serum ACE activity (Choice A) may support a diagnosis of sarcoidosis, which can cause ILD with progressive fibrosis. The test is not specific nor sensitive enough for diagnosis, and a lung
biopsy is a more appropriate diagnostic test for this patient.

Measurement of serum a,-antitrypsin concentration (Choice B) is not likely to aid in diagnosis in this case. Alpha-1 antitrypsin deficiency in the lungs causes chronic obstructive pulmonary disease with
emphysema and bronchiectasis apparent on imaging.

Serum antinuclear antibody assay (Choice C) may be included as part of the diagnostic workup to evaluate for connective tissue disease as an underlying cause but is not specific nor sensitive enough to
forego performing a lung biopsy.

Bronchoalveolar lavage (Choice D) is commonly performed in the workup of ILD, especially in patients presenting with rapid decompensation, hemoptysis, or a clinical picture suggestive of certain causes
(particularly infections, sarcoidosis, hypersensitivity pneumonitis, or pulmonary Langerhans cell histiocytosis) as these conditions have specific findings on fluid analysis. In this case, the patient should
undergo lung biopsy for more definitive diagnosis.

Educational Objective: Interstitial lung disease (ILD) is an insidious disorder with numerous potential causes that typically presents with progressive dyspnea and nonproductive cough. The presence of ILD
is best confirmed with CT imaging showing evidence of interstitial fibrosis. Definitive diagnosis of the underlying cause requires lung biopsy. For patients unable to tolerate biopsy or with milder symptoms, a
treatment plan is often determined using clinical risk factors, history of exposures, radiographic patterns, and adjunct testing.

https://t.me/USMLENBME2CK ts e t
Previous Next Score Report Lab Values Calculator Help pause
Exam Section 2: Item 9 of 50 National Board of Medical Examiners
Comprehensive Clinical Science Self-Assessment

✓ 9. A 2-year-old boy is brought lo the physician for a follow-up examination 2 weeks after being treated for an ear infection. He has been treated for six episodes of acute otitis media and one episode of
pneumonia since birth. Three months ago, he underwent placement of tympanostomy tubes. He currently takes no medications. He appears well. He is at the 75th percentile for length and 25th
percentile for weight. Examination shows several 0.5- to 1-cm, firm, nontender, mobile lymph nodes in the anterior cervical chain. Tympanostomy tubes are in place bilaterally without discharge.
Laboratory studies show:
Hemoglobin 12.5 g/dL
Hematocrit 37%
Leukocyte count 8100/mm3
Platelet count 257,000/mm3
Serum
lgA <7 mg/dL
lgG 720 mg/dL
lgM 75 mg/dL

Which of the following is the most appropriate next step in management?

A) Oral antibiotic therapy


B) Oral prednisone therapy
C) Intramuscular injection of immune globulin
D) Intravenous antithymocyte globulin
E) Intravenous immune globulin infusion
F) Bone marrow transplant
G) No intervention is indicated at this time
Correct Answer: G.

lgA deficiency is the most likely cause of this patient's recurrent infections. Selective lgA deficiency describes a decreased or absent concentration of circulating lgA with normal concentrations of lgG, and
normal or increased concentrations of lgM. lgA is critical for immune health and functions by binding pathogen surface antigens, targeting them for opsonization, complement activation and phagocytosis,
and coating mucosa I surfaces to prevent colonization, adhesion, and invasion by pathogenic microorganisms. There is clinical heterogeneity in the severity of lgA deficiency. Some patients remain
asymptomatic while others suffer from recurrent sinopulmonary infections or otitis media. In younger patients with severe deficiency, failure to thrive is common. Anaphylactic reactions to blood products
such as plasma, which contain donor immunoglobulin, are classic but uncommon. Treatment of lgA deficiency is directed toward the treatment of symptomatic infections. Asymptomatic patients do not
require specific monitoring.

Incorrect Answers: A, B, C, D, E, and F.

Oral antibiotic therapy (Choice A) may be considered as prophylaxis to reduce the frequency of infections in patients with selective lgA deficiency with more severe, recurrent infections. Despite widespread

https://t.me/USMLENBME2CK ts e t
Previous Next Score Report Lab Values Calculator Help pause
Exam Section 2: Item 9 of 50 National Board of Medical Examiners
Comprehensive Clinical Science Self-Assessment

Intramuscular injection of immune globulin


D) Intravenous antithymocyte globulin
E) Intravenous immune globulin infusion
F) Bone marrow transplant
G) No intervention is indicated at this time
Correct Answer: G.

lgA deficiency is the most likely cause of this patient's recurrent infections. Selective lgA deficiency describes a decreased or absent concentration of circulating lgA with normal concentrations of lgG, and
normal or increased concentrations of lgM. lgA is critical for immune health and functions by binding pathogen surface antigens, targeting them for opsonization, complement activation and phagocytosis,
and coating mucosa I surfaces to prevent colonization, adhesion, and invasion by pathogenic microorganisms. There is clinical heterogeneity in the severity of lgA deficiency. Some patients remain
asymptomatic while others suffer from recurrent sinopulmonary infections or otitis media. In younger patients with severe deficiency, failure to thrive is common. Anaphylactic reactions to blood products
such as plasma, which contain donor immunoglobulin, are classic but uncommon. Treatment of lgA deficiency is directed toward the treatment of symptomatic infections. Asymptomatic patients do not
require specific monitoring.

Incorrect Answers: A, B, C, D, E, and F.

Oral antibiotic therapy (Choice A) may be considered as prophylaxis to reduce the frequency of infections in patients with selective lgA deficiency with more severe, recurrent infections. Despite widespread
use, routine antibiotic prophylaxis in this population is not supported by high-quality evidence.

Oral prednisone therapy (Choice B) is not useful for the management of selective lgA deficiency, which results from a failure of B lymphocyte maturation rather than an autoimmune process. Prednisone
therapy would exacerbate immunodeficiency and render this patient more susceptible to infection.

Intramuscular injection of immune globulin (Choice C) is incorrect. Immune globulin replacement is not routinely recommended for the treatment of selective lgA deficiency. Immune globulin replacement
may be considered in patients with lgA deficiency who also demonstrate an associated lgG deficiency or other qualitative defects in antibody production. Immune globulin is administered subcutaneously or
via intravenous infusion (Choice E).

Intravenous antithymocyte globulin (Choice D) results in depletion of circulating T lymphocytes and is useful for the prevention of acute rejection following solid organ transplantation. It plays no role in the
management of selective lgA deficiency, which represents a failure of B lymphocyte maturation and would worsen immunodeficiency in this patient.

Bone marrow transplant (Choice F) is not necessary for the treatment of lgA deficiency. In general, the prognosis of selective lgA deficiency is excellent, and the risks of bone marrow transplantation
generally exceed the potential benefits.

Educational Objective: Selective lgA deficiency describes a decreased or absent concentration of circulating lgA and presents with variable findings including asymptomatic patients, recurrent
sinopulmonary infections or otitis media, failure to thrive, or anaphylactic reactions to blood products. Treatment of lg A deficiency is directed toward the treatment of symptomatic infections. Asymptomatic
patients do not require specific monitoring.

https://t.me/USMLENBME2CK ti
Previous Next Score Report Lab Values Calculator Help pause
Exam Section 2: Item 10 of 50 National Board of Medical Examiners®
Comprehensive Clinical Science Self-Assessment

"I 10. A 42-year-old woman comes to the physician because of nausea and vomiting and abdominal cramping and swelling over the past 12 hours. She takes an oral
contraceptive. She underwent an appendectomy 5 years ago. Abdominal examination shows distention and moderate tenderness without guarding. An x-ray of the
abdomen is shown. Which of the following is the most likely diagnosis?

A) Adynamic ileus
B) Mechanical obstruction of the small intestine
C) Pelvic abscess
D) Perforated abdominal viscus
E) Regional enteritis

Correct Answer: B.

Adhesions are the most common cause of small bowel obstruction (SBO) in patients with a prior history of intra-abdominal surgery. Obstruction of the small bowel occurs as a result of partial or complete
blockage of the small bowel lumen from either intraluminal or extraluminal obstruction. It typically presents with nausea, vomiting, and abdominal pain and distention. Partial SBO allows a degree of
continued flatus with diminished stooling, and complete SBO results in obstipation (no passage of stool or flatus). SBO has many potential causes, with formation of peritoneal adhesions following a previous
intra-abdominal surgery being the most frequent. On physical examination, SBO typically presents with abdominal tenderness, distension, tympany to percussion, and borborygmi (high-pitched, hyperactive
bowel sounds). Potential complications of SBO include bowel rupture and/or necrosis, which can result in sudden-onset peritoneal abdominal signs, such as rebound tenderness and guarding. Abdominal x-
rays typically show dilated small bowel loops and can also have multiple air-fluid levels. Management requires bowel rest and intravenous hydration followed by surgical intervention if the obstruction does
not reduce on its own. Partial SBO may resolve with monitoring, supportive care, and bowel rest, whereas complete obstruction or any complicated partial obstruction generally requires management through
exploratory laparotomy.

Incorrect Answers: A, C, D, and E.

Adynamic ileus (Choice A) refers to intestinal hypomotility without a mechanical obstruction, most commonly occurring after surgery or in the setting of electrolyte abnormalities (eg, end-stage kidney disease
resulting in hyperkalemia). This typically presents with diffuse small and large bowel dilatation. In a patient with prior abdominal surgery, a mechanical obstruction secondary to peritoneal adhesions is more
likely.

Pelvic abscess (Choice C) can cause abdominal pain and fever but would not be visible on abdominal x-rays. This patient's severely dilated bowel loops on abdominal x-rays are more consistent with SBO.

Perforated abdominal viscus (Choice D) would cause peritonitis, guarding, and rigidity on abdominal examination. While SBOs can be complicated by necrosis and perforation, ii is rare, and this patient does
not show signs of perforation on examination. On imaging, perforation would show free intraperitoneal air.

« https://t.me/USMLENBME2CK ts e «
Previous Next Score Report Lab Values Calculator Help pause
Exam Section 2: Item 10 of 50 National Board of Medical Examiners
Comprehensive Clinical Science Self-Assessment

C) Pelvic abscess
D) Perforated abdominal viscus
E) Regional enteritis

Correct Answer: B.

Adhesions are the most common cause of small bowel obstruction (SBO) in patients with a prior history of intra-abdominal surgery. Obstruction of the small bowel occurs as a result of partial or complete
blockage of the small bowel lumen from either intraluminal or extraluminal obstruction. It typically presents with nausea, vomiting, and abdominal pain and distention. Partial S80 allows a degree of
continued flatus with diminished stooling, and complete SBO results in obstipation (no passage of stool or flatus). S80 has many potential causes, with formation of peritoneal adhesions following a previous
intra-abdominal surgery being the most frequent. On physical examination, S80 typically presents with abdominal tenderness, distension, tympany to percussion, and borborygmi (high-pitched, hyperactive
bowel sounds). Potential complications of SBO include bowel rupture and/or necrosis, which can result in sudden-onset peritoneal abdominal signs, such as rebound tenderness and guarding. Abdominal x-
rays typically show dilated small bowel loops and can also have multiple air-fluid levels. Management requires bowel rest and intravenous hydration followed by surgical intervention if the obstruction does
not reduce on its own. Partial SBO may resolve with monitoring, supportive care, and bowel rest, whereas complete obstruction or any complicated partial obstruction generally requires management through
exploratory laparotomy.

Incorrect Answers: A, C, D, and E.

Adynamic ileus (Choice A) refers to intestinal hypomotility without a mechanical obstruction, most commonly occurring after surgery or in the setting of electrolyte abnormalities (eg, end-stage kidney disease
resulting in hyperkalemia). This typically presents with diffuse small and large bowel dilatation. In a patient with prior abdominal surgery, a mechanical obstruction secondary to peritoneal adhesions is more
likely.

Pelvic abscess (Choice C) can cause abdominal pain and fever but would not be visible on abdominal x-rays. This patient's severely dilated bowel loops on abdominal x-rays are more consistent with S80.

Perforated abdominal viscus (Choice D) would cause peritonitis, guarding, and rigidity on abdominal examination. While SBOs can be complicated by necrosis and perforation, ii is rare, and this patient does
not show signs of perforation on examination. On imaging, perforation would show free intraperitoneal air.

Regional enteritis (Choice E) can present with crampy abdominal pain, watery or bloody diarrhea, and abdominal distension. This patient's abdominal x-rays show diffuse small bowel loop dilation, making
S80 more likely.

Educational Objective: Obstruction of the small bowel occurs from partial or complete blockage of the small bowel lumen, and typically presents with nausea, vomiting, and abdominal pain. Abdominal x-rays
typically show dilated small bowel loops.

« https://t.me/USMLENBME2CK ts e «
Previous Next Score Report Lab Values Calculator Help pause
Exam Section 2: Item 11 of 50 National Board of Medical Examiners
Comprehensive Clinical Science Self-Assessment

"I 11. A 22-year-old woman comes to the physician because of a 2-day history of severe shortness of breath. Over the past week, she has been taking ibuprofen for fever, chills, and muscle pain. She has no
history of serious illness and takes no other medications. Her temperature is 38.4C (101.1°F), pulse is 105/min and irregular, respirations are 28/min, and blood pressure is 70/50 mm Hg. Examination
shows jugular venous distention. Crackles are heard throughout both lung fields. An S, gallop is heard. Pulses are decreased in the upper and lower extremities. An x-ray of the chest shows
cardiomegaly and perihilar fluid. An ECG shows frequent premature ventricular contractions. Echocardiography shows cardiomegaly with normal valve function and an ejection fraction of 18%; her
estimated cardiac output is 1.4 Umin (N=4-5). Which of the following is the most likely diagnosis?

A) Acute myocarditis
B) Acute pericarditis
C) Constrictive pericarditis
D) lschemic cardiomyopathy
E) Pericardial tamponade
F) Superior vena cava syndrome
Correct Answer: A.

Severe acute myocarditis is the most likely diagnosis in this young patient. Myocarditis refers to inflammation of the myocardium. There are numerous causes, including infections (eg, coxsackievirus,
parvovirus B19), toxins, medications (eg, doxorubicin), and connective tissue disease (eg, systemic lupus erythematosus, sarcoidosis). This patient has experienced a viral prodrome prior to presentation.
Acute myocarditis has a broad clinical presentation, ranging from mild asymptomatic disease to severe heart failure. Patients are also at risk for arrhythmias as a result of disruption of the cardiac conduction
system. Acute myocarditis may be difficult to distinguish from pericarditis and acute coronary syndrome as all of these disorders tend to present with chest pain, ECG changes, and increased serum cardiac
biomarkers. Echocardiography can aid in making the diagnosis and guiding treatment. This patient's severe ejection fraction reduction is consistent with a disease process affecting the myocardium.

Incorrect Answers: B, C, D, E, and F.

Acute pericarditis (Choice B) can also present with dyspnea and chest pain in the context of fever and recent viral illness. Pericarditis typically displays characteristic ECG findings with widespread,
noncontiguous ST elevations and PR depressions. A normal ejection fraction is also expected on echocardiography, and fluid may be observed in the pericardia! space. Pericardia! effusion may progress to
pericardia I tamponade (Choice E) if expansion of the myocardium and ventricular filling are impaired. Fluid will likely be visible in the pericardium on echocardiography as well as reduced ventricular
movement.

Constrictive pericarditis (Choice C) typically presents with sharp chest pain that is aggravated by inspiration and alleviated by sitting and leaning forward. A coarse, sandpaper-like friction rub may be heard
on cardiac auscultation. Physical examination findings also include fever, hypotension, tachycardia, pulsus paradoxus, and the Kussmaul sign (an increase in jugular venous distension during inspiration).

lschemic cardiomyopathy (Choice 0) is much more common in older patients unless there is a family history of early cardiac disease. It is most often caused by chronic coronary artery disease. Wall motion
abnormalities are often noted on echocardiography corresponding to regions affected by ischemic damage.

Superior vena cava (SVC) syndrome (Choice F) is caused by obstruction of the SVC and subsequent venous congestion of the head, neck, and upper extremities. Patients typically develop distinct swelling
of the face and neck, increased Jugular venous pressure, upper extremity swelling, distended chest vein collaterals, and conjunctiva! edema.

https://t.me/USMLENBME2CK ts e t
Previous Next Score Report Lab Values Calculator Help pause
Exam Section 2: Item 11 of 50 National Board of Medical Examiners
Comprehensive Clinical Science Self-Assessment

estimated cardiac output is 1.4 Umin (N=4-5). Which of the following is the most likely diagnosis?

A) Acute myocarditis
B) Acute pericarditis
C) Constrictive pericarditis
D) lschemic cardiomyopathy
E) Pericardial tamponade
F) Superior vena cava syndrome
Correct Answer: A.

Severe acute myocarditis is the most likely diagnosis in this young patient. Myocarditis refers to inflammation of the myocardium. There are numerous causes, including infections (eg, coxsackievirus,
parvovirus B19), toxins, medications (eg, doxorubicin), and connective tissue disease (eg, systemic lupus erythematosus, sarcoidosis). This patient has experienced a viral prodrome prior to presentation.
Acute myocarditis has a broad clinical presentation, ranging from mild asymptomatic disease to severe heart failure. Patients are also at risk for arrhythmias as a result of disruption of the cardiac conduction
system. Acute myocarditis may be difficult to distinguish from pericarditis and acute coronary syndrome as all of these disorders tend to present with chest pain, ECG changes, and increased serum cardiac
biomarkers. Echocardiography can aid in making the diagnosis and guiding treatment. This patient's severe ejection fraction reduction is consistent with a disease process affecting the myocardium.

Incorrect Answers: B, C, D, E, and F.

Acute pericarditis (Choice B) can also present with dyspnea and chest pain in the context of fever and recent viral illness. Pericarditis typically displays characteristic ECG findings with widespread,
noncontiguous ST elevations and PR depressions. A normal ejection fraction is also expected on echocardiography, and fluid may be observed in the pericardia! space. Pericardia! effusion may progress to
pericardia I tamponade (Choice E) if expansion of the myocardium and ventricular filling are impaired. Fluid will likely be visible in the pericardium on echocardiography as well as reduced ventricular
movement.

Constrictive pericarditis (Choice C) typically presents with sharp chest pain that is aggravated by inspiration and alleviated by sitting and leaning forward. A coarse, sandpaper-like friction rub may be heard
on cardiac auscultation. Physical examination findings also include fever, hypotension, tachycardia, pulsus paradoxus, and the Kussmaul sign (an increase in jugular venous distension during inspiration).

lschemic cardiomyopathy (Choice 0) is much more common in older patients unless there is a family history of early cardiac disease. It is most often caused by chronic coronary artery disease. Wall motion
abnormalities are often noted on echocardiography corresponding lo regions affected by ischemic damage.

Superior vena cava (SVC) syndrome (Choice F) is caused by obstruction of the SVC and subsequent venous congestion of the head, neck, and upper extremities. Patients typically develop distinct swelling
of the face and neck, increased Jugular venous pressure, upper extremity swelling, distended chest vein collaterals, and conjunctiva! edema.

Educational Objective: Acute myocardilis should be suspected in a patient presenting with new, unexplained heart failure with reduced ejection fraction, especially in the setting of a recent viral prodrome.
Other causes include toxins, medications, and autoimmune disease. Treatment is supportive and directed at the underlying cause.

https://t.me/USMLENBME2CK ts e t
Previous Next Score Report Lab Values Calculator Help pause
Exam Section 2: Item 12 of 50 National Board of Medical Examiners
Comprehensive Clinical Science Self-Assessment

✓ 12. A study is conducted to determine if a hemoglobin A le greater than 15% predicts diabetic retinopathy. Results show that 88% of all patients without diabetic retinopathy had a hemoglobin A 1c below
15%. Which of the following is the most accurate interpretation of these results?

A) False-negative rate is 88%


B) False-positive rate is 88%
C) Negative predictive value is 88%
D) Positive predictive value is 88%
E) Power of the study design is 88%
F) Sensitivity is 88%
G) Specificity is 88%
Correct Answer: G.

Specificity describes the proportion of people who do not have the disease who test negative and is calculated as: true negatives/ (true negatives + false positives). As specificity increases, the false
positive rate decreases. A highly specific test allows the clinician and the patient to be confident that a positive test confirms the presence of disease. A test with poor specificity, if positive, does not provide
strong evidence or confidence that the patient does, in fact, have the disease. In contrast, sensitivity is the ability of a test to detect a disease if it is present. A test is described as sensitive if it has a high
likelihood of disease detection, and therefore a low likelihood of a false negative (Type 2 error). High sensitivity is useful in ruling out a disease. In the study described, a hemoglobin A1c greater than 15%
was present in 12% (false positive rate) and not present in 88% (true negative rate) of patients without diabetic retinopathy.

Incorrect Answers: A, B, C, D, E, and F.

False-negative (FN) rate (Choice A) represents the portion of patients who test negative but have the disease under study. This scenario provides insufficient information for the calculation of the FN rate.

False-positive {FP) rate (Choice 8) represents the portion of patients who test positive but do not have the disease under study. In this scenario, this would represent patients with a hemoglobin A1c greater
than 15% who lack diabetic retinopathy (12%).

Negative predictive value {NPV) is 88% {Choice C) is incorrect. Negative predictive value is the likelihood that a person does not have a disease, given a negative test, and can be calculated as NPV = true
negatives (TN)/{TN + FN). The results of this study do not allow for the calculation of NPV.

Positive predictive value (PPV) is 88% {Choice D) is incorrect. PPV is the likelihood that a person has a disease, given a positive test, and can be calculated as PPV = true positives (TP)/(TP + FP). The
results of this study do not allow for the calculation of PPV.

Power of the study design {Choice E) is dependent upon the number of participants enrolled in the study. This scenario provides insufficient information to allow a calculation of the study's power.

Sensitivity {Choice F) describes the ability of a test to detect a disease if ii is present. A negative result from a highly sensitive test indicates a low likelihood that the disease is present. Sensitivity is
calculated by dividing TP by the sum of TP and FN, or TP/(TP + FN). In this scenario, true positives would represent patients who possess diabetic retinopathy who have a hemoglobin A,, qreater than

https://t.me/USMLENBME2CK ti e
Previous Next Score Report Lab Values Calculator Help pause
Exam Section 2: Item 12 of 50 National Board of Medical Examiners
Comprehensive Clinical Science Self-Assessment

C) Negative predictive value is 88%


D) Positive predictive value is 88%
E) Power of the study design is 88%
F) Sensitivity is 88%
G) Specificity is 88%
Correct Answer: G.

Specificity describes the proportion of people who do not have the disease who test negative and is calculated as: true negatives/ (true negatives + false positives). As specificity increases, the false
positive rate decreases. A highly specific test allows the clinician and the patient to be confident that a positive test confirms the presence of disease. A test with poor specificity, if positive, does not provide
strong evidence or confidence that the patient does, in fact, have the disease. In contrast, sensitivity is the ability of a test to detect a disease if it is present. A test is described as sensitive if it has a high
likelihood of disease detection, and therefore a low likelihood of a false negative (Type 2 error). High sensitivity is useful in ruling out a disease. In the study described, a hemoglobin A1c greater than 15%
was present in 12% (false positive rate) and not present in 88% (true negative rate) of patients without diabetic retinopathy.

Incorrect Answers: A, B, C, D, E, and F.

False-negative (FN) rate (Choice A) represents the portion of patients who test negative but have the disease under study. This scenario provides insufficient information for the calculation of the FN rate.

False-positive {FP) rate (Choice B) represents the portion of patients who test positive but do not have the disease under study. In this scenario, this would represent patients with a hemoglobin A1c greater
than 15% who lack diabetic retinopathy (12%).

Negative predictive value {NPV) is 88% (Choice C) is incorrect. Negative predictive value is the likelihood that a person does not have a disease, given a negative test, and can be calculated as NPV = true
negatives (TN)/(TN + FN). The results of this study do not allow for the calculation of NPV.

Positive predictive value {PPV) is 88% (Choice D) is incorrect. PPV is the likelihood that a person has a disease, given a positive test, and can be calculated as PPV = true positives (TP)/(TP + FP). The
results of this study do not allow for the calculation of PPV.

Power of the study design (Choice E) is dependent upon the number of participants enrolled in the study. This scenario provides insufficient information to allow a calculation of the study's power.

Sensitivity (Choice F) describes the ability of a test to detect a disease if ii is present. A negative result from a highly sensitive test indicates a low likelihood that the disease is present. Sensitivity is
calculated by dividing TP by the sum of TP and FN, or TP/(TP + FN). In this scenario, true positives would represent patients who possess diabetic relinopathy who have a hemoglobin A1c greater than
15%. This scenario provides insufficient information for the calculation of sensitivity.

Educational Objective: Specificity describes the proportion of people who do not have the disease who test negative and is calculated as: true negatives I (true negatives+ false positives). In the study
described, a hemoglobin A, greater than 15% was present in 12% (false positive rate) and not present in 88% (true negative rate) of patients without diabetic retinopathy.

https://t.me/USMLENBME2CK ti e
Previous Next Score Report Lab Values Calculator Help pause
Exam Section 2: Item 13 of 50 National Board of Medical Examiners
Comprehensive Clinical Science Self-Assessment

"I 13. A 13-year-old girl is brought to the physician by her mother because of a 3-day history of pain and pressure in her face, itchy eyes, and decreased hearing in both ears. She has not had fever, chills,
muscle pain, or nausea. Acetaminophen has not relieved her symptoms. She takes no other medications. One year ago, she had a similar episode and was successfully treated with azithromycin
therapy at an urgent care center, and her mother requests another prescription for azithromycin. The patient's two brothers have well-controlled asthma. Her vital signs are within normal limits.
Examination shows a supple neck with a few shotty lymph nodes in the upper cervical chain. The conjunctivae have a cobblestone appearance. The inferior nasal turbinates are edematous. The
tympanic membranes are injected bilaterally. There is moderate mucus in the posterior pharynx. The tonsils are not enlarged. The lungs are clear to auscultation. Which of the following is the most
appropriate next step in management?

A) CT scan of the head


B) MRI of the sinuses
C) Ceftriaxone therapy
D) Loratadine therapy
E) Prednisone therapy
Correct Answer: D.

Allergic rhinitis is a common cause of upper respiratory infection-like symptoms in children, teens, and adults. Frequent allergens include pollen, dust mites, mold, and animal dander. It commonly presents
with rhinorrhea, sneezing, and cough. It may be associated with allergic conjunctivitis, as well, which presents with itchy, watering eyes. Physical examination may disclose inflamed nasal turbinates,
cobblestoning" of the posterior pharynx and upper eyelids, retracted and injected tympanic membranes, and infraorbital darkening. Chronic nasal obstruction can lead to impairment of sinus drainage, which
can lead to the development of sinusitis. Bacterial inoculation can also occur, leading to bacterial sinusitis with fever, facial pain, purulent rhinorrhea, and cough. Risk factors include a personal or family
history of atopy, early exposure to allergens, being the firstborn child, and childhood use of antibiotics. The diagnosis is clinical, and treatment includes a second-generation antihistamine, as well as a
glucocorticoid nasal spray, to reduce inflammation and allow appropriate nasal and sinus drainage. This patient presents with facial pain likely caused by sinus obstruction in the setting of nasal inflammation,
further evidenced by her injected tympanic membranes with reduced hearing. However, she has no signs of bacterial sinusitis, such as fever or purulent nasal discharge. Thus, in conjunction with the allergic
conjunctivitis she is experiencing, the most likely cause of her symptoms is allergic rhinitis, and she should be treated with an oral antihistamine such as loratadine.

Incorrect Answers: A, B, C, and E.

CT scan of the head (Choice A) is not necessary to make the clinical diagnosis of allergic rhinitis in this patient. CT scan can be used to look for intracranial abnormalities that may be causing facial pain or to
assist in the diagnosis of bacterial sinusitis if the diagnosis is unclear. This patient has clear signs of an allergic cause, thus making treatment with loratadine a more appropriate next step.

MRI of the sinuses (Choice B) may be used in the evaluation of recurrent or chronic sinusitis. It can be especially useful in the diagnosis of sinus tumors or fungal infections. This patient has uncomplicated
allergic sinusitis, making an MRI unnecessary.

Ceflriaxone therapy (Choice C) is used in the treatment of a variety of infections, such as community-acquired pneumonia, bacterial meningitis, and acute otitis media. This patient's itchy, watery eyes,
edematous turbinates, and injected tympanic membranes are more consistent with an allergic cause than an infectious cause.

Prednisone therapy (Choice E) is often prescribed for the treatment of autoimmune and inflammatory disorders. While it may reduce the inflammation associated with this patient's allergic rhinitis, prednisone
has a variety of adverse effects and would not be the first-line therapy. Her initial treatment should include an antihistamine.

https://t.me/USMLENBME2CK ts e t
Previous Next Score Report Lab Values Calculator Help pause
Exam Section 2: Item 13 of 50 National Board of Medical Examiners
Comprehensive Clinical Science Self-Assessment

appropriate next step in management?

A) CT scan of the head


B) MRI of the sinuses
C) Ceftriaxone therapy
D) Loratadine therapy
E) Prednisone therapy
Correct Answer: D.

Allergic rhinitis is a common cause of upper respiratory infection-like symptoms in children, teens, and adults. Frequent allergens include pollen, dust mites, mold, and animal dander. It commonly presents
with rhinorrhea, sneezing, and cough. It may be associated with allergic conjunctivitis, as well, which presents with itchy, watering eyes. Physical examination may disclose inflamed nasal turbinates,
"cobblestoning" of the posterior pharynx and upper eyelids, retracted and injected tympanic membranes, and infraorbilal darkening. Chronic nasal obstruction can lead to impairment of sinus drainage, which
can lead lo the development of sinusitis. Bacterial inoculation can also occur, leading to bacterial sinusitis with fever, facial pain, purulent rhinorrhea, and cough. Risk factors include a personal or family
history of atopy, early exposure to allergens, being the firstborn child, and childhood use of antibiotics. The diagnosis is clinical, and treatment includes a second-generation antihistamine, as well as a
glucocorticoid nasal spray, to reduce inflammation and allow appropriate nasal and sinus drainage. This patient presents with facial pain likely caused by sinus obstruction in the setting of nasal inflammation,
further evidenced by her injected tympanic membranes with reduced hearing. However, she has no signs of bacterial sinusitis, such as fever or purulent nasal discharge. Thus, in conjunction with the allergic
conjunctivitis she is experiencing, the most likely cause of her symptoms is allergic rhinitis, and she should be treated with an oral antihistamine such as loratadine.

Incorrect Answers: A, B, C, and E.

CT scan of the head (Choice A) is not necessary to make the clinical diagnosis of allergic rhinitis in this patient. CT scan can be used to look for intracranial abnormalities that may be causing facial pain or to
assist in the diagnosis of bacterial sinusitis if the diagnosis is unclear. This patient has clear signs of an allergic cause, thus making treatment with loratadine a more appropriate next step.

MRI of the sinuses (Choice B) may be used in the evaluation of recurrent or chronic sinusitis. It can be especially useful in the diagnosis of sinus tumors or fungal infections. This patient has uncomplicated
allergic sinusitis, making an MRI unnecessary.

Ceflriaxone therapy (Choice C) is used in the treatment of a variety of infections, such as community-acquired pneumonia, bacterial meningitis, and acute otitis media. This patient's itchy, watery eyes,
edematous turbinates, and injected tympanic membranes are more consistent with an allergic cause than an infectious cause.

Prednisone therapy (Choice E) is often prescribed for the treatment of autoimmune and inflammatory disorders. While it may reduce the inflammation associated with this patient's allergic rhinitis, prednisone
has a variety of adverse effects and would not be the first-line therapy. Her initial treatment should include an antihistamine.

Educational Objective: Allergic rhinitis commonly presents with rhinorrhea, sneezing, cough, and nasal congestion, the latter of which can lead to sinusitis with facial pain and obstructed tympanic
membranes with reduced hearing. It is commonly associated with allergic conjunctivitis and asthma. Physical examination will show inflamed nasal turbinates, "cobblestoning" of the posterior pharynx and
upper eyelids, retracted and injected tympanic membranes, and infraorbital darkening. Treatment includes a second-generation antihistamine and a glucocorticoid nasal spray.

https://t.me/USMLENBME2CK ts e t
Previous Next Score Report Lab Values Calculator Help pause
Exam Section 2: Item 14 of 50 National Board of Medical Examiners
Comprehensive Clinical Science Self-Assessment

✓ 14. A 76-year-old woman is scheduled to undergo right knee arthroscopy at an ambulatory surgery center. Before the procedure begins, the surgical team confirms the identity of the patient, the procedure
being performed, the side and site of the procedure, the patient's position, and the availability of needed equipment. The procedure is then completed without complications, and the patient is
discharged home 4 hours later. Which of the following is most likely increased at this center as a result of the pre-procedure actions of the surgical team?

A) Efficiency
B) Patient satisfaction
C) Patient-centered care
D) Reliability
Correct Answer: D.

Despite best efforts, surgical reliability remains prone to errors, including performance of incorrect procedures or surgery performed at the wrong site or on the wrong patient. The Joint Commission Universal
Protocol for Preventing Wrong Site, Wrong Procedure, Wrong Person Surgery was developed lo reduce such errors and improve surgical reliability. Steps include preoperative verification of the patient's
identity, the procedure to be performed, and the surgical site. The patient should be involved in the verification process whenever possible, and any discrepancies or questions should be addressed. The use
of a checklist can assist in ensuring a standardized verification process. The surgical site should be marked by the surgeon who will perform the procedure, and the surgical mark should be both unambiguous
and positioned such that it will be visible after the surgical drape is placed. Additionally, a final verification "time out" should be performed prior to induction of anesthesia to verify the patient's identity, the
procedure, and the surgical site.

Incorrect Answers: A, B, and C.

Efficiency (Choice A) is incorrect. Preprocedure verification requires several minutes and slightly reduces efficiency. This loss of efficiency is more than offset by the benefits of increasing reliability and
reducing the risk for performing incorrect procedures.

Patient satisfaction (Choice B) and patient-centered care (Choice C) are important components of high-quality health care, but are impacted less directly by preoperative verification procedures, which have
their most direct impact on reliability.

Educational Objective: The Joint Commission Universal Protocol for Preventing Wrong Site, Wrong Procedure, Wrong Person Surgery was developed to reduce surgical errors and improve surgical reliability.
Preprocedure actions include preoperative verification of the patient's identity, the procedure to be performed, and the surgical site.

https://t.me/USMLENBME2CK ti
Previous Next Score Report Lab Values Calculator Help pause
Exam Section 2: Item 15 of 50 National Board of Medical Examiners
Comprehensive Clinical Science Self-Assessment

"I 15. Immediately after extubation following a total thyroidectomy, a 72-year-old woman has stridor and shortness of breath. She has a history of transient ischemic attacks and coronary artery disease. Her
only medication is aspirin. She appears to be in distress and is gasping for air. Her pulse is 100/min, respirations are 35/min, and blood pressure is 170/100 mm Hg. Examination shows an intact
surgical incision and no ecchymosis. Breath sounds are decreased bilaterally. Arterial blood gas analysis on 40% oxygen by face mask shows:
pH 7.3
Pco 60 mm Hg
Po, 200 mm Hg
O saturation 99%

Which of the following is the most likely diagnosis?

A) Cerebral infarction
B) Extrinsic tracheal compression
C) Laryngeal nerve paralysis
D) Pulmonary embolism
E) Tension pneumothorax
Correct Answer: C.

Thyroidectomy is a common surgical procedure, often used in the management of hyperthyroidism, thyroid cancer, and goiter. The thyroid gland is situated anterior to the trachea and consists of two lobes
that are connected by a smaller isthmus. The thyroid is located near a variety of vascular and neurologic structures. The recurrent laryngeal nerves run posteriorly to the two lobes bilaterally. They provide
muscle innervation for both abduction and adduction of the vocal cords, as well as sensation to the region below the glottis. Special care is taken to identify these nerves during thyroidectomy to avoid nerve
injury, often with direct intraoperative nerve monitoring. Unilateral recurrent laryngeal nerve paralysis can lead to voice hoarseness caused by the inability to abduct the vocal cord on one side, whereas
bilateral paralysis can lead to airway obstruction as both vocal cords remain in a medial position. This patient has respiratory distress with audible stridor, indicating an upper airway obstruction, the likelihood
of which is further reinforced by decreased breath sounds bilaterally caused by poor air movement through the obstruction. Given the onset of this patient's symptoms immediately after surgery, the most
likely cause of this presentation is bilateral recurrent laryngeal nerve paralysis leading to vocal cord paralysis and airway obstruction.

Incorrect Answers: A, B, D, and E.

Cerebral infarction (Choice A) would most likely lead to unilateral motor weakness or sensory changes. It would be unlikely to cause respiratory distress with stridor as a result of laryngeal nerve dysfunction
in the absence of any other symptoms. Although this patient has a history of transient ischemic attacks, her symptoms are most consistent with laryngeal nerve paralysis sustained during the procedure.

Extrinsic tracheal compression (Choice 8) can lead to airway compromise following a thyroidectomy. This is most commonly caused by surgical bleeding that leads to an expanding hematoma in an enclosed
space, thereby compressing the trachea. This patient has no evidence of hematoma on examination, making this an unlikely cause of her symptoms.

Pulmonary embolism (Choice 0) can cause the acute onset of dyspnea, tachypnea, tachycardia, and, occasionally, hemoptysis. However, pulmonary emboli are more likely to present several days after
surgery, rather than immediately following extubation. Additionally, pulmonary emboli are not associated with the stridor or decreased bilateral breath sounds exhibited by this patient.

https://t.me/USMLENBME2CK ti
Previous Next Score Report Lab Values Calculator Help pause
Exam Section 2: Item 15 of 50 National Board of Medical Examiners
Comprehensive Clinical Science Self-Assessment

Which of the following is the most likely diagnosis?

A) Cerebral infarction
B) Extrinsic tracheal compression
C) Laryngeal nerve paralysis
D) Pulmonary embolism
E) Tension pneumothorax
Correct Answer: C.

Thyroidectomy is a common surgical procedure, often used in the management of hyperthyroidism, thyroid cancer, and goiter. The thyroid gland is situated anterior to the trachea and consists of two lobes
that are connected by a smaller isthmus. The thyroid is located near a variety of vascular and neurologic structures. The recurrent laryngeal nerves run posteriorly to the two lobes bilaterally. They provide
muscle innervation for both abduction and adduction of the vocal cords, as well as sensation to the region below the glottis. Special care is taken to identify these nerves during thyroidectomy to avoid nerve
injury, often with direct intraoperative nerve monitoring. Unilateral recurrent laryngeal nerve paralysis can lead to voice hoarseness caused by the inability to abduct the vocal cord on one side, whereas
bilateral paralysis can lead to airway obstruction as both vocal cords remain in a medial position. This patient has respiratory distress with audible strider, indicating an upper airway obstruction, the likelihood
of which is further reinforced by decreased breath sounds bilaterally caused by poor air movement through the obstruction. Given the onset of this patient's symptoms immediately after surgery, the most
likely cause of this presentation is bilateral recurrent laryngeal nerve paralysis leading to vocal cord paralysis and airway obstruction.

Incorrect Answers: A, B, D, and E.

Cerebral infarction (Choice A) would most likely lead to unilateral motor weakness or sensory changes. II would be unlikely lo cause respiratory distress with strider as a result of laryngeal nerve dysfunction
in the absence of any other symptoms. Although this patient has a history of transient ischemic attacks, her symptoms are most consistent with laryngeal nerve paralysis sustained during the procedure.

Extrinsic tracheal compression (Choice 8) can lead to airway compromise following a thyroidectomy. This is most commonly caused by surgical bleeding that leads to an expanding hematoma in an enclosed
space, thereby compressing the trachea. This patient has no evidence of hematoma on examination, making this an unlikely cause of her symptoms.

Pulmonary embolism (Choice 0) can cause the acute onset of dyspnea, tachypnea, tachycardia, and, occasionally, hemoptysis. However, pulmonary emboli are more likely to present several days after
surgery, rather than immediately following extubation. Additionally, pulmonary emboli are not associated with the stridor or decreased bilateral breath sounds exhibited by this patient.

Tension pneumothorax (Choice E) commonly presents with dyspnea, hypoxia, tachycardia, hypotension, tracheal deviation, and unilaterally absent breath sounds. II can rapidly lead to cardiac arrest and
death if not identified. This patient has bilateral breath sounds and is hypertensive, making a tension pneumothorax unlikely.

Educational Objective: The recurrent laryngeal nerves are responsible for providing the majority of motor innervation lo the muscles of the larynx, as well as the sensation to the tissues below the glottis.
These nerves course posterior to the thyroid bilaterally, putting them at risk for sustaining injury during a thyroidectomy. This is often avoided by using direct intraoperative nerve monitoring. However, if
inadvertently injured, unilateral injury will lead to voice hoarseness while bilateral injury can lead to severe upper airway obstruction.

https://t.me/USMLENBME2CK ti
Previous Next Score Report Lab Values Calculator Help pause
Exam Section 2: ltem 16 of 50 National Board of Medical Examiners
Comprehensive Clinical Science Self-Assessment

✓ 16. A 46-year-old man with hypertension comes lo the physician for a routine examination. When the physician enters the room, the patient says the office staff were talking about him in the waiting
room. The patient has declined antihypertensive therapy in the past; today, he declines it again, saying "it does not matter because the world is going to end soon." He does not drink alcohol or use
illicit drugs. He lives alone and has minimal contact with his family. He says he has one friend and spends his free time playing video games. He works at home for an accounting firm. He appears
fearful and older than his stated age. He wears an overcoat and woolen hat although the outside temperature is 80F. His pulse is 80/min and regular, and blood pressure is 140/98 mm Hg. Limited
physical examination shows no abnormalities. On mental status examination, he has a euthymic mood and odd affect. He makes minimal eye contact with the physician and appears anxious when
questioned. He says he has not had anxiety or changes in mood. Cognition is intact, and answers to questions are appropriate. His thought process is linear. He reports no hallucinations. Results of
laboratory studies are within the reference ranges. Which of the following is the most likely diagnosis?

A) Autism spectrum disorder


B) Delusional disorder
C) Oppositional defiant disorder
0) Schizophrenia
E) Schizotypal personality disorder
Correct Answer: E.

Schizotypal personality disorder is one of the cluster A (the odd or eccentric cluster) personality disorders according to the OSM-5. Personality disorders are persistently maladaptive ways of relating to the
self and to society that typically appear by early adulthood. Schizotypal personality disorder is characterized by odd behavior and thinking, and a constricted affect. Patients with schizotypal personality
disorder may have strange, overly metaphorical or magical thinking, though they fluctuate in the degree of their conviction about these beliefs, which differentiates these beliefs from the fixed delusions of
schizophrenia. The strange beliefs may also manifest as an eccentric appearance. Patients may experience somatosensory illusions or other abnormal perceptions. Patients with schizotypal personality
disorder may appear to lack interest in cultivating relationships and, as a result of an inability to interpret others' motivations, may be deeply distrustful and anxious around others.

Incorrect Answers: A, B, C, and D.

Autism spectrum disorder (Choice A) is a neurodevelopmental disorder characterized by a broad range of signs and symptoms involving restricted and repetitive patterns of behavior or activities, difficulties
with social interaction, and difficulties with communication. It typically manifests during childhood by age 1 or 2 years. This patient's eccentric appearance and beliefs are more consistent with schizotypal
personality disorder.

Delusional disorder (Choice B) is characterized by the presence of one or more delusions (ie, fixed, false beliefs) for a month or longer without other psychotic symptoms. Delusional disorder is not
characterized by an eccentric appearance, social indifference, or constricted affect.

Oppositional defiant disorder (Choice C) manifests in children as a persistent pattern of angry mood, defiant behavior, and vindictiveness that causes distress and impaired functioning.

Schizophrenia (Choice D) is characterized by at least two of the following five symptoms: delusions, hallucinations, disorganized speech, disorganized behavior, or negative symptoms (eg, flat affect,
apathy, alogia) during a period of at least 6 months. This patient does not experience hallucinations and does not display disorganized thoughts, speech, or behavior.

Educational Objective: Schizotypal personality disorder manifests as an enduring pattern of strange or eccentric beliefs and behaviors, social detachment or suspiciousness, and abnormal perceptual
et - it ,._ - ,_ - let 4 4 4 Aloof

https://t.me/USMLENBME2CK ts e t
Previous Next Score Report Lab Values Calculator Help pause
Exam Section 2: ltem 16 of 50 National Board of Medical Examiners
Comprehensive Clinical Science Self-Assessment

fearful and older than his stated age. He wears an overcoat and woolen hat although the outside temperature is 80F. His pulse is 80/min and regular, and blood pressure is 140/98 mm Hg. Limited
physical examination shows no abnormalities. On mental status examination, he has a euthymic mood and odd affect. He makes minimal eye contact with the physician and appears anxious when
questioned. He says he has not had anxiety or changes in mood. Cognition is intact, and answers to questions are appropriate. His thought process is linear. He reports no hallucinations. Results of
laboratory studies are within the reference ranges. Which of the following is the most likely diagnosis?

A) Autism spectrum disorder


B) Delusional disorder
C) Oppositional defiant disorder
0) Schizophrenia
E) Schizotypal personality disorder
Correct Answer: E.

Schizotypal personality disorder is one of the cluster A (the odd or eccentric cluster) personality disorders according to the OSM-5. Personality disorders are persistently maladaptive ways of relating to the
self and to society that typically appear by early adulthood. Schizotypal personality disorder is characterized by odd behavior and thinking, and a constricted affect. Patients with schizotypal personality
disorder may have strange, overly metaphorical or magical thinking, though they fluctuate in the degree of their conviction about these beliefs, which differentiates these beliefs from the fixed delusions of
schizophrenia. The strange beliefs may also manifest as an eccentric appearance. Patients may experience somatosensory illusions or other abnormal perceptions. Patients with schizotypal personality
disorder may appear to lack interest in cultivating relationships and, as a result of an inability to interpret others' motivations, may be deeply distrustful and anxious around others.

Incorrect Answers: A, B, C, and D.

Autism spectrum disorder (Choice A) is a neurodevelopmental disorder characterized by a broad range of signs and symptoms involving restricted and repetitive patterns of behavior or activities, difficulties
with social interaction, and difficulties with communication. It typically manifests during childhood by age 1 or 2 years. This patient's eccentric appearance and beliefs are more consistent with schizotypal
personality disorder.

Delusional disorder (Choice B) is characterized by the presence of one or more delusions (ie, fixed, false beliefs) for a month or longer without other psychotic symptoms. Delusional disorder is not
characterized by an eccentric appearance, social indifference, or constricted affect.

Oppositional defiant disorder (Choice C) manifests in children as a persistent pattern of angry mood, defiant behavior, and vindictiveness that causes distress and impaired functioning.

Schizophrenia (Choice D) is characterized by at least two of the following five symptoms: delusions, hallucinations, disorganized speech, disorganized behavior, or negative symptoms (eg, flat affect,
apathy, alogia) during a period of at least 6 months. This patient does not experience hallucinations and does not display disorganized thoughts, speech, or behavior.

Educational Objective: Schizotypal personality disorder manifests as an enduring pattern of strange or eccentric beliefs and behaviors, social detachment or suspiciousness, and abnormal perceptual
experiences such as illusions. Social indifference distinguishes schizotypal disorder from DSM-5 cluster Band C personality disorders, while eccentric beliefs and behavior differentiate the disorder from the
other cluster A personality disorders.

https://t.me/USMLENBME2CK ts e t
Previous Next Score Report Lab Values Calculator Help pause
Exam Section 2: Item 17 of 50 National Board of Medical Examiners
Comprehensive Clinical Science Self-Assessment

"I 17. A 4-year-old girl is brought lo the office by her mother because of a 1-week history of vaginal itching. The mother says her daughter is scratching her vaginal area frequently and the area appears
irritated. The patient has not had fever, pain with urination, or urinary frequency. She has been toilet-trained for 1 year. The mother is not concerned about sexual abuse. When questioned alone, the
patient says that no one has touched her inappropriately. Examination shows erythema of the vaginal area. There is no vaginal discharge, foul odor, bleeding, tears, or ecchymoses. Which of the
following is the most appropriate next step in management?

A) Begin antibiotic therapy


B) Begin antifungal therapy
C) Order evaluation by child protective services
D) Recommend that the patient switch to cotton underwear and avoid bubble baths
E) Schedule gynecologic examination
Correct Answer: D.

Vulvovaginitis presents with vaginal itching and erythema, and it can have a variety of causes. When present in a prepubertal child, a careful investigation into the possibility of sexual abuse should occur. Other
causes of vulvovaginitis in a child include poor hygiene, foreign bodies (especially toilet paper), occlusive clothing, and exposure to irritants. less common causes include infections such as pinworms,
Streptococcus pyogenes, and Escherichia coli. Foreign bodies and infections are frequently associated with foul odor and vaginal discharge. Initial treatment includes hygienic improvement and the avoidance
of irritants, such as recommending that the patient switch lo cotton underwear and avoid bubble baths, which is the most appropriate next step in management of this patient's nonspecific vulvovaginitis. If the
vulvovaginitis is recurrent or does not respond to conservative measures, further evaluation with vaginal cultures may be appropriate.

Incorrect Answers: A, B, C, and E.

Begin antibiotic therapy (Choice A) and begin antifungal therapy (Choice B) are not the most appropriate next step. This patient has nonspecific vulvovaginitis with no signs of infection such as foul odor or
vaginal discharge. It is most appropriate to begin with conservative management that decreases exposure to irritants.

Order evaluation by child protective services (Choice C) would be inappropriate. Sexually transmitted infections can cause vulvovaginitis, and children presenting with this pathology should be carefully
evaluated for other signs of physical or sexual abuse. However, there is no clinical evidence of abuse.

Schedule gynecologic examination (Choice E) is not necessary. This patient has nonspecific vaginitis, likely caused by exposure to irritants. Gynecologic examination is frequently avoided in children of her age
unless the diagnosis remains unclear.

Educational Objective: Vulvovaginitis in a prepubertal child presents with vaginal itching and erythema, as well as foul odor or vaginal discharge based on the cause. Causes include poor hygiene, foreign
bodies, occlusive clothing, exposure to irritants, and infection. Children presenting with vulvovaginitis should be carefully evaluated for signs and symptoms of physical or sexual abuse. Treatment of
nonspecific vulvovaginitis is initially conservative, with recommendations to improve hygiene and limit exposure to irritants such as bubble baths or synthetic materials.

https://t.me/USMLENBME2CK ti
Previous Next Score Report Lab Values Calculator Help pause
Exam Section 2: ltem 18 of 50 National Board of Medical Examiners
Comprehensive Clinical Science Self-Assessment

"I 18. An 87-year-old man comes to the physician because of a 2-day history of visual changes in his right eye. He notices that the lines of his daily crossword puzzle look curved, and the blinds in his
apartment appear wavy. He wears magnifying lenses for reading. There is no pain in his eye, no photophobia, and no history of trauma to the area. He had cataract removal from the right eye 5 years
ago. There is no history of serious illness. On examination, visual acuity is 20/200 in his right eye and 20/100 in his left eye. The right pupil is irregular, but it is reactive to light. The left pupil is round
and reactive. This examination finding is unchanged since his cataract operation. Palpation of both globes through closed eyelids shows no abnormalities. His lenses appear clear. Which of the
following is the most appropriate next step in diagnosis?

A) Dilated funduscopic examination


B) Slit-lamp examination
C) Visual field testing
D) CT scan of the head
E) Temporal artery biopsy
Correct Answer: A.

Age-related macular degeneration (AMO) is an acquired retinal degeneration characterized by dysfunction of the retinal pigment epithelium and accumulation of macular drusen, the hallmark of AMO. Drusen
represent yellow, subretinal, extracellular deposits of lipoproteinaceous material. The underlying cause is not completely understood, but a combination of environmental and multiple genetic risk factors is
likely involved, including genes associated with complement regulation and autophagy. AMO is common and is a leading cause of nonreversible blindness in developed countries. Early stages of the disease
may be asymptomatic or characterized by insidious blurring or metamorphopsia (a visual distortion in which straight lines appear curved). In advanced stages, profound vision loss may result from retinal
geographic atrophy or choroidal neovascularization with associated macular edema, exudation, or hemorrhage. Risk factors for AMO include age, smoking, and genetics. The diagnosis is established by a
dilated funduscopic examination to evaluate the retina. Antioxidant and vitamin supplementation are helpful for delaying the progression of nonexudative (dry) AMO, while exudative (wet) AMO is treated with
intravitreal inJections of monoclonal antibodies against vascular endothelial growth factor.

Incorrect Answers: B, C, D, and E.

Slit-lamp examination (Choice B) is a method for examining ocular structures under high magnification. This technique may be used to examine structures within the anterior segment of the eye, such as the
cornea, iris, or lens, or within the posterior segment (including the retina and optic nerve), usually as part of a dilated funduscopic examination.

Visual field testing (Choice C) is a method for detecting visual scotomas or visual field loss. It is most useful in the diagnosis of glaucoma and other disorders of the optic nerve, such as idiopathic intracranial
hypertension. It plays little rote in the diagnosis of AMO.

CT scan of the head (Choice D) is useful for identification of orbital pathology, including mass lesions and fractures of the orbital bones. While this patient's irregular right pupil may suggest an orbital mass
lesion, it more likely represents postsurgical changes to the pupil following cataract surgery. Orbital masses are typically accompanied by other signs and symptoms, such as proptosis, ptosis, or diplopia. CT
scan of the head plays no role in the diagnosis of AMO.

Temporal artery biopsy (Choice E) is useful for the diagnosis of temporal arteritis, a small-vessel vasculitis with a predilection for the arteries of the head and neck that may lead to acute, irreversible
blindness. Temporal arteritis typically presents in patients older than age 65 with constitutional symptoms of fever, chills, and weight loss, as well as scalp tenderness, new headaches, jaw claudication, and
chest pain. Temporal arteritis may present concomitantly with polymyalgia rheumatica. Suspicion of temporal arteritis should lead to the immediate initiation of high-dose corticosteroid therapy.

https://t.me/USMLENBME2CK ts e t
Previous Next Score Report Lab Values Calculator Help pause
Exam Section 2: ltem 18 of 50 National Board of Medical Examiners
Comprehensive Clinical Science Self-Assessment

A) Dilated funduscopic examination


B) Slit-lamp examination
C) Visual field testing
D) CT scan of the head
E) Temporal artery biopsy
Correct Answer: A.

Age-related macular degeneration (AMO) is an acquired retinal degeneration characterized by dysfunction of the retinal pigment epithelium and accumulation of macular drusen, the hallmark of AMO. Drusen
represent yellow, subretinal, extracellular deposits of lipoproleinaceous material. The underlying cause is not completely understood, but a combination of environmental and multiple genetic risk factors is
likely involved, including genes associated with complement regulation and autophagy. AMO is common and is a leading cause of nonreversible blindness in developed countries. Early stages of the disease
may be asymptomatic or characterized by insidious blurring or metamorphopsia (a visual distortion in which straight lines appear curved). In advanced stages, profound vision loss may result from retinal
geographic atrophy or choroidal neovascularization with associated macular edema, exudation, or hemorrhage. Risk factors for AMO include age, smoking, and genetics. The diagnosis is established by a
dilated funduscopic examination to evaluate the retina. Antioxidant and vitamin supplementation are helpful for delaying the progression of nonexudative (dry) AMO, while exudative (wet) AMO is treated with
intravitreal inJections of monoclonal antibodies against vascular endothelial growth factor.

Incorrect Answers: B, C, D, and E.

Slit-lamp examination (Choice B) is a method for examining ocular structures under high magnification. This technique may be used to examine structures within the anterior segment of the eye, such as the
cornea, iris, or lens, or within the posterior segment (including the retina and optic nerve), usually as part of a dilated funduscopic examination.

Visual field testing (Choice C) is a method for detecting visual scotomas or visual field loss. It is most useful in the diagnosis of glaucoma and other disorders of the optic nerve, such as idiopathic intracranial
hypertension. II plays little role in the diagnosis of AMO.

CT scan of the head (Choice D) is useful for identification of orbital pathology, including mass lesions and fractures of the orbital bones. While this patient's irregular right pupil may suggest an orbital mass
lesion, it more likely represents postsurgical changes to the pupil following cataract surgery. Orbital masses are typically accompanied by other signs and symptoms, such as proptosis, ptosis, or diplopia. CT
scan of the head plays no role in the diagnosis of AMO.

Temporal artery biopsy (Choice E) is useful for the diagnosis of temporal arteritis, a small-vessel vasculitis with a predilection for the arteries of the head and neck that may lead to acute, irreversible
blindness. Temporal arteritis typically presents in patients older than age 65 with constitutional symptoms of fever, chills, and weight loss, as well as scalp tenderness, new headaches, jaw claudication, and
chest pain. Temporal arteritis may present concomitantly with polymyalgia rheumatica. Suspicion of temporal arteritis should lead to the immediate initiation of high-dose corticosteroid therapy.

Educational Objective: Age-related macular degeneration (AMO) is an acquired retinal degeneration characterized by dysfunction of the retinal pigment epithelium and accumulation of macular drusen. Age
is the most important risk factor for AMO, along with smoking and genetics. Early stages of the disease may be asymptomatic or characterized by insidious blurring or metamorphopsia. In advanced stages,
vision loss may result from retinal geographic atrophy or choroidal neovascularization with macular edema, exudation, or hemorrhage. The diagnosis is established with a dilated funduscopic examination.

https://t.me/USMLENBME2CK ts e t
Previous Next Score Report Lab Values Calculator Help pause
Exam Section 2: Item 19 of 50 National Board of Medical Examiners
Comprehensive Clinical Science Self-Assessment

V 4g A27-year-old woman comes to the office for a health maintenance examination. She says she feels well. She has no history of serious illness and takes no medications. Vital signs are within normal
limits. Examination shows no lymphadenopathy. A 2-cm nodule is palpated in the left lobe of the thyroid gland. The remainder of the examination shows no abnormalities. Results of laboratory studies,
including measurement of serum thyroid-stimulating hormone concentration, are within the reference ranges. Which of the following is the most appropriate next step in management?

A) 131] scintigraphy
B)Levothyroxine therapy and reexamination in 6 weeks
C) Reexamination of the thyroid gland in 6 months
D) Repeat measurement of serum thyroid-stimulating hormone concentration in 6 weeks
E) Ultrasonography-guided fine-needle aspiration biopsy
Correct Answer: E.

Discovery of a palpable thyroid mass by a patient at home or during a routine clinical visit is a common occurrence, and the diagnostic workup focuses on ruling out malignancy. Risk factors for thyroid
malignancy include young age (the prevalence of benign thyroid nodules increases with age), radiation exposure to the head or neck, and a family history of thyroid cancer or associated congenital neoplastic
syndrome (eg, multiple endocrine neoplasia, type 2). All patients with a new nodule should be evaluated with serum TSH measurement and thyroid ultrasonography. Ultrasonography-guided fine-needle
aspiration biopsy can be performed at the same time, which is recommended for patients with risk factors for malignancy or for nodules with concerning ultrasound features. Subsequent workup and
management are dependent upon the results of the TSH measurement, ultrasound characteristics of the nodule, and biopsy results.

Incorrect Answers: A, B, C, and D.

1314 scintigraphy (Choice A), also known as a radioactive iodine uptake scan, is recommended if the TSH concentration is low, indicating primary hyperthyroidism, as hyperfunctioning nodules are rarely
malignant and do not require fine-needle aspiration biopsy. The uptake pattern guides further management.

Levothyroxine therapy and reexamination in 6 weeks (Choice B), reexamination of the thyroid gland in 6 months (Choice C), and repeat measurement of serum thyroid-stimulating hormone concentration in 6
weeks (Choice D) are not appropriate management options given the importance of ruling out malignancy, especially in a young patient.

Educational Objective: Palpable thyroid masses are a common incidental discovery on physical examination and imaging. Diagnostic workup is necessary to exclude malignancy. All patients should have their
TSH concentration checked and ultrasonography of the thyroid gland performed. Fine-needle aspiration biopsy is indicated if the TSH is normal or increased, the patient has risk factors for thyroid malignancy,
or the nodule has concerning features on ultrasonography.

https://t.me/USMLENBME2CK ti
Previous Next Score Report Lab Values Calculator Help pause
Exam Section 2: Item 20 of 50 National Board of Medical Examiners°
Comprehensive Clinical Science Self-Assessment

✓ 20. A17-year-old boy is brought to the emergency department by his mother because of a 20-minute episode of severe chest pain and sweating
that started 1 hour ago while he was watching television. On arrival, he is alert and repeatedly expresses concern that something is "very I aVR
wrong." He reports that during the episode, his heart was "pounding," he felt short of breath, and he was sure he was having a heart attack
and was going to die. One month ago, he had a similar episode that occurred in the evening 30 minutes after his high school football
practice. Al that lime, results of laboratory studies and an ECG obtained in the emergency department showed no abnormalities. After the
initial episode, he quit the football team, and he has continued to limit his physical activity because he is afraid of bringing on another
episode. He reports no insomnia or changes in appetite. His mother states that he was diagnosed with an innocent heart murmur at the age 11 aVL
of 6 months, but it has not been noted on examinations since the age of 1 year. His paternal grandfather died of a myocardial infarction
3 years ago at the age of 68 years. The patient takes no medications. He does not smoke cigarettes, drink alcohol, or use illicit drugs. He
performs well in school. He is 180 cm (5 ft 11 in) tall and weighs 73 kg (161 lb); 8MI is 23 kg/m2. His temperature is 37.0"C (98.6°F), and
jg4 EE pg[EA pg
blood pressure is 130/83 mm Hg. No murmurs are heard on cardiac examination. The remainder of the examination shows no abnormalities.
ECG is shown. Which of the following is the most likely diagnosis? pp
�,..!ill!!! .l!!!l!r I aVF
A) Acute stress disorder iii.lliis
8) Delusional disorder
C) Generalized anxiety disorder
D) Hypertrophic cardiomyopathy
E) Idiopathic recurrent pericarditis
F) Panic disorder
G) Paroxysmal supraventricular tachycardia
Correct Answer: G.

Paroxysmal supraventricular tachycardia (PSVT) refers to an abnormally rapid heart rate and rhythm originating from above the ventricles, typically with a regular rhythm and a narrow ORS complex on
ECG. Patients with PSVT can manifest with acute symptoms of lightheadedness, palpitations, diaphoresis, and syncope. PSVT is typically caused by structural defects in the His-Purkinje system causing a
re-entrant pathway, such as in Wolff-Parkinson-White syndrome, atrioventricular nodal tachycardia, or atriovenlricular nodal re-entrant tachycardia. Or PSVT may be provoked by alcohol, nicotine, caffeine,
or psychological stress. In patients who are hemodynamically unstable, cardioversion is indicated in the management of narrow complex tachycardia. For patients who are hemodynamically stable,
attempts can be made to terminate the narrow complex tachyarrhythmia by vagal maneuvers (eg, ice or cold water over the face), or with adenosine or nodal blocking agents.

Incorrect Answers: A, 8, C, D, E, and F.

Acute stress disorder (Choice A) refers to symptoms of intrusive thoughts, hyperarousal, dissociation, avoidance, and/or depressed mood within 1 month of a traumatic event such as a life-threatening
motor vehicle collision.

Delusional disorder (Choice 8) features the presence of one or more delusions for 1 month or longer with no other psychotic symptoms such as hallucinations, thought disorder, psychosomatic symptoms,
or mood disorder. Delusions are characterized by a persistent idea or belief that can have undue influence on a patient's life.

« https://t.me/USMLENBME2CK ts e t
Previous Next Score Report Lab Values Calculator Help pause
Exam Section 2: ltem 20 of 50 National Board of Medical Examiners
Comprehensive Clinical Science Self-Assessment

D) Hypertrophic cardiomyopathy
E) Idiopathic recurrent pericarditis
F) Panic disorder
G) Paroxysmal supraventricular tachycardia

Correct Answer: G.

Paroxysmal supraventricular tachycardia (PSVT) refers to an abnormally rapid heart rate and rhythm originating from above the ventricles, typically with a regular rhythm and a narrow ORS complex on
ECG. Patients with PSVT can manifest with acute symptoms of lightheadedness, palpitations, diaphoresis, and syncope. PSVT is typically caused by structural defects in the His-Purkinje system causing a
re-entrant pathway, such as in Wolff-Parkinson-White syndrome, atrioventricular nodal tachycardia, or atrioventricular nodal re-entrant tachycardia. Or PSVT may be provoked by alcohol, nicotine, caffeine,
or psychological stress. In patients who are hemodynamically unstable, cardioversion is indicated in the management of narrow complex tachycardia. For patients who are hemodynamically stable,
attempts can be made to terminate the narrow complex tachyarrhythmia by vagal maneuvers (eg, ice or cold water over the face), or with adenosine or nodal blocking agents.

Incorrect Answers: A, B, C, D, E, and F.

Acute stress disorder (Choice A) refers to symptoms of intrusive thoughts, hyperarousal, dissociation, avoidance, and/or depressed mood within 1 month of a traumatic event such as a life-threatening
motor vehicle collision.

Delusional disorder (Choice B) features the presence of one or more delusions for 1 month or longer with no other psychotic symptoms such as hallucinations, thought disorder, psychosomatic symptoms,
or mood disorder. Delusions are characterized by a persistent idea or belief that can have undue influence on a patient's life.

Generalized anxiety disorder (Choice C) features excessive and persistent worrying that may be associated with sympathetic hyperactivity.

Hypertrophic cardiomyopathy (Choice D) classically presents with dyspnea, chest pain, syncope, or sudden cardiac death in an exercising young athlete with a potential family history of a similar event.
Physical examination findings include a systolic murmur loudest in the lower left sternal border, which is quieter with increased preload and afterload, and a laterally displaced point of maximal impulse.

Idiopathic recurrent pericarditis (Choice E) refers to repeated episodes of pericarditis, with a repeat episode recurring about 6 weeks after the resolution of the prior episode. Pericarditis classically presents
with substernal chest pain that is often pleuritic, worsens when lying down, and improves with leaning forward. On examination, pericarditis discloses a diastolic friction rub, described as a harsh sound
heard in diastole.

Panic disorder (Choice F) is characterized by recurrent panic attacks that are unexpected and associated with worry about future panic attacks or avoidance of panic attack triggers. Panic attacks feature
acute fear or anxiety that peaks within minutes and is associated with symptoms of sympathetic hyperactivity, such as tachycardia, heart palpitations, sweating, and trembling along with shortness of
breath, chest pain, dizziness, gastrointestinal distress, and paresthesias.

Educational Objective: Paroxysmal supraventricular tachycardia (PSVT) refers to an abnormally rapid heart rate and rhythm originating from above the ventricles, typically with a narrow ORS complex on
ECG. PSVT is typically caused by structural defects in the His-Purkinje system causing a re-entrant pathway.

https://t.me/USMLENBME2CK ti
Previous Next Score Report Lab Values Calculator Help pause
Exam Section 2: Item 21 of 50 National Board of Medical Examiners
Comprehensive Clinical Science Self-Assessment

"I 21. A72-year-old man comes to the physician for a routine health maintenance examination. Abdominal examination shows a nontender, pulsatile mass in the epigastrium. Abdominal ultrasonography shows
a 7.2-cm aneurysm. The physician reviews several studies of patients with abdominal aneurysms that are greater than 7 cm in size. The rupture rate is estimated to be 20% per year. Following rupture of
an abdominal aneurysm, one half of patients die before reaching a hospital. For patients who do reach a hospital, the perioperative mortality rate is approximately 50%. The mortality rate for patients who
undergo elective repair of an aneurysm is 5%. After discussing the risks and benefits of surgical repair of the aneurysm, the patient decides not to undergo an operation. Which of the following best
represents this patient's risk for mortality from rupture of the aneurysm during the next year?

A) 5%
B) 10%
C) 15%
0) 20%
E) 50%
Correct Answer: C.

Correct statistical interpretation is important for conveying accurate information to patients. In this scenario, a study demonstrates a 20% annual rate of rupture for abdominal aortic aneurysms greater than 7
cm in diameter. Among ruptured aneurysms, the study identified a 50% mortality rate prior lo hospital evaluation, in addition to a 50% perioperative mortality rate among the remaining 50% of patients who do
reach a hospital. In effect, 75% (50% prehospital, and half of the remaining 50% who survived to surgery) of those who experience aneurysmal rupture will die; however, the risk for rupture is 20% per year,
representing a 15% overall risk of mortality from aneurysmal rupture in a given year. This is represented by the calculation: (10.5 (prehospital risk of death) + 10.50.5 (perioperative mortality))0.2 (annual
risk for rupture)= 0.15.

Incorrect Answers: A, B, 0, and E.

5% (Choice A) is incorrect, and represents an incorrect mathematical calculation: 0.20 x 0.5 x 0.5 = 0.05, or 5%.

10% (Choice B) is incorrect, and represents an incorrect mathematical calculation: 0.20 x 0.5 = 0.1, or 10%. This calculation fails to include perioperative mortality.

20% (Choice 0) is incorrect, and represents the annual rate of aneurysmal rupture.

50% (Choice E) is incorrect, and represents the mortality rate prior to hospital admission among patients with ruptured aneurysms.

Educational Objective: Correct statistical interpretation is important for conveying accurate information to patients. In this scenario, 75% of those who experience aneurysmal rupture will die, representing a
15% overall risk of mortality from aneurysmal rupture in a given year.

https://t.me/USMLENBME2CK ti
Previous Next Score Report Lab Values Calculator Help pause
Exam Section 2: Item 22 of 50 National Board of Medical Examiners
Comprehensive Clinical Science Self-Assessment

"I 22. A 58-year-old man with an 8-year history of schizoaffective disorder, manic type, is brought to the clinic for a follow-up examination. Two weeks ago, he was hospitalized for a manic episode.
Risperidone was added to his medication regimen; the dosage was increased to the maximum recommended prior to discharge. At that time, neurologic examination showed mild cogwheel rigidity.
There was no evidence of tremors. Muscle strength was 5/5. He has no other history of serious medical or psychiatric illness. His other medications are valproic acid and clonazepam. Today,
examination shows drooling, masked facies, and decreased blinking. He has a slow, shuffling gait with decreased arm swing. The remainder of the examination shows no abnormalities. Mental status
examination shows normal speech and a mildly anxious mood. There is no evidence of psychosis. Which of the following is the most appropriate next step in management?

A) Addition of diphenhydramine to the medication regimen


B) Addition of lithium carbonate to the medication regimen
C) Cognitive behavioral therapy
D) Decreasing the dosage of risperidone
E) Increasing the dosage of clonazepam
F) Psychotherapy
Correct Answer: D.

Risperidone is an antipsychotic that is an antagonist of the dopamine receptor, which blocks dopamine signaling nonspecifically across different brain areas. Blocking dopamine signaling in the limbic system
decreases psychotic symptoms, while blocking dopamine signaling in the basal ganglia leads lo medication-induced parkinsonism, including bradykinesia, tremor, and cogwheel rigidity. In patients with
extrapyramidal adverse effects, decreasing the dosage is necessary to reduce the adverse effects, and cessation of the offending drug may be necessary.

Incorrect Answers: A, B, C, E, and F.

Addition of diphenhydramine to the medication regimen (Choice A) can help with extra pyramidal adverse effects associated with antipsychotics, such as dystonia, akathisia, and torticollis. The most
appropriate step is to decrease the dose of risperidone to help reduce parkinsonism symptoms as diphenhydramine would only offer transient benefit, if any, in the case of such subacute to late-presenting
symptoms.

Addition of lithium carbonate to the medication regimen (Choice B) is an option to treat bipolar disorder. While this patient has manic episodes, this patient's symptoms are caused by adverse effects of
anlidopaminergic medications, and the addition of lithium would not be appropriate at this time.

Cognitive behavioral therapy (Choice C) would be appropriate in many cases of major depressive disorder, anxiety disorder, specific phobias, illness anxiety disorders, and other nonpsychotic diseases. In
this case, the patient's symptoms are caused by an adverse effect of an antipsychotic medication, and cognitive behavioral therapy would not play an immediate role.

Increasing the dosage of clonazepam (Choice E) and psychotherapy (Choice F) would not help treat any extrapyramidal adverse effects of risperidone or other antipsychotics.

Educational Objective: Most antipsychotic medications antagonize dopamine receptors and therefore decrease psychotic symptoms while also leading to medication-induced parkinsonism and other
extrapyramidal effects, such as dystonia, akathisia, and tardive dyskinesia. While mediations such as diphenhydramine and benzodiazepines may be useful for acute management, decreasing the dose of
the antipsychotic or changing agents is most appropriate.

https://t.me/USMLENBME2CK ts e t
Previous Next Score Report Lab Values Calculator Help pause
Exam Section 2: Item 23 of 50 National Board of Medical Examiners
Comprehensive Clinical Science Self-Assessment

A67-year-old man with metastatic lung cancer comes to the physician because of malaise and lethargy during the past 3 days. Medications are ibuprofen and lisinopril. He appears cachectic but is
oriented to person, place, and time. His temperature is 37°C (98.6°F), pulse is 102/min, respirations are 13/min, and blood pressure is 105/75 mm Hg. Examination shows decreased skin turgor. The
remainder of the examination shows no abnormalities. Laboratory studies show:
Hemoglobin 10.6 g/dL
Hematocrit 36%
Leukocyte count 11,100/mm3
Platelet count 226,000/mm3
Serum
Na 136 mEq/L
K' 3.6 mEq/L
Cl- 96 mEq/L
HCO 3 28 mEq/L
Ca? 11.8 mg/dL
Urea nitrogen 22 mg/dL
Glucose 98 mg/dL
Creatinine 0.9 mg/dL
Phosphorus 2.5 mg/dL
Albumin 2.9 g/dL

Which of the following is the most appropriate initial pharmacotherapy?

A) Calcitonin
B) Hydrocortisone
C) Plicamycin
0) 0.9%Saline
E) Zoledronic acid
Correct Answer: D.

Lung cancer is often complicated by hypercalcemia caused by paraneoplastic parathyroid hormone-related peptide (PTHrP) production. This phenomenon frequently occurs in the setting of squamous cell
lung cancer and may also occur in squamous cell carcinoma of the head and neck, as well as breast and kidney cancers. PTHrP acts similarly to parathyroid hormone, causing an increase in osteoclast
activity via G protein-coupled receptors. It causes increased absorption of calcium in the intestine and renal tubules. Bone pain is a common manifestation. Other symptoms include abdominal discomfort,
muscle pain and weakness, polyuria, and psychiatric disturbances. Hypercalcemia is associated with hypophosphatemia (caused by renal wasting of phosphate), subperiosteal bone resorption,
nephrolithiasis, and end-stage kidney disease. Management is determined by the severity of calcium imbalance and symptoms. Patients with mild hypercalcemia (<12 mg/dL) with minimal or no symptoms
may not require treatment. This patient, with an albumin-corrected calcium concentration of >12 mg/dL and altered mental status, requires treatment. Immediate treatment is with intravenous normal saline.
This may be followed by the administration of calcitonin and a bisphosphonate, such as zoledronic acid. Patients should be monitored closely to evaluate for volume status and electrolyte disturbances.

https://t.me/USMLENBME2CK ts e t
Previous Next Score Report Lab Values Calculator Help pause
Exam Section 2: Item 23 of 50 National Board of Medical Examiners
Comprehensive Clinical Science Self-Assessment

Creatinine 0.9 mg/dL


Phosphorus 2.5 mg/dL
Albumin 2.9 g/dL

Which of the following is the most appropriate initial pharmacotherapy?

A) Calcitonin
B) Hydrocortisone
C) Plicamycin
0) 0.9%Saline
E) Zoledronic acid
Correct Answer: D.

Lung cancer is often complicated by hypercalcemia caused by paraneoplastic parathyroid hormone-related peptide {PTHrP) production. This phenomenon frequently occurs in the setting of squamous cell
lung cancer and may also occur in squamous cell carcinoma of the head and neck, as well as breast and kidney cancers. PTHrP acts similarly to parathyroid hormone, causing an increase in osteoclast
activity via G protein-coupled receptors. It causes increased absorption of calcium in the intestine and renal tubules. Bone pain is a common manifestation. Other symptoms include abdominal discomfort,
muscle pain and weakness, polyuria, and psychiatric disturbances. Hypercalcemia is associated with hypophosphatemia (caused by renal wasting of phosphate), subperiosteal bone resorption,
nephrolithiasis, and end-stage kidney disease. Management is determined by the severity of calcium imbalance and symptoms. Patients with mild hypercalcemia (<12 mg/dL) with minimal or no symptoms
may not require treatment. This patient, with an albumin-corrected calcium concentration of >12 mg/dL and altered mental status, requires treatment. Immediate treatment is with intravenous normal saline.
This may be followed by the administration of calcitonin and a bisphosphonate, such as zoledronic acid. Patients should be monitored closely to evaluate for volume status and electrolyte disturbances.

Incorrect Answers: A, B, C, and E.

Calcitonin (Choice A) and zoledronic acid (Choice E) are useful for the acute management of hypercalcemia but should be initiated after intravenous volume resuscitation. Calcitonin inhibits osteoclast
activity and reabsorption of calcium within the renal tubules. Zoledronic acid is a bisphosphonate that also inhibits osteoclast activity.

Hydrocortisone (Choice B) is a glucocorticoid that is useful for the management of Addisonian crisis in the setting of adrenal insufficiency. It does not play a role in the management of hypercalcemia
associated with malignancy.

Plicamycin (Choice C) is an antibiotic that is effective in the management of hypercalcemia caused by the inhibition of osteoclasts. Plicamycin is rarely used because of its adverse effects, including
thrombocytopenia, abnormal bleeding, hepatotoxicity, and renal toxicity.

Educational Objective: Lung cancer, typically squamous cell, is associated with hypercalcemia caused by parathyroid hormone-related peptide production. Symptoms of hypercalcemia include bone pain,
muscle weakness, central nervous system and psychiatric disturbances, abdominal discomfort, polyuria, and renal calculi formation. Initial treatment is with intravenous fluid resuscitation.

https://t.me/USMLENBME2CK ts e t
Previous Next Score Report Lab Values Calculator Help pause
National Board of Medical Examiners
Comprehensive Clinical Science Self-Assessment

"I 24. A 4 7-year-old man is brought to the emergency department because of severe left groin pain since he fell 2 hours ago. He is unable to walk because of the pain. He does not have pain in other joints.
He has a 20-year history of ulcerative colitis. His only medication is daily prednisone. On examination, any attempts at range of motion of the left hip elicit pain. Muscle strength is intact distally and in
the right lower extremity. Distal pulses are 2+. An x-ray of the pelvis and left lower hip shows moderate loss of joint space and diffuse osteopenia. Which of the following is the most appropriate next
step in management?

A) Observation only
B) DEXA scan
C) MRI of the hip
D) Nonsteroidal anti-inflammatory drug therapy
E) Aspiration of the hip joint
F) Total hip arthroplasty
Correct Answer: C.

Osteoporosis and osteopenia are metabolic bone diseases characterized by diffusely low bone density. The most common risk factors are age, smoking, chronic inflammatory disease, corticosteroid use,
and estrogen-depleting conditions in women. Such demineralization and low bone density is often asymptomatic but increases the risk for fractures, most commonly involving the hip, vertebrae, and distal
radius. In this patient, diffuse osteopenia can limit the appearance and detection of fractures on x-rays. An occult hip fracture is likely in the setting of osteopenia or osteoporosis, and advanced imaging, such
as an MRI of the hip, is the most appropriate next step in management. Calcium and vitamin D supplementation are the first line for treatment and prevention of osteoporosis. Reduction of active
osteoporotic risk factors should also be pursued. Weight-bearing exercises are recommended to slow deterioration of bone mineral density. Bisphosphonates inhibit osteoclast activity. They are utilized once
an individual is diagnosed with osteoporosis to reduce further loss of bone mineral density.

Incorrect Answers: A, B, D, E, and F.

Observation only (Choice A) would not be appropriate, as this patient has severe uncontrolled pain in the setting of trauma, raising suspicion for a fracture. If a hip fracture is present, he would need urgent
surgical management.

DEXA scan (Choice B) is used to evaluate for osteoporosis, which is defined as a bone mineral density less than 2.5 standard deviations compared to a young, normal population (T-score). It can be useful
as screening and for serial measurements in the outpatient setting. However, in a patient with suspicion of a hip fracture, a DEXA scan is not appropriate.

Nonsteroidal anti-inflammatory drug therapy (Choice 0) would be appropriate in the management of hip strain or sprain. This patient's inability to walk and traumatic pain in the setting of osteoporosis is more
likely the result of a hip fracture.

Aspiration of the hip joint (Choice E) would be indicated if there was suspicion for a septic arthritis of the hip. Septic arthritis typically presents with atraumatic pain of a joint, fever, and inability to range the
hip. This patient's hip pain with range of motion in the setting of trauma is more likely the result of a hip fracture.

Total hip arthroplasty (Choice F) may be necessary for this patient but would be premature prior to the diagnosis of a hip fracture. Advanced imaging, such as MRI, should be pursued to evaluate for
traumatic patholoav in this patient.

https://t.me/USMLENBME2CK ts e t
Previous Next Score Report Lab Values Calculator Help pause
Exam Section 2: Item 24 0f 50 National Board of Medical Examiners
Comprehensive Clinical Science Self-Assessment

A) Observation only
B) DEXA scan
C) MRI of the hip
D) Nonsteroidal anti-inflammatory drug therapy
E) Aspiration of the hip joint
F) Total hip arthroplasty
Correct Answer: C.

Osteoporosis and osteopenia are metabolic bone diseases characterized by diffusely low bone density. The most common risk factors are age, smoking, chronic inflammatory disease, corticosteroid use,
and estrogen-depleting conditions in women. Such demineralization and low bone density is often asymptomatic but increases the risk for fractures, most commonly involving the hip, vertebrae, and distal
radius. In this patient, diffuse osteopenia can limit the appearance and detection of fractures on x-rays. An occult hip fracture is likely in the setting of osteopenia or osteoporosis, and advanced imaging, such
as an MRI of the hip, is the most appropriate next step in management. Calcium and vitamin D supplementation are the first line for treatment and prevention of osteoporosis. Reduction of active
osteoporotic risk factors should also be pursued. Weight-bearing exercises are recommended to slow deterioration of bone mineral density. Bisphosphonates inhibit osteoclast activity. They are utilized once
an individual is diagnosed with osteoporosis to reduce further loss of bone mineral density.

Incorrect Answers: A, B, D, E, and F.

Observation only (Choice A) would not be appropriate, as this patient has severe uncontrolled pain in the setting of trauma, raising suspicion for a fracture. If a hip fracture is present, he would need urgent
surgical management.

DEXA scan (Choice B) is used to evaluate for osteoporosis, which is defined as a bone mineral density less than 2.5 standard deviations compared to a young, normal population (T-score). It can be useful
as screening and for serial measurements in the outpatient setting. However, in a patient with suspicion of a hip fracture, a DEXA scan is not appropriate.

Nonsteroidal anti-inflammatory drug therapy (Choice D) would be appropriate in the management of hip strain or sprain. This patient's inability to walk and traumatic pain in the setting of osteoporosis is more
likely the result of a hip fracture.

Aspiration of the hip joint (Choice E) would be indicated if there was suspicion for a septic arthritis of the hip. Septic arthritis typically presents with atraumatic pain of a joint, fever, and inability to range the
hip. This patient's hip pain with range of motion in the setting of trauma is more likely the result of a hip fracture.

Total hip arthroplasty (Choice F) may be necessary for this patient but would be premature prior to the diagnosis of a hip fracture. Advanced imaging, such as MRI, should be pursued to evaluate for
traumatic pathology in this patient.

Educational Objective: Osteoporosis is often asymptomatic but increases the risk for fractures, most commonly involving the hip, vertebrae, and distal radius. Osteopenia can limit the detection of occult
fractures on x-rays, and when they are suspected, advanced imaging with MRI or CT scan should be pursued.

https://t.me/USMLENBME2CK ts e t
Previous Next Score Report Lab Values Calculator Help pause
Exam Section 2: Item 25 of 50 National Board of Medical Examiners
Comprehensive Clinical Science Self-Assessment

✓ 25. A 74-year-old woman is brought to the clinic by her family because of a 7-month history of progressive confusion and urinary incontinence. During the past year, she has had multiple falls; she
sustained wrist fractures 1 year ago and a left tibia fracture 3 months ago. She has hypertension, osteoporosis, and depression. She is evasive when asked how much alcohol she drinks. She
appears frail. Vital signs are within normal limits. Physical examination shows a wide-based gait and marked ataxia. She is unable to attempt the Romberg test. Cranial nerve examination shows
nystagmus and an inability to move her eyes horizontally. On cognitive testing, she recalls zero of three objects after 5 minutes. Laboratory studies show:
Hemoglobin 10 g/dl
Mean corpuscular volume 102 m?
Serum
Vitamin B, (cobalamin) 980 pg/mL (N=160--950)
Folate 3.1 ng/mL (N=2.7-17)

Which of the following is the most likely diagnosis?

A) Folic acid deficiency


B) Lewy body dementia
C) Normal-pressure hydrocephalus
0) Progressive supranuclear palsy
E) Wernicke-Korsakoff syndrome
Correct Answer: E.

Thiamine, or vitamin B,, deficiency causes Wernicke encephalopathy and Korsakoff dementia. The classic triad of Wernicke encephalopathy presents with encephalopathy, ataxia, and oculomotor
dysfunction (lateral gaze palsy and nystagmus). Wernicke encephalopathy is reversible with treatment but, if allowed to progress, may result in Korsakoff dementia. Korsakoff dementia is an irreversible
condition that includes psychosis, anterograde and retrograde amnesia, and the consequent tendency to confabulate, or verbalize false stories or memories, to compensate for the inability to remember.
Thiamine deficiency is common in patients with alcohol use disorder because of poor nutritional intake, thiamine malabsorption, and impaired cellular utilization of thiamine. MRI may show atrophy of the
mammillary bodies. To prevent Wernicke-Korsakoff syndrome, all patients with sustained heavy alcohol use should be given thiamine supplementation, and patients presenting with acute intoxication
should generally be given parenteral thiamine. While serum thiamine concentration may be decreased, this is not useful in management as it cannot be checked quickly. Thus, the diagnosis is clinical and
should be rapidly treated with thiamine administration.

Incorrect Answers: A, B, C, and D.

Folic acid deficiency (Choice A) presents with megaloblastic anemia and is often seen in patients with malnutrition, alcohol use disorder, and patients taking antifolate medications (eg, phenytoin,
methotrexate).

Lewy body dementia (Choice B) is characterized by fluctuating alterations in cognitive function, parkinsonism, visual hallucinations, and REM sleep behavior disorder.

Normal-pressure hydrocephalus (Choice C) is a type of communicating (nonobstructive) hydrocephalus in which the ventricles are enlarged, leading to damage of the white matter tracts connecting the
+ 4% 4 ,4% ,e

https://t.me/USMLENBME2CK ts e t
Previous Next Score Report Lab Values Calculator Help pause
Exam Section 2: Item 25 of 50 National Board of Medical Examiners
Comprehensive Clinical Science Self-Assessment

Which of the following is the most likely diagnosis?

A) Folic acid deficiency


B) Lewy body dementia
C) Normal-pressure hydrocephalus
D) Progressive supranuclear palsy
E) Wernicke-Korsakoff syndrome
Correct Answer: E.

Thiamine, or vitamin B,, deficiency causes Wernicke encephalopathy and Korsakoff dementia. The classic triad of Wernicke encephalopathy presents with encephalopathy, ataxia, and oculomotor
dysfunction (lateral gaze palsy and nystagmus). Wernicke encephalopathy is reversible with treatment but, if allowed to progress, may result in Korsakoff dementia. Korsakoff dementia is an irreversible
condition that includes psychosis, anterograde and retrograde amnesia, and the consequent tendency to confabulate, or verbalize false stories or memories, to compensate for the inability to remember.
Thiamine deficiency is common in patients with alcohol use disorder because of poor nutritional intake, thiamine malabsorption, and impaired cellular utilization of thiamine. MRI may show atrophy of the
mammillary bodies. To prevent Wernicke-Korsakoff syndrome, all patients with sustained heavy alcohol use should be given thiamine supplementation, and patients presenting with acute intoxication
should generally be given parenteral thiamine. While serum thiamine concentration may be decreased, this is not useful in management as it cannot be checked quickly. Thus, the diagnosis is clinical and
should be rapidly treated with thiamine administration.

Incorrect Answers: A, B, C, and D.

Felic acid deficiency (Choice A) presents with megaloblastic anemia and is often seen in patients with malnutrition, alcohol use disorder, and patients taking antifolate medications (eg, phenytoin,
methotrexate)

Lewy body dementia (Choice B) is characterized by fluctuating alterations in cognitive function, parkinsonism, visual hallucinations, and REM sleep behavior disorder.

Normal-pressure hydrocephalus (Choice C) is a type of communicating (nonobstructive) hydrocephalus in which the ventricles are enlarged, leading to damage of the white matter tracts connecting the
frontal lobes and basal ganglia. It presents with the triad of cognitive impairment, urinary incontinence, and gait instability.

Progressive supranuclear palsy (Choice D) is a parkinsonian syndrome that typically presents with gait instability, oculomotor deficits (classically, vertical gaze palsy), frontal lobe deficits, bradykinesia, and
muscle rigidity. Nystagmus and lateral gaze palsy would be atypical.

Educational Objective: Chronic alcohol use disorder and associated malnutrition may result in thiamine deficiency, which can cause Wernicke-Korsakoff syndrome. The syndrome is characterized by altered
mental status, ophthalmoplegia, ataxia, psychosis, and anterograde and retrograde amnesia. Thiamine supplementation can prevent the development of Wernicke-Korsakoff syndrome in patients with
alcohol use disorder.

https://t.me/USMLENBME2CK ts e t
Previous Next Score Report Lab Values Calculator Help pause
Exam Section 2: ltem 26 of 50 National Board of Medical Examiners
Comprehensive Clinical Science Self-Assessment

Eight days after he was involved in a motor vehicle collision in which he was the restrained driver, a hospitalized 28-year-old man has hypertension. He underwent extensive reconstructive orthopaedic
operations for injuries to his left clavicle, shoulder, and femur and exploratory laparotomy for a ruptured spleen. Urine toxicology screening on admission was positive for °-tetrahydrocannabinol,
cocaine, and opiates. He was intubated and mechanically ventilated during the procedures and was extubated on hospital day 4. He receives oral acetaminophen-oxycodone as needed. His blood
pressure has been 180/110 mm Hg; it decreases to 150/80 mm Hg when he receives his pain medication. An abdominal bandage covers the surgical incision. The nurse tells the physician that the
patient rates his pain as a 10on a 10-point scale and is constantly asking for pain medications. Which of the following is the most appropriate next step in pharmacotherapy for this patient's
hypertension?

A) Continued administration of acetaminophen-oxycodone every 4 hours


B) Continuous intravenous labetalol infusion with titration
C) Intravenous nitroprusside therapy
D) Patient-controlled morphine
E) Switching from acetaminophen-oxycodone to lramadol
Correct Answer: D.

Postoperative pain control is a vital component of caring for the surgical patient. Inadequately treated pain can lead to hypertension and patient dissatisfaction as well as delayed recovery caused by
decreased ambulation and increased hospital length of stay. This can be exceptionally challenging in patients taking opioids prior to surgery as they often have an increased tolerance to opioids and require
larger doses than the opioid-naive patient. The classic approach to perioperative pain control includes a multimodal technique, including acetaminophen, local anesthetics, nonsteroidal anti-inflammatory
drugs, and oral and intravenous opioids. Patient-controlled analgesia is a method of providing pain control frequently used in the opioid-tolerant population or for those with significant postoperative pain. It
involves programming a pump to deliver a small bolus dose of parenteral opioid in response to the patient pressing a button with a lockout interval and maximum dose to prevent overdose. This allows the
patient to control the frequency of analgesic medication rather than be reliant on the nurse for pain medication distribution and improves patient satisfaction. Additionally, it allows the practitioner to calculate
a total opioid requirement that may be utilized to transition the patient to an effective oral regimen. This patient has hypertension likely related to his ongoing significant pain in the setting of previous opioid
use, given that his blood pressure improves after pain medication administration. Therefore, the most appropriate pharmacotherapy to improve his hypertension should be aimed at treating his pain.

Incorrect Answers: A, B, C, and E.

Continued administration of acetaminophen-oxycodone every 4 hours (Choice A) may improve this patient's pain temporarily but does not seem to provide lasting benefit, as he continues to report pain.
While it briefly decreases his blood pressure, it likely does not provide lasting pain control or is an inadequate dose to control pain. Transitioning to patient-controlled morphine is likely to adequately control
pain and therefore improve blood pressure.

Continuous intravenous labetalol infusion with titration (Choice B) and intravenous nitroprusside therapy (Choice C) are used to treat severe hypertension. This patient's hypertension is likely secondary to
inadequately treated pain. Therefore, improving pain control is more likely to improve blood pressure with minimum risk for adverse effects.

Switching from acetaminophen-oxycodone to tramadol (Choice E) would likely worsen blood pressure, as tramadol is a weaker opioid than oxycodone. Thus, tramadol would provide even less adequate pain
control. Poor pain control is likely to lead to worsening blood pressure.

Educational Objective: Inadequately treated pain in the postoperative patient can lead to hypertension, delayed recovery, and patient dissatisfaction. Patient-controlled analgesia is an option for treating
moderate to severe pain, especially in previously opioid-tolerant patients or in patients unable to take oral pain medications. It provides small boluses of parenteral opioids with lockout intervals and
maximum doses to prevent overdose. It can also allow the practitioner to calculate a total opioid requirement that can be utilized for transitioning the patient to an adequate oral regimen.

https://t.me/USMLENBME2CK ti
Previous Next Score Report Lab Values Calculator Help pause
Exam Section 2: Item 27 of 50 National Board of Medical Examiners
Comprehensive Clinical Science Self-Assessment

"I 27. A 37-year-old primigravid woman at 16 weeks' gestation comes lo the physician lo discuss the results of amniocentesis. She has an IQ of 70. She is employed as a housekeeping assistant in a hotel and
has been married to a coworker for 6 months. She has no family history of serious illness. Karyotype analysis of the fetus showed trisomy 21. Her pregnancy has been otherwise uncomplicated. She is
informed of the abnormalities associated with the fetal genotype and still wants to continue the pregnancy. Which of the following is the most appropriate next step in management?

A) Schedule the patient for ongoing prenatal care


B) Discuss the situation with the patient's husband
C) Try to convince the patient to change her decision
D) Seek a court-appointed guardian
E) Refer the patient for psychiatric evaluation to determine competence
Correct Answer: A.

To meet the criteria for decisional capacity, patients must express a consistent choice, demonstrate an understanding of the risks and benefits of the choice, demonstrate an appreciation of the significance of
the choice for their personal circumstances, and illustrate an ability to reason through the options. Capacity assessments should be based on the individual patient and the particular medical decision being
made. Certain populations are at increased risk for limitations of capacity, such as patients with dementia or psychiatric disorders. IQ scoring was previously used as a definitional criterion for intellectual
disability but has been replaced in the DSM-5 by a scale based on degrees of impairment in a variety of adaptive skills (practical, social, conceptual). This patient has a low IQ score but functions independently
and does not meet criteria for intellectual disability. Even among patients with intellectual disability, capacity cannot be assumed to be globally lacking but must be assessed on a case-by-case basis. This
patient should be scheduled for ongoing prenatal care in accordance with her expressed preference to continue her pregnancy.

Incorrect Answers: B, C, D, and E.

Discussing the situation with the patient's husband (Choice B) and seeking a court-appointed guardian (Choice D) are both incorrect. This patient has expressed a preference to continue her pregnancy and
has not demonstrated a lack of decisional capacity.

Trying to convince the patient to change her decision (Choice C) is inappropriate, may impair the patient-physician relationship, and may be viewed as condescending.

Referring the patient for psychiatric evaluation to determine competence (Choice E) is incorrect. Capacity assessments are decision-specific and based on a discussion with the patient in order to assess
understanding, appreciation, reasoning, and choice, and can be conducted by any physician. In contrast, competency is a global assessment that determines the ability of an individual to participate in legal
proceedings. Competency assessments must be made by a judge, rather than a psychiatrist.

Educational Objective: Capacity assessments should be based on the individual patient and the particular medical decision being made. They are based on a discussion with the patient in order to assess
understanding, appreciation, reasoning, and choice. IQ scoring is no longer used as a criterion for intellectual disability. Among patients with intellectual disability, capacity assessment must be made on a case-
by-case basis and cannot be assumed to be lacking.

https://t.me/USMLENBME2CK ti e
Previous Next Score Report Lab Values Calculator Help pause
Exam Section 2: Item 28 of 50 National Board of Medical Examiners
Comprehensive Clinical Science Self-Assessment

"I 28. A 29-year-old male US Navy veteran comes to the local Veterans Affairs clinic because of a 2-week history of purulent drainage from a small opening at the site of amputation of his left arm. For the past
10 days, he has had increasing pain, tenderness, and redness at the wound site. Thirteen months ago, the patient sustained a traumatic amputation of the left arm below the elbow when an improvised
explosive device detonated while he was on a combat mission in the Middle East. Management at that time consisted of aggressive surgical debridement of the residual limb, which has healed slowly; a
small part of the wound appeared to heal incompletely. The patient appears well. Vital signs are within normal limits. Examination of the left upper extremity shows a 3-mm opening in the scar with
surrounding erythema; the site is draining purulent material. The remainder of the examination shows no abnormalities. Which of the following studies is most likely to confirm the diagnosis?

A) Bone biopsy of the wound site


B) Complete blood count with differential
C) Erythrocyte sedimentation rate and serum C-reactive protein concentration
D) X-ray of the left upper extremity residual limb
Correct Answer: A.

This patient likely has chronic osteomyelitis with adjacent soft tissue infection and a draining sinus tract to the skin. The bone was likely seeded at the time of his limb injury and treatment, especially given the
evidence that part of the wound experienced poor healing. Chronic osteomyelitis can present as an indolent infection with gradually increasing pain at the affected site, low-grade fever, chills, and unexplained
weight loss. X-rays may show lytic lesions and cortical destruction, and laboratory studies generally show increased serum inflammatory markers (eg, C-reactive protein). A bone biopsy of the wound site is
needed to confirm the diagnosis, with the additional benefit of identifying the causal organism and testing for antibiotic susceptibilities.

Incorrect Answers: B, C, and D.

Complete blood count with differential (Choice B) should be checked and may provide evidence of infection with leukocytosis and left shift on the differential. However, it will not confirm the presence of
osteomyelitis.

Erythrocyte sedimentation rate and serum C-reactive protein concentration (Choice C) should also be checked, as these inflammatory markers are expected to be increased in the setting of infection. They are
useful in monitoring the progression of treatment and in making management decisions, as they should decrease with successful therapy.

X-ray of the left upper extremity residual limb (Choice 0) is not recommended in the evaluation of acute osteomyelitis but may show findings in chronic cases that suggest bone involvement, such as soft tissue
edema, bone loss, and periosteal reaction. However, plain radiography is not specific enough to confirm the diagnosis.

Educational Objective: Osteomyelitis is an infection of bone and the marrow cavity, and can present acutely or as a chronic, indolent infection. Laboratory studies will show an increase in inflammatory markers
such as erythrocyte sedimentation rate and serum C-reactive protein concentration, and x-rays may show lytic lesions and cortical destruction. Bone biopsy and culture is recommended to confirm diagnosis
and tailor antibiotics to the causative organism.

https://t.me/USMLENBME2CK ts e t
Previous Next Score Report Lab Values Calculator Help pause
Exam Section 2: Item 29 of 50 National Board of Medical Examiners
Comprehensive Clinical Science Self-Assessment

"I 29. A 2-year-old girl is brought lo the physician because of a 2-week history of difficulty walking and intermittent crossing of the eyes. She also has had three to four episodes of watery diarrhea daily
during this time. She has not had fever or vomiting. She has no history of serious illness and receives no medications. She is alert but irritable and tearful; she clings to her mother during the
examination. She is at the 50th percentile for length, 25th percentile for weight, and 50th percentile for head circumference. Her pulse is 100/min, respirations are 20/min, and blood pressure is
120/80 mm Hg. The abdomen is soft and nontender; a smooth, firm mass is palpated over the right upper quadrant. Neurologic examination shows rapid, chaotic, multidirectional saccades of the eyes.
There are Jerking motions of the extremities, which are exacerbated by intentional movement. There is truncal ataxia, and the patient is unable to sit or walk without assistance. Which of the following is
the most likely underlying cause of this patient's symptoms?

A) Hepatoblastoma
B) Medulloblastoma
C) Nephroblastoma (Wilms tumor)
D) Neuroblastoma
E) Non-Hodgkin lymphoma
F) Rhabdomyosarcoma
Correct Answer: D.

Opsoclonus myoclonus syndrome (OMS) typically presents in children between age 1 to 3 years with the acute onset of truncal ataxia, myoclonus, and opsoclonus, a form of saccadic intrusion. Ocular
saccades are rapid, conjugate, ballistic ocular movements that occur in order to rapidly and precisely change the point of gaze fixation. Saccades are normal ocular movements that may be voluntarily
initiated, although saccadic abnormalities occur in various pathologic conditions. Opsoclonus is characterized by rapid, arrhythmic, multivectorial ocular saccades that occur without an intersaccadic interval.
These features distinguish this entity from nystagmus and other saccadic disorders. Patients also typically display irritability, developmental regression, and behavioral or sleep disturbances. The underlying
cause of OMS is unclear, but it often occurs as a paraneoplastic phenomenon in association with neuroblastoma. Neuroblastoma is a neoplasm of neuroendocrine cells of the sympathetic nervous system
and is associated with overamplification of the MYCN oncogene. It may arise from any sympathetic nervous tissue and is the most common tumor of the adrenal gland that occurs in children. Adrenal
involvement classically presents with abdominal pain and distension, weight loss, and failure to thrive. Physical examination may disclose a firm, nodular mass that crosses the midline and diastolic
hypertension. Diagnosis is suggested by the presence of increased catecholamine metabolites in the serum and urine. Treatment and prognosis are dependent on the stage of the tumor at diagnosis and the
presence or absence of MYCN amplification.

Incorrect Answers: A, B, C, E, and F.

Hepatoblastoma (Choice A) is a neoplasm of hepatocyte precursor cells that occurs in children, frequently in association with Beckwith-Wiedemann syndrome. It is not commonly associated with OMS.

Medulloblastoma (Choice 8) is the most common malignant brain tumor in children, often presenting in patients age 5 to 9 years with signs of increased intracranial pressure (eg, headache, nausea,
vomiting) and gait ataxia or incoordination. It is not a common cause of OMS.

Nephroblastoma (Wilms tumor) (Choice C) is the most common pediatric renal malignancy and is characterized by a large, often palpable, unilateral flank mass. It is associated with Beckwith-Wiedemann
syndrome but is not commonly associated with OMS.

Non-Hodakin lymphoma (Choice E) encompasses a heterogeneous aroup of neoplasms and is amona the most common malianancies in children. Patients tvpically present with constitutional svmptoms.

https://t.me/USMLENBME2CK ts e t
Previous Next Score Report Lab Values Calculator Help pause
Exam Section 2: Item 29 of 50 National Board of Medical Examiners
Comprehensive Clinical Science Self-Assessment

C) Nephroblastoma (Wilms tumor)


D) Neuroblastoma
E) Non-Hodgkin lymphoma
F) Rhabdomyosarcoma
Correct Answer: D.

Opsoclonus myoclonus syndrome (OMS) typically presents in children between age 1 to 3 years with the acute onset of truncal ataxia, myoclonus, and opsoclonus, a form of saccadic intrusion. Ocular
saccades are rapid, conjugate, ballistic ocular movements that occur in order to rapidly and precisely change the point of gaze fixation. Saccades are normal ocular movements that may be voluntarily
initiated, although saccadic abnormalities occur in various pathologic conditions. Opsoclonus is characterized by rapid, arrhythmic, multivectorial ocular saccades that occur without an intersaccadic interval.
These features distinguish this entity from nystagmus and other saccadic disorders. Patients also typically display irritability, developmental regression, and behavioral or sleep disturbances. The underlying
cause of OMS is unclear, but it often occurs as a paraneoplastic phenomenon in association with neuroblastoma. Neuroblastoma is a neoplasm of neuroendocrine cells of the sympathetic nervous system
and is associated with overamplification of the MYCN oncogene. It may arise from any sympathetic nervous tissue and is the most common tumor of the adrenal gland that occurs in children. Adrenal
involvement classically presents with abdominal pain and distension, weight loss, and failure to thrive. Physical examination may disclose a firm, nodular mass that crosses the midline and diastolic
hypertension. Diagnosis is suggested by the presence of increased catecholamine metabolites in the serum and urine. Treatment and prognosis are dependent on the stage of the tumor at diagnosis and the
presence or absence of MYCN amplification.

Incorrect Answers: A, B, C, E, and F.

Hepatoblastoma (Choice A) is a neoplasm of hepatocyte precursor cells that occurs in children, frequently in association with Beckwith-Wiedemann syndrome. It is not commonly associated with OMS.

Medulloblastoma (Choice 8) is the most common malignant brain tumor in children, often presenting in patients age 5 to 9 years with signs of increased intracranial pressure (eg, headache, nausea,
vomiting) and gait ataxia or incoordination. It is not a common cause of OMS.

Nephroblastoma (Wilms tumor) (Choice C) is the most common pediatric renal malignancy and is characterized by a large, often palpable, unilateral flank mass. It is associated with Beckwith-Wiedemann
syndrome but is not commonly associated with OMS.

Non-Hodgkin lymphoma (Choice E) encompasses a heterogeneous group of neoplasms and is among the most common malignancies in children. Patients typically present with constitutional symptoms,
including fever, night sweats, and weight loss, as well as lymphadenopathy, decreased appetite, or dyspnea. Non-Hodgkin lymphoma is not a common cause of OMS.

Rhabdomyosarcoma (Choice F) is a rare, aggressive neoplasm of mesenchymal skeletal muscle precursors that may occur in the orbit or genitourinary tract. Orbital rhabdomyosarcoma typically presents in
children age 5 to 7 years with proptosis and globe displacement. Rhabdomyosarcoma does not typically produce paraneoplastic phenomena such as OMS.

Educational Objective: Opsoclonus myoclonus syndrome (OMS) typically presents in children between age 1 to 3 years with the acute onset of truncal ataxia, myoclonus, and opsoclonus. Opsoclonus is
characterized by rapid, arrhythmic, multivectorial ocular saccades that occur without an intersaccadic interval. OMS often occurs as a paraneoplastic phenomenon that, in children, frequently indicates an
underlying neuroblastoma.

https://t.me/USMLENBME2CK ts e t
Previous Next Score Report Lab Values Calculator Help pause
Exam Section 2: Item 30 of 50 National Board of Medical Examiners
Comprehensive Clinical Science Self-Assessment

"I 30. Three days after admission to the hospital for acute bacterial endocarditis, a 62-year-old man has shortness of breath that began 5 minutes ago while walking for the first time since admission. On
admission, he had a 10-day history of fever; examination showed splinter hemorrhages and normal heart sounds without murmurs. His current pulse is 110/min, respirations are 24/min, and blood
pressure is 100/50 mm Hg. Crackles are heard over both lung bases. S, and Sare soft; there is an S A new grade 2/6, early diastolic murmur is heard at the apex and the left sternal border, and a
grade 4/6, systolic ejection murmur is heard best in the carotids and at the left sternal border. Which of the following is the most appropriate next step in management?

A) Echocardiography
B) Ventilation-perfusion lung scans
C) Intravenous furosemide therapy
D) Intravenous heparin therapy
E) Sublingual nitroglycerin therapy
Correct Answer: A.

The development of a new murmur in the setting of acute infective endocarditis is highly concerning for valve damage, and this patient should be evaluated with echocardiography promptly. Complications
from infective endocarditis occur by two main mechanisms: septic embolization with seeding of distal sites and direct injury to cardiac tissue. Cardiac complications include heart failure caused by valvular
insufficiency, perivalvular abscess, pericarditis, tamponade, and intracardiac fistula formation. Valvular insufficiency as a result of direct inflammatory damage is the most common cardiac complication
associated with infective endocarditis and may require surgical intervention if severe.

Incorrect Answers: B, C, D, and E.

Ventilation-perfusion lung scans (Choice B) are useful in evaluating for pulmonary embolism in patients who cannot tolerate CT angiography (usually because of renal disease preventing the use of contrast,
pregnancy, or a contrast allergy). Septic embolization to the lungs often occurs in infective endocarditis affecting the tricuspid or pulmonary valves.

Intravenous furosemide therapy (Choice C) is indicated for volume overload states and is commonly utilized in treatment of heart failure exacerbations and valvulopathy. However, in the setting of
endocarditis, a new murmur should first be evaluated with echocardiography.

Intravenous heparin therapy (Choice 0) is indicated for treatment of acute thromboembolic disease, which is a complication of infective endocarditis. This patient's physical examination findings are more
suggestive of cardiac valve dysfunction.

Sublingual nitroglycerin therapy (Choice E) is not indicated, as this patient's clinical setting and examination findings are not suggestive of cardiac ischemia.

Educational Objective: Valve damage is a common complication of infective endocarditis, and the clinician should maintain a low threshold for evaluating new murmurs and cardiac symptoms with
echocardiography in management of these cases. Valvular insufficiency as a result of direct inflammatory damage is the most common cardiac complication associated with infective endocarditis and may
require surgical intervention if severe.

https://t.me/USMLENBME2CK ti
Previous Next Score Report Lab Values Calculator Help pause
Exam Section 2: ltem 31 of 50 National Board of Medical Examiners
Comprehensive Clinical Science Self-Assessment

✓ 31 . A study is proposed to investigate the effectiveness of a third-generation cephalosporin for the treatment of streptococcal pharyngitis. The study will include children between the ages of 5 and 10 years.
They will be randomly assigned to receive treatment with a cephalosporin, penicillin, or a placebo. Informed consent will be obtained from one parent. The principal investigator offers a $50 finder's fee
to any medical resident who identifies a possible study participant. Which of the following features of this study is of greatest potential concern?

A) Consent only obtained from one parent


B) Use of finder's fees to recruit subjects
C) Use of minors as subjects
D) Use of placebo as treatment
Correct Answer: D.

The use of a placebo allows for the concealment of the administration of the experimental drug from the subject or from the clinician and helps demonstrate the efficacy of the experimental drug by accounting
for the beneficial effects that the subject may experience secondary to the administration of any intervention by the clinician. While the use of a placebo can lend validity to a well-designed clinical trial, it
poses ethical dilemmas when effective treatments exist for the disease under study (eg, penicillins for streptococcal pharyngitis). Many researchers advocate that trials be designed to compare the
experimental drug against the current standard of care, rather than a placebo. The ethical dilemma is unambiguous in situations such as the one presented in this case, where an effective, standard treatment
regimen exists, and where the administration of a placebo could cause harm. In such scenarios, the use of a placebo would be unethical and should be strictly avoided.

Incorrect Answers: A, B, and C.

Consent only obtained from one parent (Choice A) is incorrect. Under United States federal guidelines, consent from both parents is generally required but may be waived under particular circumstances,
such as when one parent has sole legal responsibility for the child, or when one parent is deceased, unknown, incompetent, or not reasonably available.

Use of finder's fees to recruit subjects (Choice B) is incorrect. Finder's fees are often viewed as unethical because of their propensity to create conflict of interest and are illegal under antikickback statutes in
particular jurisdictions. However, the use of a placebo in this study represents a greater concern because of the potential for harm to study participants who are randomized to the placebo.

Use of minors as subjects (Choice C) is incorrect. While children are considered a vulnerable population because of their inability to give informed consent, some conditions or treatments may be best studied
in pediatric populations where the disease under study is more prevalent, such as in the example of streptococcal pharyngitis.

Educational Objective: The use of a placebo may be inappropriate in situations where an effective treatment is known to exist. A placebo should not be used in situations where its use could cause harm. In
such circumstances, the experimental drug should be compared with the standard treatment, rather than a placebo.

https://t.me/USMLENBME2CK ts e t
Previous Next Score Report Lab Values Calculator Help pause
Exam Section 2: ltem 32 of 50 National Board of Medical Examiners
Comprehensive Clinical Science Self-Assessment

"I 32. A 27-year-old woman comes to the physician for a follow-up examination. She has a 5-month history of temperatures to 38.3C (101F) and a cough productive of phlegm and mucous plugs. She has
severe asthma treated with high doses of prednisone. She has had numerous visits to the emergency department since childhood because of uncontrolled symptoms. Expiratory wheezes are heard
bilaterally. Laboratory studies show:
Hemoglobin 14.6 g/dL
Leukocyte count 8700/mm3
Segmented neutrophils 60%
Eosinophils 18%
Lymphocytes 22%
Serum lgE 23,000 IU/mL

An x-ray of the chest shows patchy bilateral upper lobe infiltrates that have shifted in location since an x-ray was taken 4 months ago. A CT scan of the chest shows central bronchiectasis without
lymphadenopathy. This patient is most likely to have an immediate response to which of the following on skin testing?

A) Animal dander
B) Aspergillus fumigatus
C) Bacillus subtilis
D) C/adosporium species
E) Isocyanate
F) Thermophilic actinomycetes
Correct Answer: B.

Allergic bronchopulmonary aspergillosis is a hypersensitivity reaction caused by the presence of Aspergillus fumigatus organisms in the airways. The clinical syndrome is associated with transitory pulmonary
infiltrates, peripheral eosinophilia, and bronchiectasis. Patients typically present with constitutional symptoms, including generalized fatigue and mild fever, progressive cough, and asthma-like symptoms that
persist despite treatment. Brown or black sputum and hemoptysis may be noted. Laboratory studies are notable for eosinophilia and increased serum lgE concentrations. Skin testing for a response to
Aspergi/lus antigens and Aspergil/us-specific antibody serum assays can aid in diagnosis. CT scan of the chest is the gold standard for diagnosis and is characterized by central bronchiectasis and signs of
mucoid impaction. Treatment consists of systemic corticosteroids, antifungal agents, and anti-lgE immunologies depending on the severity and chronicity of the syndrome.

Incorrect Answers: A, C, D, E, and F.

Animal dander (Choice A) is a common cause of asthma and allergic rhinitis. It would be unusual to be so refractory to treatment in a patient who is able to avoid exposure. It is not associated with transitory
pulmonary infiltrates or bronchiectasis.

Bacillus subtilis (Choice C), colloquially known as grass or hay bacillus, is the most studied gram-positive bacteria and is used for production of bacitracin. Bacitracin is a common allergen (and topical
antibiotic) that causes contact dermatitis.

https://t.me/USMLENBME2CK ti e
Previous Next Score Report Lab Values Calculator Help pause
Exam Section 2: Item 32 of 50 National Board of Medical Examiners
Comprehensive Clinical Science Self-Assessment
.. 4

A) Animal dander
B) Aspergillus fumigatus
C) Bacillus subtilis
D) C/adosporium species
E) Isocyanate
F) Thermophilic actinomycetes
Correct Answer: B.

Allergic bronchopulmonary aspergillosis is a hypersensitivity reaction caused by the presence of Aspergillus fumigatus organisms in the airways. The clinical syndrome is associated with transitory pulmonary
infiltrates, peripheral eosinophilia, and bronchiectasis. Patients typically present with constitutional symptoms, including generalized fatigue and mild fever, progressive cough, and asthma-like symptoms that
persist despite treatment. Brown or black sputum and hemoptysis may be noted. Laboratory studies are notable for eosinophilia and increased serum lgE concentrations. Skin testing for a response lo
Aspergil/us antigens and Aspergillus-specific antibody serum assays can aid in diagnosis. CT scan of the chest is the gold standard for diagnosis and is characterized by central bronchiectasis and signs of
mucoid impaction. Treatment consists of systemic corticosteroids, antifungal agents, and anti-lgE immunologics depending on the severity and chronicity of the syndrome.

Incorrect Answers: A, C, D, E, and F.

Animal dander (Choice A) is a common cause of asthma and allergic rhinitis. It would be unusual to be so refractory to treatment in a patient who is able to avoid exposure. It is not associated with transitory
pulmonary infiltrates or bronchiectasis.

Bacillus subtilis (Choice C), colloquially known as grass or hay bacillus, is the most studied gram-positive bacteria and is used for production of bacitracin. Bacitracin is a common allergen (and topical
antibiotic) that causes contact dermatitis.

C/adosponum species (Choice 0) are the most common black mold organisms that grow in moist indoor environments, and inhalation of spores can cause hypersensitivity symptoms, including asthma,
allergic rhinitis, and hypersensitivity pneumonitis. This patient's presentation is more consistent with allergic bronchopulmonary aspergillosis.

lsocyanates (Choice E) are organic compounds widely used in manufacturing and are implicated in causing occupational asthma. Symptoms are worse when the patient is at the workplace and improve
when they are at home.

Thermophitic actinomycetes (Choice F) are ubiquitous soil and plant matter organisms associated with hypersensitivity pneumonitis in farmers and cattle workers (Farmer's lung). Patients develop chronic
bronchitis and asthma symptoms.

Educational Objective: Allergic bronchopulmonary aspergillosis is a hypersensitivity reaction caused by Aspergil/us fumigatus, and patients with asthma or cystic fibrosis are at increased risk. Patients
present with chronic cough that may be productive of dark-colored sputum and hemoptysis. Characteristic findings include transitory pulmonary infiltrates, peripheral eosinophilia, and bronchiectasis.

https://t.me/USMLENBME2CK ti
Previous Next Score Report Lab Values Calculator Help pause
Exam Section 2: ltem 33 of 50 National Board of Medical Examiners
Comprehensive Clinical Science Self-Assessment

"I 33. A 4-year-old boy undergoes splenectomy lo control bleeding from injuries sustained in a motor vehicle collision. His postoperative course is uncomplicated, and he receives pneumococcal and
meningococcal vaccinations in preparation for his discharge home. After discharge, which of the following is the most appropriate follow-up intervention to decrease this patient's risk for subsequent
infection?

A) Avoidance of influenza vaccinations


B) Ciprofloxacin prophylaxis
C) Felic acid supplementation
D) Penicillin prophylaxis
E) No further intervention is indicated
Correct Answer: D.

Asplenia can occur because of congenital anomalies, splenectomy after trauma, or because of chronic vasoocclusion and infarction in conditions such as sickle cell disease. The spleen removes opsonized
pathogens and infected cells by mechanical filtration in the sinusoids and by phagocytosis by splenic macrophages. Additionally, the spleen houses immunoglobulin-producing B lymphocytes. The spleen plays
an important role in the clearance of encapsulated pathogens such as Haemophilus influenzae, Neisseria meningitidis, and Streptococcus pneumoniae, among others. Thus, patients lacking a spleen are at
risk for infection with encapsulated bacteria, and antibiotic prophylaxis against these organisms is often indicated, typically with penicillin or amoxicillin.

Incorrect Answers: A, B, C, and E.

Avoidance of influenza vaccinations (Choice A) would not be indicated, as patients with functional asplenia are immunocompromised and at increased risk for infections.

Ciprofloxacin prophylaxis (Choice B) is not typically used for antibiotic prophylaxis in asplenia. While fluoroquinolones can be used in patients with a penicillin allergy, they are not the most appropriate initial
follow-up intervention.

Folic acid supplementation (Choice C) is recommended in pregnancy to decrease the risk for neural tube defects. It is also commonly warranted in patients with rapid cellular turnover, such as acute leukemia
and sickle cell disease, to offset the consumption of folic acid in cell division. It is not indicated in the treatment of asplenia.

No further intervention (Choice E) would be harmful as patients with asplenia are at increased risk for infection for encapsulated bacteria, and patients should be treated with vaccination and antibiotic
prophylaxis against encapsulated organisms.

Educational Objective: Splenectomy increases the risk for infection with encapsulated organisms such as Haemophilus influenzae, Neisseria meningitidis, and Streptococcus pneumoniae. Patients with
impaired splenic function often require antibiotic prophylaxis, typically with penicillin or amoxicillin.

https://t.me/USMLENBME2CK ts e t
Previous Next Score Report Lab Values Calculator Help pause
Exam Section 2: Item 34 of 50 National Board of Medical Examiners
Comprehensive Clinical Science Self-Assessment

"I 34. A 22-year-old nulligravid woman with cystic fibrosis comes lo the physician because she would like lo conceive within the next year. She has not been hospitalized in the past 3 years. Her disease is
well controlled with antibiotics, vigorous bronchial drainage, bronchodilators, and nutritional support including pancreatic enzyme replacement. She is 155 cm (5 ft 1in) tall and weighs 38 kg (83 lb);
BMI is 16 kg/m. Her pulse is 96/min, and blood pressure is 138/86 mm Hg. Examination shows no other abnormalities except for a prolonged expiratory phase. Her husband is healthy and has no
family history of cystic fibrosis. She should be told that she will most likely require which of the following during pregnancy?

A) Additional antibiotic therapy


8) Insulin therapy
C) Nutritional supplementation
D) Oxygen therapy
E) Multiple hospital admissions
Correct Answer: C.

Cystic fibrosis is a genetic disorder caused by abnormalities in chloride ion transport that causes thickened secretions in a variety of organ systems, most notably the respiratory tract and the gastrointestinal
system. This leads to the common presenting symptoms of recurrent pneumonia, sinusitis, pancreatic insufficiency, and pancreatitis. Most cystic fibrosis cases are diagnosed on neonatal screening
performed shortly after birth. Malnutrition is common in patients with cystic fibrosis, as their pancreatic insufficiency causes inadequate absorption of protein, fat, and vitamins. This patient has signs of long-
term malnutrition, such as small stature and low BMI. This malnutrition is likely to be worsened in pregnancy as her nutritional requirements increase to provide the developing fetus with appropriate
nutrients. Thus, she is most likely to require nutritional supplementation during her pregnancy to prevent complications such as fetal growth restriction.

Incorrect Answers: A, B, D, and E.

Additional antibiotic therapy (Choice A) would be dependent on the patient's pulmonary status, rather than on pregnancy itself. It is possible that she develops an infection during pregnancy, but she will not
necessarily be at increased risk for its development. Additionally, her cystic fibrosis is well controlled now, with no recent hospitalizations or infections, making the likelihood of needing future antibiotics lower.

Insulin therapy (Choice 8) is used to treat diabetes mellitus, which can occur during pregnancy. Diabetes mellitus is also common in cystic fibrosis as a result of pancreatic endocrine dysfunction. However, it
more frequently presents at a younger age. As this patient does not currently have cystic fibrosis-related diabetes mellitus, she is not more likely to require insulin therapy than a parturient patient without
cystic fibrosis.

Oxygen therapy (Choice 0) is unlikely lo be necessary in this patient. She has good baseline lung function given her underlying disease and has had no recent infections or hospitalizations. Thus, pregnancy
itself will not increase her risk for needing supplemental oxygen therapy.

Multiple hospital admissions (Choice E) are not more likely for this patient. Many patients are hospitalized during their pregnancy for a variety of obstetric reasons. This patient's well-controlled cystic fibrosis
is unlikely lo lead to additional hospitalizations.

Educational Objective: Cystic fibrosis is a genetic disorder caused by abnormalities in chloride ion transport that causes thickened secretions in a variety of organ systems, notably the respiratory tract and
the gastrointestinal system. Malnutrition is common in patients with cystic fibrosis, as pancreatic insufficiency causes inadequate absorption of protein, fat, and vitamins. This can be worsened in
hypermetabolic states, such as those associated with pregnancy.

https://t.me/USMLENBME2CK ts e t
Previous Next Score Report Lab Values Calculator Help pause
Exam Section 2: ltem 35 of 50 National Board of Medical Examiners
Comprehensive Clinical Science Self-Assessment

"I 35. A 6-month-old girl is brought to the emergency department by her parents because of a 2-day history of temperatures to 39.4°C (103.0"F), irritability, decreased oral intake, and vomiting. She has no
history of serious illness and receives no medications. She has not received any vaccinations. She appears acutely ill and is lethargic. Temperature is 38.9C (102.0F), pulse is 170/min, respirations
are 60/min, and blood pressure is 70/45 mm Hg. Pulse oximetry on room air shows an oxygen saturation of 95%. Examination discloses a bulging fontanel and decreased responsiveness. Lumbar
puncture is performed. Results of cerebrospinal fluid analysis are shown:
RBC 2/mm3
WBC 800/mm3
Neutrophils 88%
Monocytes 12%
Glucose 15 mg/dL
Protein 150 mg/dl

Broad-spectrum antibiotic and corticosteroid therapy is initiated. Which of the following is the most likely long-term complication of this patient's condition?

A) Epidural abscess
8) Hearing loss
C) Obstructive hydrocephalus
D) Seizure disorder
E) Subdural effusion
Correct Answer: 8.

Bacterial meningitis in infants commonly presents with fever or hypothermia, lethargy, irritability, a bulging fontanelle, poor feeding, and seizures. Leukocytosis is common on laboratory examination, and
evaluation of the cerebrospinal fluid will show an increased white blood cell count with predominant neutrophils, increased protein, and decreased glucose. Thorough evaluation also includes a complete
blood count, blood culture, and urine culture to evaluate for other causes of sepsis, as well as culture of the cerebrospinal fluid to determine the causative organism. Common causes of neonatal meningitis
include Escherichia coli, Streptococcus aga/actiae, and Listeria monocytogenes. Based on age though, alternative bacterial pathogens should be considered. Haemophilus influenzae commonly causes
meningitis in children aged 6 months to 6 years, especially unvaccinated children. Meningitis is commonly treated with broad-spectrum antibiotics, and complications include lifelong neurologic deficits, such
as developmental delay and hearing loss. Hearing loss is one of the most common long-term complications of bacterial meningitis and may be transient or permanent.

Incorrect Answers: A, C, D, and E.

Epidural abscess (Choice A) can be an acute complication of meningitis but would not be a long-term complication after the treatment of meningitis.

Obstructive hydrocephalus (Choice C) can be a complication of Chiari malformations that presents as a large head circumference and bulging anterior fontanelle as a result of downward displacement of the
cerebellar vermis and tonsils. It can also result from subarachnoid hemorrhage blood products circulating within the subarachnoid space and also meningitis caused by inflammation and scarring of the dura,
however, is less common than hearing loss in this setting.

https://t.me/USMLENBME2CK ts e t
Previous Next Score Report Lab Values Calculator Help pause
Exam Section 2: ltem 35 of 50 National Board of Medical Examiners
Comprehensive Clinical Science Self-Assessment

Monocytes 12%
Glucose 15 mg/dL
Protein 150 mg/dL

Broad-spectrum antibiotic and corticosteroid therapy is initiated. Which of the following is the most likely long-term complication of this patient's condition?

A) Epidural abscess
8) Hearing loss
C) Obstructive hydrocephalus
D) Seizure disorder
E) Subdural effusion
Correct Answer: 8.

Bacterial meningitis in infants commonly presents with fever or hypothermia, lethargy, irritability, a bulging fontanelle, poor feeding, and seizures. Leukocytosis is common on laboratory examination, and
evaluation of the cerebrospinal fluid will show an increased white blood cell count with predominant neutrophils, increased protein, and decreased glucose. Thorough evaluation also includes a complete
blood count, blood culture, and urine culture to evaluate for other causes of sepsis, as well as culture of the cerebrospinal fluid to determine the causative organism. Common causes of neonatal meningitis
include Escherichia coli, Streptococcus agalactiae, and Listeria monocytogenes. Based on age though, alternative bacterial pathogens should be considered. Haemophi/us influenzae commonly causes
meningitis in children aged 6 months to 6 years, especially unvaccinated children. Meningitis is commonly treated with broad-spectrum antibiotics, and complications include lifelong neurologic deficits, such
as developmental delay and hearing loss. Hearing loss is one of the most common long-term complications of bacterial meningitis and may be transient or permanent.

Incorrect Answers: A, C, D, and E.

Epidural abscess (Choice A) can be an acute complication of meningitis but would not be a long-term complication after the treatment of meningitis.

Obstructive hydrocephalus (Choice C) can be a complication of Chiari malformations that presents as a large head circumference and bulging anterior fontanelle as a result of downward displacement of the
cerebellar vermis and tonsils. It can also result from subarachnoid hemorrhage blood products circulating within the subarachnoid space and also meningitis caused by inflammation and scarring of the dura,
however, is less common than hearing loss in this setting.

Seizure disorder (Choice 0) can occur during active or resolved meningitis but is a less common long-term complication than hearing loss.

Subdural effusion (Choice E) can occur in some children with bacterial meningitis, but they produce few symptoms and often do not require treatment.

Educational Objective: Bacterial meningitis in infants commonly presents with fever or hypothermia, lethargy, irritability, a bulging fontanelle, poor feeding, and seizures. Hearing loss is a common transient or
permanent long-term complication of pediatric bacterial meningitis.

https://t.me/USMLENBME2CK ts e t
Previous Next Score Report Lab Values Calculator Help pause
Exam Section 2: Item 36 of 50 National Board of Medical Examiners
Comprehensive Clinical Science Self-Assessment

"I 36. A 16-year-old girl is brought to the physician by her mother because she has never had a menstrual period. She is 147 cm (4 fl 10 in) tall and weighs 52 kg (115 lb); BMI is 24 kg/m2. Her temperature is
37.1C (98.8F), pulse is 70/min, respirations are 14/min, and blood pressure is 110/60 mm Hg. Breast and pubic hair development is sexual maturity rating stage 1, and there is scant axillary hair.
Pelvic examination shows no abnormalities. Her serum follicle-stimulating hormone concentration is 62 mlU/mL. Which of the following is the most appropriate next step in management?

A) Observation only
B) Karyotype analysis
C) Hysterosalpingography
D) MRI of the sella turcica
E) Gonadotropin-releasing hormone stimulation test
Correct Answer: B.

Gonadal dysgenesis 45,X (Turner syndrome) is a common cause of primary amenorrhea, which is secondary to gonadal (ovarian) dysgenesis in the setting of a 45,XO genotype. Other signs of Turner
syndrome include a short stature, cystic hygroma or webbed neck, swelling in the hands and/or feet, coarctation of the aorta, bicuspid aortic valve, fused kidney, and infertility. Additionally, decreased
estrogen concentrations may cause failure to develop mature secondary sexual characteristics of breast and pubic hair development. Patients may also develop vaginal atrophy or dryness related to low
estrogen concentrations, similar to postmenopausal patients. Vital signs will be within normal limits. Diagnosis is made with a karyotype analysis, which would be the most appropriate next step in the
management of this patient's condition. Other laboratory values that may aid in diagnosis are increased concentrations of luteinizing hormone and follicle-stimulating hormone caused by the lack of negative
feedback from estrogen, but these findings are nonspecific.

Incorrect Answers: A, C, D, and E.

Observation only (Choice A) is inappropriate. This patient has primary amenorrhea with physical characteristics classic of Turner syndrome and should undergo karyotype analysis to confirm the diagnosis.

Hysterosalpingography (Choice C) can be used in the evaluation of congenital uterine anomalies or infertility. While some structural abnormalities of the uterus are associated with amenorrhea, these are
often evident on pelvic examination. Additionally, congenital uterine anomalies would not affect stature or cause lack of secondary sexual characteristics.

MRI of the sella turcica (Choice 0) can be used to evaluate the pituitary gland. Lesions in the pituitary gland can cause hypopituitarism, which presents with fatigue, ovarian dysfunction, and short stature.
While this patient demonstrates some of these features, hypopituitarism would be associated with a decreased, rather than an increased, follicle-stimulating hormone concentration.

Gonadotropin-releasing hormone stimulation test (Choice E) is used to evaluate central precocious puberty. Precocious puberty refers to the onset of secondary sexual characteristics before the age of 8
years in females or 9 years in males. This patient has delayed, rather than precocious, pubertal development.

Educational Objective: Gonadal dysgenesis 45,X (Turner syndrome) is the most common cause of primary amenorrhea, which is a result of ovarian dysgenesis. Other signs include the absence of
secondary sexual characteristics, short stature, cystic hygroma or webbed neck, swelling in the hands and/or feet, coarctation of the aorta, bicuspid aortic valve, fused kidney, and infertility. Luteinizing
hormone and follicle-stimulating hormone concentrations are typically increased as a result of the absence of negative feedback from estrogen. Diagnosis is made with a karyotype analysis.

https://t.me/USMLENBME2CK ts e t
Previous Next Score Report Lab Values Calculator Help pause
Exam Section 2: Item 37 of 50 National Board of Medical Examiners°
Comprehensive Clinical Science Self-Assessment

"I 37. A 7-year-old boy is brought to the physician because of a 4-day history of a dry, mildly itchy rash. The rash began as a small, pink, oval area
on his chest and progressed to scaly, red patches over his arms, legs, and body. His mother applied calamine lotion and a topical
corticosteroid cream without relief. He is currently taking amoxicillin for streptococcal pharyngitis diagnosed 4 days ago. He has no other
history of rash or serious illness. He is alert and cooperative and in no acute distress. The pharynx is pink, and the tonsils are slightly
enlarged with no exudate. The neck is supple with no adenopathy. A photograph of the rash over the trunk is shown. Which of the following is
the most likely diagnosis?

A) Drug allergy
B) Pityriasis rosea
C) Scarlet fever
D) Seborrheic dermatitis
E) Tinea corporis

Correct Answer: B.

Pityriasis rosea is a cutaneous eruption characterized by the acute onset of a pruritic patch, or "herald patch," that is followed by the development of numerous smaller macules and patches. It commonly
involves the trunk and proximal extremities. The individual patches of pityriasis rosea are oval in shape, and their longitudinal axis is often lined up with the relaxed skin tension lines bilaterally on the trunk.
They are typically scaly and have a rim around the leading edge of any new lesion. In patients with darker skin types, pityriasis rosea may take on a papular appearance or may demonstrate
hyperpigmentation. It is usually induced by a viral infection including human herpesvirus 6 and 7, though often the exact infection is not determined. Pityriasis rosea self-resolves within 6 to 8 weeks without
treatment, though topical corticosteroids may be used for symptomatic management of pruritus.

Incorrect Answers: A, C, D and E.

Drug allergy (Choice A) may occur in the setting of antibiotic use, particularly those in the penicillin and sulfonamide classes. It may present with urticaria or erythema multiforme, which is characterized by
targetoid lesions on the palms, soles, trunk, extremities, and oral mucosa, as well as fever, myalgias, and arthralgias.

Scarlet fever (Choice C) can present with an erythematous and swollen tongue, rash, and desquamation of hands and feet. The rash of scarlet fever is characteristically a rough, papular, diffuse body rash.

Seborrheic dermatitis (Choice D) presents with chronic erythema, flaking, and pruritus of the skin, typically on the scalp or intertriginous folds. Malassezia yeast is thought to play a role in its pathogenesis.

Tinea corporis (Choice E) is caused by a superficial dermatophyte infection of the skin, most often Trichophyton rubrum. The classic appearance is a round, annular patch or plaque with a scaly border.

« https://t.me/USMLENBME2CK ts e t
Previous Next Score Report Lab Values Calculator Help pause
Exam Section 2: ltem 37 of 50 National Board of Medical Examiners"
Comprehensive Clinical Science Self-Assessment

A) Drug allergy
B) Pityriasis rosea

C) Scarlet fever
D) Seborrheic dermatitis
E) Tinea corporis

Correct Answer: B.

Pityriasis rosea is a cutaneous eruption characterized by the acute onset of a pruritic patch, or "herald patch," that is followed by the development of numerous smaller macules and patches. It commonly
involves the trunk and proximal extremities. The individual patches of pityriasis rosea are oval in shape, and their longitudinal axis is often lined up with the relaxed skin tension lines bilaterally on the trunk.
They are typically scaly and have a rim around the leading edge of any new lesion. In patients with darker skin types, pityriasis rosea may take on a papular appearance or may demonstrate
hyperpigmentation. It is usually induced by a viral infection including human herpesvirus 6 and 7, though often the exact infection is not determined. Pityriasis rosea self-resolves within 6 to 8 weeks without
treatment, though topical corticosteroids may be used for symptomatic management of pruritus.

Incorrect Answers: A, C, D and E.

Drug allergy (Choice A) may occur in the setting of antibiotic use, particularly those in the penicillin and sulfonamide classes. It may present with urticaria or erythema multiforme, which is characterized by
targetoid lesions on the palms, soles, trunk, extremities, and oral mucosa, as well as fever, myalgias, and arthralgias.

Scarlet fever (Choice C) can present with an erythematous and swollen tongue, rash, and desquamation of hands and feet. The rash of scarlet fever is characteristically a rough, papular, diffuse body rash.

Seborrheic dermatitis (Choice D) presents with chronic erythema, flaking, and pruritus of the skin, typically on the scalp or intertriginous folds. Ma/assezia yeast is thought to play a role in its pathogenesis.

Tinea corporis (Choice E) is caused by a superficial dermatophyle infection of the skin, most often Trichophyton rubrum. The classic appearance is a round, annular patch or plaque with a scaly border.
Treatment is with a topical antifungal, such as clotrimazole, or systemic antifungal, if widespread.

Educational Objective: Pityriasis rosea is characterized by the acute onset of a pruritic patch followed by the development of numerous smaller macules and patches in the setting of a recent viral illness. The
individual patches of pityriasis rosea are oval in shape, scaly, and have a rim of scale around the leading edge. This patient should be reassured that the condition will self-resolve in 6 to 8 weeks and can be
treated symptomatically with topical corticosteroids in the meantime.

« https://t.me/USMLENBME2CK ts e t
Previous Next Score Report Lab Values Calculator Help pause
Exam Section 2: Item 38 of 50 National Board of Medical Examiners
Comprehensive Clinical Science Self-Assessment

"I 38. A 3-month-old girl is brought to the physician because of constipation. She was born at home at term following an uncomplicated pregnancy and weighed 3118 g (6 lb 14 0z) She has not had routine
well-child examinations. She is unable to hold her head up, does not roll over, and does not smile. She is at the 10th percentile for length, 5th percentile for weight, and 75th percentile for head
circumference. The posterior fontanel is 2 cm in diameter. The tongue is large and protuberant. Which of the following is most likely to confirm the diagnosis?

A) Diet history
B) Thyroid function tests
C) Upper gastrointestinal series
D) Cranial ultrasonography
E) Rectal biopsy
Correct Answer: B.

Congenital hypothyroidism commonly presents in the first few months of life with feeding difficulties, lethargy, constipation, myxedematous facies, macroglossia, large fontanelles, hypotonia, and hypothermia.
Infants with this disorder may also present with goiter or congenital cardiac, renal, or gastrointestinal malformations. Thyroid dysfunction can be caused by a variety of mechanisms, including thyroid
dysgenesis, resistance to TSH, abnormalities in thyroid hormone transport, iodine deficiency, maternal antibodies, or antithyroid medications. Congenital hypothyroidism is typically identified on routine newborn
screening. Thyroid function tests will confirm the diagnosis. It is treated with thyroid hormone supplementation (eg, levothyroxine) and can lead to intellectual disability or neurologic issues (eg, hearing loss,
ataxia) if untreated.

Incorrect Answers: A, C, D, and E.

Diet history (Choice A) may be useful in identifying hypothyroidism secondary to iodine deficiency. Iodine deficiency in infancy is rare in the developed world, as adequate quantities of iodine are usually found
in breast milk and infant formula.

Upper gastrointestinal series (Choice C) and rectal biopsy (Choice E) are useful in the assessment of constipation caused by gastric motility disorders or obstruction, particularly congenital megacolon
(Hirschsprung disease). Congenital mega colon (Hirsch sprung disease) is caused by the congenital absence of the distal portion of the colonic myenteric plexus. This often leads to an inability to pass
meconium within the first few days of life and may result in a large bowel obstruction.

Cranial ultrasonography (Choice D) is useful for the diagnosis of intracranial hemorrhage in infants or certain structural abnormalities such as agenesis of the corpus callosum. It is not useful for the diagnosis
of congenital hypothyroidism.

Educational Objective: Congenital hypothyroidism presents in the first few months of life with feeding difficulties, lethargy, constipation, myxedematous facies, macroglossia, large fontanelles, hypotonia, and
hypothermia. It is typically identified on routine newborn screening. Thyroid function tests will confirm the diagnosis.

https://t.me/USMLENBME2CK ti
Previous Next Score Report Lab Values Calculator Help pause
Exam Section 2: ltem 39 of 50 National Board of Medical Examiners
Comprehensive Clinical Science Self-Assessment

✓ 39. A 32-year-old man comes to the office for a routine examination. He was diagnosed with HIV infection 7 years ago and began antiretroviral therapy 6 years ago. He received the appropriate
immunizations at the time of diagnosis. He has tolerated his medication regimen well. He used illicit intravenous drugs for 5 years but stopped 7 years ago. He is in a monogamous relationship with one
female partner; they use condoms consistently. Examination shows no abnormalities. His CD4+ T-lymphocyte count is 645/mm3 (Norma1>500), and plasma HIV viral load is 1000 copies/mL. Which of
the following immunizations is most appropriate to administer at this time?

A) Hepatitis A
B) Hepatitis 8
C) Human papillomavirus
D) Meningococcal
E) Pneumococcal
Correct Answer: E.

Per the Advisory Committee on Immunization Practices (ACIP) guidelines, the pneumococcal vaccine is the most appropriate immunization to administer at this lime. For immunocompetent adults, a single
dose of the pneumococcal polysaccharide (PPSV23) vaccine is recommended at age 65 years or older. Current guidelines recommend shared decision making between the patient and clinician regarding the
addition of pneumococcal conjugate (PCV13) doses. Special situations in which the PPSV23 vaccine is recommended for patients younger than 65 years include significant chronic medical conditions (eg,
diabetes, congestive heart failure, pulmonary disease) or immunocompromised status. For patients with immunocompromising conditions, a PCV13 dose followed by a series of PPSV23 doses is
recommended.

Incorrect Answers: A, 8, C, and D.

Patients with HIV are at increased risk for severe disease and mortality from infection, and the patient presumably already received hepatitis A (Choice A), hepatitis B (Choice 8), human papillomavirus
(Choice C), and meningococcal (Choice D) vaccines. These are recommended and were likely administered at the lime of diagnosis, along with the first dose in the pneumococcal series. The hepatitis A and
B vaccines may be given in combination or separately as a two- or three-dose series during the course of 1 to18 months. Meningococcal vaccination is recommended to be repeated every 5 years for patients
who remain at an increased risk for exposure.

Educational Objective: Pneumococcal vaccination is recommended for all adult patients with HIV regardless of age. The initial series consists of a PCV13 dose followed by a PPSV23 dose at least 8 weeks
later. Revaccination with PPSV23 is recommended at least 5 years after the initial series, and again at age 65 years.

https://t.me/USMLENBME2CK ti e
Previous Next Score Report Lab Values Calculator Help pause
Exam Section 2: Item 40 of 50 National Board of Medical Examiners
Comprehensive Clinical Science Self-Assessment

"I 40. A 24-year-old man with protein C deficiency comes to the physician because of a 1-week history of yellowing of his skin. Three months ago, he had a mesenteric infarction and required resection of all
but 45 cm of the proximal small bowel. The remainder of the jejunum was anastomosed to the colon. He receives total parenteral nutrition at home. He takes no medications. Examination today shows
generalized jaundice. Laboratory studies show:
Leukocyte count 7000/mm?
Serum
Bilirubin, total 3.6 mg/dL
Direct 2.6 mg/dL
Alkaline phosphatase 150 U/L
AST 20 U/L
ALT 17 U/L

Which of the following is the most likely cause of this patient's hyperbilirubinemia?

A) Biliary stricture
B ) Cholestatic hepatic disease
C) Hemolysis
D) Hepatitis A
E) Hepatitis C
Correct Answer: B.

Cholestatic hepatic disease is identified by a disproportionate rise in bilirubin and alkaline phosphatase compared to AST and ALT. Cholestasis is accompanied by Jaundice, visible at a bilirubin concentration
of 2 mg/dL or greater, and pruritus. Biliary motility is triggered by oral intake and neurohormonal control stimulated in part by feeding and gastric motility. A lack of oral intake, such as in cases of total
parenteral nutrition (TPN), with resultant limited gastric emptying will also cause limited biliary drainage and limited gallbladder contractility. In addition, TPN can cause bile duct inflammation, fibrosis, and
biliary obstruction, which describes the clinical picture of cholestasis. TPN-induced cholestasis is more common in patients with a short bowel. Cholestasis is defined by increased indirect and/or direct
bilirubin caused by the inability to excrete bilirubin. Typically, liver transaminases and alkaline phosphatase concentrations are not increased. Management of this condition involves discontinuing TPN and
providing enteral feeding.

Incorrect Answers: A, C, D, and E.

Biliary strictures (Choice A) create physical obstructions to emptying and are often seen in the setting of biliary duct disease, such as primary biliary cirrhosis (PBC) or primary sclerosing cholangitis (PSC).
Eventually, biliary strictures lead to hepatic fibrosis and end-stage liver failure with resultant ascites, splenomegaly, esophageal varices, and hepatic encephalopathy. They would not necessarily be
correlated with administration of TPN and limited oral intake independent of other hepatobiliary pathology such as PBC, PSC, or other biliary inflammatory disease states.

Hemolysis (Choice C) leads to the release and subsequent degradation of hemoglobin, a byproduct of which is bilirubin. It leads to a largely unconjugated hyperbilirubinemia barring any concomitant
downstream biliary obstruction.

https://t.me/USMLENBME2CK ts e t
Previous Next Score Report Lab Values Calculator Help pause
Exam Section 2: Item 40 of 50 National Board of Medical Examiners
Comprehensive Clinical Science Self-Assessment

Which of the following is the most likely cause of this patient's hyperbilirubinemia?

A) Biliary stricture
B ) Cholestatic hepatic disease
C) Hemolysis
D) Hepatitis A
E) Hepatitis C
Correct Answer: B.

Cholestatic hepatic disease is identified by a disproportionate rise in bilirubin and alkaline phosphatase compared to AST and ALT. Cholestasis is accompanied by Jaundice, visible at a bilirubin concentration
of 2 mg/dl or greater, and pruritus. Biliary motility is triggered by oral intake and neurohormonal control stimulated in part by feeding and gastric motility. A lack of oral intake, such as in cases of total
parenteral nutrition (TPN), with resultant limited gastric emptying will also cause limited biliary drainage and limited gallbladder contractility. In addition, TPN can cause bile duct inflammation, fibrosis, and
biliary obstruction, which describes the clinical picture of cholestasis. TPN-induced cholestasis is more common in patients with a short bowel. Cholestasis is defined by increased indirect and/or direct
bilirubin caused by the inability to excrete bilirubin. Typically, liver transaminases and alkaline phosphatase concentrations are not increased. Management of this condition involves discontinuing TPN and
providing enteral feeding.

Incorrect Answers: A, C, D, and E.

Biliary strictures (Choice A) create physical obstructions to emptying and are often seen in the setting of biliary duct disease, such as primary biliary cirrhosis (PBC) or primary sclerosing cholangitis (PSC).
Eventually, biliary strictures lead to hepatic fibrosis and end-stage liver failure with resultant ascites, splenomegaly, esophageal varices, and hepatic encephalopathy. They would not necessarily be
correlated with administration of TPN and limited oral intake independent of other hepatobiliary pathology such as PBC, PSC, or other biliary inflammatory disease states.

Hemolysis (Choice C) leads to the release and subsequent degradation of hemoglobin, a byproduct of which is bilirubin. It leads to a largely unconjugated hyperbilirubinemia barring any concomitant
downstream biliary obstruction.

Hepatitis A virus (HAV) (Choice 0) causes a self-limited infection that presents with nausea, vomiting, abdominal pain, icterus, hepatomegaly, and right upper quadrant tenderness that develops during
several weeks following the ingestion of poorly cooked, improperly handled, or raw foods, such as shellfish. HAV is diagnosed by the serologic detection of anti-HAV lgM antibodies.

Hepatitis C infection (Choice E) can present in both acute and chronic forms. The acute variant typically presents with symptoms of fatigue, nausea, vomiting, jaundice, and right upper quadrant discomfort. If
chronic infection ensues, it can eventually lead to cirrhosis and end-stage liver disease. Both hepatitis A and hepatitis C would cause transaminitis (generally a hepatocellular injury pattern, with
transaminases increased out of proportion to alkaline phosphatase and bilirubin), which is not seen in this patient.

Educational Objective: Total parenteral nutrition (TPN), especially in patients with a short bowel and absent oral intake, can cause cholestatic hepatic disease, characterized by increased bilirubin and
jaundice. Management of this condition involves discontinuing TPN and providing enteral feeding.

https://t.me/USMLENBME2CK ts e t
Previous Next Score Report Lab Values Calculator Help pause
Exam Section 2: Item 41 of 50 National Board of Medical Examiners
Comprehensive Clinical Science Self-Assessment

✓ 41. Eight weeks after an uncomplicated vaginal delivery of a healthy full-term newborn, a 27-year-old woman, gravida 1, para 1, is brought to the physician by her husband because of a change in
personality and depressed mood over the past 10 days. She has not had headaches. Her menses have not resumed despite the discontinuation of breast-feeding 5 weeks ago. Her husband says
that she has no interest in sexual intercourse. She does not use illicit drugs. She has had a 2.7-kg (6-lb) weight gain since delivery. She is now 175 cm (5 ft 9 in) tall and weighs 74 kg (164 lb); BMI is
24 kg/m?. Physical and neurologic examinations show no abnormalities. On mental status examination, she has a flat affect and is slow to respond to questioning. She spontaneously cries during the
interview and examination. She has no suicidal ideation. Which of the following is the most appropriate next step in management?

A) MRI of the brain


B) Antipsychotic therapy
C) Cyclic progestin therapy
D) Estrogen therapy
E) Selective serotonin reuptake inhibitor therapy
Correct Answer: E.

Postpartum depression (PPD), now known as major depressive disorder with peripartum onset, presents with the typical symptoms of major depressive disorder (eg, depressed mood, decreased energy,
anhedonia, sleep disturbances, appetite disturbances, poor concentration) and may be delayed by as many as 12 months after delivery. Risk factors include sleep disturbance, childcare stress, decreased
estrogen concentrations, breast-feeding difficulty, body image dissatisfaction, and poor postpartum social support. Consequences can be severe and include impaired bonding with the child, developmental
delays in the child, child neglect, suicide, and infanticide. The relative prevalence and severe consequences of PPD necessitate proactive screening for depression in primary care settings. Initial treatment
of moderate to severe cases is with selective serotonin reuptake inhibitors, which pose minimal risk to breast-feeding infants.

Incorrect Answers: A, B, C, and D.

MRI of the brain (Choice A) is unnecessary. MRI may show an empty sella in the setting of postpartum pituitary necrosis, which may present with a depressed mood. However, patients with postpartum
pituitary necrosis typically have a history of hemorrhage or hypovolemic shock during or shortly after delivery, and typically present with evidence of hypopituitarism, including agalactorrhea, hypothyroidism,
and adrenal insufficiency.

Antipsychotic therapy (Choice B) is useful for the treatment of postpartum psychosis, which may present with hallucinations, delusions, or bizarre or disorganized thinking, speech, or behavior. This patient
does not display symptoms of psychosis.

Cyclic progestin therapy (Choice C) may actually increase the risk for PPD and should be used with caution in the postpartum period. By contrast, estrogen therapy (Choice D) is supported by preclinical
and limited clinical data as useful for the treatment of PPD. However, this therapy is not yet supported by evidence from randomized clinical trials and remains experimental at this time.

Educational Objective: Postpartum depression presents with symptoms of major depressive disorder in the 12 months following delivery and can be associated with serious consequences such as suicide
and infanticide. Any postpartum patient who demonstrates severe symptoms or expresses suicidal or homicidal ideation should be referred for emergency psychiatric evaluation. Initial treatment of
moderate to severe cases is with selective serotonin reuptake inhibitors.

https://t.me/USMLENBME2CK ts e t
Previous Next Score Report Lab Values Calculator Help pause
Exam Section 2: Item 42 of 50 National Board of Medical Examiners
Comprehensive Clinical Science Self-Assessment

"I 42. A 32-year-old woman, gravida 3, para 2, at 37 weeks' gestation is admitted to the hospital in labor. Contractions occur every 3 minutes. She has gained 2.3 kg (5 lb) during the past week and has had
occasional blurred vision. She has been otherwise healthy. Her temperature is 37C (98.6F), pulse is 80/min, respirations are 20/min, and blood pressure is 150/110 mm Hg. Physical examination
shows retinal arteriolar spasm and pedal edema. The fundus is midway between the umbilicus and xiphoid. The cervix is 3 cm dilated and 100% effaced; the vertex is at O station. Laboratory studies
show:
Hemoglobin 12.5 g/dL
Leukocyte count 8000/mm3
Platelet count 200,000/mm3
Serum creatinine 1.2 mg/dL
Urine protein 2+

Which of the following is the most likely complication during labor?

A) Abruptio placentae
B) Amniotic fluid embolism
C) Arrest of labor
D) Maternal death
E) Uterine rupture
Correct Answer: A.

This patient's findings of hypertension, vision changes, weight gain, and lower extremity edema occurring after the 20th week of gestation are consistent with a diagnosis of preeclampsia. Preeclampsia is
caused by an abnormality of the placental spiral arteries and poor trophoblastic invasion, which leads to placental hypoperfusion and the subsequent production of vasoactive hormones that cause
generalized endothelial dysfunction, hypercoagulability, and vasospasm. Preeclampsia may lead to uteroplacental insufficiency that puts the fetus at risk for fetal growth restriction. If untreated, preeclampsia
can progress to eclampsia, characterized by maternal seizures, which can have profoundly adverse outcomes. Additionally, preeclampsia increases the risk for abruptio placentae, or premature separation of
the placenta from the uterus, which can lead to fetal exsanguination. Abruptio placentae most commonly presents with vaginal bleeding, severe uterine pain, and tetanic contractions. Thus, the most likely
complication during labor for this patient is abruptio placentae based on her underlying preeclampsia.

Incorrect Answers: B, C, D, and E.

Amniotic fluid embolism (Choice B) is a rare complication of childbirth and occurs during or immediately after delivery as a result of amniotic fluid entering the maternal circulation. It presents with shortness
of breath, altered mental status, hypoxia, and hypotension. Complications include disseminated intravascular coagulopathy, cardiovascular collapse, and maternal and/or fetal death. Preeclampsia is not
generally a risk factor for amniotic fluid embolism.

Arrest of labor (Choice C) occurs when there is no cervical change for greater than 4 hours with adequate contractions or 6 hours with inadequate contractions. These cases are typically managed with
cesarean delivery. This patient is multiparous, which puts her at a decreased risk for arrest of labor, and she has no other risk factors that would predispose her to it.

https://t.me/USMLENBME2CK ts e t
Previous Next Score Report Lab Values Calculator Help pause
Exam Section 2: Item 42 of 50 National Board of Medical Examiners
Comprehensive Clinical Science Self-Assessment

Which of the following is the most likely complication during labor?

A) Abruptio placentae
B) Amniotic fluid embolism
C) Arrest of labor
D) Maternal death
E) Uterine rupture
Correct Answer: A.

This patient's findings of hypertension, vision changes, weight gain, and lower extremity edema occurring after the 20th week of gestation are consistent with a diagnosis of preeclampsia. Preeclampsia is
caused by an abnormality of the placental spiral arteries and poor trophoblastic invasion, which leads to placental hypoperfusion and the subsequent production of vasoactive hormones that cause
generalized endothelial dysfunction, hypercoagulability, and vasospasm. Preeclampsia may lead to uteroplacental insufficiency that puts the fetus at risk for fetal growth restriction. If untreated, preeclampsia
can progress to eclampsia, characterized by maternal seizures, which can have profoundly adverse outcomes. Additionally, preeclampsia increases the risk for abruptio placentae, or premature separation of
the placenta from the uterus, which can lead to fetal exsanguination. Abruptio placentae most commonly presents with vaginal bleeding, severe uterine pain, and tetanic contractions. Thus, the most likely
complication during labor for this patient is abruptio placentae based on her underlying preeclampsia.

Incorrect Answers: B, C, D, and E.

Amniotic fluid embolism (Choice B) is a rare complication of childbirth and occurs during or immediately after delivery as a result of amniotic fluid entering the maternal circulation. It presents with shortness
of breath, altered mental status, hypoxia, and hypotension. Complications include disseminated intravascular coagulopathy, cardiovascular collapse, and maternal and/or fetal death. Preeclampsia is not
generally a risk factor for amniotic fluid embolism.

Arrest of labor (Choice C) occurs when there is no cervical change for greater than 4 hours with adequate contractions or 6 hours with inadequate contractions. These cases are typically managed with
cesarean delivery. This patient is multiparous, which puts her at a decreased risk for arrest of labor, and she has no other risk factors that would predispose her to it.

Maternal death (Choice 0) can occur in severe cases of preeclampsia. If eclampsia occurs, it can lead to maternal stroke, cardiac arrest, renal failure, and death. However, this patient's preeclampsia is likely
to be treated with magnesium, which would prevent the development of seizures and the associated consequences. Thus, abruptio placentae would be a more likely outcome than maternal death.

Uterine rupture (Choice E) is a dangerous obstetric complication where the uterine wall tears, presenting with acute pain and both maternal and fetal hemodynamic instability. The sudden loss of fetal station
is a suggestive finding. Risk factors include prior cesarean delivery or uterine surgery.

Educational Objective: Preeclampsia is caused by abnormalities of the placental spiral arteries and poor trophoblastic invasion, which leads to placental hypoperfusion and production of vasoactive
hormones that cause generalized endothelial dysfunction, hypercoagulability, and vasospasm. Preeclampsia may lead to uteroplacental insufficiency that puts the fetus at risk for fetal growth restriction. It
also increases the risk for abruptio placentae, which presents with vaginal bleeding, severe uterine pain, and tetanic contractions.

https://t.me/USMLENBME2CK ts e t
Previous Next Score Report Lab Values Calculator Help pause
Exam Section 2: Item 43 of 50 National Board of Medical Examiners
Comprehensive Clinical Science Self-Assessment

"I 43. A 32-year-old man is brought to the physician by his wife because of a gradual change in personality during the past 2 years. She describes him as less motivated and more self-centered. During the
past year, he has had several angry outbursts that were triggered by minor events. The patient says that he does not believe his personality has changed and that he does not recall these events. He
is alert and attentive. On examination, eye movements are full, but saccadic eye movements are noticeably slow. Smooth pursuit and vestibular ocular movements are normal. He has occasional
involuntary rapid, jerk-like movements of the trunk and extremities, but he is unaware of these movements. The remainder of the examination shows no abnormalities. Which of the following is the
most appropriate next step in diagnosis?

A) EEG
B) Follow-up examination in 1 month
C) Genetic testing
D) Lumbar puncture
E) Measurement of serum ceruloplasmin concentration
F) Psychiatric assessment
Correct Answer: C.

Genetic testing will likely confirm a diagnosis of Huntington disease (HD), which is a hereditary, progressive neurodegenerative disorder typically arising in the third to sixth decades of life. The disease
usually presents with an insidious onset of psychiatric (eg, irritability, depression, psychosis) and cognitive symptoms. Motor symptoms (eg, chorea, hypotonia, dystonia, loss of voluntary control of
movements) typically appear later in the disease course. People with HD typically die 10 to 40 years after disease onset. Chorea, a key HD symptom, can be initially experienced as restlessness. Chorea is
distinguished by irregular, unpredictable muscle movements that tend to flow from one muscle group to another. Treatment is symptomatic and can include tetrabenazine or deutetrabenazine, which target
chorea by depleting dopamine stores. HD is caused by expansion of a trinucleotide repeat (CAG) on chromosome 4, which occurs when a series of three nucleotides in repetition are replicated incorrectly
and too many repeats are included in the replicated product. HD also demonstrates anticipation, a genetic inheritance phenomenon in which future generations are affected at a younger age or with more
severe symptoms.

Incorrect Answers: A, B, D, E, and F.

EEG (Choice A) is unlikely to provide useful information in this case. The patient's involuntary movements are more consistent with chorea than with epilepsy.

Follow-up examination in 1 month (Choice B) is not appropriate at this time given the impact of the patient's symptoms and abnormalities noted on neurologic examination. Obtaining a diagnosis is important
so appropriate management can be initiated.

Lumbar puncture (Choice D) can be useful in patients presenting with nonspecific cognitive, behavioral, or neurologic symptoms to rule out central nervous system (CNS) infections such as neurosyphilis. It
can also aid in the diagnosis of CNS malignancies, multiple sclerosis, Guillain-Barr~ syndrome, and subarachnoid hemorrhage.

Measurement of serum ceruloplasmin concentration (Choice E) is used in the diagnosis of hepatolenticular degeneration (Wilson disease), which can be difficult to distinguish clinically from HD. Patients
typically present between childhood and early 30s. Neurologic symptoms are frequently encountered and include dysarthria, gait abnormalities, dystonia, tremor, parkinsonism, chorea, or athetosis. Kayser-
Fleischer rings are a classical finding on ophthalmologic examination.

https://t.me/USMLENBME2CK ts e t
Previous Next Score Report Lab Values Calculator Help pause
Exam Section 2: Item 43 of 50 National Board of Medical Examiners
Comprehensive Clinical Science Self-Assessment

A) EEG
B) Follow-up examination in 1 month
C) Genetic testing
D) Lumbar puncture
E) Measurement of serum ceruloplasmin concentration
F) Psychiatric assessment
Correct Answer: C.

Genetic testing will likely confirm a diagnosis of Huntington disease (HD), which is a hereditary, progressive neurodegenerative disorder typically arising in the third to sixth decades of life. The disease
usually presents with an insidious onset of psychiatric (eg, irritability, depression, psychosis) and cognitive symptoms. Motor symptoms (eg, chorea, hypotonia, dystonia, loss of voluntary control of
movements) typically appear later in the disease course. People with HD typically die 10 to 40 years after disease onset. Chorea, a key HD symptom, can be initially experienced as restlessness. Chorea is
distinguished by irregular, unpredictable muscle movements that tend lo flow from one muscle group to another. Treatment is symptomatic and can include tetrabenazine or deutetrabenazine, which target
chorea by depleting dopamine stores. HD is caused by expansion of a trinucleotide repeat (CAG) on chromosome 4, which occurs when a series of three nucleotides in repetition are replicated incorrectly
and too many repeats are included in the replicated product. HD also demonstrates anticipation, a genetic inheritance phenomenon in which future generations are affected at a younger age or with more
severe symptoms.

Incorrect Answers: A, B, D, E, and F.

EEG (Choice A) is unlikely to provide useful information in this case. The patient's involuntary movements are more consistent with chorea than with epilepsy.

Follow-up examination in 1 month (Choice B) is not appropriate at this time given the impact of the patient's symptoms and abnormalities noted on neurologic examination. Obtaining a diagnosis is important
so appropriate management can be initiated.

Lumbar puncture (Choice 0) can be useful in patients presenting with nonspecific cognitive, behavioral, or neurologic symptoms lo rule out central nervous system (CNS) infections such as neurosyphilis. It
can also aid in the diagnosis of CNS malignancies, multiple sclerosis, Guillain-Barr~ syndrome, and subarachnoid hemorrhage.

Measurement of serum ceruloplasmin concentration (Choice E) is used in the diagnosis of hepatolenticular degeneration (Wilson disease), which can be difficult lo distinguish clinically from HD. Patients
typically present between childhood and early 30s. Neurologic symptoms are frequently encountered and include dysarthria, gait abnormalities, dystonia, tremor, parkinsonism, chorea, or athetosis. Kayser-
Fleischer rings are a classical finding on ophthalmologic examination.

Psychiatric assessment (Choice F) may benefit the patient if the diagnostic workup for organic disease is inconclusive. Laboratory and genetic testing for neurologic disorders should be performed first.

Educational Objective: Huntington disease is a hereditary, progressive neurodegenerative disorder associated with expansion of a trinucleotide repeat (CAG) on chromosome 4. It presents with insidious
onset of psychiatric and cognitive symptoms, followed by motor symptoms (eg, chorea, hypotonia, dystonia, loss of voluntary control of movements). Diagnosis can be confirmed with genetic testing.

https://t.me/USMLENBME2CK ts e t
Previous Next Score Report Lab Values Calculator Help pause
Exam Section 2: Item 44 of 50 National Board of Medical Examiners
Comprehensive Clinical Science Self-Assessment

"I 44. A 12-year-old girl is brought to the physician by her mother because of a 2-month history of increasing, constant, throbbing pain and pressure in the pelvic area. She has no history of serious illness
and takes no medications. She has never had a menstrual period. She follows a normal diet and takes gymnastics lessons weekly. She is at the 50th percentile for height, weight, and BMI. Vital signs
are within normal limits. Breast and pubic hair development are sexual maturity rating (SMR) stage 3. Examination of the external genitalia shows a bluish bulge of tissue between a normal-appearing
urethra and anus. Serum studies show:
Follicle-stimulating hormone 12 mlU/ml
luteinizing hormone 15 mlU/mL
Estradiol 50 pg/ml (N=30-400)
Prolactin 10 ng/mL

Which of the following is the most likely diagnosis?

A) Bartholin duct cyst


8 ) Cervical stenosis
C) Im perforate hymen
D) Ovarian cancer
E ) Sarcoma botryoides
Correct Answer: C.

The hymen is a thin membranous band of tissue that covers the introitus, or vaginal opening, in young females. In most cases, this tissue degrades and stretches sufficiently enough to allow drainage of
blood and discharge by the time patients reach puberty. However, an imperforate hymen can occur when the hymen fails to regress. This results in the trapping of blood in a patient who has begun
menstruation. The most common presentation is primary amenorrhea and cyclical, abdominopelvic pain. Classically, a hematocolpos, or a vaginal canal distended by blood, will be evident on examination.
The diagnosis can be confirmed with ultrasonography, and treatment is with surgical hymenotomy.

Incorrect Answers: A, 8, D, and E.

Bartholin duct cyst (Choice A) may present with a bulge of tissue on pelvic examination. A cyst results from an obstructed Bartholin gland, which is a gland located on each side of the introitus that provides
secretions to lubricate the vagina. Bartholin duct cysts do not cause amenorrhea or abdominopelvic pain.

Cervical stenosis (Choice 8), or narrowing of the cervical os at the entrance to the uterus, can cause symptoms like the ones seen in this patient. If the os was completely stenosed, it would result in blood
accumulating in the uterus, causing persistent lower abdominal pain. However, cervical stenosis would not cause the bulging mucosal tissue seen on the examination of this patient.

Ovarian cancer (Choice 0) is uncommon in premenstrual patients. When present, it is often a germ cell tumor or sex cord-stromal tumor. Patients commonly present with abdominopelvic pain, bloating, and
nausea, with ultrasonography showing an ovarian mass. While these tumors can cause hormonal fluctuations, they are not associated with hematocolpos.

Sarcoma botryoides (Choice E) is a rare malignancy formed by striated muscle cells. It is often found in infants and children under the age of 3 years and presents with vaginal bleeding. Its name comes
fee#le erweile wwerweeefeleeiel4ieerte le etiefie ewsheiele le ewreeheel=ewe ereefee#lie veliewweelierweeetie suitl reweelee veLielie er wliLeh elieeweee

https://t.me/USMLENBME2CK ts e t
Previous Next Score Report Lab Values Calculator Help pause
Exam Section 2: Item 44 of 50 National Board of Medical Examiners
Comprehensive Clinical Science Self-Assessment

Estradiol 50 pg/ml (N=30-400)


Prolactin 10 ng/mL

Which of the following is the most likely diagnosis?

A) Bartholin duct cyst


B ) Cervical stenosis
C) Im perforate hymen
D) Ovarian cancer
E ) Sarcoma botryoides
Correct Answer: C.

The hymen is a thin membranous band of tissue that covers the introitus, or vaginal opening, in young females. In most cases, this tissue degrades and stretches sufficiently enough to allow drainage of
blood and discharge by the time patients reach puberty. However, an imperforate hymen can occur when the hymen fails to regress. This results in the trapping of blood in a patient who has begun
menstruation. The most common presentation is primary amenorrhea and cyclical, abdominopelvic pain. Classically, a hematocolpos, or a vaginal canal distended by blood, will be evident on examination.
The diagnosis can be confirmed with ultrasonography, and treatment is with surgical hymenotomy.

Incorrect Answers: A, B, D, and E.

Bartholin duct cyst (Choice A) may present with a bulge of tissue on pelvic examination. A cyst results from an obstructed Bartholin gland, which is a gland located on each side of the introitus that provides
secretions to lubricate the vagina. Bartholin duct cysts do not cause amenorrhea or abdominopelvic pain.

Cervical stenosis (Choice 8), or narrowing of the cervical os at the entrance to the uterus, can cause symptoms like the ones seen in this patient. If the os was completely stenosed, it would result in blood
accumulating in the uterus, causing persistent lower abdominal pain. However, cervical stenosis would not cause the bulging mucosal tissue seen on the examination of this patient.

Ovarian cancer (Choice 0) is uncommon in premenstrual patients. When present, it is often a germ cell tumor or sex cord-stromal tumor. Patients commonly present with abdominopelvic pain, bloating, and
nausea, with ultrasonography showing an ovarian mass. While these tumors can cause hormonal fluctuations, they are not associated with hematocolpos.

Sarcoma botryoides (Choice E) is a rare malignancy formed by striated muscle cells. It is often found in infants and children under the age of 3 years and presents with vaginal bleeding. Its name comes
from the grape-like appearance of polypoid tissue. This patient is outside the expected age range for this malignancy and is presenting with amenorrhea, making this an unlikely diagnosis.

Educational Objective: An imperforate hymen occurs when a thin band of membranous tissue over the introitus fails to regress during childhood. It presents during puberty with apparent amenorrhea and
cyclical abdominopelvic pain. Classically, a hematocolpos, or a vaginal canal distended by blood, will be evident on examination. The diagnosis can be confirmed with ultrasonography, and treatment is with
surgical hymenotomy.

https://t.me/USMLENBME2CK ti
Previous Next Score Report Lab Values Calculator Help pause
Exam Section 2: Item 45 of 50 National Board of Medical Examiners
Comprehensive Clinical Science Self-Assessment

"I 45. A 9-year-old girl is brought lo the physician by her father for a well-child examination. He is concerned that she is beginning to develop breasts at an early age. Menarche has not occurred. Her
mother's menarche began at the age of 12 years. The patient is at the 75th percentile for height and 50th percentile for weight. Breast development is sexual maturity rating (SMR) stage 2, and pubic
hair development is SMR stage 1. The remainder of the examination shows no abnormalities. Which of the following is the most appropriate next step in management?

A) Reassurance
B) Measurement of serum estrogen concentration
C) Measurement of serum follicle-stimulating hormone concentration
D) X-ray of the wrist to assess bone age
E) Transvaginal ultrasonography
F) Pelvic examination
Correct Answer: A.

Puberty refers to the normal hormonal and physiologic changes that occur as children age. The most common age range for the onset of puberty in females is age 9 to 12 years, although this can vary
markedly between patients based on factors such as body weight, genetics, and ethnicity. Precocious puberty refers to the onset of sexual secondary characteristics before age 8 in females or age 9 in
males. It can be idiopathic or caused by a variety of etiologies, such as central nervous system lesions, ovarian or germ cell tumors, adrenal disorders, or genetic defects, such as McCune-Albright
syndrome. Normal pubertal development in females often starts with breast development, followed by pubic hair development and menarche. This patient is progressing through normal pubertal milestones
at an appropriate age, and no further workup is currently necessary, making reassurance the most appropriate next step in management.

Incorrect Answers: B, C, D, E, and F.

Measurement of serum estrogen concentration (Choice B) and the measurement of serum follicle-stimulating hormone concentration (Choice C) may be appropriate in the evaluation of precocious puberty,
in addition to a thorough history and physical examination. Additional considerations for evaluation include measurement of bone age and serum concentrations of luteinizing hormone and testosterone. This
patient is progressing through normal pubertal development and further evaluation is not necessary.

X-ray of the wrist to assess bone age (Choice 0) is often used in the evaluation of precocious puberty. Advancements in bone age greater than two standard deviations from chronologic age are more likely
to correspond to central or peripheral precocious puberty. Further determination of bone age in this patient is not necessary as she is progressing through normal puberty.

Transvaginal ultrasonography (Choice E) and pelvic examination (Choice F) are invasive ways to examine the patient's genitalia and reproductive system. While imaging of the pelvis, more commonly with
abdominal ultrasonography, and examination of the external genitalia to determine sexual maturity rating are frequently utilized in the evaluation of precocious puberty, these measures are invasive and
unnecessary in this patient. Additionally, these invasive examinations are not commonly recommended in patients her age, even in the setting of precocious puberty.

Educational Objective: Normal pubertal development most commonly occurs between ages 9 and 12 years in females. Puberty typically begins with breast development, followed by pubic hair development
and menarche. The development of secondary sexual characteristics prior to age 8 in females is referred to as precocious puberty and warrants further evaluation with a thorough history and physical
examination as well as laboratory evaluation. Normal pubertal development does not require additional workup.

https://t.me/USMLENBME2CK ts e t
Previous Next Score Report Lab Values Calculator Help pause
Exam Section 2: Item 46 of 50 National Board of Medical Examiners
Comprehensive Clinical Science Self-Assessment

"I 46. A57-year-old man comes to the physician because of a 1-week history of shortness of breath and swelling of his legs. He says his shortness of breath occurs most frequently when he lies down.
During the past month, he has had the sensation of abdominal fullness and pulling and has noticed a progressive increase in his belt size. Five years ago, he received the diagnosis of bleeding
esophageal varices and alcoholic cirrhosis. He underwent sclerosis of the varices and has abstained from alcohol since then. His temperature is 36.9°C (98.4°F). The abdomen is moderately tense
and nontender with shifting dullness. There is 3+ pitting edema of the lower extremities. Paracentesis is performed. Laboratory studies show:
Serum
Protein, total 2.8 g/dL
Albumin 1.8 g/dL
Peritoneal fluid
Leukocyte count 900/mm3
Segmented neutrophils 67%
Lymphocytes 28%
Protein, total 1.8 g/dL
Albumin 1 g/dl

Which of the following is the most appropriate next step in management?

A) Echocardiography
B) Ultrasonography of the hepatic veins
C) Antibiotic therapy
D) Laparoscopy
E ) Liver biopsy
F) CT angiography of the liver
Correct Answer: C.

Spontaneous bacterial peritonitis (SBP) occurs most frequently in cirrhotic states, although ii can occur in any setting causing ascites. SBP is an infection within ascitic fluid in the peritoneal cavity, most often
from translocated enteric flora such as Escherichia coli. The classic presentation includes fever with diffuse abdominal pain and tenderness in a patient with ascites. If there is clinical concern for SBP, the
ascites fluid should be sampled through a paracentesis. Laboratory analysis of the ascitic fluid should include Gram stain, culture, and cell counts. A total leukocyte count of greater than 1000/mm3, or
greater than 250 neutrophils/mm3, is generally diagnostic of SBP since culture results are variably positive. Antibiotics with gram-negative rod coverage (eg, third-generation cephalosporins) are considered
first-line therapy.

Incorrect Answers: A, B, D, E, and F.

Echocardiography (Choice A) does not have a defined role in the evaluation of liver disease or SBP as it solely evaluates the heart and pericardium. If heart failure was also suspected, echocardiography
would be appropriate.

https://t.me/USMLENBME2CK ti
Previous Next Score Report Lab Values Calculator Help pause
Exam Section 2: Item 46 of 50 National Board of Medical Examiners
Comprehensive Clinical Science Self-Assessment
• I I I I

A) Echocardiography
B) Ultrasonography of the hepatic veins
C) Antibiotic therapy
D) Laparoscopy
E ) Liver biopsy
F) CT angiography of the liver
Correct Answer: C.

Spontaneous bacterial peritonitis (SBP) occurs most frequently in cirrhotic states, although it can occur in any setting causing ascites. SBP is an infection within ascitic fluid in the peritoneal cavity, most often
from translocated enteric flora such as Escherichia coli. The classic presentation includes fever with diffuse abdominal pain and tenderness in a patient with ascites. If there is clinical concern for SBP, the
ascites fluid should be sampled through a paracentesis. Laboratory analysis of the ascitic fluid should include Gram stain, culture, and cell counts. A total leukocyte count of greater than 1000/mm3, or
greater than 250 neutrophils/mm3, is generally diagnostic of SBP since culture results are variably positive. Antibiotics with gram-negative rod coverage (eg, third-generation cephalosporins) are considered
first-line therapy.

Incorrect Answers: A, B, D, E, and F.

Echocardiography (Choice A) does not have a defined role in the evaluation of liver disease or SBP as it solely evaluates the heart and pericardium. If heart failure was also suspected, echocardiography
would be appropriate.

Ultrasonography of the hepatic veins (Choice B) can be helpful in diagnosing Budd-Chiari syndrome, which is characterized by acute onset of severe right upper quadrant pain, congestive hepatopathy, and
liver failure caused by obstruction of the hepatic veins by thrombosis. However, in cases with evidence of SBP, administration of antibiotics should not be delayed.

Laparoscopy (Choice 0) would not be appropriate in cases of ascites without suspicion for a competing diagnosis warranting its use (eg, bowel perforation, cholecystitis).

Liver biopsy (Choice E) is useful in the diagnosis of conditions such as cirrhosis, hepatitis, and hepatocellular carcinoma. While it can be helpful in diagnosing the cause of cirrhosis, it should not delay
antibiotic administration when SBP is known.

CT angiography of the liver (Choice F) would not be helpful in the diagnosis or treatment of SBP. CT angiography is used in a variety of clinical presentations to assess the patency, course, and integrity of
blood vessels, most commonly in thromboembolic disease, atherosclerotic disease, or bleeding.

Educational Objective: Spontaneous bacterial peritonitis is diagnosed when paracentesis fluid shows a total leukocyte count of greater than 1000/mm or greater than 250 neutrophils/mm3. If diagnosed,
antibiotic therapy should be initiated.

https://t.me/USMLENBME2CK ti
Previous Next Score Report Lab Values Calculator Help pause
Exam Section 2: Item 47 of 50 National Board of Medical Examiners
Comprehensive Clinical Science Self-Assessment

"I 47. An asymptomatic 62-year-old man comes for a routine health maintenance examination. Physical examination shows splenomegaly. Laboratory studies show:
Hematocrit 42%
Leukocyte count 46,000/mm?
Segmented neutrophils 35%
Bands 15%
Basophils 1%
Lymphocytes 16%
Monocytes 5%
Myeloblasts 1%
Promyelocytes 2%
Metamyelocytes 13%
Myelocytes 12%
Platelet count 650,000/mm?

Leukocyte alkaline phosphatase activity is decreased. Examination of bone marrow shows many immature white blood cell precursors. Which of the following is the most likely diagnosis?

A) Agnogenic myeloid metaplasia


B) Chronic myelogenous leukemia
C) Chronic myelomonocytic leukemia
D) Erythroleukemia
E) Essential thrombocytosis
F) Myelodysplasia
G) Polycythemia vera
Correct Answer: B.

The patient most likely has a diagnosis of chronic myelogenous leukemia (CML), which commonly presents as an incidental laboratory finding in asymptomatic patients. The chronic phase may also present
with fatigue, night sweats, weight loss, splenomegaly, and fever. CML is defined by the Philadelphia (Ph 1) chromosome (translocation between chromosomes 9 and 22). Typical laboratory findings in CML
include hyperleukocytosis, which is present in this case, with an increase in nearly all the myeloid cell lines. Leukostasis, a condition in which hyperleukocytosis leads to end-organ damage from occlusion of
capillaries by malignant, nondistensible cells, is uncommon in CML because malignant cells are well-differentiated and pliable. This contrasts with conditions such as acute myeloid leukemia and acute
lymphocytic leukemia in which circulating blasts, which are large and not particularly pliable, easily lodge in capillaries to cause pulmonary and neurologic complications. Treatment of CML is primarily with
tyrosine kinase inhibitors such as imatinib or dasatinib and routine laboratory monitoring.

Incorrect Answers: A, C, D, E, F, and G.

https://t.me/USMLENBME2CK ts e t
Previous Next Score Report Lab Values Calculator Help pause
Exam Section 2: Item 47 of 50 National Board of Medical Examiners
Comprehensive Clinical Science Self-Assessment

D) Erythroleukemia
E) Essential thrombocytosis
F) Myelodysplasia
G) Polycythemia vera
Correct Answer: B.

The patient most likely has a diagnosis of chronic myelogenous leukemia (CML), which commonly presents as an incidental laboratory finding in asymptomatic patients. The chronic phase may also present
with fatigue, night sweats, weight loss, splenomegaly, and fever. CML is defined by the Philadelphia (Ph 1) chromosome (translocation between chromosomes 9 and 22). Typical laboratory findings in CML
include hyperleukocytosis, which is present in this case, with an increase in nearly all the myeloid cell lines. leukostasis, a condition in which hyperleukocytosis leads to end-organ damage from occlusion of
capillaries by malignant, nondistensible cells, is uncommon in CML because malignant cells are well-differentiated and pliable. This contrasts with conditions such as acute myeloid leukemia and acute
lymphocytic leukemia in which circulating blasts, which are large and not particularly pliable, easily lodge in capillaries to cause pulmonary and neurologic complications. Treatment of CML is primarily with
tyrosine kinase inhibitors such as imalinib or dasatinib and routine laboratory monitoring.

Incorrect Answers: A, C, D, E, F, and G.

Agnogenic myeloid metaplasia (Choice A), also called idiopathic or primary myelofibrosis, is the least common of the chronic myeloproliferative disorders and is associated with increased leukocyte alkaline
phosphatase activity. The patient's serum discloses decreased leukocyte alkaline phosphatase activity.

Chronic myelomonocytic leukemia (Choice C) is a hematologic disorder that displays both neoplastic and proliferative behavior. laboratory studies are notable for a predominant monocytosis, which is not
present in this patient's sample.

Erythroleukemia (Choice 0) is a subtype of acute myeloid leukemia characterized by proliferation of immature erythrocytes. leukopenia is disclosed on laboratory studies rather than leukocytosis as in this
case, as well as anemia and thrombocytopenia.

Essential thrombocytosis (Choice E) is characterized by abnormal platelet proliferation, which are often dysfunctional leading to hemorrhagic and thrombotic complications. Bone marrow examination would
be expected to show proliferation of megakaryocytes rather than white blood cell precursors.

Myelodysplasia (Choice F) is frequently encountered in older individuals or in those who previously received cytotoxic chemotherapy. Findings on bone marrow aspirate include abnormal appearing cells of
multiple lineages, which are not present on this patient's bone marrow biopsy.

Polycythemia vera (Choice G) is associated with JAK2 mutations that lead to unregulated myeloproliferation and erythrocytosis. Presenting symptoms are often related to increased blood viscosity and
include headaches, visual disturbances, and erythromelalgia (burning pain in the digits). It is also associated with pruritus caused by hot showers.

Educational Objective: Chronic myelogenous leukemia is a common myelo proliferative disorder encountered in older adults, caused by the reciprocal translocation between chromosomes 9 and 22 known as
the Philadelphia (Ph1) chromosome. In the chronic phase, patients present with nonspecific symptoms and have laboratory studies that disclose proliferation of white blood cells and their precursors.

https://t.me/USMLENBME2CK ts e t
Previous Next Score Report Lab Values Calculator Help pause
Exam Section 2: Item 48 of 50 National Board of Medical Examiners
Comprehensive Clinical Science Self-Assessment

✓ 48. A 5-year-old boy is brought to the office for a follow-up examination 2 days after he was discharged from the hospital following an episode of pyelonephritis. Medications are cefixime and
acetaminophen. He is at the 50th percentile for height and 25th percentile for weight; BMI is at the 10th percentile. Temperature is 37.6C (99.6°F), pulse is 120/min, respirations are 22/min, and
blood pressure is 90/60 mm Hg. Physical examination shows no other abnormalities. Results of laboratory studies from his hospital admission showed a leukocyte count within the reference range
and urine positive for leukocyte esterase; urine culture grew Klebsiel/a pneumoniae susceptible to cephalosporins. Kidney ultrasonography showed mild hydronephrosis. Which of the following is the
most appropriate next step in management?

A) Continuous antibiotic prophylaxis


B) Kidney and bladder ultrasonography
C) Renal scintigraphy using dimercaptosuccinic acid (DMSA)
D) Repeat urine culture
E) Voiding cystourethrography
Correct Answer: E.

Among children who present with an initial episode of febrile urinary tract infection (UTI), it is essential to identify factors that may predispose the patient to recurrent infections. Recurrent UTI increases the
risk for renal scarring, hydronephrosis, and chronic kidney disease. Vesicoureteral reflux (VUR) is characterized by incomplete closure of the vesicoureteral junction during voiding with retrograde flow of
urine from the bladder to the ureters. VUR allows for retrograde transport of bacteria to the upper urinary tract and is a significant risk factor for pyelonephritis. Voiding cystourethrography (VCUG) is
indicated for children with a first febrile UTI if there is evidence of hydronephrosis, renal scarring, or other findings that are suggestive of VUR. VCUG involves instillation of contrast dye into the bladder via
catheterization, which allows for visualization of reflux during voiding.

Incorrect Answers: A, B, C, and D.

Continuous antibiotic prophylaxis (Choice A) is indicated for children with VUR, although this remains controversial. Antibiotics are not routinely given prior to the diagnosis of VUR with VCUG.

Kidney and bladder ultrasonography (Choice B) is useful for identifying hydronephrosis and abnormal ureteral architecture but is not reliable for the detection of VUR.

Renal scintigraphy using dimercaptosuccinic acid (DMSA) (Choice C) is a method of detecting renal scarring via the intravenous injection of a radiotracer. It is not recommended as the initial test to
diagnose sequelae from VUR because it involves greater radiation exposure and is more expensive.

Repeat urine culture (Choice D) is not useful for identifying VUR or risk-stratification of recurrent pyelonephritis.

Educational Objective: Vesicoureteral reflux (VUR) is characterized by incomplete closure of the vesicoureteral junction during voiding with retrograde flow of urine from the bladder to the ureters. VUR is a
significant risk factor for pyelonephritis, which may have long-lasting sequelae including renal scarring, hydronephrosis, and chronic kidney disease. Voiding cystourethrography is indicated for children with
unexplained urinary tract infection if there is evidence of hydronephrosis, renal scarring, or other findings that are suggestive of VUR.

https://t.me/USMLENBME2CK ts e t
Previous Next Score Report Lab Values Calculator Help pause
Exam Section 2: Item 49 of 50 National Board of Medical Examiners°
Comprehensive Clinical Science Self-Assessment

"I 49. A 16-year-old girl is brought to the emergency department immediately after she collapsed on her way to the bathroom at home. She has anorexia nervosa and has had an 18-kg (40-lb) weight loss
during the past 16 months. Menarche was at the age of 11 years, but her last menstrual period was 14 months ago. She does not drink alcohol or use illicit drugs. On arrival, she appears cachectic. She
is 165 cm (5 fl 5 in) tall and weighs 36 kg (80 lb); BMI is 13 kg/m2. Her temperature is 36.4C (97.6F), pulse is 40/min, respirations are 18/min, and blood pressure is 85/55 mm Hg. Physical examination
shows fine hair over the skin. An ECG shows ST-segment depression and an increased U wave. This patient is most likely to have which of the following sets of serum electrolyte findings?

Na K CI HCO 3
(mEq/L) (mEq/L) (mEq/L) (mEq/L)
A) 125 3.2 102 25
B) 125 6.8 102 27
C) 140 23 94 28
0) 140 6.8 94 28
E) 155 3.2 102 26
Correct Answer: C.

Anorexia nervosa is characterized by an intense fear of gaining weight, decreased self-worth related to body weight, and restriction of food or binge eating and purging behavior. Patients with anorexia nervosa
often have a BMI less than 17 kg/m (below the 5" percentile). As a result of insufficient nutrition and weight loss, the hypothalamus decreases gonadotropin-releasing hormone secretion, which leads to
amenorrhea and pubertal delay or arrest. The physical examination can show dry, scaly skin, and fine hair or hair loss. Patients with the binge-eating/purging type of anorexia nervosa will also have tooth decay
from gastric hydrochloric acid erosion of enamel and scars on the knuckles secondary to abrasions from incisors when inducing vomiting. Losses of gastric hydrochloric acid lead to hypochloremia and
metabolic alkalosis. Gastric losses of potassium also result in hypokalemia. However, this is further exacerbated by renal losses of potassium as a result of alkalosis. In severe cases, signs of hypovolemia
such as tachycardia and hypotension may be present. Treatment of anorexia is through a combined medical and psychiatric approach and involves correcting fluid and electrolyte derangements alongside
behavioral and pharmacologic therapy.

Incorrect Answers: A, B, D, and E.

Hyponatremia (Choices A and B) and hypernatremia (Choice E) are not generally features of anorexia nervosa apart from severe volume depletion or water consumption. Hyponatremia can occur in syndrome
of inappropriate antidiuretic hormone. Severe hyponatremia may be accompanied by seizures, confusion, coma, or death. In addition, renal excretion of potassium leads lo hypokalemia, not normokalemia
(Choices A and E) or hyperkalemia (Choices Band D).

Educational Objective: Patients with anorexia nervosa (binge-eating/purging type) have low BMI, pubertal delay or arrest, and purging behavior leading to hypovolemia and electrolyte disturbances of
hypokalemia, hypochloremia, and metabolic alkalosis.

« https://t.me/USMLENBME2CK ts e t
Previous Next Score Report Lab Values Calculator Help pause
Exam Section 2: Item 50 of 50 National Board of Medical Examiners
Comprehensive Clinical Science Self-Assessment

"I 50. A 20-hour-old female newborn was noted to have jaundice 1 hour ago. She was born at term to a 29-year-old woman, gravida 2, para 2, following an uncomplicated pregnancy and spontaneous vaginal
delivery. The mother emigrated from Ethiopia 2 years ago. She received routine prenatal care. Examination of the patient shows Jaundice to the level of the umbilicus. There is conjunctival icterus. No
other abnormalities are noted. The maternal blood group is 0, Rh-positive. Results of laboratory studies, including the patient's blood group, are pending. Which of the following laboratory findings is
most likely to be decreased in this patient?

A) Hematocrit
B) Leukocyte count
C) Platelet count
D) Serum direct bilirubin concentration
E) Serum indirect bilirubin concentration
Correct Answer: A.

Hemolytic disease of the newborn can be caused by red blood cell antigen alloimmunization in the mother. While this most commonly involves the Rh{D) antigen, hemolysis can also occur as a result of the
incompatibility of other red blood cell antigens, such as the A and 8 antigens. This is of particular concern in mothers with blood group O blood who have antibodies against A and B. This incompatibility can
result in hemolytic anemia of the newborn, as seen in this patient, caused by the transposition of these lgG antibodies across the placenta and their subsequent attack of newborn erythrocytes. Thus, this
patient with profound jaundice born to a multiparous mother with blood group O blood is most likely to have decreased hematocrit on laboratory evaluation as a result of hemolysis.

Incorrect Answers: B, C, D, and E.

Leukocyte count (Choice B) would not be decreased in this patient. As the maternal antibodies are directed against A and 8 antigens on red blood cells, the white blood cells should not be affected.

Platelet count (Choice C) would not be decreased in this patient. As the maternal antibodies are directed against A and B antigens on red blood cells, the platelet count should not be affected.

Serum direct bilirubin concentration (Choice 0) and serum indirect bilirubin concentration (Choice E) would likely be increased in the setting of hemolysis. However, given the newborn's limited conjugation
system, the indirect bilirubin concentration would be increased to a greater degree than the direct concentration. The increased bilirubin concentration in this patient's serum is responsible for her jaundice.

Educational Objective: Hemolytic disease of the newborn can occur in children born to mothers with blood group O blood as a result of the transposition of lgG antibodies against A and B antigens across the
placenta and their subsequent attack of newborn erythrocytes. Unlike Rh(D) isoimmunization, ABO incompatibility more commonly affects the newborn than the fetus. It presents postnatally with hemolysis and
subsequent jaundice and anemia.

https://t.me/USMLENBME2CK ti e
Previous Next Score Report Lab Values Calculator Help pause
Exam Section 3: Item 1 of 50 National Board of Medical Examiners
Comprehensive Clinical Science Self-Assessment

✓ 1. A 48-year-old man is admitted to the hospital for management of acute respiratory failure. He has a 4-day history of fever, an increase in thick, yellow-green sputum production, and diffuse muscular
aches. During the past 2 years, he has produced 2 lo 3 tablespoons of yellow sputum daily. He has smoked two packs of cigarettes daily for 30 years. He is 183 cm (6 ft) tall and weighs 134 kg
(295 lb); BMI is 40 kg/m?. On admission, his temperature is 38.6°C (101.5°F), pulse is 120/min, respirations are 30/min, and blood pressure is 170/100 mm Hg. Pulse oximetry on room air shows an
oxygen saturation of 60%. On admission, an AP portable x-ray shows bilateral diffuse interstitial infiltrates without consolidation. He does not respond to high-flow oxygen therapy. He is intubated and
mechanically ventilated. Examination shows hyperresonance to percussion and wheezes bilaterally. Ceftriaxone and levofloxacin are administered, and his symptoms improve. On hospital day 4, he is
extubated. His temperature is 37 .6°C (99.7F), pulse is 100/min, respirations are 22/min, and blood pressure is 150/96 mm Hg. Pulse oximetry while sitting shows an oxygen saturation greater than
95%. Six hours later, the patient's oxygen saturation is 86%. The patient is lying in bed and says he feels fine. Physical examination shows no wheezing, crackles, or rhonchi. There is no peripheral
edema. A repeat chest x-ray shows improvement. Which of the following is the most appropriate next step in management to improve this patient's oxygen saturation?

A) Increase the oxygen concentration of inspired air


B) Increase the positive end-expiratory pressure
C) Place the patient in Trendelenburg position
D) Place the patient's head on three pillows
E) Return the patient to a sitting position
Correct Answer: E.

Intubation and mechanical ventilation are indicated in patients who are apneic, hypoxic, or hypercapnic; not responding to less invasive means; or are unable to protect their airway (eg, lethargic, no gag or
cough reflex). After intubation and mechanical ventilation, patients may have a component of respiratory insufficiency caused by prolonged dependence on a ventilator. Respiratory muscles can atrophy,
leading to respiratory insufficiency. In this patient, hypoxia is not caused by pulmonary edema, infection, or chronic obstructive pulmonary disease (COPO) exacerbation, and is likely caused by respiratory
muscle atrophy after several days of intubation. Additionally, this patient has morbid obesity, which can cause respiratory insufficiency through mechanisms such as chest wall restriction when supine and
suppression of central respiratory drive. Exercises such as pulmonary physiotherapy, repositioning to a sitting or standing position, and coughing can improve oxygenation and respiratory muscle use.

Incorrect Answers: A, B, C, and D.

Increasing the oxygen concentration of inspired air (Choice A), such as supplying oxygen via nasal cannula, would be a temporary measure to improve oxygenation but would not address the underlying
respiratory insufficiency. The patient requires respiratory exercises and support to improve respiratory function.

Increasing the positive end-expiratory pressure (Choice 8), such as with noninvasive positive pressure, would be indicated in cases of respiratory distress secondary lo pulmonary edema or COPD
exacerbation. This patient has no respiratory distress, wheezing, or rales.

Placing the patient in Trendelenburg position (Choice C) would not be helpful, as intra-abdominal contents and gravity would decrease lung compliance, worsening respiratory insufficiency.

Placing the patient's head on three pillows (Choice 0) would be helpful in the management of orthopnea in cases of pulmonary edema, such as a heart failure exacerbation, provided that the torso was also
somewhat elevated. This patient does not have rales, making this diagnosis unlikely.

Educational Objective: Mechanical ventilation can lead to respiratory muscle atrophy and insufficiency. Exercises such as physiotherapy, coughing, repositioning, and moving can improve respiratory
sufficiency after mechanical ventilation.

https://t.me/USMLENBME2CK ts e t
Previous Next Score Report Lab Values Calculator Help pause
Exam Section 3: Item 2 of 50 National Board of Medical Examiners
Comprehensive Clinical Science Self-Assessment

"I 2. A 24-year-old female medical student sustains a deep laceration of her left index finger while assisting the surgeon during an operation on a patient with cervical cancer. The surgical patient has a
history of multiple sexual partners and intravenous drug use; it is not yet known if she is HIV positive. Examination of the medical student shows a 2-cm, bleeding laceration; the bleeding stops with
compression. Which of the following is the most appropriate next step in management pending results of HIV testing on the surgical patient?

A) Acyclovir therapy
B) Antiretroviral therapy
C) Corticosteroid therapy
D) Nafcillin therapy
E) No treatment is indicated
Correct Answer: B.

Occupational exposure to HIV is common in healthcare personnel as a result of needlestick injuries and other surgical injuries, such as small lacerations or punctures. The rate of seroconversion following
accidental HIV exposure is low (estimated at approximately 0.25 to 0.35%). The risk for seroconversion is dependent upon several factors, including the patient's HIV viral load, the depth of injury, puncture
with a hollow-bored as opposed to solid needle, or injury with a visibly contaminated instrument. Immediately following exposure, the area should be copiously washed with soap and water. Baseline and
serial serologic testing for HIV should be performed at the time of exposure and up to 6 months following exposure. If unknown, the patient's HIV status should be determined immediately. Post-exposure
prophylaxis should be instituted as soon as possible following a known or possible exposure to HIV. A variety of regimens are frequently employed, commonly a multidrug antiretroviral regimen including two
nucleoside/nucleotide reverse transcriptase inhibitors, such as tenofovir or emtricitabine, and an integrase inhibitor, such as dolutegravir or raltegravir. All needlestick or surgical injuries should be
reported promptly so that appropriate testing and follow up can be arranged.

Incorrect Answers: A, C, D, and E.

Acyclovir therapy (Choice A) is incorrect. Acyclovir is an inhibitor of viral DNA polymerase. It has little to no activity against HIV and is used primarily in the management of herpesvirus infections.

Corticosteroid therapy (Choice C) is incorrect. Administration of corticosteroids will result in immune suppression and is not helpful for reducing HIV transmission.

Nafcillin therapy (Choice 0) is incorrect. Prophylactic systemic antibiotic therapy is not necessary following occupational injuries among healthcare personnel and is not useful for preventing transmission of
HIV.

No treatment (Choice E) is incorrect. While the risk for seroconversion following a surgical or needle-stick exposure to HIV is low, the use of post-exposure prophylaxis is supported by high-quality evidence
and minimizes the risk for HIV transmission.

Educational Objective: Occupational exposure to HIV is common in healthcare personnel as a result of needlestick injuries and surgical injuries, such as lacerations or punctures. The rate of seroconversion
following accidental HIV exposure is low. Immediately following exposure, the area should be washed with soap and water. Post-exposure prophylaxis should be instituted following a known or possible
exposure to HIV. Multidrug antiretroviral regimens including two nucleoside/nucleotide reverse transcriptase inhibitors and an integrase inhibitor are frequently employed.

https://t.me/USMLENBME2CK ts e t
Previous Next Score Report Lab Values Calculator Help pause
Exam Section 3: ltem 3 of 50 National Board of Medical Examiners
Comprehensive Clinical Science Self-Assessment

✓ 3. A 10-month-old female infant is brought lo the office by her parents for a follow-up examination 3 weeks after undergoing staged repair of a left complete unilateral cleft lip and palate. The procedure
and recovery were uncomplicated, and the infant was discharged home 2 days later. The parents report she is feeding well with a traditional bottle nipple and now also consumes some soft baby
foods. She has not had fever, pain, or nasal regurgitation. She receives no medications. Today, she is at the 25th percentile for length and weight. Vital signs are normal. Physical examination
discloses a well-healing surgical incision on the left upper vermilion and philtrum; the palate also appears to be healing well without wound dehiscence or palatal holes. This patient's parents should be
informed that she is at increased risk for developing which of the following?

A) Chronic colonization with group A streptococcus


B) Dyslexia
C) Peritonsillar abscess
D) Severe myopia
E) Velopharyngeal insufficiency
Correct Answer: E.

Cleft lip and cleft palate may occur in either complete (extending through the nares) or incomplete (not involving the nares) fashions and may occur unilaterally or bilaterally. Cleft lip occurs from a failure of
fusion of the maxillary and nasal prominences. Cleft palate occurs from a failure of fusion of the palatine prominences. Cleft lip and palate occur as isolated developmental anomalies or in the setting of
chromosomal abnormalities (eg, Palau syndrome or trisomy 13) or syndromes of midfacial hypoplasia (eg, Treacher Collins syndrome). Treatment requires surgical repair. Velopharyngeal insufficiency
(VPI) is a common sequela of cleft palate, with or without repair. VPI is defined as an inability to properly close the velopharyngeal port separating the nasopharynx and oropharynx during the production of
speech, which is frequently caused by a shortened palate or persistent palatal defects. VPI can lead to high-pitched, hypernasal, or whistling speech with reduced intelligibility. Persistent or severe VPI may
require additional surgical repair.

Incorrect Answers: A, B, C, and D.

Chronic colonization with group A streptococcus (Choice A) is typically not more common in children with cleft lip and palate. Some evidence has suggested that Staphylococcus aureus is more common in
these children.

Dyslexia (Choice B) is incorrect. There is some evidence that children with cleft lip and palate, especially in association with syndromic causes, have higher rates of language disorders, including dyslexia,
although this remains controversial.

Peritonsillar abscess (Choice C) is a localized collection of pus in the peritonsillar space and may occur following tonsillitis. It is not a common consequence of cleft lip and palate repair.

Severe myopia (Choice 0) is less likely. While Stickler syndrome may present with myopia as well as cleft lip and palate, this is rare. VPI is a more common consequence of cleft palate either with or
without surgical repair.

Educational Objective: Velopharyngeal insufficiency (VPI) is a common sequela of cleft palate, with or without surgical repair. VPI is defined as an inability to properly close the velopharyngeal port
separating the nasopharynx and oropharynx during the production of speech, which is frequently caused by a shortened palate or persistent palatal defects. VPI can lead to high-pitched, hypemasal, or
whistling speech with reduced intelligibility.

https://t.me/USMLENBME2CK ti
Previous Next Score Report Lab Values Calculator Help pause
Exam Section 3: Item 4 of 50 National Board of Medical Examiners
Comprehensive Clinical Science Self-Assessment

"I 4. A22-year-old man is brought to the physician by his parents because he has become newly preoccupied with religion during the past year. He lives with his parents and attends a community college.
Two months ago, he stopped attending his classes. His parents report that he spends most of the day in his room alone reading religious scripture. He shows no interest in activities he once enjoyed, no
longer spends lime with his friends, masturbates frequently and obviously, and spends most of the night walking around his room laughing and talking to himself. He has no history of serious illness and
takes no medications. Vital signs are within normal limits. Physical examination shows no abnormalities. On mental status examination, he has a depressed mood and animated affect. His speech is
pressured, and responses to questions are tangential. He reports hearing a voice during the past 2 months that he believes is "the voice of God." He says that he feels confused and has thoughts about
ending his life because he cannot understand if God truly wants him to become the next Messiah. Which of the following is the most likely diagnosis?

A) Delusional disorder
B) Obsessive-compulsive disorder
C) Schizoid personality disorder
D) Schizophrenia
E) Schizotypal personality disorder
Correct Answer: D.

According to the DSM-5, patients with schizophrenia demonstrate at least two of the following five symptoms: delusions, hallucinations, disorganized speech, disorganized behavior, or negative symptoms
(eg, flat affect, apathy, alogia). Many patients also illustrate prodromal periods of strange behavior and social withdrawal. To meet diagnostic criteria, patients must demonstrate functional deficits, and the
total duration of symptoms must exceed 6 months. The cause is thought to include dopamine dysregulation, and brain imaging typically shows decreased cortical volume and enlarged ventricles. The
treatment of schizophrenia is with antipsychotics, which can be divided into two groups: typical (haloperidol, fluphenazine, chlorpromazine) and atypical (risperidone, quetiapine, olanzapine).

Incorrect Answers: A, B, C, and E.

Delusional disorder (Choice A) features the presence of one or more delusions for a month or longer with no other symptoms of psychosis. The presence of hallucinations and disordered speech and
behavior are more consistent with a diagnosis of schizophrenia.

Obsessive-compulsive disorder (Choice B) manifests with recurrent, intrusive urges or thoughts (obsessions) that the patient tries to relieve or neutralize by repetitive rituals (compulsions). Common
symptom complexes revolve around cleanliness, symmetry, harm to self or others, taboo thoughts or ideas, and completeness. Symptoms are time-consuming, cause significant distress, and markedly
impair functioning.

Schizoid personality disorder (Choice C), a cluster A personality disorder (the odd or eccentric cluster), manifests as extreme social detachment and a cold, restricted affect. In contrast to patients with
schizophrenia, they do not typically display disorganized speech and behavior or experience hallucinations.

Schizotypal personality disorder (Choice E) is a cluster A personality disorder characterized by odd behavior and/or magical thinking and a constricted affect. Delusions, hallucinations, and disorganized
speech and behavior, as in this patient, would be atypical for patients with schizotypal personality disorder.

Educational Objective: Schizophrenia is characterized by at least 6 months exhibiting two of the following five symptoms: delusions, hallucinations, disorganized speech, disorganized behavior, or negative
symptoms (eg, flat affect, apathy, alogia).

https://t.me/USMLENBME2CK ts e t
Previous Next Score Report Lab Values Calculator Help pause
Exam Section 3: Item 5 of 50 National Board of Medical Examiners
Comprehensive Clinical Science Self-Assessment

"I 5. A 26-year-old primigravid woman has spontaneous rupture of the membranes at 30 weeks' gestation. She is afebrile and has no contractions. A sterile speculum examination shows a closed cervix and
a pool of clear fluid in the vagina. Which of the following is the most likely complication?

A) Amniotic band syndrome


B) Fetal deformation
C) Fetal hypoplastic lungs
D) Preterm delivery
E) Umbilical cord occlusion
Correct Answer: D.

Premature rupture of membranes refers to the rupture of the amniotic sac before the onset of labor (regular contractions plus cervical change). If it occurs before 37 weeks' gestation, it is termed preterm
premature rupture of membranes (PPROM). Patients typically report a gush of fluid or steady leakage of fluid from the vagina. Initial evaluation includes a sterile speculum examination of the vagina, which
may show pooling of amniotic fluid or leakage of fluid from the cervical os. Nitrazine testing of the fluid will show a pH of 7.0 to 7.3, and terning will be seen on microscopic examination. Ultrasonography can
also be utilized to determine the amount of residual amniotic fluid, which would be decreased in the setting of the rupture of membranes. Risk factors include previous premature rupture of membranes, a
vaginal or cervical infection, and cigarette smoking. Patients with PPROM are at increased risk for preterm delivery, as the rupture of membranes usually precipitates the onset of labor. Thus, these patients
are treated with betamethasone to accelerate fetal lung maturity. Antibiotics are commonly administered prophylactically as a result of an increased risk for chorioamnionitis. The most likely complication in
this patient presenting with PPROM is preterm delivery.

Incorrect Answers: A, B, C, and E.

Amniotic band syndrome (Choice A) occurs when a band of amniotic tissue compresses a portion of the developing fetus, which can cause limb defects and craniofacial abnormalities. It is often diagnosed
prenatally on ultrasonography. Premature rupture of membranes is not a risk factor for its development.

Fetal deformation (Choice B) includes a variety of anatomic defects. While some of these may predispose the patient lo preterm delivery, fetal defects are not a risk factor for PPROM. Similarly, PPROM is
not known to cause fetal defects.

Fetal hypoplastic lungs (Choice C) can occur as a result of PPROM, as oligohydramnios prevents the development of the lungs with intrauterine fetal breathing. However, this is more common in neonates
that are born at the extremes of prematurity and generally requires an amount time with oligo- or -anhydramnios beyond the duration from onset of PPROM lo delivery. Additionally, this patient is not in labor
which provides additional time for betamethasone to improve fetal lung maturity.

Umbilical cord occlusion (Choice E) can lead to fetal hypoxia and death if the compression is severe and ongoing. While severe oligohydramnios in the setting of rupture of membranes can predispose a
fetus to umbilical cord compression, preterm labor is a more likely outcome of this patient's presentation.

Educational Objective: Preterm premature rupture of membranes (PPR OM) refers to the rupture of the amniotic sac before the onset of labor and before 37 weeks' gestation. Risk factors include previous
premature rupture of membranes, a vaginal or cervical infection, and cigarette smoking. Patients with PPROM are at increased risk for preterm delivery, as the rupture of membranes usually precipitates the
onset of labor. For this reason, these patients are treated with betamethasone to accelerate fetal lung maturity.

https://t.me/USMLENBME2CK ts e t
Previous Next Score Report Lab Values Calculator Help pause
Exam Section 3: Item 6 of 50 National Board of Medical Examiners°
Comprehensive Clinical Science Self-Assessment

6. An 8-year-old African American girl with sickle cell disease is brought to the physician by her mother because of a 12-hour history of cough and difficulty breathing. Her medications are penicillin and
folic acid. Her temperature is 39C (102.2F), pulse is 130/min, respirations are 40/min, and blood pressure is 105/60 mm Hg. Pulse oximetry on room air shows an oxygen saturation of 94%. On
pulmonary examination, crackles are heard in the upper lobes. Breath sounds are decreased at the lung bases bilaterally. The remainder of the examination shows no abnormalities. An x-ray of the
chest is shown. Which of the following is the most appropriate next step in diagnosis?

A) Anti-Mycop/asma pneumoniae lgG titer


B) Rapid antigen detection test for respiratory syncytial virus
C) Blood culture
D) Spiral CT scan of the chest
E ) Thoracentesis
Correct Answer: C.

« https://t.me/USMLENBME2CK ts e t
Previous Next Score Report Lab Values Calculator Help pause
Exam Section 3: Item 6 of 50 National Board of Medical Examiners
Comprehensive Clinical Science Self-Assessment

A) Anti-Mycop/asma pneumoniae lgG titer


B) Rapid antigen detection test for respiratory syncytial virus
C) Blood culture
D) Spiral CT scan of the chest
E ) Thoracentesis
Correct Answer: C.

Patients with sickle cell disease (SCO) are at increased risk for rapidly progressive, life-threatening infections as a result of hyposplenia and accompanying immunocompromise. Acute chest syndrome (ACS)
occurs as a result of vaso-occlusion within the pulmonary microvasculature and is diagnosed in patients with SCO who present with a new pulmonary infiltrate in the setting of fever or other new respiratory
symptoms, including cough, shortness of breath, increased work of breathing, tachypnea, or hypoxemia. ACS is a leading cause of mortality in patients with SCO. While ACS is a distinct clinical syndrome, it
is commonly precipitated by another illness such as bacterial or viral pneumonia, asthma exacerbation with hypoventilation, or fat emboli. Plain film x-rays typically show segmental or subsegmental
consolidation, or a pleural effusion, with predilection for involvement of the lower lobes. It is both impractical and nearly impossible to distinguish ACS from bacterial pneumonia in patients with SCO. In
patients with SCO who present with fever or ACS, obtaining blood cultures is essential to promptly identify bacteremia. Empiric antibiotic therapy should be initiated immediately after blood cultures are
obtained and should not be delayed pending further workup. Further treatment involves analgesia, maintaining normoxia with supplemental oxygen or the use of noninvasive positive pressure ventilation,
judicious use of fluids, and aggressive pulmonary hygiene measures, including the use of an incentive spirometer.

Incorrect Answers: A, B, D, and E.

Anti-Mycop/asma pneumoniae lgG titer (Choice A) is useful for identifying prior exposure to Mycop/asma pneumoniae. While patients with SCO are at increased risk for M. pneumoniae infection,
mycoplasma infections are covered by empiric therapy, and antibody tilers are often unnecessary.

Rapid antigen detection test for respiratory syncytial virus (Choice B) may be useful for the diagnosis of bronchiolitis, which presents with a viral prodrome of fever, nasal congestion, and rhinorrhea followed
by cough, dyspnea, and lethargy. While evaluating for possible viral causes of ACS is important, obtaining blood cultures is more important, as bacteremia and sepsis can be acutely life-threatening.

Spiral CT scan of the chest (Choice 0) is useful for the identification of pulmonary embolism, which is often not evident on plain film x-rays. While adults with SCO are at increased risk for venous
thromboembolism in the setting of ACS, this complication is less frequent in children and if there is concern, it can be assessed after blood cultures are obtained.

Thoracentesis (Choice E) is useful for the diagnosis and management of a pleural effusion. While this patient's chest x-ray shows small pleural effusions, obtaining blood cultures is a more important
immediate step for preventing morbidity and mortality.

Educational Objective: Patients with sickle cell disease are at increased risk for rapidly progressive, life-threatening infections as a result of hyposplenia and accompanying immunocompromise.
Differentiating bacterial pneumonia from acute chest syndrome (ACS) is often impossible. Patients who present with fever or suspected ACS should be emergently evaluated with a blood culture, followed
immediately by empiric antibiotic therapy.

https://t.me/USMLENBME2CK ti
Previous Next Score Report Lab Values Calculator Help pause
Exam Section 3: Item 7 of 50 National Board of Medical Examiners
Comprehensive Clinical Science Self-Assessment

"I 7. A 58-year-old man is admitted lo the hospital for treatment of anemia. Two days ago, he had excessive blood loss following colon polyp removal via colonoscopy. Today, his hemoglobin concentration is
6.5 g/dl and hematocrit is 19.5%. Two months ago, in order to decrease transfusion errors, a blood lock system was implemented in which the patient ID must be entered into the lock to open the blood
container; the container cannot be opened without the correct ID. Requiring staff to enter the patient's ID to open the blood lock is best described as which of the following?

A) An affordance
B) A forcing function
C) An interruption in care
D) Mapping of the design of the blood container
E) A policy leading to delays in care
Correct Answer: B.

A forcing function is a design feature that prevents a desired action from occurring if a prerequisite action is not performed first. Forcing functions reduce human error by anticipating mistakes that are likely lo
occur and requiring a verification step or other intermediary step to reduce the likelihood of such an error occurring. In this case, a common error in blood transfusion involves transfusion of the incorrect blood
type or blood product, or administration of a product to the wrong patient. The blood container forcing function requires verification of patient identity before allowing the blood product to be removed. Forcing
health care staff to perform this verification reduces the likelihood of transfusion error.

Incorrect Answers: A, C, D, and E.

An affordance (Choice A) is defined in human factors engineering as the functional potential of an individual or object within its environment. Within health care, this term is often applied to the design of a
medical instrument to describe the way in which the instrument's design suggests or conveys how the instrument should be used.

An interruption in care (Choice C) and a policy leading to delays in care (Choice E) are incorrect. While forcing functions delay the immediate administration of care, they do so deliberately in order to reduce
the likelihood of human error. Verification processes maximize patient safety and reflect a normal part of the administration of care, rather than an interruption or delay.

Mapping of the design of the blood container (Choice 0) is incorrect. Process mapping is a method of describing the function of a process or device, in this instance a blood container, such that the steps of its
use and the potential opportunities for error can be identified.

Educational Objective: A forcing function is a design feature that prevents a desired action from occurring if a prerequisite action is not performed first. Forcing functions reduce human error by anticipating
mistakes that are likely to occur and requiring a verification step or other intermediary step to reduce the likelihood of such an error occurring.

https://t.me/USMLENBME2CK ts e t
Previous Next Score Report Lab Values Calculator Help pause
Exam Section 3: Item 8 of 50 National Board of Medical Examiners
Comprehensive Clinical Science Self-Assessment

"I 8. Twenty-four hours after admission to the hospital for treatment of temperatures to 38.5°C(101.3F) and abdominal pain, a 22-year-old man has a decrease in his temperature. Two weeks ago, he went
on a 5-day trip to South America and had diarrhea for 7 days. He has no previous history of serious illness. On admission, a CT scan of the abdomen showed a 5-cm mass in the right lobe of the liver;
the mass was filled with fluid. He has received metronidazole, ampicillin, and gentamicin since admission. His current temperature is 37.4"C (99.3°F), and blood pressure is 120/75 mm Hg. The
abdomen is flat with moderate right upper quadrant tenderness and guarding. There are no peritoneal signs. A serum liter for antibodies to Entamoeba histolytica is positive at 1:128. Which of the
following is the most appropriate next step in management?

A) Continue the current antibiotic regimen for 10 days


B) Discontinue ampicillin and gentamicin and continue metronidazole
C) Perform examination of the stool for ova and parasites
D) Perform CT scan-guided aspiration of the mass
E) Perform surgical drainage of the mass
Correct Answer: B.

Infection with Entamoeba histolytica leads to amoebiasis, or amoebic dysentery. E. histolytica is an amoebic parasite that forms cysts, which can be transmitted through the fecal-oral route, typically through
ingestion of contaminated water. Amoebic cysts mature to trophozoites in the large bowel, which may invade the colonic mucosa and interstitium, leading to symptoms of abdominal pain and bloody diarrhea,
or may enter the bloodstream and cause hepatic amoebic abscesses, which present with fever and right upper quadrant pain. The diagnosis can be made by identifying cysts on stool microscopy or
peripheral blood smear, which may show trophozoites with endosomes containing red blood cells, as well as by antibody assay or polymerase chain reaction. Colonoscopy shows mucosal ulcers that are
often flask-shaped. Treatment of hepatic amoebic abscesses involves combination therapy with metronidazole or tinidazole as well as an antiluminal agent such as paromomycin or iodoquinol. In this
otherwise stable patient with a confirmed diagnosis, it is most appropriate to discontinue broad-spectrum antibiotic treatment with ampicillin and gentamicin.

Incorrect Answers: A, C, D, and E.

Continuing the current antibiotic regimen for 10 days (Choice A) is unnecessary. In general, uncomplicated amoebic liver abscesses can be treated with amoebicidal agents alone.

Performing examination of the stool for ova and parasites (Choice C) is a useful method of diagnosing amoebiasis, but it is unnecessary for this patient in whom the diagnosis has already been confirmed
with a serum antibody assay.

Performing CT scan-guided aspiration (Choice D) or surgical drainage (Choice E) of the mass is unnecessary given this patient's improvement with conservative management. Surgical drainage may be
necessary for large abscesses greater than 5 cm in diameter, abscesses in the left lobe of the liver (which is associated with an increased risk for peritoneal rupture), or patients who fail to respond to
antibiotic therapy.

Educational Objective: Infection with Entamoeba histolytica leads to amoebiasis, which can lead to bloody diarrhea and hepatic liver abscesses. Treatment of hepatic amoebic abscesses involves
combination therapy with metronidazole or tinidazole as well as an antiluminal agent such as paromomycin or iodoquinol. Surgical drainage is often unnecessary.

https://t.me/USMLENBME2CK ti
Previous Next Score Report Lab Values Calculator Help pause
Exam Section 3: Item 9 of 50 National Board of Medical Examiners°
Comprehensive Clinical Science Self-Assessment

"I 9. A previously healthy 23-year-old woman is brought to the emergency department 20 minutes after the sudden onset of shortness of breath,
moderate abdominal pain, nausea, and an itchy rash over the abdomen. Her symptoms began after she ate a sugar cookie given to her by a
friend. The patient had a peanut allergy as a child and before eating the cookie, she asked her friend to taste it. Neither she nor her friend tasted or
smelled peanuts. The patient's only medication is an oral contraceptive. Her last menstrual period was 1 week ago. She has smoked one pack of
cigarettes daily for 5 years. She drinks three 12-oz beers on weekends. She has been staying at different friends' homes during the past 2 months
while searching for an apartment and has been using unfamiliar laundry detergents. She is in mild respiratory distress but is able to speak in full
sentences. Her pulse is 110/min, respirations are 28/min, and blood pressure is 100/60 mm Hg. Pulse oximetry on room air shows an oxygen
saturation of 94 %. A photograph of the rash is shown. Scattered wheezes are heard bilaterally. Cardiac examination shows no abnormalities.
Abdominal examination shows mild, diffuse tenderness to palpation; there is no rebound or guarding. Which of the following is the most
appropriate initial pharmacotherapy?

A) Albuterol
B) Diphenhydramine
C) Epinephrine
D) Furosemide
E) Nitroglycerin
F) Prednisone

Correct Answer: C.

An anaphylactic reaction is a hypersensitivity reaction to an exogenous factor, most commonly medications, foods, or insect stings. II is mediated by activation of lgE, which leads to the release of histamine
and other cytokines from mast cells and basophils. This subsequently leads to acute inflammation and profound vasodilation in multiple organ systems, generally the cardiovascular, respiratory,
gastrointestinal, and integumentary systems. Symptoms include the rapid onset of bronchospasm with dyspnea or wheezing, urticaria, flushing, nausea and emesis, edematous lips and tongue, and
hypotension with tachycardia. When severe and untreated, ii can progress to circulatory collapse with cardiovascular arrest or death caused by asphyxiation and airway obstruction. Treatment includes the
administration of epinephrine, antihistamines, bronchodilators, and glucocorticoids, as well as intravenous fluids. Severe cases may require endotracheal intubation and the administration of vasopressors.
This patient with a history of peanut allergy consumed a cookie that likely contained traces of peanuts, leading to an anaphylactic reaction with her symptoms of dyspnea, abdominal pain, nausea, urticaria,
and wheezing. Therefore, the most appropriate initial treatment is epinephrine, as it treats the underlying mechanisms leading to the manifestations of anaphylaxis. Epinephrine causes vasoconstriction,
bronchodilation, and decreased cytokine release. Delays to epinephrine administration increase the risk of death.

Incorrect Answers:A.B D) F and F


« https://t.me/USMLENBME2CK ts e t
Previous Next Score Report Lab Values Calculator Help pause
Exam Section 3: Item 9 of 50 National Board of Medical Examiners"
Comprehensive Clinical Science Self-Assessment

F) Prednisone •

Correct Answer: C.

An anaphylactic reaction is a hypersensitivity reaction to an exogenous factor, most commonly medications, foods, or insect stings. II is mediated by activation of lgE, which leads to the release of histamine
and other cytokines from mast cells and basophils. This subsequently leads to acute inflammation and profound vasodilation in multiple organ systems, generally the cardiovascular, respiratory,
gastrointestinal, and integumentary systems. Symptoms include the rapid onset of bronchospasm with dyspnea or wheezing, urticaria, flushing, nausea and emesis, edematous lips and tongue, and
hypotension with tachycardia. When severe and untreated, ii can progress to circulatory collapse with cardiovascular arrest or death caused by asphyxiation and airway obstruction. Treatment includes the
administration of epinephrine, antihistamines, bronchodilators, and glucocorticoids, as well as intravenous fluids. Severe cases may require endotracheal intubation and the administration of vasopressors.
This patient with a history of peanut allergy consumed a cookie that likely contained traces of peanuts, leading to an anaphylactic reaction with her symptoms of dyspnea, abdominal pain, nausea, urticaria,
and wheezing. Therefore, the most appropriate initial treatment is epinephrine, as it treats the underlying mechanisms leading to the manifestations of anaphylaxis. Epinephrine causes vasoconstriction,
bronchodilation, and decreased cytokine release. Delays to epinephrine administration increase the risk of death.

Incorrect Answers: A, B, D, E, and F.

Albuterol (Choice A) and diphenhydramine (Choice B) are used in the treatment of anaphylaxis, as are other antihistamines, such as famotidine, and glucocorticoids, such as methylprednisolone. However,
the initial pharmacotherapy chosen for treating anaphylaxis should be epinephrine.

Furosemide (Choice D) is a loop diuretic most used in the management of congestive heart failure. An acute exacerbation of congestive heart failure can present with dyspnea, tachypnea, and hypoxia
caused by pulmonary edema. However, this patient is young and has no cardiac medical history, making new congestive heart failure a less likely diagnosis than anaphylaxis.

Nitroglycerin (Choice E) can be used to treat the anginal chest pain that occurs with myocardial infarction, as it decreases preload to the heart by causing venodilation. This consequently leads to decreased
myocardial demand and decreased chest pain. This patient has wheezing and urticaria, which are less likely to be seen with a myocardial infarction than with anaphylaxis. Additionally, the administration of
nitroglycerin would likely lead to a decrease in this patient's already low blood pressure.

Prednisone (Choice F) may be useful in the treatment of allergic or autoimmune reactions. However, intravenous glucocorticoids, such as methylprednisolone, are more often chosen in the acute
management of anaphylaxis given a more rapid onset of action and avoidance of aspiration risks with oral administration. Epinephrine is the most important pharmacotherapy in the treatment of anaphylaxis.

Educational Objective: Anaphylaxis is an lgE-mediated hypersensitivity reaction that leads to extensive degranulation of mast cells and basophils, which results in acute multisystem inflammation and
vasodilation. Symptoms include bronchospasm with associated dyspnea or wheezing, urticaria, flushing, nausea and emesis, edematous lips and tongue, and hypotension with tachycardia. Treatment
includes epinephrine, antihistamines, bronchodilators, glucocorticoids, and fluids.

« https://t.me/USMLENBME2CK ts e t
Previous Next Score Report Lab Values Calculator Help pause
Exam Section 3: Item 10 of 50 National Board of Medical Examiners
Comprehensive Clinical Science Self-Assessment

"I 10. A67-year-old man comes to the physician because he has noticed blood in his urine for the past 2 weeks. He has no prior history of blood in his urine. He has not had pain or any other symptoms. He
has hypertension controlled with a diuretic and hypercholesterolemia controlled with a lipid-lowering agent. His blood pressure is 120/76 mm Hg. Examination shows no abnormalities. Urinalysis shows
RBCs that are too numerous to count. Which of the following is the most appropriate next step in diagnosis?

A) Renal scintigraphy
B) Retrograde urethrography
C) MRI of the pelvis
D) Cystoscopy
E) Renal biopsy
Correct Answer: D.

Bladder cancer is the most common malignant tumor of the urinary tract, a majority of which are transitional cell carcinoma. Risk factors include schistosomiasis, chronic cyclophosphamide use, aniline dye
exposure, and tobacco smoking. Patients typically present with painless, gross hematuria. Diagnosis is confirmed with cystoscopy and biopsy, which is the most sensitive approach for the identification of small
mucosal tumors. Transitional cell carcinoma in situ refers to a lesion of precursor malignant cells that are confined to the epithelial lining of the bladder and have not yet invaded the subepithelial connective
tissue or muscle. Treatment requires resection of the cells with negative margins, which can be achieved through cystoscopic resection.

Incorrect Answers: A, B, C, and E.

Renal blood flow scan, also known as renal scintigraphy (Choice A), involves the use of an injected radiotracer to evaluate and quantify the flow of blood to the kidney.

Retrograde urethrography (Choice B) involves the injection of contrast into the urethra to evaluate for the extravasation of contrast, indicating urethral discontinuity in the setting of potential urethral trauma.
Urethral injuries typically present after straddle injuries or secondary to pelvic fractures in trauma.

MRI of the pelvis (Choice C) can help evaluate the extent of metastatic disease but would not be the next step in diagnosis. Diagnosis of bladder cancer begins with cystoscopy.

Renal biopsy (Choice E) is used in the diagnosis of nephrotic and nephritic syndromes, as well as with interstitial nephritis or to evaluate the extent of renal involvement in systemic lupus erythematosus. While
renal cell carcinoma also causes hematuria, it typically also presents with flank pain and weight loss. It is generally diagnosed with contrast-enhanced CT scan or MRI followed by biopsy.

Educational Objective: Bladder cancer typically presents with painless, gross hematuria. Diagnosis involves cystoscopy for direct visualization and identification of mucosal tumors.

https://t.me/USMLENBME2CK ts e t
Previous Next Score Report Lab Values Calculator Help pause
Exam Section 3: Item 11 of 50 National Board of Medical Examiners°
Comprehensive Clinical Science Self-Assessment

11. A57-year-old man comes to the emergency department because of a 3-week history of headaches that have been increasing in severity and frequency. He also has a 2-week history of progressive
blurred vision in both eyes. He has not had pain in the eyes. His last visit to a physician was 10 years ago because of pain in his fingers; al that visit, he was told he had high blood pressure. His
temperature is 37°C (98.6°F), pulse is 90/min, respirations are 12/min, and blood pressure is 200/110 mm Hg. Repeat blood pressure measurements are 210/112 mm Hg and 214/114 mm Hg. Findings
on funduscopic examination are shown. Examination shows ulnar deviation of both hands and several swollen interphalangeal Joints. His fasting serum glucose concentration is 160 mg/dL. Which of
the following is the most likely underlying cause of this patient's symptoms?

A) Diabetes mellitus
B) Glioblastoma multiforme
C) Hypertension
D) Idiopathic inlracranial hypertension
E ) Rheumatoid arthritis
Correct Answer: C.

Hypertensive retinopathy results from chronically increased blood pressure with subsequent microvasculopathy of the retinal arterial vasculature. Patients often report generalized blurry vision, although may
be asymptomatic, and may also report associated symptoms such as headache or chest pain. Typical funduscopic findings include constricted retinal arterioles, retinal hemorrhage, nerve fiber layer
infarctions, hard exudates, and lobular choroidal infarctions. The retinal hemorrhages typically occur in the inner retina and adopt a linear configuration, which allows them to be distinguished from the round
hemorrhages that occur in the middle retinal layers in the setting of diabetic retinopathy. Retinal edema and hard exudates typically conform to the radial orientation of the nerve fibers with respect to the
fovea, producing a characteristic macular star appearance. Optic disc edema may occur with disc hemorrhages and obscuration of the disc margin, mimicking papilledema. This patient's examination
findinas includina retinal hemorrhaaes nerve fiber infarctions hard exudates with macular star confiauration and bilateral optic disc edema are characteristic of severe hvoertensive retinopathy Patients
« https://t.me/USMLENBME2CK ts e t
Previous Next Score Report Lab Values Calculator Help pause
Exam Section 3: Item 11 of 50 National Board of Medical Examiners

. re-'>
the following is the most likely underlying cause of this patient's symptoms?
Comprehensive Clinical Science Self-Assessment
e!tr '-"- ---·-·· . .- - .. . - - -- -· . . .. - - .

A) Diabetes mellitus
B) Glioblastoma multiforme
C) Hypertension
D) Idiopathic intracranial hypertension
E ) Rheumatoid arthritis
Correct Answer: C.

Hypertensive retinopathy results from chronically increased blood pressure with subsequent microvasculopathy of the retinal arterial vasculature. Patients often report generalized blurry vision, although may
be asymptomatic, and may also report associated symptoms such as headache or chest pain. Typical funduscopic findings include constricted retinal arterioles, retinal hemorrhage, nerve fiber layer
infarctions, hard exudates, and lobular choroidal infarctions. The retinal hemorrhages typically occur in the inner retina and adopt a linear configuration, which allows them lo be distinguished from the round
hemorrhages that occur in the middle retinal layers in the setting of diabetic retinopathy. Retinal edema and hard exudates typically conform to the radial orientation of the nerve fibers with respect to the
fovea, producing a characteristic macular star appearance. Optic disc edema may occur with disc hemorrhages and obscuration of the disc margin, mimicking papilledema. This patient's examination
findings, including retinal hemorrhages, nerve fiber infarctions, hard exudates with macular star configuration, and bilateral optic disc edema, are characteristic of severe hypertensive retinopathy. Patients
with hypertensive crisis and evidence of end-organ damage should be managed emergently for appropriate blood pressure control.

Incorrect Answers: A, B, D, and E.

Diabetes mellitus (Choice A) causes diabetic retinopathy, which is characterized by round retinal hemorrhages, hard exudates, and retinal nerve fiber layer infarctions. The presence of linear retinal
hemorrhages and bilateral optic disc edema is not consistent with diabetic retinopathy.

Glioblastoma multiforme (Choice B) is an aggressive primary brain astrocytoma that carries a poor prognosis. Patients present with seizures, headache, and focal neurologic deficits. While papilledema may
be suggestive of increased intracranial pressure in the setting of a central nervous system lesion, this patient's linear retinal hemorrhages, hard exudates, and nerve fiber layer infarctions suggest that his
optic disc edema is caused by hypertension, and therefore does not represent papilledema.

Idiopathic intracranial hypertension (IIH) (Choice D), or pseudotumor cerebri, may also present with papilledema. IIH refers to an idiopathic increase in intracranial pressure that typically occurs in women of
childbearing age with obesity and presents with headaches that may be postural, visual deficits, nausea, and vomiting. Retinal findings, such as hemorrhages, hard exudates, and nerve fiber layer infarctions
are not typical of IIH.

Rheumatoid arthritis (Choice E) commonly produces findings within the anterior segment of the eye such as dry eye, episcleritis, scleritis, or sterile corneal ulceration.

Educational Objective: Hypertensive retinopathy is characterized by constricted retinal arterioles, linear retinal hemorrhages, retinal nerve fiber layer infarctions, lobular choroidal infarctions, and retinal hard
exudates. Optic disc edema may occur and can mimic papilledema. Patients with suspected hypertensive crisis should be managed emergently for appropriate blood pressure control.

https://t.me/USMLENBME2CK ti
Previous Next Score Report Lab Values Calculator Help pause
Exam Section 3: Item 12 of 50 National Board of Medical Examiners
Comprehensive Clinical Science Self-Assessment

"I 12. A case-control study is conducted to assess the effect of dietary fat intake on the occurrence of sudden cardiac death. One hundred fifty cases and controls are matched by age, sex assigned at birth,
race, and history of hypertension, hypercholesterolemia, and cigarette smoking. Dietary intake history of the cases is obtained from the next of kin. Results show an odds ratio of 1.6 with a 95%
confidence interval of 1.2 t0 1.9. The authors conclude that there is a relationship between dietary fat intake and sudden cardiac death. Which of the following raises the most concern about the
authors' conclusion?

A) Differential bias in assessing dietary fat


B) Inadequate sample size
C) Lack of blinding
D) Misclassification of cases and controls
E) Weak statistical association
Correct Answer: A.

Differential bias occurs when there is an unequal classification of exposure to a risk factor between groups with or without the outcome under study. In this scenario, the family members of patients who died
of sudden cardiac death are more likely to recall dietary exposure to fatty foods, in comparison to the family members of patients whose next of kin has not died of sudden cardiac death. Differential bias
differs from nondifferential bias, in which case there is an equal risk for exposure misclassification between groups with or without the outcome under study. Differential bias may lead to skewing of the odds
ratio (OR) toward or away from the null, depending on the experimental design. In this scenario, a higher rate of misclassification of exposure to fatty foods in the group that experienced sudden cardiac
death may lead to an erroneously high OR, skewing the results of the study.

Incorrect Answers: B, C, D, and E.

Inadequate sample size (Choice 8) is incorrect. The appropriate sample size for a given experiment is dependent upon several factors, including the desired power, expected margin of error, confidence
interval, and standard deviation.

Lack of blinding (Choice C) may occur in retrospective, case-control studies, but blinding may be accomplished by careful design of the questionnaire used to obtain information from subjects or next of kin,
as in this scenario. We are unable to say whether precautions have been taken regarding blinding in this case. This scenario is more concerning for the risk for differential bias when recalling exposure
status.

Misclassification of cases and controls (Choice D) may occur when subjects are inappropriately classified by outcome status. The outcome status under study in this scenario, sudden cardiac death, is
relatively unambiguous and does not raise significant concern for misclassification.

Weak statistical association (Choice E) is a result of poor correlation between the outcome and exposure status. In this scenario, there is a strong statistical association between exposure to fatty foods and
sudden cardiac death. However, the design of the experiment raises concern that this relationship may be affected by differential bias, leading to an erroneous result.

Educational Objective: Differential bias occurs when there is an unequal classification of exposure to a risk factor between groups with or without the outcome under study. In this scenario, the family
members of patients who died of sudden cardiac death are more likely to recall dietary exposure to fatty foods, in comparison to the family members of patients whose next of kin has not died of sudden
cardiac death. Differential bias may lead to skewing of the odds ratio toward or away from the null and may affect the results of the study.

https://t.me/USMLENBME2CK ts e t
Previous Next Score Report Lab Values Calculator Help pause
Exam Section 3: ltem 13 of 50 National Board of Medical Examiners
Comprehensive Clinical Science Self-Assessment

✓ 13. A67-year-old woman comes to the physician because of a 2-month history of hoarseness. She has not had any other symptoms. She has no history of serious illness and takes no medications. She
has smoked one pack of cigarettes daily for 45 years. Examination, including examination of the oropharynx, shows no abnormalities. Which of the following is the most appropriate next step in
management?

A) Follow-up examination in 6 weeks


B) X-rays of the maxillary sinuses
C) CT scan of the chest
D) Bronchodilator therapy
E) Indirect laryngoscopy
Correct Answer: E.

Laryngeal carcinoma may present with hoarseness, vocal changes, persistent cough, stridor, or dysphagia, and is more common in patients with a history of smoking. The vast majority of laryngeal
carcinomas are squamous, and location in the glottic or supraglottic region is frequent. Supraglottic lesions are likely to induce dysphagia, whereas hoarseness is an early symptom of glottic lesions as a
result of involvement of the vocal cords. The diagnosis is established by direct tumor visualization, which may be accomplished by indirect laryngoscopy. Thorough examination of the neck is also essential to
determine the extent of the tumor and to identify associated lymphadenopathy, which may indicate nodal metastasis. Distant metastases may also occur. CT scan or PET scan may be useful for
characterizing metastatic disease. Early stage laryngeal carcinoma is associated with a high survival rate, although locally extensive or metastatic disease is more difficult lo treat. Treatment involves surgical
excision, radiation therapy, and/or chemotherapy.

Incorrect Answers: A, B, C, and D.

Follow-up examination in 6 weeks (Choice A) is incorrect. Early diagnosis and treatment of laryngeal carcinoma is associated with superior outcomes and a higher rate of survival.

X-rays of the maxillary sinuses (Choice B) are useful for the identification of sinusitis, which typically presents with headache, facial pain, postnasal drip, or halitosis.

CT scan of the chest (Choice C) is an important component of the staging of laryngeal carcinoma and other cancers of the head and neck. Identification of the primary tumor through direct visualization is the
first step in diagnosis and should be performed before a CT scan.

Bronchodilator therapy (Choice D) is useful for the treatment of asthma and chronic obstructive pulmonary disease, which may occur in smokers. These diseases are characterized by airway disease with
subsequent wheezing. Hoarseness would be atypical and is indicative of laryngeal pathology.

Educational Objective: Laryngeal carcinoma is common in patients with a history of smoking and may present with hoarseness, vocal changes, persistent cough, stridor, or dysphagia. The diagnosis is
established by direct tumor visualization, which may be accomplished by indirect laryngoscopy.

https://t.me/USMLENBME2CK ti
Previous Next Score Report Lab Values Calculator Help pause
Exam Section 3: Item 14 of 50 National Board of Medical Examiners
Comprehensive Clinical Science Self-Assessment

✓ 14. A 57-year-old man is admitted to the hospital because of fever, shortness of breath, and cough productive of yellow sputum. He has prostate cancer metastatic to the bone; his only symptom from the
cancer is occasional back pain well controlled with oral acetaminophen and hydrocodone. At his last appointment 6 months ago, his serum prostate-specific antigen assay was stable, and his disease
had not progressed. He plays golf several times each week and enjoys traveling with his wife. His advance directive states that he does not want intubation or mechanical ventilation unless he has an
acute illness with a reasonable chance of recovery to a good quality of life. He is lethargic and unable to communicate. His temperature is 38.9°C (102°F), pulse is 100/min, respirations are 30/min,
and blood pressure is 148/92 mm Hg. Pulmonary examination shows shallow, rapid breathing. He is using accessory muscles of respiration. lnspiratory crackles are heard at the right upper and
lower lung fields. Arterial blood gas analysis on 100% oxygen shows:
pH 7.36
Pco, 45 mm Hg
Po, 42 mm Hg

An x-ray of the chest shows diffuse right-sided infiltrates. Which of the following is the most appropriate next step in management?

A) Consult the hospital ethics committee


B) Begin noninvasive positive pressure ventilation by face mask
C) Continue oxygen by face mask and administer intravenous morphine for comfort
D) Intubate and begin mechanical ventilation
Correct Answer: D.

Regardless of advance directives or prior code status documentation, physicians need to assess all patients' code status during each hospitalization, as the context of each hospitalization differs. The
patient or the patient's decision maker (eg, legal guardian or durable power of attorney) should be given the opportunity to update the patient's code status during each hospitalization. This patient is unable
to communicate, and his acute hypoxia necessitates an urgent decision by the physician as it represents an emergent and immediate life threat that is potentially reversible. Based on his advance directive
and current clinical condition, intubation and mechanical ventilation is appropriate given his reasonable chance of recovery with appropriate treatment.

Incorrect Answers: A, B, and C.

Consulting the hospital ethics committee (Choice A) is unnecessary since the patient has a clear advance directive outlining his wishes. This would also delay appropriate emergency care that is required to
sustain the patient's life.

Beginning noninvasive positive pressure ventilation by face mask (Choice B) and continuing oxygen by face mask and administering intravenous morphine for comfort (Choice C) are both inappropriate and
represent a misinterpretation of the patient's advance directive. This patient with lobar pneumonia has a good chance of recovery with treatment and should be immediately intubated. Noninvasive positive
pressure ventilation is a reasonable option for patients who are not at risk for airway compromise prior to intubation; however, that is not the case with this patient. His mental status, hypoxic respiratory
failure, and increased work of breathing together indicate that invasive mechanical ventilation is most appropriate.

Educational Objective: Physicians should assess a patient's code status during each hospitalization. The patient or the patient's decision maker (eg, legal guardian or durable power of attorney) should be
given the opportunity to update the patient's code status during each hospitalization.

https://t.me/USMLENBME2CK ti
Previous Next Score Report Lab Values Calculator Help pause
Exam Section 3: Item 15 of 50 National Board of Medical Examiners
Comprehensive Clinical Science Self-Assessment

"I 15. A 77-year-old man comes to the physician because of a 3-day history of passing air and fecal material during urination. He has a long-standing history of intermittent constipation. His last examination
was 20 years ago. He has smoked two packs of cigarettes daily for 55 years. He is 183 cm (6 ft) tall and weighs 70 kg (155 lb); BMI is 21 kg/m2. Vital signs are within normal limits. Abdominal
examination shows mild distention. Bowel sounds are decreased. Rectal examination shows a diffusely enlarged prostate. There is no stool in the rectal vault. Laboratory studies show:
Hemoglobin 14 g/dl
Leukocyte count 9600/mm3
Platelet count 444,000/mm3
Urine
WBC >100/hpf
Flora mixed

Which of the following is the most likely cause of this patient's symptoms?

A) Appendiceal abscess
B) Diverticular disease
C) Inflammatory bowel disease
D) Prostate cancer
E) Transitional cell carcinoma of the bladder
Correct Answer: B.

The layers of the gastrointestinal wall, from the luminal surface to the exterior of the bowel, include the mucosa, submucosa, muscularis propria, and serosa. Diverticulosis, also known as diverticular
disease, is characterized by sac-like protrusions of the colonic wall that are hypothesized to develop secondary to increased intraluminal pressure. This predisposes the herniation of mucosa and submucosa
through the muscularis propria of the colonic wall. Risk factors for the development of diverticulosis include abnormal colonic motility, constipation, high dietary intake of red meat and low dietary fiber,
physical inactivity, and obesity. Complications of diverticular disease include lower gastrointestinal bleeding and diverticulitis. Rarely, diverticular disease can lead to the development of a colovesical fistula in
which there is a connection between the rectum and the urinary bladder. This can lead to pneumaturia and passage of fecal material during urination, as well as the development of urinary tract infections,
seen in this patient with mixed flora and numerous leukocytes on urinalysis. Diagnosis involves contrast CT imaging or direct visualization. Treatment involves antibiotics and surgical repair.

Incorrect Answers: A, C, D, and E.

Appendiceal abscess (Choice A) can be a complication of untreated appendicitis. Appendicitis and appendiceal abscess would present with right lower quadrant abdominal pain and tenderness, and
generally fever, nausea, vomiting. It would not cause pneumaturia or passage of fecal material during urination.

Inflammatory bowel disease (Choice C) comprises both Crohn disease and ulcerative colitis. Symptoms include chronic abdominal pain, weight loss, bloody diarrhea, abdominal bloating, and tenesmus.
While colovesical fistulas can be a complication of inflammatory bowel disease, most patients diagnosed with inflammatory bowel disease are younger than age 30 years. This patient does not have a history
of abdominal pain or bloody diarrhea, both of which would be expected in the setting of inflammatory bowel disease.

https://t.me/USMLENBME2CK ts e t
Previous Next Score Report Lab Values Calculator Help pause
Exam Section 3: Item 15 of 50 National Board of Medical Examiners
Comprehensive Clinical Science Self-Assessment

Which of the following is the most likely cause of this patient's symptoms?

A) Appendiceal abscess
B) Diverticular disease
C) Inflammatory bowel disease
D) Prostate cancer
E) Transitional cell carcinoma of the bladder
Correct Answer: B.

The layers of the gastrointestinal wall, from the luminal surface to the exterior of the bowel, include the mucosa, submucosa, muscularis propria, and serosa. Diverticulosis, also known as diverticular
disease, is characterized by sac-like protrusions of the colonic wall that are hypothesized to develop secondary to increased inlraluminal pressure. This predisposes the herniation of mucosa and submucosa
through the muscularis propria of the colonic wall. Risk factors for the development of diverticulosis include abnormal colonic motility, constipation, high dietary intake of red meat and low dietary fiber,
physical inactivity, and obesity. Complications of diverticular disease include lower gastrointestinal bleeding and diverticulitis. Rarely, diverticular disease can lead to the development of a colovesical fistula in
which there is a connection between the rectum and the urinary bladder. This can lead to pneumaturia and passage of fecal material during urination, as well as the development of urinary tract infections,
seen in this patient with mixed flora and numerous leukocytes on urinalysis. Diagnosis involves contrast CT imaging or direct visualization. Treatment involves antibiotics and surgical repair.

Incorrect Answers: A, C, D, and E.

Appendiceal abscess (Choice A) can be a complication of untreated appendicitis. Appendicitis and appendiceal abscess would present with right lower quadrant abdominal pain and tenderness, and
generally fever, nausea, vomiting. It would not cause pneumaturia or passage of fecal material during urination.

Inflammatory bowel disease (Choice C) comprises both Crohn disease and ulcerative colitis. Symptoms include chronic abdominal pain, weight loss, bloody diarrhea, abdominal bloating, and tenesmus.
While colovesical fistulas can be a complication of inflammatory bowel disease, most patients diagnosed with inflammatory bowel disease are younger than age 30 years. This patient does not have a history
of abdominal pain or bloody diarrhea, both of which would be expected in the setting of inflammatory bowel disease.

Prostate cancer (Choice D) or hyperplasia can lead to urethral obstruction when outflow from bilateral kidneys is obstructed by an underlying pathology. It can lead to postrenal azotemia and difficulty
urinating but would not cause pneumaturia or urination of fecal material without concomitant fistula. This patient's prostate examination is consistent with benign prostatic hyperplasia, not malignancy.

Transitional cell carcinoma of the bladder (Choice E), also known as urothelial carcinoma, typically presents in patients, especially men, who smoke cigarettes. It can present with hematuria and urinary tract
obstruction. It is not as likely as diverticular disease to cause fistulation, especially in early or "silent" disease.

Educational Objective: Diverticular disease can lead to the development of a colovesical fistula in which there is an abnormal connection between the rectum and the urinary bladder This can lead to
complications of urine leakage through the rectum and associated urinary tract infections with pneumaturia.

https://t.me/USMLENBME2CK ti
Previous Next Score Report Lab Values Calculator Help pause
Exam Section 3: Item 16 of 50 National Board of Medical Examiners
Comprehensive Clinical Science Self-Assessment

✓ 16. A25-year-old man comes to the physician 2 hours after the sudden onset of sharp right buttock pain that radiates to the back of his knee. The pain worsens when he sits for a prolonged period or
drives. He is able to walk. Use of over-the-counter ibuprofen has provided mild relief. He has no history of serious illness and takes no other medications. On examination, ipsilateral and contralateral
straight-leg raise testing is positive. Muscle strength is normal throughout. Sensation is intact throughout. Which of the following is the most appropriate next step in diagnosis?

A) Electromyography and nerve conduction studies


B) MRI of the lumbosacral spine
C) Myelography
D) X-ray of the lumbosacral spine
E) No further testing is indicated
Correct Answer: E.

Lumbar disc herniation and piriformis syndrome are both common causes of sciatica. Disc herniation occurs most often following chronic disc degeneration combined with a minor traumatic event such as
bending and lifting a heavy object causing an acute increase in the intradiscal pressure. Disc herniation typically presents as acute back pain along with symptoms of lumbosacral radiculopathy, including
acute extremity pain and/or weakness in a dermalomal and myotomic distribution with respect to the impinged spinal nerve. Piriformis syndrome presents similarly but with pain reproduced with palpation of
the piriformis on the affected side. Regardless of cause, sciatica is diagnosed clinically, through a combination of history and examination maneuvers, including straight-leg raise testing, in which flexion of
the hip while the leg is held in extension at the knee reproduces pain in the lower back that generally shoots down the leg. Lumbar imaging with MRI or CT scan is useful for patients with severe motor or
sensory dysfunction but is not required for most patients with mild or moderate symptoms without an objective neurologic deficit and in whom there is no concern for spinal cord impingement. Trial of
therapy is generally appropriate before imaging in such cases. Patients with evidence of muscular weakness may benefit from nerve conduction studies or electromyography. Most patients with sciatica
demonstrate spontaneous improvement with conservative management, including analgesia, physical therapy, and activity as tolerated. Prolonged bed rest is not necessary for most patients and may
worsen pain and prolong recovery.

Incorrect Answers: A, B, C, and D.

Electromyography and nerve conduction studies (Choice A) are used to differentiate myopathic and neuropathic causes of extremity weakness and would not be used in this patient with normal muscle
strength and sensation.

MRI of the lumbosacral spine (Choice B) is indicated for evaluation of acute onset neurologic deterioration from cauda equina syndrome. Cauda equina syndrome occurs from acute compression of the
cauda equina nerve roots in the lumbosacral spinal canal, most commonly from an acute disc herniation, or a traumatic or pathologic vertebral column fracture. This results in bilateral lower extremity
weakness, loss of deep tendon reflexes, bowel or bladder dysfunction (eg, overflow urinary incontinence), and saddle anesthesia. This patient does not have any signs of cauda equina syndrome, so MRI is
not indicated at this time.

Myelography (Choice C) involves examination of the spinal cord using x-ray or CT scan following injection of an intrathecal contrast medium. It has largely been replaced by MRI because of improved
image accuracy, absence of radiation, and noninvasive technique.

X-ray of the lumbosacral spine (Choice D) may be useful for the diagnosis of vertebral compression fractures, which commonly occur in elderly patients with low bone mineral density, especially among
postmenopausal females. This patient's age, history, and radiation of pain are more suggestive of disc herniation or piriformis syndrome.

https://t.me/USMLENBME2CK ts e t
Previous Next Score Report Lab Values Calculator Help pause
National Board of Medical Examiners
Comprehensive Clinical Science Self-Assessment

B) MRI of the lumbosacral spine


C) Myelography
D) X-ray of the lumbosacral spine
E) No further testing is indicated
Correct Answer: E.

Lumbar disc herniation and piriformis syndrome are both common causes of sciatica. Disc herniation occurs most often following chronic disc degeneration combined with a minor traumatic event such as
bending and lifting a heavy object causing an acute increase in the intradiscal pressure. Disc herniation typically presents as acute back pain along with symptoms of lumbosacral radiculopathy, including
acute extremity pain and/or weakness in a dermalomal and myotomic distribution with respect lo the impinged spinal nerve. Piriformis syndrome presents similarly but with pain reproduced with palpation of
the piriformis on the affected side. Regardless of cause, sciatica is diagnosed clinically, through a combination of history and examination maneuvers, including straight-leg raise testing, in which flexion of
the hip while the leg is held in extension at the knee reproduces pain in the lower back that generally shoots down the leg. Lumbar imaging with MRI or CT scan is useful for patients with severe motor or
sensory dysfunction but is not required for most patients with mild or moderate symptoms without an objective neurologic deficit and in whom there is no concern for spinal cord impingement. Trial of
therapy is generally appropriate before imaging in such cases. Patients with evidence of muscular weakness may benefit from nerve conduction studies or electromyography. Most patients with sciatica
demonstrate spontaneous improvement with conservative management, including analgesia, physical therapy, and activity as tolerated. Prolonged bed rest is not necessary for most patients and may
worsen pain and prolong recovery.

Incorrect Answers: A, B, C, and D.

Electromyography and nerve conduction studies (Choice A) are used to differentiate myopathic and neuropathic causes of extremity weakness and would not be used in this patient with normal muscle
strength and sensation.

MRI of the lumbosacral spine (Choice B) is indicated for evaluation of acute onset neurologic deterioration from cauda equina syndrome. Cauda equina syndrome occurs from acute compression of the
cauda equina nerve roots in the lumbosacral spinal canal, most commonly from an acute disc herniation, or a traumatic or pathologic vertebral column fracture. This results in bilateral lower extremity
weakness, loss of deep tendon reflexes, bowel or bladder dysfunction (eg, overflow urinary incontinence), and saddle anesthesia. This patient does not have any signs of cauda equina syndrome, so MRI is
not indicated at this time.

Myelography (Choice C) involves examination of the spinal cord using x-ray or CT scan following injection of an intrathecal contrast medium. It has largely been replaced by MRI because of improved
image accuracy, absence of radiation, and noninvasive technique.

X-ray of the lumbosacral spine (Choice D) may be useful for the diagnosis of vertebral compression fractures, which commonly occur in elderly patients with low bone mineral density, especially among
postmenopausal females. This patient's age, history, and radiation of pain are more suggestive of disc herniation or piriformis syndrome.

Educational Objective: Sciatica is often caused by lumbar disc herniation. It typically presents with acute back pain along with symptoms of lumbosacral radiculopathy, including acute extremity pain and/or
weakness in a dermatomal and myotomic distribution with respect to the impinged nerve, generally radiating down the back of the leg. It is typically a self-limited process, with most patients demonstrating
spontaneous improvement with conservative management.

https://t.me/USMLENBME2CK ts e t
Previous Next Score Report Lab Values Calculator Help pause
Exam Section 3: Item 17 of 50 National Board of Medical Examiners
Comprehensive Clinical Science Self-Assessment

"I 17. A 16-year-old girl is brought to the physician by her mother because of an 8-month history of irregular menstrual periods, acne, and facial hair. Menarche was at the age of 12 years. Menses occur at
irregular 35- to 42-day intervals; they last 1 week, and the flow is heavy. Moderate cramps on the first 2 days are relieved with ibuprofen therapy. The patient had recurrent urinary tract infections until
the age of 10 years. She has no other history of serious illness and takes no other medications. She is 163 cm (5 fl 4 in) tall and weighs 77 kg (170 lb); BMI is 29 kg/m?. Her temperature is 36.9°C
(98.4°F), pulse is 78/min, respirations are 20/min, and blood pressure is 128/84 mm Hg. Examination shows scattered papules, pustules, and comedones over the face. There is sparse, coarse hair
over the chin. The neck is supple; there is no thyromegaly. Breast examination shows no masses. Breast and pubic hair development are sexual maturity rating (SMR) stage 5. Serum studies are most
likely to show an increased concentration of which of the following?

A) Follicle-stimulating hormone
B) 11a-Hydroxyprogesterone
C) 21a-Hydroxyprogesterone
D) Luteinizing hormone
E) Thyroid-stimulating hormone
Correct Answer: D.

Polycystic ovarian syndrome (PCOS) presents with irregular menses, menorrhagia, infertility, acne, and hirsutism, often in patients with an increased body weight. It requires two of the following for
diagnosis: oligo/anovulation, polycystic ovaries on ultrasonography, and clinical or biochemical signs of hyperandrogenism. It is also associated with signs of insulin resistance, such as acanlhosis nigricans
and increased blood glucose concentrations. PCOS is caused by an abnormal hypothalamic hormonal feedback response, which leads to increased luteinizing hormone in comparison to follicle-stimulating
hormone, as well as markedly increased androgen (eg, testosterone) production from the theca lutein cells. Treatment includes weight loss, combined oral contraceptives for menstrual regulation, and
spironolactone or ketoconazole for hirsutism.

Incorrect Answers: A, 8, C, and E.

Follicle-stimulating hormone (Choice A) may be increased in menopause or in central precocious puberty. Its role in the hypothalamic-pituitary-gonadal axis in females is to stimulate the production of follicles
in the ovary to support an egg for development. It is not increased to the same degree as luteinizing hormone in PCOS.

Both 11a-hydroxy progesterone (Choice 8) and 21a-hydroxyprogesterone (Choice C) are precursors to aldosterone and thus carry a degree of mineralocorticoid activity. Increased mineralocorticoid activity
would be expected to produce hypertension, and/or hypokalemia or metabolic alkalosis depending on hormonal activity of the precursor. It would not explain this patient's ovulatory dysfunction or
hyperandrogenism.

Thyroid-stimulating hormone (Choice E) can be used to diagnose both hypothyroidism and hyperthyroidism. It is increased in primary hypothyroidism. Hypothyroidism is associated with fatigue, weight gain,
ovulatory dysfunction, constipation, and weight gain. While hypothyroidism could explain this patient's irregular menses with menorrhagia, as well as her increased body weight, hypothyroidism does not
cause hyperandrogenism.

Educational Objective: Polycystic ovarian syndrome presents with menstrual irregularities, signs of androgen excess, obesity, insulin resistance, and polycystic ovaries on ultrasonography. It is caused by an
abnormal hypothalamic hormonal feedback response, which leads to increased luteinizing hormone and increased androgen production. Treatment includes weight loss, combined oral contraceptives for
menstrual regulation, and spironolactone or keloconazole for hirsutism.

https://t.me/USMLENBME2CK ts e t
Previous Next Score Report Lab Values Calculator Help pause
Exam Section 3: Item 18 of 50 National Board of Medical Examiners
Comprehensive Clinical Science Self-Assessment

✓ 18. A 49-year-old woman comes to the physician for preoperative examination before she undergoes cholecystectomy. During the past 3 years, she has had recurrent cholecystitis and multiple
gallstones. She has not had shortness of breath, difficulty swallowing, or hoarseness. She has rheumatoid arthritis with morning stiffness and widespread Joint swelling. Her medications are
methotrexate, folic acid, and prednisone. Vital signs are within normal limits. Examination shows nodules over the extensor aspect of the upper extremities. There is swelling of the wrists,
metacarpophalangeal Joints of the hands, and knees; there are swan neck deformities of the fingers. The remainder of the examination shows no abnormalities. Which of the following is the most
appropriate next step in management to decrease this patient's risk for operative complications?

A) Discontinuation of methotrexate
B) Discontinuation of prednisone
C) Indirect laryngoscopy
D) Pulmonary function testing
E) X-rays of the cervical spine
Correct Answer: E.

Rheumatoid arthritis (RA) is an autoimmune disorder characterized by systemic inflammation that prominently affects synovial joints. It is chronic and progressive, often leading to significant morbidity.
Symptoms include the gradual onset of symmetric arthralgia and joint swelling, as well as morning stiffness, commonly in the fingers and wrists. Chronic disease can progress to affect proximal joints and
the cervical spine and leads to significant Joint damage and deformities. Severe disease can cause systemic disease, such as interstitial lung disease, vasculitis, pleuritis, pericarditis, anemia, and
subcutaneous nodules. Atlantoaxial involvement is common in chronic disease and may be associated with subluxation with consequent spinal cord injury or radiculopathy. Evaluation of the cervical spine
with flexion and extension x-rays in patients with chronic RA is appropriate prior to general anesthesia, to evaluate for atlantoaxial instability and inform the technique for intubation. Direct laryngoscopy,
often used to intubate patients for general anesthesia, involves significant neck extension and flexion, which could lead to catastrophic neurologic damage in the setting of atlantoaxial instability. With
significant instability, intubation with in-line stabilization is needed. Therefore, the most appropriate next step in management in this case is x-rays of the cervical spine to evaluate risk for direct
laryngoscopy.

Incorrect Answers: A, B, C, and D.

Discontinuation of methotrexate (Choice A) is not necessary. Methotrexate is used in the treatment of RA because of its anti-inflammatory and immunosuppressive actions. While it can cause hepatotoxicity,
pulmonary disease, renal injury, and increased risk for infection, its importance in managing this patient's RA outweighs its risk.

Discontinuation of prednisone (Choice B) is not necessary. II acts as an anti-inflammatory and immunosuppressive agent in the treatment of RA. Discontinuation of chronic prednisone therapy can cause
adverse effects, as long-standing prednisone therapy can cause adrenal insufficiency if discontinued. Thus, chronic prednisone therapy may necessitate stress-dose steroids prior to this patient's
procedure, and discontinuation of prednisone may be harmful.

Indirect laryngoscopy (Choice C) may benefit this patient. However, this patient should not proceed to general anesthesia without evaluation of cervical spine stability, as indirect laryngoscopy can also
require neck extension, leading to neurologic damage in the setting of severe instability.

Pulmonary function testing (Choice D) is not indicated. While RA can cause interstitial lung disease when chronic and severe, this patient has no elements in the history or physical examination that raise
the concern for lung disease. With no symptoms, evaluation of this patient's lung function with pulmonary function testing is unlikely to change anesthetic management or decrease risk for perioperative
4

https://t.me/USMLENBME2CK ts e t
Previous Next Score Report Lab Values Calculator Help pause
Exam Section 3: Item 18 of 50 National Board of Medical Examiners
Comprehensive Clinical Science Self-Assessment

A) Discontinuation of methotrexate
B) Discontinuation of prednisone
C) Indirect laryngoscopy
D) Pulmonary function testing
E) X-rays of the cervical spine
Correct Answer: E.

Rheumatoid arthritis (RA) is an autoimmune disorder characterized by systemic inflammation that prominently affects synovial joints. It is chronic and progressive, often leading to significant morbidity.
Symptoms include the gradual onset of symmetric arthralgia and joint swelling, as well as morning stiffness, commonly in the fingers and wrists. Chronic disease can progress lo affect proximal joints and
the cervical spine and leads to significant Joint damage and deformities. Severe disease can cause systemic disease, such as interstitial lung disease, vasculitis, pleuritis, pericarditis, anemia, and
subcutaneous nodules. Atlantoaxial involvement is common in chronic disease and may be associated with subluxation with consequent spinal cord injury or radiculopathy. Evaluation of the cervical spine
with flexion and extension x-rays in patients with chronic RA is appropriate prior to general anesthesia, to evaluate for atlantoaxial instability and inform the technique for intubation. Direct laryngoscopy,
often used to intubate patients for general anesthesia, involves significant neck extension and flexion, which could lead to catastrophic neurologic damage in the setting of atlantoaxial instability. With
significant instability, intubation with in-line stabilization is needed. Therefore, the most appropriate next step in management in this case is x-rays of the cervical spine to evaluate risk for direct
laryngoscopy.

Incorrect Answers: A, B, C, and D.

Discontinuation of methotrexate (Choice A) is not necessary. Methotrexate is used in the treatment of RA because of its anti-inflammatory and immunosuppressive actions. While ii can cause hepatotoxicity,
pulmonary disease, renal injury, and increased risk for infection, its importance in managing this patient's RA outweighs its risk.

Discontinuation of prednisone (Choice B) is not necessary. It acts as an anti-inflammatory and immunosuppressive agent in the treatment of RA. Discontinuation of chronic prednisone therapy can cause
adverse effects, as long-standing prednisone therapy can cause adrenal insufficiency if discontinued. Thus, chronic prednisone therapy may necessitate stress-dose steroids prior to this patient's
procedure, and discontinuation of prednisone may be harmful.

Indirect laryngoscopy (Choice C) may benefit this patient. However, this patient should not proceed to general anesthesia without evaluation of cervical spine stability, as indirect laryngoscopy can also
require neck extension, leading to neurologic damage in the setting of severe instability.

Pulmonary function testing (Choice D) is not indicated. While RA can cause interstitial lung disease when chronic and severe, this patient has no elements in the history or physical examination that raise
the concern for lung disease. With no symptoms, evaluation of this patient's lung function with pulmonary function testing is unlikely to change anesthetic management or decrease risk for perioperative
complications.

Educational Objective: Rheumatoid arthritis (RA) is a chronic, progressive, autoimmune disease that primarily leads to destruction of synovial joints. Atlantoaxial involvement is common in severe disease
and can lead to atlantoaxial instability with consequent spinal cord injury and radiculopathy. Therefore, patients with chronic RA should be evaluated with cervical spine x-rays prior to undergoing intubation,
as direct laryngoscopy often utilizes neck extension that could lead to catastrophic neurologic damage in the setting of severe instability.

https://t.me/USMLENBME2CK ts e t
Previous Next Score Report Lab Values Calculator Help pause
Exam Section 3: Item 19 of 50 National Board of Medical Examiners
Comprehensive Clinical Science Self-Assessment

✓ 19. A 32-year-old woman, gravida 1, para 1, comes to the physician because of spontaneous discharge from her nipples during the past 3 months. She discontinued breast-feeding her child 1 year ago.
She takes no medications. Menses occur at regular 28-day intervals. On physical examination, a milky white discharge can be expressed from both nipples. Examination shows no other
abnormalities. Her fasting serum prolactin concentration is 40 ng/ml. The most appropriate next step in diagnosis is measurement of which of the following serum concentrations?

A) Cortisol
B) Dopamine
C) Estrogen
0) Gonadotropin-releasing hormone
E) Thyroid-stimulating hormone
Correct Answer: E.

An abnormal increase in serum prolactin is defined as hyperprolaclinemia and causes non physiologic discharge of milk from the nipple (galactorrhea) and growth of glandular breast tissue. A diagnosis of
hyperprolactinemia is made when the serum prolactin concentration is greater than 30 ng/ml in premenopausal women or greater than 20 ng/ml in men and postmenopausal women. After cessation of
breast-feeding, serum prolactin concentrations typically return to prepregnancy levels. Common causes of hyperprolactinemia include hyperfunctioning anterior pituitary microadenomas, hypothyroidism,
and dopamine-blocking medications, as prolactin is negatively regulated by dopamine. Prolactin is positively regulated by the TSH-releasing hormone (TRH) and, in states of hypothyroidism with
upregulated TRH, galactorrhea can also occur. The initial workup should involve measurement of TSH concentration and evaluation of any dopamine-blocking medications the patient may be taking. If
these steps are unremarkable, further diagnostic evaluation should include MRI of the brain to assess for the presence of a pituitary adenoma. Treatment of hypothyroidism, if present, will resolve
hyperprolactinemia and the patient's symptoms.

Incorrect Answers: A, B, C, and D.

Cortisol (Choice A) measurement is useful in the workup of hyperaldosteronism and adrenal insufficiency, though the serum concentration fluctuates widely throughout the day. It is also used to evaluate for
corticotropin deficiency to test for panhypopituitarism, which results in prolactin deficiency and an inability to lactate after delivery.

Dopamine (Choice B) is the primary regulator of prolactin secretion from the anterior pituitary gland, but serum concentrations do not correspond to the level of regulatory activity. Assessment for
hypothyroidism and dopamine-blocking medications is more useful.

Estrogen (Choice C) and gonadotropin-releasing hormone (Choice 0) are generally not useful in the evaluation of galactorrhea. The evaluation for pituitary abnormalities, especially if there is concern for
hypersecretion, includes checking insulin-like growth factor 1, corticotropin, follicle-stimulating hormone, and luteinizing hormone.

Educational Objective: Hyperprolactinemia is defined as a serum prolactin concentration greater than 30 ng/ml in premenopausal women or greater than 20 ng/ml in men and postmenopausal women.
Galactorrhea is a common presenting symptom. Initial workup should rule out hypothyroidism and renal insufficiency as causes of an increased serum prolactin concentration, and MRI of the pituitary gland
should be obtained.

https://t.me/USMLENBME2CK ts e t
Previous Next Score Report Lab Values Calculator Help pause
Exam Section 3: Item 20 of 50 National Board of Medical Examiners
Comprehensive Clinical Science Self-Assessment

"I 20. A57-year-old man comes to the physician for a follow-up examination. One year ago, he sustained a myocardial infarction. He has coronary artery disease, hypertension, and type 2 diabetes mellitus.
His medications are atorvastatin, lisinopril, metformin, metoprolol, and aspirin. He has smoked one-half pack of cigarettes daily for 25 years. He is 178 cm (5 ft 10 in) tall and weighs 97 kg (213 lb); BMI
is 31 kg/m? He feels well. His pulse is 62/min, and blood pressure is 145/78 mm Hg. Examination shows no other abnormalities. Laboratory studies show a hemoglobin A,_ of 6.5% and serum
LDL-cholesterol concentration of 110 mg/dl. Serum electrolyte, urea nitrogen, glucose, and creatinine concentrations are within the reference ranges. The physician recommends smoking cessation.
Addition of which of the following is the most appropriate next step in pharmacotherapy?

A) Clonidine
B ) Exenatide
C) Hydrochlorothiazide
D) Rosiglitazone
E) Sitagliptin
F) Spironolactone
Correct Answer: C.

Patients with a history of acute coronary syndrome are at increased risk for future adverse cardiovascular events. Management focuses on control of modifiable risk factors to reduce this risk. Patients should
be counseled on smoking cessation, healthy diet, and regular exercise. Common comorbid conditions that contribute to increased risk include hypertension, diabetes mellitus, metabolic syndrome, and
hyperlipidemia. The American Heart Association recommends a blood pressure goal of less than 140/90 mm Hg for adults, unless the patient also has diabetes mellitus or chronic renal disease, in which
case the goal is less than 130/80 mm Hg. This patient has a blood pressure above goal and addition of a second antihypertensive agent, such as hydrochlorothiazide, is likely to provide the most benefit at
this time.

Incorrect Answers: A, B, D, E, and F.

Clonidine (Choice A) is an alpha-adrenergic agonist primarily used to treat attention deficit hyperactivity disorder in children and to manage tics in Tourette disorder. It is also an antihypertensive that is
typically reserved for use in certain clinical situations, as other antihypertensives such as thiazide diuretics have a lower risk for adverse effects.

Exenatide (Choice B) and sitagliptin (Choice E) are second-line antihyperglycemic agents in the glucagon-like peptide 1 (GLP-1) receptor agonist and dipeptidyl peptidase 4 (OPP-4) inhibitor classes
respectively. Both are reasonable choices lo add for patients who are not meeting glycemic goals with melformin monotherapy and lifestyle changes. A hemoglobin A,, of 6.5% is an acceptable
concentration for a patient with known cardiovascular disease, as tighter glycemic control has a higher risk for adverse effects from hypoglycemia with no additional benefit.

Rosiglitazone (Choice 0) is a second-line antihyperglycemic agent in the thiazolidinedione class. It is not recommended for use because of concerns about increased risk for major adverse cardiovascular
events.

Spironolactone (Choice F) is an aldosterone receptor antagonist used as a potassium-sparing diuretic, most commonly in patients receiving loop diuretics. It is not indicated at this time.

Educational Objective: Secondary prevention for cardiovascular disease focuses on modifiable risk factors, especially appropriate management of comorbidities such as hypertension. A blood pressure goal
of less than 140/90 mm Hg for adults is recommended, unless the patient also has diabetes mellitus or chronic renal disease, in which case the goal is less than 130/80 mm Hg.

https://t.me/USMLENBME2CK ti e
Previous Next Score Report Lab Values Calculator Help pause
Exam Section 3: Item 21 of 50 National Board of Medical Examiners
Comprehensive Clinical Science Self-Assessment

"I 21. A 60-year-old woman with osteoarthritis of the right knee comes to the office to discuss treatment options, including total knee replacement. Medications are naproxen and 81-mg aspirin. Vital signs are
within normal limits. Examination shows edema of the right knee; range of motion of the right knee is decreased. Results of laboratory studies are within the reference ranges. The patient asks the
physician about the success and complication rates associated with total knee replacement at the local university hospital. Which of the following types of studies is most appropriate for the physician to
reference to answer this patient's question?

A) Case-control study
B ) Cross-sectional study
C) Institutional case series
D) Randomized controlled trial
Correct Answer: C.

A case series study is a descriptive study that describes the history, possible exposures, clinical findings, and treatment results of a group of patients with a similar diagnosis. Case series studies are
nonanalytic and do not imply a cause-and-effect relationship. They do not test a hypothesis nor are they randomized. They are useful in characterizing the natural history of a disease or response to treatment
and are also useful for evaluating the outcomes of a particular procedure or patient population at a single institution. They are useful in describing rare diseases, as the small population size may not permit
conduction of a larger cohort, or randomized trials with sufficient power.

Incorrect Answers: A, B, and D.

Case-control studies (Choice A) group patients according to disease or procedure outcome status and analyze the odds of exposure to a particular hazard. The appropriate statistical measure of case-control
studies is the odds ratio (OR).

Cross-sectional studies (Choice B) examine a sample population at a single point in time and describe the prevalence of a particular disease or risk factor. In contrast to cohort studies, which group patients
according to exposure status, and case-control studies, which group patients according to disease outcome status, cross-sectional studies analyze both disease states and exposures simultaneously.

Randomized controlled trials (Choice 0) are a type of prospective, interventional study design wherein patients are randomly assigned to receive a particular intervention. The intervention may be compared
against placebo therapy or standard therapy, depending on the study design and the disease being studied. When combined with blinding procedures, randomized control studies form the gold standard of
medical research and generate more powerful evidence than other forms of observational or descriptive studies.

Educational Objective: A case series study is a descriptive study that describes the history, possible exposures, clinical findings, and treatment results of a group of patients with a similar diagnosis. Case series
studies are useful for studying the outcomes or natural history of a particular group, procedure, or treatment.

https://t.me/USMLENBME2CK ts e t
Previous Next Score Report Lab Values Calculator Help pause
Exam Section 3: Item 22 of 50 National Board of Medical Examiners
Comprehensive Clinical Science Self-Assessment

22. A72-year-old woman comes to the office with her son because of a "strange mole" on her back that she first noticed 6 months ago. The patient's son says he thinks it has enlarged during the past
month. The patient says the mole is mildly itchy but not painful, and she has not had bleeding from the area. She has not had any weight loss or changes in appetite. Medical history is unremarkable,
and she takes no medications. The patient has spent a lot of time outside by her son's new swimming pool during the past summer months. Vital signs are within normal limits. A photograph of the
lesion is shown. The remainder of the examination shows no abnormalities. Which of the following is the most appropriate diagnostic study at this time?

A) Culture of the lesion


B) Full-thickness biopsy of the lesion
C) KOH testing of a scraping of the lesion
D) Shave biopsy of the lesion
E) No diagnostic study is indicated
Correct Answer: E.

Seborrheic keratosis is a benign proliferation of the epidermis. It is characterized histologically by a benign, acanthotic proliferation of keratinocytes with absence of cellular atypia. The lesions are typically
round, raised, brown or tan in color, and sharply demarcated with an adherent or stuck-on appearance. Seborrheic keratosis is common in middle-aged and older adults, and it is associated with increasing
age. While seborrheic keratosis is benign, the sudden appearance of numerous seborrheic keratosis lesions on the back and trunk, known as the Leser-Tr~lat sign, can be a paraneoplastic manifestation of
gastrointestinal malignancy. The diagnosis of seborrheic keratosis is established based on clinical appearance, and no diagnostic study is indicated. Treatment is not necessary, although lesions that are
cosmetically bothersome may be removed with cryotherapy or simple excision.

https://t.me/USMLENBME2CK ti
Previous Next Score Report Lab Values Calculator Help pause
Exam Section 3: Item 22 of 50 National Board of Medical Examiners
Comprehensive Clinical Science Self-Assessment

22. A72-year-old woman comes to the office with her son because of a "strange mole" on her back that she first noticed 6 months ago. The patient's son says he thinks it has enlarged during the past
month. The patient says the mole is mildly itchy but not painful, and she has not had bleeding from the area. She has not had any weight loss or changes in appetite. Medical history is unremarkable,
and she takes no medications. The patient has spent a lot of time outside by her son's new swimming pool during the past summer months. Vital signs are within normal limits. A photograph of the
lesion is shown. The remainder of the examination shows no abnormalities. Which of the following is the most appropriate diagnostic study at this time?

A) Culture of the lesion


B) Full-thickness biopsy of the lesion
C) KOH testing of a scraping of the lesion
D) Shave biopsy of the lesion
E) No diagnostic study is indicated
Correct Answer: E.

Seborrheic keratosis is a benign proliferation of the epidermis. It is characterized histologically by a benign, acanthotic proliferation of keratinocytes with absence of cellular atypia. The lesions are typically
round, raised, brown or tan in color, and sharply demarcated with an adherent or stuck-on appearance. Seborrheic keratosis is common in middle-aged and older adults, and it is associated with increasing
age. While seborrheic keratosis is benign, the sudden appearance of numerous seborrheic keratosis lesions on the back and trunk, known as the Leser-Tr~lat sign, can be a paraneoplastic manifestation of
gastrointestinal malignancy. The diagnosis of seborrheic keratosis is established based on clinical appearance, and no diagnostic study is indicated. Treatment is not necessary, although lesions that are
cosmetically bothersome may be removed with cryotherapy or simple excision.

Incorrect Answers: A, B, C, and D.

Culture of the lesion (Choice A) is not necessary. Seborrheic keratosis is a result of proliferation of keratinocytes and is not infectious in cause.

Full-thickness biopsy of the lesion (Choice B) and shave biopsy of the lesion (Choice D) are methods of obtaining tissue for histopathologic analysis during the diagnosis of benign or malignant cutaneous
neoplasms. Shave biopsy may be adequate for initial diagnosis of certain malignancies, such as basal cell carcinoma. Other techniques, such as punch biopsy or excisional biopsy, are more appropriate
when determination of Breslow thickness is necessary. Biopsy of seborrheic keratosis is not usually necessary.

KOH testing of a scraping of the lesion (Choice C) is useful for identification of fungal elements in the setting of tinea corporis or versicolor. Seborrheic keratosis does not have an underlying fungal cause.

Educational Objective: Seborrheic keratosis is a benign proliferation of the epidermal keralinocytes. Lesions are typically round, raised, brown or tan in color, and sharply demarcated with an adherent or
stuck-on appearance. The main risk factor for seborrheic keratosis is increasing age. The diagnosis is established based on clinical appearance, and no diagnostic study is indicated.

https://t.me/USMLENBME2CK ti
Previous Next Score Report Lab Values Calculator Help pause
Exam Section 3: Item 23 of 50 National Board of Medical Examiners
Comprehensive Clinical Science Self-Assessment

"I 23. A 27-year-old woman is brought to the emergency department because of a 2-day history of fever, pain with urination, and progressive generalized weakness. She has systemic lupus erythematosus
complicated by nephritis. Current medications include prednisone and cyclophosphamide. Her temperature is 39.4C (103°F), pulse is 122/min, respirations are 24/min, and blood pressure is
88/62 mm Hg. Examination shows warm, flushed skin. There is moderate supra pubic and right flank tenderness. Her leukocyte count is 18,000/mm3 (80% segmented neutrophils, 5% bands,
1% eosinophils, 1% basophils, 10% lymphocytes, and 3% monocytes). Urinalysis shows 100 WBC/hpf. A Gram stain shows numerous gram-negative rods. Intravenous administration of piperacillin
and tazobactam and 2 liters of 0.9% saline is started. Thirty minutes later, her pulse is 124/min, and blood pressure is 92/68 mm Hg. Which of the following is the most appropriate next step in
management?

A) Administration of ciprofloxacin
B) Administration of gentamicin
C) Administration of hydrocortisone
D) Administration of metoprolol
E) Cardioversion
Correct Answer: C.

This patient's hypotension and tachycardia have failed to improve with appropriate initiation of intravenous antibiotics and fluid resuscitation. She may benefit from the addition of a vasopressor; however, her
history of chronic corticosteroid use for systemic lupus erythematosus should prompt the physician to consider acute adrenal insufficiency and adrenal crisis precipitated by severe infection. Adrenal crisis is
a life-threatening emergency, and prompt administration of hydrocortisone is the most appropriate next step in management. In secondary adrenal insufficiency, as with chronic exogenous steroid use, an
acute stressor suddenly increases physiologic demand for cortisol, and the adrenal glands are unable to meet the demand. This results in inadequate vascular tone to maintain blood pressure. Survival
dramatically increases with early recognition and management of adrenal crisis.

Incorrect Answers: A, B, D, and E.

Administration of ciprofloxacin (Choice A) is not appropriate given this patient's state of septic shock. Broad gram-positive and enteric gram-negative coverage, which also covers Pseudomonas, is
recommended. Numerous organisms that cause complicated urinary tract infections and pyelonephritis have resistance to fluoroquinolones, and they should not be used for empiric treatment in a sick
patient.

Administration of gentamicin (Choice 8) is not necessary and would not add significant additional coverage in this setting to justify the risk for adverse effects, especially in a patient with underlying kidney
disease, given the known nephrotoxicity of aminoglycosides.

Administration of metoprolol (Choice 0) is contraindicated in the setting of shock, as it impairs the body's physiological attempt to compensate for reduced tissue perfusion.

Cardioversion (Choice E) is not indicated, as the most likely explanation for the patient's tachycardia is sinus tachycardia in the setting of shock. This should be permitted to continue, as the increased heart
rate (and therefore cardiac output) is compensating for diminished systemic vascular resistance in maintaining mean arterial pressure.

Educational Objective: Adrenal crisis is a life-threatening emergency that can occur in the setting of chronic adrenal insufficiency when an acute stressor causes a rapid increase in physiologic demand for
cortisol that the adrenal glands are unable to meet. Initial symptoms are often vague and nonspecific, but hypotension refractory to fluid resuscitation in patients at risk should prompt the physician to
administer hydrocortisone.

https://t.me/USMLENBME2CK ti
Previous Next Score Report Lab Values Calculator Help pause
Exam Section 3: Item 24 0f 50 National Board of Medical Examiners
Comprehensive Clinical Science Self-Assessment

✓ 24. A 72-year-old man comes to the physician because of a 3-month history of progressive shortness of breath, nonproductive cough, and fatigue. During this period, he has had early satiety resulting in
a 2.3-kg (5-lb) weight loss. He has no known sick contacts. He has a 20-year history of chronic obstructive pulmonary disease and a 10-year history of hypertension. Current medications are an
inhaled bronchoditator and an oral thiazide diuretic. He has smoked one pack of cigarettes daily for 50 years. He is in mild respiratory distress. He is 165 cm (5 ft 5 in) tall and weighs 52 kg (115 lb);
BMI is 19 kg/m?. His temperature is 37°C (98.6°F), pulse is 103/min, respirations are 20/min, and blood pressure is 130/90 mm Hg. Pulse oximetry on room air shows an oxygen saturation of 87%.
Examination shows no scleral icterus. There is jugular venous distention. Breath sounds are distant. There is a right ventricular heave and a loud S 'l: A grade 2/6 systolic murmur is heard best over
the left sternal border. The liver is tender, smooth, and pulsatile. There is no peripheral edema. His hematocrit is 50%, leukocyte count is 4500/mm3, and platelet count is 200,000/mm3. Which of the
following is the most appropriate next step in management?

A) Digoxin therapy
B) Furosemide therapy
C) Incentive spirometry
D) Continuous positive airway pressure
E) Home oxygen therapy
Correct Answer: E.

The most appropriate next step in management is home oxygen therapy. This patient presents with right-sided heart failure secondary lo chronic obstructive pulmonary disease (COPD). This is referred to
as cor pulmonale (meaning "pulmonary heart" in Latin). Chronic lung disease is associated with pulmonary hypertension, and, similar to systemic hypertension causing left-sided heart failure, over time the
right side of the heart fails as a result of the increased demands of pumping against elevated pulmonary arterial resistance. Patients present with signs of venous congestion, which are seen in this patient
with jugular venous distension and a pulsatile liver. Structural changes to the right ventricle are evident with a palpable heave on physical examination and a loud S heart sound as the pulmonic valve
slaps shut under increased pulmonary pressures. Mortality in severe COPD is improved by smoking cessation and home oxygen therapy. In addition to improving peripheral oxygenation, supplemental
oxygen reduces the extent of vasoconstriction that contributes lo increased pulmonary vascular pressures, reducing strain on the right ventricle.

Incorrect Answers: A, B, C, and D.

Digoxin therapy (Choice A) is used in the treatment of heart failure and certain arrhythmias. In heart failure, the role of digoxin is lo improve symptoms by augmenting cardiac output; however, it does not
confer long-term survival benefits. For patients with COPD, smoking cessation and home oxygen therapy demonstrate the greatest improvements in symptoms and survival.

Furosemide therapy (Choice B) is a mainstay of heart failure treatment and may be required for this patient if he continues to have symptoms associated with venous congestion. Home oxygen therapy
should be initiated first.

Incentive spirometry (Choice C) is useful to prevent pulmonary complications in postoperative patients and improve ventilation in patients with atelectasis. It does not have a significant role in routine
management of COPD.

Continuous positive airway pressure (Choice D) is beneficial for acute exacerbations where a patient is hypoventilating as a result of airway collapse and at risk for respiratory failure. It also aids in heart
failure by decreasing afterload through equalization of thoracic pressures. It is not indicated in this current setting.

https://t.me/USMLENBME2CK ti e
Previous Next Score Report Lab Values Calculator Help pause
Exam Section 3: Item 24 0f 50 National Board of Medical Examiners

I I I •
Comprehensive Clinical Science Self-Assessment
. ..
.-'. . . ' 2
the left sternal border. The liver is tender, smooth, and pulsatile. There is no peripheral edema. His hematocrit is 50%, leukocyte count is 4500/mm3, and platelet count is 200,000/mm3. Which of the
following is the most appropriate next step in management?

A) Oigoxin therapy
B) Furosemide therapy
C) Incentive spirometry
0) Continuous positive airway pressure
E) Home oxygen therapy
Correct Answer: E.

The most appropriate next step in management is home oxygen therapy. This patient presents with right-sided heart failure secondary to chronic obstructive pulmonary disease (COPO). This is referred to
as cor pulmonale (meaning "pulmonary heart" in Latin). Chronic lung disease is associated with pulmonary hypertension, and, similar to systemic hypertension causing left-sided heart failure, over time the
right side of the heart fails as a result of the increased demands of pumping against elevated pulmonary arterial resistance. Patients present with signs of venous congestion, which are seen in this patient
with jugular venous distension and a pulsatile liver. Structural changes to the right ventricle are evident with a palpable heave on physical examination and a loud S heart sound as the pulmonic valve
slaps shut under increased pulmonary pressures. Mortality in severe COPO is improved by smoking cessation and home oxygen therapy. In addition to improving peripheral oxygenation, supplemental
oxygen reduces the extent of vasoconstriction that contributes to increased pulmonary vascular pressures, reducing strain on the right ventricle.

Incorrect Answers: A, B, C, and 0.

Oigoxin therapy (Choice A) is used in the treatment of heart failure and certain arrhythmias. In heart failure, the role of digoxin is to improve symptoms by augmenting cardiac output; however, it does not
confer long-term survival benefits. For patients with COPO, smoking cessation and home oxygen therapy demonstrate the greatest improvements in symptoms and survival.

Furosemide therapy (Choice B) is a mainstay of heart failure treatment and may be required for this patient if he continues to have symptoms associated with venous congestion. Home oxygen therapy
should be initiated first.

Incentive spirometry (Choice C) is useful to prevent pulmonary complications in postoperative patients and improve ventilation in patients with atelectasis. It does not have a significant role in routine
management of COPO.

Continuous positive airway pressure (Choice 0) is beneficial for acute exacerbations where a patient is hypoventilating as a result of airway collapse and at risk for respiratory failure. It also aids in heart
failure by decreasing afterload through equalization of thoracic pressures. It is not indicated in this current setting.

Educational Objective: Chronic obstructive pulmonary disease is characterized by chronic, progressive deterioration in pulmonary function. In severe cases, patients develop right-sided heart failure as a
result of chronically increased pulmonary vascular pressures. Supplemental oxygen and smoking cessation provide the strongest mortality benefits, and smoking cessation results in a decreased rate of
pulmonary function decline as compared with patients who continue smoking.

https://t.me/USMLENBME2CK ti e
Previous Next Score Report Lab Values Calculator Help pause
Exam Section 3: Item 25 of 50 National Board of Medical Examiners°
Comprehensive Clinical Science Self-Assessment

IIII II II III II III I III W' III III IIIII WII III III II INII
[y(I[ IMIINII I[III(III [II III[III[ [III[NII [III1[ (I[1(/1Ill [/I[]
III III III /III II II III III III IIIIIIIIIII III III IL III
II IA III]! I! ,INIA!IWI.I!UNI,I!! IIIIIIMI
iilllllllllllllllllllllllllllllllllillllllllllllllllllllllllllllllllillllllllllllllllllllllllll
25. Four days after admission to the hospital for treatment of a myocardial infarction, a 72-year-old woman has the sudden onset of shortness of breath and confusion. She has atrial fibrillation, type 2
diabetes mellitus, essential hypertension, and hypercholesterolemia. Her medications are diltiazem, quinapril, metformin, and simvastatin. She has smoked one pack of cigarettes daily for 50 years
and has drunk one glass of wine daily for 52 years. She is anxious and oriented to person but not to place or time. Her temperature is 35°C (95°F), pulse is 116/min and irregularly irregular,
respirations are 30/min, and blood pressure is 92/58 mm Hg. Pulse oximetry on 100% oxygen by face mask shows an oxygen saturation of 91%. The skin is cool to the touch. Examination shows
jugular venous distention. Decreased breath sounds are heard in the lung bases bilaterally. Scattered expiratory wheezes and crackles are heard bilaterally. A rub is heard in the lower lobe of the left
lung. There is 2+ pretibial and pedal edema bilaterally. Laboratory studies show:
Hemoglobin 11.6 g/dL
Leukocyte count 11,000/mm3
Segmented neutrophils 65%
Bands 3%
Lymphocytes 30%
Monocytes 2%
Serum
Na 136 mEq/L
K 3.7 mEq/L
cl- 105 mEq/L
HCO. 3 14 mEq/L
Glucose 86 mg/dL
Creatine kinase 68 U/L
Lactic acid 3.6 mEq/L (N=0.5-2)

Arterial blood gas analysis shows:


pH 7.29
Pco, 30mm Hg
Po 64mm Hg

An ECG is shown. A chest x-ray shows cardiomegaly and pulmonary infiltrates, and pleural effusions bilaterally. Which of the following is the most appropriate next step in diagnosis?

A) Cardiac catheterization
« https://t.me/USMLENBME2CK ts e t
Previous Next Score Report Lab Values Calculator Help pause
Exam Section 3: Item 25 of 50 National Board of Medical Examiners
Comprehensive Clinical Science Self-Assessment

pH 7.29
Pco, 30mm Hg
Po 64 mm Hg

An ECG is shown. A chest x-ray shows cardiomegaly and pulmonary infiltrates, and pleural effusions bilaterally. Which of the following is the most appropriate next step in diagnosis?

A) Cardiac catheterization
B) CT angiography
C) Echocardiography
D) Measurement of serum brain natriuretic peptide concentration
E) Radionuclide myocardial scan
Correct Answer: C.

This patient is in a state of cardiogenic shock with tachycardia, borderline hypotension, cool extremities, and evidence of pulmonary and venous congestion on physical examination and imaging. It is
important to rapidly identify and treat the cause, and echocardiography provides diagnostic information quickly at the bedside. Acute and subacute complications of myocardial infarction include papillary
muscle rupture with subsequent mitral regurgitation, dysrhythmias, cardiogenic shock, pulmonary edema, ventricular wall rupture, and postinfarction pericarditis. Echocardiography can identify most causes
of cardiogenic shock at the bedside and guide further management.

Incorrect Answers: A, B, D, and E.

Cardiac catheterization (Choice A) may be indicated if echocardiography shows evidence of stent thrombosis or features of acute coronary syndrome requiring invasive intervention. Assessment of other
causes with echocardiography should be performed first.

CT angiography (Choice B) in this scenario is time-consuming and unlikely to provide actionable diagnostic information. Coronary CT angiography is useful in the outpatient setting as a noninvasive method
of identifying obstructive coronary artery disease and patients at risk for major adverse cardiovascular events.

Measurement of serum brain natriuretic peptide concentration (Choice 0) will not provide useful information in management of the patient's acute shock. It is expected to be increased in the setting of heart
failure. This patient is displaying clear clinical signs of cardiogenic shock with recent ischemic damage to the myocardium, and urgent, definitive diagnostic testing is needed.

Radionuclide myocardial scan (Choice E) compares myocardial perfusion at rest and during exertion. It is indicated in the outpatient workup of angina, as well as assessment of myocardial viability for
patients who have suffered an ischemic injury to guide revascularization management decisions.

Educational Objective: Cardiogenic shock can occur in the setting of acute myocardial infarction, severe valvular dysfunction, severe heart failure, heart block, or arrhythmia. Signs and symptoms include
hypotension, cool and pale extremities, jugular venous distension, and pulmonary edema. Echocardiography is indicated to determine the underlying cause.

https://t.me/USMLENBME2CK ts e t
Previous Next Score Report Lab Values Calculator Help pause
Exam Section 3: Item 26 of 50 National Board of Medical Examiners
Comprehensive Clinical Science Self-Assessment

✓ 26. A 2-month-old girl is brought to the clinic by her mother for a well-child examination. The mother says her daughter is exclusively breast-fed; ii takes 1 hour to complete a feeding, and her daughter
sweats during feedings. The patient is at the 10th percentile for weight; she was at the 50th percentile at her last examination 6 weeks ago. Other growth parameters have remained stable at the 50th
percentile. Vital signs are within normal limits. The lungs are clear to auscultation. On cardiac examination, a grade 3/6 systolic murmur is heard best at the lower left sternal border. The liver edge is
palpated 5 cm below the right costal margin. Which of the following is the most likely cause of this patient's failure to thrive?

A) Atrial septal defect


B) Congenital hypothyroidism
C) Fabry disease
D) Glycogen storage disease, type II (Pompe disease)
E) Inadequate breast milk supply
F) Inadequate breast-feeding technique
G) Ventricular septal defect
Correct Answer: G.

Ventricular septal defects (VSDs) are among the most common congenital heart malformations, and they may occur in isolation or in association with complex structural abnormalities. The genetic cause of
VSDs is heterogeneous and includes known associations with well-defined syndromes such as Down, DiGeorge, and Holt-Oram. It may also occur in isolation and via nonspecific, random mutations and
individual point mutations that are de nova. VSOs are characterized by a holosystolic murmur best heard in the lower left sternal border and may have an associated palpable thrill. VSDs result in a left-to-
right shunt, which increases pulmonary blood flow. They are acyanotic lesions. If uncorrected, the increased pulmonary blood flow may result in pulmonary artery remodeling and consequent pulmonary
arterial hypertension, which may cause a reversal of the shunt from right to left. This process is referred to as Eisenmenger syndrome. Clinically, fatigue with feeding, sweating with feeding, hepatomegaly,
and the presence of a murmur may be the only findings.

Incorrect Answers: A, B, C, D, E, and F.

Atrial septa I defect (Choice A) results in a left-to-right shunt with abnormal flow of blood from the left atrium to the right atrium, resulting in relative volume overload of the right atrium and ventricle, which
presents as a fixed, split S,, and low-grade physiologic ejection murmur on cardiac auscultation.

Congenital hypothyroidism (Choice B) is frequently asymptomatic at birth, though may present with lethargy, hypothermia, feeding problems, hoarse cry, myxedematous facies, macroglossia, hypotonia,
umbilical hernia, prolonged jaundice, and dry skin, along with intellectual disability in childhood.

Fabry disease (Choice C) is an X-linked lysosomal storage disorder that results from deficiency of alpha-galactosidase A. Ocular manifestations include cataracts and retinal vascular tortuosity. Cardiac
manifestations include left and right ventricular hypertrophy, and conduction defects. This patient lacks other stigmata of the condition, and is genetically female, making presentation of such rare.

Glycogen storage disease, type II (Pompe disease) (Choice 0) is an inherited recessive disease that causes cardiomegaly, cardiomyopathy, hepatomegaly, and hypotonia.

Inadequate breast milk supply (Choice E) and inadequate breast-feeding technique (Choice F) can lead to breast milk or breast-feeding jaundice. Neither condition presents with a heart murmur or fatigue

https://t.me/USMLENBME2CK ti
Previous Next Score Report Lab Values Calculator Help pause
Exam Section 3: Item 26 of 50 National Board of Medical Examiners
Comprehensive Clinical Science Self-Assessment
" I • I " "

B) Congenital hypothyroidism
C) Fabry disease
D) Glycogen storage disease, type II (Pompe disease)
E) Inadequate breast milk supply
F) Inadequate breast-feeding technique
G) Ventricular septal defect
Correct Answer: G.

Ventricular septal defects (VSDs) are among the most common congenital heart malformations, and they may occur in isolation or in association with complex structural abnormalities. The genetic cause of
VSDs is heterogeneous and includes known associations with well-defined syndromes such as Down, DiGeorge, and Holt-Oram. It may also occur in isolation and via nonspecific, random mutations and
individual point mutations that are de nova. VSOs are characterized by a holosystolic murmur best heard in the lower left sternal border and may have an associated palpable thrill. VSDs result in a left-to-
right shunt, which increases pulmonary blood flow. They are acyanotic lesions. If uncorrected, the increased pulmonary blood flow may result in pulmonary artery remodeling and consequent pulmonary
arterial hypertension, which may cause a reversal of the shunt from right to left. This process is referred to as Eisenmenger syndrome. Clinically, fatigue with feeding, sweating with feeding, hepatomegaly,
and the presence of a murmur may be the only findings.

Incorrect Answers: A, B, C, D, E, and F.

Atrial septal defect (Choice A) results in a left-to-right shunt with abnormal flow of blood from the left atrium to the right atrium, resulting in relative volume overload of the right atrium and ventricle, which
presents as a fixed, split S,, and low-grade physiologic ejection murmur on cardiac auscultation.

Congenital hypothyroidism (Choice B) is frequently asymptomatic at birth, though may present with lethargy, hypothermia, feeding problems, hoarse cry, myxedematous facies, macroglossia, hypotonia,
umbilical hernia, prolonged jaundice, and dry skin, along with intellectual disability in childhood.

Fabry disease (Choice C) is an X-linked lysosomal storage disorder that results from deficiency of alpha-galactosidase A. Ocular manifestations include cataracts and retinal vascular tortuosity. Cardiac
manifestations include left and right ventricular hypertrophy, and conduction defects. This patient lacks other stigmata of the condition, and is genetically female, making presentation of such rare.

Glycogen storage disease, type II (Pompe disease) (Choice 0) is an inherited recessive disease that causes cardiomegaly, cardiomyopathy, hepatomegaly, and hypotonia.

Inadequate breast milk supply (Choice E) and inadequate breast-feeding technique (Choice F) can lead to breast milk or breast-feeding jaundice. Neither condition presents with a heart murmur or fatigue
with feeding.

Educational Objective: Ventricular septal defect is characterized by a holosystolic murmur best heard in the lower left sternal border. Large defects may require repair to prevent persistent shunting and
infantile heart failure.

https://t.me/USMLENBME2CK ti
Previous Next Score Report Lab Values Calculator Help pause
Exam Section 3: Item 27 of 50 National Board of Medical Examiners
Comprehensive Clinical Science Self-Assessment

A67-year-old man comes to the emergency department because of a 12-hour history of increasingly severe generalized, cramping pain that began in his left groin and spread to his abdomen. The
patient rates the pain as a 10 on a 10-point scale. He previously has noted a hernia in the left groin area. He says, "Normally I can push it back in, but this time I couldn't. I was going to wail and make
an appointment with my primary care doctor, but I couldn't tolerate the pain." He has hypertension treated with verapamil and hydrochlorothiazide. The patient appears uncomfortable. Temperature is
37.4C(99.3F), pulse is 96/min, respirations are 20/min, and blood pressure is 140/95 mm Hg. Cardiac examination discloses an S,gallop. Bowel sounds are increased, and the abdomen is soft with
moderate, diffuse tenderness. A tender mass is present in the left groin extending into the scrotal sac, and the skin overlying the groin is erythematous. Results of laboratory studies are shown:
Blood Arterial blood gas analysis
Hematocrit 45% Po 75 mm Hg
Hemoglobin 15 g/dL Pco 34 mm Hg
WBC 18,000/mm3 pH 7.35
Neutrophils, segmented 70%
Neutrophils, bands 20%
Lymphocytes 8%
Monocytes 2%

The physician contacts the on-call surgeon to see the patient. The surgeon instructs the physician to attempt reduction of the hernia and to call back if unsuccessful. Which of the following is the most
appropriate next step in management?

A) Administer hydromorphone and attempt manual reduction of the hernia


B) Contact the hospital administrator and report an Emergency Medical Treatment and Active Labor Act violation
C) Inform the surgeon that the patient needs to be evaluated
D) Obtain CT scan of the abdomen and call the surgeon back if there are signs of bowel obstruction
Correct Answer: C.

An incarcerated or strangulated hernia represents a surgical emergency. Strangulation presents with severe pain in the hernia site, often with overlying reactive erythema, fever, leukocytosis, and a tender,
irreducible mass on examination. In extremis, sepsis and septic shock can arise from necrotic bowel. Immediate surgical exploration is necessary. On-call physicians have an obligation to act ethically, to
promote the patient's best interests, and to respond to calls for patient care in a timely fashion. Additionally, on-call physicians are required under the Emergency Medical Treatment and Active Labor Act
(EMTALA) to appear to perform an in-person evaluation if summoned by another physician. Failure to perform an in-person evaluation in this scenario deviates from the standard of care, may result in harm,
and represents negligence, which may be interpreted as medical malpractice. When encountering a colleague who is neglecting their duties while on-call, the consulting physician should remind their
colleague of this obligation and escalate the demand for the patient to be seen or to locate another surgeon if necessary. Physicians should follow appropriate guidelines for reporting EMTALA violations.

Incorrect Answers: A, B, and D.

Administering hydromorphone and attempting manual reduction of the hernia (Choice A) and obtaining a CT scan of the abdomen (Choice 0) are both inappropriate. This patient demonstrates clear clinical
signs of hernia strangulation. Manual reduction will not be successful in this case, and a CT scan is unlikely to provide additional useful information. The next step in management is surgical exploration.

Contacting the hospital administrator and reporting an EMTALA violation (Choice B) may be necessary if the patient is not seen. The consulting physician's priority should be to remind the on-call physician of ....

https://t.me/USMLENBME2CK ts e t
Previous Next Score Report Lab Values Calculator Help pause
Exam Section 3: Item 27 of 50 National Board of Medical Examiners
Comprehensive Clinical Science Self-Assessment

Hematocrit 45% Po, 75 mm Hg


Hemoglobin 15 g/dl Pco, 34 mm Hg
WBC 18,000/mm? pH 7.35
Neutrophils, segmented 70%
Neutrophils, bands 20%
Lymphocytes 8%
Monocytes 2%

The physician contacts the on-call surgeon to see the patient. The surgeon instructs the physician to attempt reduction of the hernia and to call back if unsuccessful. Which of the following is the most
appropriate next step in management?

A) Administer hydromorphone and attempt manual reduction of the hernia


B) Contact the hospital administrator and report an Emergency Medical Treatment and Active Labor Act violation
C) Inform the surgeon that the patient needs to be evaluated
D) Obtain CT scan of the abdomen and call the surgeon back if there are signs of bowel obstruction
Correct Answer: C.

An incarcerated or strangulated hernia represents a surgical emergency. Strangulation presents with severe pain in the hernia site, often with overlying reactive erythema, fever, leukocytosis, and a tender,
irreducible mass on examination. In extremis, sepsis and septic shock can arise from necrotic bowel. Immediate surgical exploration is necessary. On-call physicians have an obligation to act ethically, to
promote the patient's best interests, and to respond to calls for patient care in a timely fashion. Additionally, on-call physicians are required under the Emergency Medical Treatment and Active Labor Act
(EMTALA) to appear to perform an in-person evaluation if summoned by another physician. Failure to perform an in-person evaluation in this scenario deviates from the standard of care, may result in harm,
and represents negligence, which may be interpreted as medical malpractice. When encountering a colleague who is neglecting their duties while on-call, the consulting physician should remind their
colleague of this obligation and escalate the demand for the patient to be seen or to locate another surgeon if necessary. Physicians should follow appropriate guidelines for reporting EMTALA violations.

Incorrect Answers: A, B, and D.

Administering hydromorphone and attempting manual reduction of the hernia (Choice A) and obtaining a CT scan of the abdomen (Choice 0) are both inappropriate. This patient demonstrates clear clinical
signs of hernia strangulation. Manual reduction will not be successful in this case, and a CT scan is unlikely to provide additional useful information. The next step in management is surgical exploration.

Contacting the hospital administrator and reporting an EMTALA violation (Choice B) may be necessary if the patient is not seen. The consulting physician's priority should be to remind the on-call physician of
their duty to see the patient, or to locate another surgeon if the on-call surgeon continues in their refusal to see the patient.

Educational Objective: On-call physicians have an obligation to act ethically, promote the patient's best interests, and to respond to calls for patient care in a timely fashion. Additionally, on-call physicians are
required under the Emergency Medical Treatment and Active Labor Act (EMTALA) to appear to perform an in-person evaluation if summoned by another physician. Failure to perform an in-person evaluation
in this scenario deviates from the standard of care and represents negligence, which may be interpreted as medical malpractice.

https://t.me/USMLENBME2CK ts e t
Previous Next Score Report Lab Values Calculator Help pause
Exam Section 3: Item 28 of 50 National Board of Medical Examiners
Comprehensive Clinical Science Self-Assessment

"I 28. A 55-year-old man comes to the office because of a 2-month history of moderate right shoulder pain. The patient says the pain occurs whenever he raises his right arm straight up in front of him or up
from his side. He also reports decreased strength in his right shoulder. He says he injured his right shoulder 35 years ago during a college football game when he was hit by another player and landed
shoulder-first on the ground. After the injury, he had pain in the shoulder that resolved in 3 days without treatment. He has not had any subsequent injuries to the shoulder. Medical history otherwise is
unremarkable, and he takes no routine medications. Vital signs are normal. Physical examination discloses inability to resist downward pressure on the right arm when it is held straight out in front with
the thumb pointing downward. Which of the following imaging studies of the right shoulder is most likely to confirm the diagnosis?

A) AP and lateral x-rays


B) Arthrography
C) MRI
D) Ultrasonography
E) No imaging studies are necessary to confirm the diagnosis
Correct Answer: C.

The rotator cuff is the group of muscles and tendons that act to seat the humeral head directly into the center of the glenoid during motion of the shoulder. These muscles also act in concert with other
muscles in the shoulder to internally and externally rotate, abduct, and adduct the shoulder. These muscles include the subscapularis, teres minor, supraspinatus, and infraspinatus. The prime movements of
these muscles at the glenohumeral joint include internal rotation (subscapularis), external rotation (teres minor and infraspinatus), and abduction (supraspinatus). The coupling and antagonistic action of
these muscles seats the humeral head in the glenoid and provides a stable pivot upon which the larger, more powerful muscles of the shoulder can act, such as the latissimus dorsi, deltoid, and pectoralis
major. A rotator cuff tear typically occurs at the insertion of the tendons of the rotator cuff on the humeral head. Commonly, this occurs in the supraspinatus tendon; however, tears can occur in any of the
rotator cuff tendons or at the myotendinous Junction. Risk factors for rotator cuff tears include heavy labor, repetitive overhead lifting, diabetes mellitus, and vascular disease. Chronic disease leading to
tendon degeneration as well as acute or repetitive microtrauma are thought to contribute to eventual tendon failure. The classic presentation of a rotator cuff tear is with pain and weakness in one of the axes
of motion corresponding with the associated rotator cuff muscle. Late manifestations of a tear include clinically appreciable atrophy of the corresponding muscle. MRI is typically indicated for patients with
rotator cuff injury to delineate the extent of injury and to determine the need for or to plan for surgical intervention.

Incorrect Answers: A, B, D, and E.

AP and lateral x-rays (Choice A) of the shoulder would be indicated if acute traumatic fracture or dislocation was suspected.

Arthrography (Choice B) is a diagnostic modality to evaluate joint structure, function, and intra-articular pathologies. This patient's presentation is more consistent with an extra-articular pathology.

Ultrasonography (Choice 0) of the shoulder is limited and can evaluate for Joint effusions or acute soft tissue inflammation such as bursitis. MRI, x-ray, and CT imaging modalities are more sensitive and
specific for shoulder pathologies.

No imaging studies are necessary to confirm the diagnosis (Choice E) is incorrect, as rotator cuff injury can be suspected clinically but requires imaging for definitive diagnosis.

Educational Objective: Rotator cuff injuries typically present with pain and weakness in the direction of shoulder motion that corresponds with the action of the associated rotator cuff muscle. MRI is used to
confirm the diagnosis of rotator cuff injury.

https://t.me/USMLENBME2CK ts e t
Previous Next Score Report Lab Values Calculator Help pause
Exam Section 3: Item 29 of 50 National Board of Medical Examiners
Comprehensive Clinical Science Self-Assessment

✓ 29. A 25-year-old woman is brought to the emergency department 45 minutes after the onset of dizziness. Her symptom began after she consumed 1800 mL of water during a contest in which
participants tried to drink the most water in 15 minutes. She has no history of serious illness and takes no medications. On arrival, she is alert and fully oriented. Her pulse is 88/min and regular,
respirations are 16/min, and blood pressure is 110/80 mm Hg. Examination, including neurologic examination, shows no abnormalities. Laboratory studies show:
Serum
Na' 128 mEq/L
K+ 3.8 mEq/L
Urea nitrogen 5 mg/dl
Glucose 80 mg/dl
Creatinine 0. 7 mg/dl
Urine specific gravity 1.005

Administration of which of the following is the most appropriate next step in management?

A) ADH (vasopressin)
B) Furosemide
C) 0.9% Saline
D) 3% Saline
E) No pharmacotherapy is indicated
Correct Answer: E.

Excess free water consumption over a rapid period can lead to acute hyponatremia, which, if severe, may include neurologic manifestations including altered mental status, seizures, central nervous
system herniation syndromes, coma, and death. Acute consumption of free water results in dilution of renal interstitial concentration gradients and limits concentrating ability. Resulting serum and urine
abnormalities include hyponatremia, low serum osmolality, and low urine osmolality. Hyponatremia, if mild, does not generally cause symptoms or objective examination findings. Treatment of hyponatremia
involves treatment of any underlying disease and gradual sodium correction, typically with fluid restriction in cases of psychogenic polydipsia or syndrome of inappropriate antidiuretic hormone. A safe rate
of sodium correction is generally no more than 6 to 12 mEq/day. When hyponatremia is corrected too rapidly, extracellular sodium exerts osmotic pressure that leads to the rapid efflux of water from
neurons, which can lead to demyelination. The pons is particularly vulnerable to demyelinating injury, and the rapid correction of hyponatremia can result in central pontine myelinolysis. In cases of acute,
severe hyponatremia with neurologic symptoms of consequence (eg, seizure, coma, herniation, altered mental status), correction with hypertonic saline is appropriate.

Incorrect Answers: A, B, C, and D.

ADH (vasopressin) (Choice A) administration leads to the development of hypotonic, euvolemic hyponatremia, which would worsen the patient's condition.

Furosemide (Choice B) is a loop diuretic, which increases the delivery of Na' and K' to the collecting duct by blocking the Na/K'/2CI- c0transporter in the thick ascending loop of Henle. Loop diuretics are
associated with hypokalemia, hypocalcemia, and hypomagnesemia. In the event of hyponatremia resulting from hypervolemia, diuretics may be appropriate. However, this patient is clinically euvolemic and
does not have a precipitating condition such as heart failure to argue for the use of a diuretic.

https://t.me/USMLENBME2CK ti e
Previous Next Score Report Lab Values Calculator Help pause
Exam Section 3: Item 29 of 50 National Board of Medical Examiners
Comprehensive Clinical Science Self-Assessment

Urine specific gravity

Administration of which of the following is the most appropriate next step in management?

A) ADH (vasopressin)
B) Furosemide
C) 0.9% Saline
D) 3% Saline
E) No pharmacotherapy is indicated
Correct Answer: E.

Excess free water consumption over a rapid period can lead to acute hyponatremia, which, if severe, may include neurologic manifestations including altered mental status, seizures, central nervous
system herniation syndromes, coma, and death. Acute consumption of free water results in dilution of renal interstitial concentration gradients and limits concentrating ability. Resulting serum and urine
abnormalities include hyponatremia, low serum osmolality, and low urine osmolality. Hyponatremia, if mild, does not generally cause symptoms or objective examination findings. Treatment of hyponatremia
involves treatment of any underlying disease and gradual sodium correction, typically with fluid restriction in cases of psychogenic polydipsia or syndrome of inappropriate antidiuretic hormone. A safe rate
of sodium correction is generally no more than 6 to 12 mEq/day. When hyponatremia is corrected too rapidly, extracellular sodium exerts osmotic pressure that leads to the rapid efflux of water from
neurons, which can lead to demyelination. The pons is particularly vulnerable to demyelinating injury, and the rapid correction of hyponatremia can result in central pontine myelinolysis. In cases of acute,
severe hyponatremia with neurologic symptoms of consequence (eg, seizure, coma, herniation, altered mental status), correction with hypertonic saline is appropriate.

Incorrect Answers: A, B, C, and D.

ADH (vasopressin) (Choice A) administration leads to the development of hypotonic, euvolemic hyponatremia, which would worsen the patient's condition.

Furosemide (Choice B) is a loop diuretic, which increases the delivery of Na' and K' to the collecting duct by blocking the Na'/K'/2CI- c0transporter in the thick ascending loop of Henle. Loop diuretics are
associated with hypokalemia, hypocalcemia, and hypomagnesemia. In the event of hyponatremia resulting from hypervolemia, diuretics may be appropriate. However, this patient is clinically euvolemic and
does not have a precipitating condition such as heart failure to argue for the use of a diuretic.

0.9% saline (Choice C) can be helpful if hyponatremia does not correct with fluid restriction. However, fluid restriction is the first step in management of asymptomatic mild hyponatremia in the setting of
excess free water intake.

3% saline (Choice D) is not appropriate for this patient. Apart from its use in nonelectrolyte-related cerebral herniation syndromes, hypertonic saline is reserved for patients with severe, acutely symptomatic
hyponatremia.

Educational Objective: Excess free water intake can result in hyponatremia. Treatment of hyponatremia involves treatment of any underlying disease and gradual sodium correction.

https://t.me/USMLENBME2CK ti e
Previous Next Score Report Lab Values Calculator Help pause
Exam Section 3: Item 30 of 50 National Board of Medical Examiners°
Comprehensive Clinical Science Self-Assessment

"I 30. A previously healthy 57-year-old man is brought to the emergency department immediately after being involved in a head-on motor vehicle
collision. He was the driver and was wearing a seat belt with a shoulder strap. On arrival, he is alert and oriented to person, place, and lime.
He says that he does not think he had loss of consciousness but feels sore all over his body. His temperature is 36.7°C (98°F), pulse is
100/min, respirations are 14/min, and blood pressure is 140/70 mm Hg. Pulse oximetry on 2 Umin of oxygen via nasal cannula shows an
oxygen saturation of 99%. Physical examination and an ECG show no abnormalities. A chest x-ray is shown. Which of the following is the
most likely diagnosis?

A) Aortic disruption
B) Aspiration pneumonia
C) Perforation of the esophagus
D) Perforation of an intra-abdominal viscus
E) Pericardial effusion
F) Pneumothorax

R
Correct Answer: A.

Traumatic disruption of the aorta can occur in deceleration injuries such as motor vehicle collisions or falls. Patients can present with tearing chest pain, hypotension, hemodynamic instability, and
tachycardia. Many patients with complete transection or rupture of the aorta do not survive to evaluation and many decompensate abruptly. The gold standard of diagnosis involves CT angiography of the
aorta; however, hypotensive patients may be too unstable to tolerate a CT scan. A chest x-ray can be obtained, which may show a widened mediastinum as seen in this patient. CT angiography of the chest
can differentiate types of blunt aortic injury, including aortic dissection, aortic rupture, or pseudoaneurysm. Repair of traumatic dissection of the aorta can be performed via open repair or endovascular repair.
Such patients are often critically ill and hemodynamically unstable. Attention must be paid to hemodynamic support, reversal of coagulopathy if present, and surgical evaluation without delay for advanced
imaging in cases of high pretest probability.

Incorrect Answers: B, C, D, E, and F.

Aspiration pneumonia (Choice 8) presents as shortness of breath, cough productive of sputum, and fever, with a focal consolidation on chest x-ray, often in the superior segment of the inferior right lobe of
lung.

Perforation of the esophagus (Choice C) can occur from multiple causes, including forceful vomiting (Boerhaave syndrome), trauma, invasive malignancy, and inflammation. Translocation of gastric contents
into the mediastinum can cause rapidly progressive infection and sepsis from mediastinitis. Esophageal perforation is a surgical emergency and is diagnosed through water-soluble contrast esophagography.

Perforation of an intra-abdominal viscus (Choice 0) may lead to severe, diffuse abdominal pain, nausea, vomiting, high fever, tachycardia, and septic shock. Bowel contents in the peritoneal space cause
rapid infection in the form of peritonitis, marked by abdominal rigidity, rebound tenderness, and guarding. Perforated viscus and resultant peritonitis is a surgical emergency that requires exploratory
laparotomv
« https://t.me/USMLENBME2CK ts e t
Previous Next Score Report Lab Values Calculator Help pause
Exam Section 3: Item 30 of 50 National Board of Medical Examiners"
Comprehensive Clinical Science Self-Assessment

D) Perforation of an intra-abdominal viscus


E) Pericardial effusion
F) Pneumolhorax

R
Correct Answer: A.

Traumatic disruption of the aorta can occur in deceleration injuries such as motor vehicle collisions or falls. Patients can present with tearing chest pain, hypotension, hemodynamic instability, and
tachycardia. Many patients with complete transection or rupture of the aorta do not survive to evaluation and many decompensate abruptly. The gold standard of diagnosis involves CT angiography of the
aorta; however, hypotensive patients may be too unstable to tolerate a CT scan. A chest x-ray can be obtained, which may show a widened mediaslinum as seen in this patient. CT angiography of the chest
can differentiate types of blunt aortic injury, including aortic dissection, aortic rupture, or pseudoaneurysm. Repair of traumatic dissection of the aorta can be performed via open repair or endovascular repair.
Such patients are often critically ill and hemodynamically unstable. Attention must be paid to hemodynamic support, reversal of coagulopathy if present, and surgical evaluation without delay for advanced
imaging in cases of high pretest probability.

Incorrect Answers: B, C, D, E, and F.

Aspiration pneumonia (Choice B) presents as shortness of breath, cough productive of sputum, and fever, with a focal consolidation on chest x-ray, often in the superior segment of the inferior right lobe of
lung.

Perforation of the esophagus (Choice C) can occur from multiple causes, including forceful vomiting (Boerhaave syndrome), trauma, invasive malignancy, and inflammation. Translocation of gastric contents
into the mediastinum can cause rapidly progressive infection and sepsis from mediastinitis. Esophageal perforation is a surgical emergency and is diagnosed through water-soluble contrast esophagography.

Perforation of an intra-abdominal viscus (Choice 0) may lead to severe, diffuse abdominal pain, nausea, vomiting, high fever, tachycardia, and septic shock. Bowel contents in the peritoneal space cause
rapid infection in the form of peritonitis, marked by abdominal rigidity, rebound tenderness, and guarding. Perforated viscus and resultant peritonitis is a surgical emergency that requires exploratory
laparotomy.

Pericardia! effusion (Choice E) presents with hypotension, shortness of breath, distant heart sounds, jugular venous distention, and pulsus paradoxus if tamponade is present. Muffled heart sounds may be
the only clue if tamponade physiology is absent. Rapid decompression of the pericardia! space is required to prevent ongoing shock. X-ray imaging is generally normal in acute tamponade.

Pneumothorax (Choice F) occurs as air accumulates within the pleural space as a result of injury of the bronchopulmonary parenchyma or penetrating trauma violating the pleura. On physical examination, a
pneumothorax presents as unilateral decreased breath sounds with hyperresonance to percussion. Pneumothorax can appear on chest x-ray as radiolucent air and the absence of interstitial lung markings in
the pleural space adjacent to the lung parenchyma.

Educational Objective: Traumatic disruption of the aorta can occur in sudden deceleration injuries, and patients can present with chest pain and hemodynamic instability. Chest x-ray can show a widened
mediastinum.

« https://t.me/USMLENBME2CK ts e t
Previous Next Score Report Lab Values Calculator Help pause
Exam Section 3: Item 31 of 50 National Board of Medical Examiners
Comprehensive Clinical Science Self-Assessment

"I 31. A 77-year-old man comes to the physician because of a 2-week history of increasing back pain. Eight years ago, he was diagnosed with prostate cancer. Gonadotropin inhibitor therapy was begun,
and he was asymptomatic until 2 months ago; metastatic lesions at l3 and TB were confirmed at that time. Ibuprofen therapy sufficiently controlled the pain for 1 month, after which oxycodone (5 mg
every 4 hours) and laxatives were added to the medication regimen. At his last visit 1 month ago, he reported his pain as between O and 2 on a 10-point scale. He now describes his pain as 4 to 5 on a
10-point scale. Which of the following is the most appropriate next step in management?

A) Massage therapy and reevaluation in 1 week


B) Add oral acetaminophen therapy to the regimen
C) Begin intravenous morphine therapy
D) Begin oral propoxyphene therapy
E) Increase the oxycodone dosage by 25% to 50%
F) Switch from codeine to meperidine therapy
Correct Answer: E.

Opioids are potent analgesics that reduce the sensation of pain via delta, kappa, and particularly mu opioid receptor agonism throughout the central and peripheral nervous systems. Despite their high
potential to lead to opioid use disorder and overdose death, opioid agonists are useful for the management of pain in certain situations, such as in the perioperative period, following trauma, and in the setting
of cancer, which can produce severe, chronic pain that is often intractable to other modalities. Treatment with opioids typically employs mu opioid receptor agonists, such as morphine, oxycodone,
hydromorphone, fentanyl, or methadone. Dose titration is often necessary after beginning initial therapy. Oral opioids are typically administered on a fixed, scheduled dose. Breakthrough or progressive pain
can be treated by increasing the scheduled dose, usually by at least 25%. The percentage increase in dosage is informed by the severity of pain and assessment of other comorbidities and risk factors for
overdose. Patients with uncontrolled pain on high oral doses of opioids may benefit from transdermal patches or intravenous administration.

Incorrect Answers: A, B, C, D, and F.

Massage therapy and reevaluation in 1 week (Choice A) is incorrect. This patient presents with bone pain caused by vertebral metastases, which will not respond to massage. While nonpharmacologic
techniques of pain management play an important role in addressing pain, this strategy alone is inadequate in this scenario.

Add oral acetaminophen therapy to the regimen (Choice B) is incorrect. Acetaminophen provides a relatively small amount of analgesia in comparison to oxycodone and is likely to be inadequate for
addressing this patient's pain. Additionally, acetaminophen is associated with hepatotoxicity if used at high doses for prolonged periods.

Begin intravenous morphine therapy (Choice C) is an appropriate strategy for intractable pain but is premature at this point. A more appropriate initial strategy is to increase the dose of oxycodone.

Begin oral propoxyphene therapy (Choice 0) is incorrect. Propoxyphene was an opioid that was previously used for analgesia and as a cough suppressant. It has been removed from the market in the
United States because of an association with fatal cardiac arrhythmias.

Switch from codeine to meperidine therapy (Choice F) is incorrect. Meperidine is a pure mu opioid receptor agonist but is not recommended for use in patients with cancer because of its long half-life and
adverse effect profile (including seizures and delirium), creating a high risk for meperidine toxicity with repeated dosing.

https://t.me/USMLENBME2CK ts e t
Previous Next Score Report Lab Values Calculator Help pause
Exam Section 3: Item 31 of 50 National Board of Medical Examiners
Comprehensive Clinical Science Self-Assessment

A) Massage therapy and reevaluation in 1 week


B) Add oral acetaminophen therapy to the regimen
C) Begin intravenous morphine therapy
D) Begin oral propoxyphene therapy
E) Increase the oxycodone dosage by 25% to 50%
F) Switch from codeine to meperidine therapy
Correct Answer: E.

Opioids are potent analgesics that reduce the sensation of pain via delta, kappa, and particularly mu opioid receptor agonism throughout the central and peripheral nervous systems. Despite their high
potential to lead to opioid use disorder and overdose death, opioid agonisls are useful for the management of pain in certain situations, such as in the perioperative period, following trauma, and in the setting
of cancer, which can produce severe, chronic pain that is often intractable to other modalities. Treatment with opioids typically employs mu opioid receptor agonists, such as morphine, oxycodone,
hydromorphone, fentanyl, or methadone. Dose titration is often necessary after beginning initial therapy. Oral opioids are typically administered on a fixed, scheduled dose. Breakthrough or progressive pain
can be treated by increasing the scheduled dose, usually by at least 25%. The percentage increase in dosage is informed by the severity of pain and assessment of other comorbidities and risk factors for
overdose. Patients with uncontrolled pain on high oral doses of opioids may benefit from transdermal patches or intravenous administration.

Incorrect Answers: A, B, C, D, and F.

Massage therapy and reevaluation in 1 week (Choice A) is incorrect. This patient presents with bone pain caused by vertebral metastases, which will not respond to massage. While nonpharmacologic
techniques of pain management play an important role in addressing pain, this strategy alone is inadequate in this scenario.

Add oral acetaminophen therapy to the regimen (Choice B) is incorrect. Acetaminophen provides a relatively small amount of analgesia in comparison to oxycodone and is likely to be inadequate for
addressing this patient's pain. Additionally, acetaminophen is associated with hepatotoxicity if used at high doses for prolonged periods.

Begin intravenous morphine therapy (Choice C) is an appropriate strategy for intractable pain but is premature at this point. A more appropriate initial strategy is to increase the dose of oxycodone.

Begin oral propoxyphene therapy (Choice 0) is incorrect. Propoxyphene was an opioid that was previously used for analgesia and as a cough suppressant. It has been removed from the market in the
United States because of an association with fatal cardiac arrhythmias.

Switch from codeine to meperidine therapy (Choice F) is incorrect. Meperidine is a pure mu opioid receptor agonist but is not recommended for use in patients with cancer because of its long half-life and
adverse effect profile (including seizures and delirium), creating a high risk for meperidine toxicity with repeated dosing.

Educational Objective: Opioids are useful for the management of cancer pain, which can be severe, chronic, and intractable to other treatment modalities. Oral opioids are typically administered on a fixed,
scheduled dose. Breakthrough or progressive pain can be treated by increasing the scheduled dose, usually by at least 25%. Patients with uncontrolled pain on high oral doses of opioids may benefit from
transdermal patches or intravenous administration.

https://t.me/USMLENBME2CK ts e t
Previous Next Score Report Lab Values Calculator Help pause
Exam Section 3: ltem 32 of 50 National Board of Medical Examiners
Comprehensive Clinical Science Self-Assessment

"I 32. An 18-year-old woman comes to the physician for a follow-up examination. She has a 5-year history of asthma treated with inhaled albuterol as needed and inhaled beclomethasone daily. She has
never required treatment in the hospital for an acute episode. She says that her asthma is worse after she spends prolonged periods of time indoors and after she visits homes of friends who own cats.
Her house has hardwood floors, which she and her family clean every week. She sleeps with her windows closed and changes her bedding regularly. She does not smoke or drink alcohol.
Examination shows no abnormalities. The lungs are clear to auscultation. Skin testing shows allergies to dust mites and cat dander. The physician advises the patient to avoid direct contact with cats.
Which of the following environmental changes is likely to be most helpful in preventing exacerbations of this patient's asthma?

A) Humidifier use
B) Placement of carpet in her bedroom
C) Sleeping with the windows open
D) Use of allergen-impermeable mattress and pillow covers
E) Vacuuming the floors of her house daily
Correct Answer: D.

Preventative management of asthma involves controller therapy and avoidance of known triggers. Asthma triggers can be divided into allergenic and nonallergenic categories. Nonallergenic triggers such as
cold exposure and exercise can be anticipated. Allergenic triggers can be more difficult to control, especially in indoor environments. The most common indoor allergens are dust mites, pets, and molds. This
patient has noticed an association between her asthma symptoms and being indoors around cats. Skin testing confirms her hypersensitivity to dust mites and cat dander. One of the most effective proven
strategies for minimizing dust mite exposure is the use of an allergen-impermeable mattress and pillow covers. These have been shown to reduce asthma symptoms in sensitive patients.

Incorrect Answers: A, B, C, and E.

Humidifier use (Choice A) can help decrease asthma symptoms, particularly if cold or dry air has been identified as a trigger. This patient's positive skin testing for dust mite sensitivity indicates that use of an
allergen-impermeable mattress and pillow covers will be more effective.

Placement of carpet in her bedroom (Choice B) should be avoided, as carpeting can harbor allergens that are dispersed with movement through the room. Removal of carpeting may improve a patient's
symptoms.

Sleeping with the windows open (Choice C) should be avoided as well, as it increases exposure to smoke, pollen, and other airborne allergens that may make asthma symptoms worse while providing
limited benefit in reducing the presence of indoor allergens.

Vacuuming the floors of her house daily (Choice E) is not recommended. Though ii does reduce the burden of indoor allergens, the act of vacuuming can distribute allergens throughout the air and worsen
asthma symptoms.

Educational Objective: Use of an allergen-impermeable mattress and pillow covers is effective in reducing asthma symptoms for patients sensitive to indoor allergens. Skin testing for specific
hypersensitivities can help guide management strategies.

https://t.me/USMLENBME2CK ti
Previous Next Score Report Lab Values Calculator Help pause
Exam Section 3: ltem 33 of 50 National Board of Medical Examiners
Comprehensive Clinical Science Self-Assessment

"I 33. A 44-year-old man comes to the office because of a 6-month history of intermittent moderate headache. He has had one to two headaches weekly since residents of the neighboring apartment got a
dog that barks a lot. He says he has had recurrent thoughts about killing the dog during the past 6 months, but he has no intent or plan to harm the animal. He worries that his headaches are
punishment for these thoughts about killing; he says he realizes this does not make sense but remains concerned. He vacuums 2 hours daily because he is worried that dog hair is coming into his
apartment. He has no history of serious illness. His only medication is acetaminophen, which resolves his headaches. He drinks two to three 12-oz beers on weekends. His pulse is 72/min,
respirations are 16/min, and blood pressure is 150/105 mm Hg. Physical examination shows no other abnormalities. Mental status examination shows an anxious affect. Which of the following is the
most likely diagnosis?

A) Delusional disorder
B) Generalized anxiety disorder
C) Obsessive-compulsive disorder
D) Paranoid personality disorder
E ) Somatic symptom disorder
Correct Answer: C.

Obsessive-compulsive disorder (OCD) is an anxiety disorder typically characterized by obsessions (unwanted, intrusive thoughts that produce anxiety) that the patient attempts to neutralize with compulsions
(repetitive behaviors). By definition, the compulsions are excessive or do not realistically address the obsessive fears, and patients commonly recognize the irrationality of their compulsions. This patient's
thoughts of killing his neighbor's dog, as well as worry about dog hair entering his apartment, represent obsessions, while his excessive vacuuming represents a compulsion. Treatment includes use of
selective serotonin reuptake inhibitors (SSRls, eg, fluoxetine) or cognitive behavioral therapy with a focus on exposure to the discomfort and suppressing the compulsion after experiencing the obsession.
SSRls have the lowest adverse effect burden of antidepressant medications. Clomipramine (a tricyclic antidepressant) and venlafaxine (a selective serotonin-norepinephrine reuptake inhibitor
antidepressant) are effective second-line agents.

Incorrect Answers: A, B, D, and E.

Delusional disorder (Choice A) features one or more delusions for a month or longer with no other psychotic symptoms. This patient exhibits both obsessions and compulsions that are more consistent with a
diagnosis of OCD.

Generalized anxiety disorder (Choice B) is an anxiety disorder that features excessive worrying about a range of topics with associated functional impairment. This patient's unwanted, intrusive thoughts
about dog hair entering his apartment are more consistent with an obsession.

Paranoid personality disorder (Choice D) is a cluster A personality disorder characterized by a pervasive mistrust of others and viewing others as possessing malicious intent.

Somatic symptom disorder (Choice E) is characterized by preoccupation with one or more somatic symptoms such that these symptoms disrupt the patient's daily life. Patients persistently devote excessive
time and energy to these symptoms or related health concerns (eg, repeatedly going to the doctor).

Educational Objective: Obsessive-compulsive disorder (OCD) is an anxiety disorder typically characterized by obsessions (unwanted, intrusive thoughts that produce anxiety) that the patient attempts to
neutralize with compulsions (repetitive behaviors). OCD is commonly treated with cognitive behavioral therapy or selective serotonin reuptake inhibitor medications.

https://t.me/USMLENBME2CK ts e t
Previous Next Score Report Lab Values Calculator Help pause
Exam Section 3: Item 34 of 50 National Board of Medical Examiners
Comprehensive Clinical Science Self-Assessment

"I 34. A 27-year-old primigravid woman at 32 weeks' gestation comes lo the emergency department because of a 2-day history of progressive drooping of the right side of her face, an inability lo close her
right eye, and right-sided facial pain. She has not had loss of hearing or balance. Her pregnancy had been uncomplicated. She has no personal or family history of serious illness. Her temperature is
37°C (98.6F), pulse is 88/min, respirations are 16/min, and blood pressure is 140/82 mm Hg. Examination of the face shows paralysis on the right side and an inability to close the right eye. The
patient moves her head stiffly; there is no nuchal rigidity. Heart sounds are normal. A grade 2/6 holosystolic murmur is heard best at the apex. The abdomen is soft. The fund al height is 32 cm. There is
mild edema of the lower extremities. Muscle strength is normal. Deep tendon reflexes are 2+. Which of the following is the most appropriate next step in management?

A) Carotid ultrasonography
B) CT scan of the head
C) Corticosteroid therapy
D) Angiography of the brain
E) Nerve conduction studies
Correct Answer: C.

Complete paralysis of one side of the face with no other neurologic lesion is consistent with an ipsilateral lower motor neuron lesion of the facial nerve (cranial nerve Vil) as in Bell palsy. Bell palsy, also
known as cranial nerve VI I palsy, may be a result of viral infection or inflammation of the nerve. It presents with unilateral weakness of the facial muscles, including those of the forehead. It is differentiated
from ischemic stroke, which typically demonstrates intact forehead musculature, as these muscles are dually innervated by upper motor neurons from bilateral precentral gyri. Lesions involving the lower
motor neurons of facial expression (eg, facial nerve inflammation in Bell palsy) affect the forehead and lower face together. Bell palsy incidence and severity are increased in pregnancy, and cases typically
arise in the third trimester or postpartum period. Treatment of Bell palsy involves corticosteroid therapy and, occasionally, antivirals depending on severity. Supportive care measures such as eye patches
and artificial tears are appropriate to limit ocular complications.

Incorrect Answers: A, B, D, and E.

Carotid ultrasonography (Choice A) is useful for the diagnosis of carotid stenosis or cerebrovascular accidents but has no significant role in the diagnosis of clear cases of Bell palsy.

CT scan of the head (Choice B) should be considered for patients with focal neurologic deficits or severe, sudden onset headache, to rule out hemorrhagic stroke or subarachnoid hemorrhage. This patient
does not have a headache, and the neurologic deficits reported are consistent with Bell palsy. A CT scan would likely provide little additional diagnostic information and would expose this patient and fetus to
unnecessary radiation.

Angiography of the brain (Choice D) can be diagnostic for large vessel occlusions of cerebral arteries causing ischemic stroke. This patient's neurologic deficits are secondary to a cranial nerve palsy, not
acute ischemic stroke.

Nerve conduction studies (Choice E) are used for the diagnosis of muscular weakness or peripheral nerve palsies. Bell palsy is a clinical diagnosis, and nerve conduction studies are not typically required for
diagnosis.

Educational Objective: Bell palsy may result from viral infection or inflammation of the facial nerve and presents with unilateral weakness of the facial muscles, including those of the forehead. Corticosteroid
therapy is the mainstay of treatment.

https://t.me/USMLENBME2CK ts e t
Previous Next Score Report Lab Values Calculator Help pause
Exam Section 3: Item 35 0f 50 National Board of Medical Examiners°
Comprehensive Clinical Science Self-Assessment

35. A 73-year-old woman is brought to the clinic by her husband because of a 6-month history of progressive memory difficulty. During this time, she has misplaced objects more frequently than usual and
has had trouble following recipes. Six days ago, she became lost while walking their dog in their neighborhood. Today, she forgot that their daughter had called the previous day. The patient has type 2
diabetes mellitus and hypertension. Her medications are metformin and lisinopril. She does not smoke cigarettes or drink alcohol. Vital signs are within normal limits. Physical examination shows no
abnormalities. On mental status examination, she is oriented to person but not to place or time. She recalls zero of three objects after 5 minutes. She makes four errors when performing serial sevens.
She can name the US President and Vice President. She can name eight animals in 1 minute. The remainder of the examination shows no abnormalities. Results of a complete blood count, her
erythrocyte sedimentation rate, and results of liver and thyroid function tests are within the reference ranges. VDRL testing for syphilis is negative. MRls of the brain are shown. Which of the following
is the most appropriate next step in management?

A) Deep brain stimulation


B) Placement of a ventriculoperitoneal shunt
C) Transcranial magnetic stimulation
D) Treatment with an acetylcholine precursor

« https://t.me/USMLENBME2CK ts e t
Previous Next Score Report Lab Values Calculator Help pause
Exam Section 3: Item 35 of 50 National Board of Medical Examiners
Comprehensive Clinical Science Self-Assessment
.
'
, .. -- __ , - - - .. -. - .i
She can name the US President and Vice President. She can name eight animals in 1 minute. The remainder of the examination shows no abnormalities. Results of a complete blood count, her
erythrocyte sedimentation rate, and results of liver and thyroid function tests are within the reference ranges. VDRL testing for syphilis is negative. MRls of the brain are shown. Which of the following
is the most appropriate next step in management?

A) Deep brain stimulation


B) Placement of a ventriculoperitoneal shunt
C) Transcranial magnetic stimulation
D) Treatment with an acetylcholine precursor
E) Treatment with a cholinesterase inhibitor
F) Treatment with a postsynaptic choline receptor agonist
Correct Answer: E.

The patient is presenting with classic signs and symptoms of dementia. Dementia, Alzheimer type, is the most prevalent dementia, presenting with progressive cognitive decline that begins with short-term
memory impairment, progresses to apraxia and language abnormalities, and culminates in behavioral and personality changes preventing the patient from performing basic activities of daily living. Dementia,
Alzheimer type, is postulated to result from decreased cholinergic signaling in the cortex and basal forebrain. Acetylcholinesterase normally hydrolyzes acetylcholine in synaptic clefts. Treatment with a
cholinesterase inhibitor, such as donepezil, noncompetitively and reversibly inhibits acetylcholinesterase activity and thereby increases the amount of synaptic acetylcholine available for neurotransmission.
Though these agents may slow the rate of cognitive decline and modestly improve functionality, they are not curative.

Incorrect Answers: A, B, C, D, and F.

Deep brain stimulation (Choice A), in which electrodes are implanted into brain tissue, is an effective treatment modality for alleviating motor symptoms of Parkinson disease. It does not have a role in
treatment of dementia, Alzheimer type.

Placement of a ventriculoperitoneal shunt (Choice B) is used to treat select patients with normal pressure hydrocephalus (NPH). NPH classically presents with the triad of cognitive impairment, gait instability,
and urinary incontinence. Ventriculomegaly is characteristic, though it is a nonspecific finding that is also seen in dementia, Alzheimer type, as a result of atrophy of the brain parenchyma.

Transcranial magnetic stimulation (Choice C) is a treatment for unipolar major depressive disorder for patients that do not respond to pharmacotherapy or psychotherapy. It is important to screen patients for
depressive symptoms, as they often present with reversible cognitive impairments in older adults.

Treatment with an acetylcholine precursor (Choice 0), such as choline, and treatment with a postsynaptic choline receptor agonist (Choice F) have not been shown to be effective in treating dementia,
Alzheimer type, despite the pathophysiology of the disease being associated with acetylcholine deficiency in certain regions of the brain.

Educational Objective: Cognitive impairment in dementia, Alzheimer type, is related to decreased acetylcholine signaling in the cortex. Oonepezil and other cholinesterase inhibitors decrease degradation of
acetylcholine and therefore increase the amount of acetylcholine available for synaptic transmission, showing a modest improvement in symptoms.

https://t.me/USMLENBME2CK ti
Previous Next Score Report Lab Values Calculator Help pause
Exam Section 3: Item 36 of 50 National Board of Medical Examiners
Comprehensive Clinical Science Self-Assessment

"I 36. A previously healthy 77-year-old man comes lo the emergency department because of a 6-hour history of increasingly severe, nonradiating lower abdominal pain. He also has had urinary urgency
during this time but has not been able to void for 8 hours. He has not had fever, chills, diarrhea, nausea, or vomiting. Vital signs are within normal limits. On examination, there is fullness in the midline
from the umbilicus to the pubic symphysis; the area is tender to palpation, but there are no peritoneal signs. Rectal examination shows an enlarged prostate; sphincter tone is normal, and no masses
are palpable. Which of the following is the most likely diagnosis?

A) Bacterial prostatitis
B) Benign prostatic hyperplasia
C) Cecal volvulus
D) Prostatic adenocarcinoma
E) Sigmoid volvulus
F) Urinary tract infection
Correct Answer: B.

Benign prostatic hyperplasia (BPH) affects middle-aged and elderly male patients, and it occurs as a result of excess growth of tissue in the central zone of the prostate causing lower urinary tract
obstruction. BPH presents with urinary dribbling and incontinence, difficulty starting or maintaining a urinary stream, nocturia, the need to frequently urinate, and incomplete voiding. These symptoms are
typically insidious in onset, chronic, and progressive. Urethral or bladder outlet obstruction may lead to acute symptoms of pelvic pain along with an inability to void, and may predispose to cystitis,
hydronephrosis, or pyelonephritis. The diagnosis of BPH is made by history, physical examination, and urodynamic testing, along with ruling out alternate diagnoses (eg, prostatic malignancy). Treatment
includes supportive care, a-blockers (eg, tamsulosin), 5-a-reductase inhibitors (eg, finasteride), or surgery. Acute obstruction can be managed with catheterization in order to decompress the urinary bladder
and prevent acute complications of prolonged bladder outlet obstruction, such as bladder rupture, urothelial cell atrophy or necrosis, hydroureter, hydronephrosis, postrenal azotemia, and/or end-stage
kidney disease.

Incorrect Answers: A, C, D, E, and F.

Bacterial prostatitis (Choice A) may present with urinary retention, but also typically presents with fever, chills, malaise, dysuria, pyuria, and intense pain. The diagnosis is suspected clinically and often
requires imaging for diagnosis of prostatic inflammation or abscess as digital rectal examination is considered relatively contraindicated due to risk for hematogenous seeding with compression, though
evidence for this hypothetical risk is lacking.

Cecal volvulus (Choice C) results from twisting of the cecal mesentery and can be a cause of large bowel obstruction. Similarly, sigmoid volvulus (Choice E) results from twisting of the sigmoid mesentery,
which can lead to large bowel obstruction and potential rupture or intestinal necrosis if not treated. It typically demonstrates a coffee-bean appearance on abdominal x-rays. Neither cecal nor sigmoid
volvulus commonly leads to urinary retention.

Prostatic adenocarcinoma (Choice 0) is commonly asymptomatic but can present with urinary symptoms, such as urinary hesitancy or hematuria, or bone pain if metastatic. It does not commonly cause
urethral or bladder outlet obstruction, which occur more frequently as a result of BPH.

Urinary tract infection (Choice F) may occur in the setting of BPH and bladder outlet obstruction. This patient's acute symptoms of inability to void and pelvic pain are more suggestive of bladder outlet
obstruction secondarv lo BPH than a urinarv tract infection. which tvpicallv presents with dvsuria.

https://t.me/USMLENBME2CK ti e
Previous Next Score Report Lab Values Calculator Help pause
Exam Section 3: Item 36 of 50 National Board of Medical Examiners
Comprehensive Clinical Science Self-Assessment

A) Bacterial prostatitis
B) Benign prostatic hyperplasia
C) Cecal volvulus
D) Prostatic adenocarcinoma
E) Sigmoid volvulus
F) Urinary tract infection
Correct Answer: B.

Benign prostatic hyperplasia (BPH) affects middle-aged and elderly male patients, and it occurs as a result of excess growth of tissue in the central zone of the prostate causing lower urinary tract
obstruction. BPH presents with urinary dribbling and incontinence, difficulty starting or maintaining a urinary stream, nocturia, the need to frequently urinate, and incomplete voiding. These symptoms are
typically insidious in onset, chronic, and progressive. Urethral or bladder outlet obstruction may lead to acute symptoms of pelvic pain along with an inability to void, and may predispose to cystitis,
hydronephrosis, or pyelonephritis. The diagnosis of BPH is made by history, physical examination, and urodynamic testing, along with ruling out alternate diagnoses (eg, prostatic malignancy). Treatment
includes supportive care, a-blockers (eg, tamsulosin), 5-a-reductase inhibitors (eg, finasteride), or surgery. Acute obstruction can be managed with catheterization in order to decompress the urinary bladder
and prevent acute complications of prolonged bladder outlet obstruction, such as bladder rupture, urothelial cell atrophy or necrosis, hydroureter, hydronephrosis, postrenal azotemia, and/or end-stage
kidney disease.

Incorrect Answers: A, C, D, E, and F.

Bacterial prostatitis (Choice A) may present with urinary retention, but also typically presents with fever, chills, malaise, dysuria, pyuria, and intense pain. The diagnosis is suspected clinically and often
requires imaging for diagnosis of prostatic inflammation or abscess as digital rectal examination is considered relatively contraindicated due lo risk for hematogenous seeding with compression, though
evidence for this hypothetical risk is lacking.

Cecal volvulus (Choice C) results from twisting of the cecal mesentery and can be a cause of large bowel obstruction. Similarly, sigmoid volvulus (Choice E) results from twisting of the sigmoid mesentery,
which can lead to large bowel obstruction and potential rupture or intestinal necrosis if not treated. It typically demonstrates a coffee-bean appearance on abdominal x-rays. Neither cecal nor sigmoid
volvulus commonly leads to urinary retention.

Prostatic adenocarcinoma (Choice 0) is commonly asymptomatic but can present with urinary symptoms, such as urinary hesitancy or hematuria, or bone pain if metastatic. It does not commonly cause
urethral or bladder outlet obstruction, which occur more frequently as a result of BPH.

Urinary tract infection (Choice F) may occur in the setting of BPH and bladder outlet obstruction. This patient's acute symptoms of inability to void and pelvic pain are more suggestive of bladder outlet
obstruction secondary to BPH than a urinary tract infection, which typically presents with dysuria.

Educational Objective: Benign prostatic hyperplasia presents in middle-aged to elderly males with urinary dribbling and incontinence, hesitancy, urgency, difficulty starting or maintaining a urinary stream, and
nocturia. Urethral or bladder outlet obstruction may occur and present with acute symptoms of pelvic pain and inability to void. Treatment is with catheterization.

https://t.me/USMLENBME2CK ti e
Previous Next Score Report Lab Values Calculator Help pause
Exam Section 3: Item 37 of 50 National Board of Medical Examiners°
Comprehensive Clinical Science Self-Assessment

% A 6-year-old boy is brought to the emergency department by his parents because of a 2-hour history of severe right leg pain and refusal to walk. They say he has
intermittently reported pain in his leg during the past 2 weeks. He has no history of trauma or serious illness and receives no medications. Growth and development are
appropriate for age. His temperature is 38.1°C (100.5°F), pulse is 100/min, and blood pressure is 100/80 mm Hg. There is warmth and tenderness to palpation of the
proximal aspect of the right tibia with an associated firm mass. Distal pulses are intact. Range of motion of the right knee is limited by pain. Muscle strength in the right
lower extremity is 4/5 because of pain and 5/5 elsewhere. Sensation is intact. Laboratory studies show a hemoglobin concentration of 10 g/dL, leukocyte count of
20,000/mm3, and erythrocyte sedimentation rate of 10 mm/h. X-rays show a 6-cm, poorty defined lytic lesion in the proximal tibial metadiaphysis with adjacent periosteal
reaction. The results of a biopsy are shown. Which of the following is the most likely diagnosis?

A) Acute lymphoblastic leukemia


B) Eosinophitic granuloma
C) Ewing sarcoma
D) Osteoid osteoma
E ) Osteomyelitis

Correct Answer: C.

Ewing sarcoma is a primary bone malignancy that typically presents in childhood, adolescence, or early adulthood, and occurs frequently in the long bones of the extremities, such as the femur or tibia, or in
the axial skeleton, such as in the pelvis. Patients typically present with the insidious onset of dull pain that may be suddenly exacerbated by exercise or minor trauma. The neoplasm is derived from a bone
mesenchymal precursor and is driven by a I( 11 ;22) chromosomal translocation, which results in the formation of an oncogenic fusion protein. X-rays may show elevated periosteum and layered neocortex
formation. The diagnosis is confirmed by biopsy, which shows multiple small, round, blue cells. The prognosis is often poor for patients who are older at initial presentation, with axial skeletal involvement, or
with distant metastases. Treatment is both local, with surgical excision or radiation therapy, and systemic, usually with a combination chemotherapy regimen.

Incorrect Answers: A, B, D, and E.

Acute lymphoblastic leukemia (Choice A) is a common pediatric malignancy of 8- or T-lymphocytes that may present with malaise, fatigue, weight loss, splenomegaly, lymphadenopathy, anemia, and
thrombocytopenia, with circulating leukemic blasts on peripheral blood smear. Bone lesions are atypical.

Eosinophilic granuloma (Choice 8) is a form of Langerhans cell histiocytosis that primarily involves the bones. In children, these lytic bone lesions are commonly seen in the skull.

Osteoid osteoma (Choice D) is a benign bony tumor that commonly occurs in long bones and presents with pain and refusal to bear weight. A characteristic radiolucent nidus with a surrounding
osteosclerotic reaction is seen on x-ray and CT scan.

Osteomyelitis (Choice E) is an infection of bone and bone marrow that typically presents acutely, subacutely, or chronically from direct inoculation or hematogenous spread. Patients typically report pain,
difficulty bearing weight, and overlying erythema.

Educational Objective: Ewing sarcoma is a primary bone malignancy that typically presents in childhood, adolescence, or early adulthood with bone pain exacerbated by exercise or minor trauma. The most
frequent sites of involvement are the long bones of the extremities, such as the femur or tibia, or the axial skeleton, such as the pelvis. X-rays may show elevated periosteum and layered neocortex
formation.

« https://t.me/USMLENBME2CK ts e t
Previous Next Score Report Lab Values Calculator Help pause
Exam Section 3: Item 38 of 50 National Board of Medical Examiners
Comprehensive Clinical Science Self-Assessment

A 39-year-old man comes to the office because of a 1-month history of intermittent regurgitation and constipation. He has regurgitated undigested food four times weekly. He previously had daily bowel
movements; during the past month, he has had a bowel movement every 3 to 4 days. He also has a 1-year history of progressive difficulty swallowing. He says initially he had difficulty swallowing
solids. He now also has difficulty swallowing liquids. When he eats, he occasionally has moderate pain in the lower sternal area that radiates upward. He emigrated from Brazil 5 years ago. Vital signs
are within normal limits. Examination shows no abnormalities. A chest x-ray shows a dilated esophagus with an air-fluid level. Which of the following is the most likely diagnosis?

A) Barrett esophagus
B) Nutcracker esophagus
C) Schatzki ring
D) South American trypanosomiasis
E) Systemic sclerosis (scleroderma)
Correct Answer: D.

South American trypanosomiasis, or Chagas disease, results from infection with the parasitic, flagellate protozoan Trypanosoma cruzi T. cruzi is acquired through the bites of Reduviidae insects, a species
widely found throughout South America, Central America, and areas of the southern United States. Reduviidae insects are nocturnal and feed on sleeping human hosts. They are common in rural areas and
infest thatched roofs, cracks, or holes in poorly built housing. South American trypanosomiasis manifests with an early, acute stage characterized by mild, nonspecific symptoms of fever, malaise, and
lymphadenopathy or splenomegaly, which can last for 1 to 2 months and resolve spontaneously. There may be mild swelling and erythema at the site of the bite, commonly on the eyelids or near the mouth.
South American trypanosomiasis is typically curable if treatment is initiated promptly during the acute stage. If not detected or treated during the acute stage, chronic infection typically ensues, which is not
curable. Chronic South American trypanosomiasis is characterized by cardiac manifestations, including dilated cardiomyopathy, arrhythmia, and congestive heart failure, along with gastrointestinal
manifestations, such as constipation or megaesophagus, which can produce dysphagia, odynophagia, and weight loss. The diagnosis is confirmed with a variety of tests depending on the chronicity of
infection and may include peripheral blood smear to detect extracellular parasites, polymerase chain reaction test, and enzyme-linked immunosorbent assay. Treatment involves prolonged courses of
antiparasitic drugs, such as nifurtimox or benznidazole. Treatment of the chronic phase is supportive and oriented toward management of cardiac and gastrointestinal complications.

Incorrect Answers: A, B, C, and E.

Barrett esophagus (Choice A) is a state of premalignancy marked by intestinal metaplasia of the distal esophagus. Patients typically report a long-standing history of heartburn and may experience
odynophagia. If untreated, Barrett esophagus may progress to esophageal cancer. Esophageal dilation does not occur in Barrett esophagus.

Nutcracker esophagus (Choice B), or hypertensive peristalsis, is a disorder of esophageal dysmotility that presents with dysphagia and intermittent chest pain. It does not lead to esophageal dilation.

Schatzki ring (Choice C) consists of a ring of redundant esophageal mucosal tissue that presents with dysphagia or chest pain if a bolus of food becomes lodged against the ring. Esophageal dilation does
not typically occur.

Systemic sclerosis (scleroderma) (Choice E) presents with taut, hardened skin, Raynaud phenomenon, end-stage kidney disease, telangiectasias, cardiac fibrosis, esophageal dysmotility, and lower
gastrointestinal tract fibrosis. Esophageal manifestations of scleroderma typically result in stricture, dysphagia, and dysmotility, but esophageal dilation does not typically occur.

Educational Objective: South American trypanosomiasis (Chagas disease) results from infection with Trypanosoma cruzi, acquired through the bites of Reduviidae insects. Chronic South American
trypanosomiasis is incurable and characterized by cardiac manifestations including dilated cardiomyopathy, arrhythmia, and congestive heart failure, along with gastrointestinal manifestations such as
constipation or megaesophagus, which can produce dysphagia, odynophagia, and weight loss.

https://t.me/USMLENBME2CK ts e t
Previous Next Score Report Lab Values Calculator Help pause
Exam Section 3: Item 39 of 50 National Board of Medical Examiners
Comprehensive Clinical Science Self-Assessment

✓ 39. A hospitalized 27-year-old woman with sickle cell crisis has shortness of breath and nonproductive cough 4 hours after transfusion of 1 unit of packed red blood cells. Her only medication is
morphine. Her temperature is 38°C (100.4°F), pulse is 126/min, respirations are 28/min, and blood pressure is 98/58 mm Hg. Pulse oximetry on room air shows an oxygen saturation of 88%.
Examination shows central cyanosis. Diffuse crackles are heard bilaterally. Heart sounds are normal. Arterial blood gas analysis on room air shows:
pH 7.47
Pco, 38 mm Hg
Po, 55mm Hg

A chest x-ray shows bilateral pulmonary infiltrates. Which of the following is the most likely cause of these findings?

A) ABO incompatibility
8) Anaphylactic transfusion reaction
C) Fluid overload
0) Hemolytic transfusion reaction
E) Transfusion-related acute lung injury
Correct Answer: E.

Transfusion-related acute lung injury (TRALI) is a rare complication of blood product transfusion but a common cause of death related to transfusion. It commonly presents during or within 6 hours after a
transfusion, with symptoms including fever, cyanosis, hypotension, and hypoxemia. Physical examination may disclose crackles on auscultation of the lungs. Chest x-rays will show new bilateral pulmonary
infiltrates unrelated to cardiac dysfunction. The pathology is thought to be caused by activation of neutrophils in the pulmonary vasculature by antibodies in the transfused product, which leads lo the
release of cytokines and inflammatory mediators. Risk factors for its development include patient risk factors, such as tobacco use, fluid overload, shock, and alcohol use disorder. Platelets, plasma, and
whole blood carry the greatest risk for causing TRALI. Treatment includes discontinuation of the transfusion, oxygen supplementation in the form of noninvasive positive pressure ventilation or intubation,
and hemodynamic support.

Incorrect Answers: A, 8, C, and D.

ABO incompatibility (Choice A) occurs when a patient receives an inappropriately typed and crossed transfusion product. This leads to acute hemolysis as the patient's antibodies attack the transfused red
blood cells. It presents with fever, rigors, flank pain, hematuria, and hypotension. It does not cause hypoxemia or bilateral pulmonary infiltrates as in this patient.

Anaphylactic transfusion reaction (Choice 8) is a rare lgE-mediated transfusion reaction most seen in patients with lgA deficiency or in patients who are allergic to substances used to store blood products.
Symptoms include bronchospasm with associated dyspnea or wheezing, urticaria, flushing, nausea and emesis, edematous lips and tongue, and hypotension with tachycardia. This patient has primarily
respiratory symptoms, including hypoxia and bilateral pulmonary infiltrates, making TRALI a more likely cause of symptoms.

Fluid overload (Choice C) presents with peripheral and pulmonary edema, often after large volume fluid resuscitation. Pulmonary edema can cause dyspnea, hypoxemia, and bilateral pulmonary infiltrates
on chest x-rays, as seen in this patient. However, it is less likely to cause fever and hypotension and would be unlikely to occur after only a single blood product transfusion.

https://t.me/USMLENBME2CK ti
Previous Next Score Report Lab Values Calculator Help pause
Exam Section 3: Item 39 of 50 National Board of Medical Examiners
Comprehensive Clinical Science Self-Assessment

A chest x-ray shows bilateral pulmonary infiltrates. Which of the following is the most likely cause of these findings?

A) ABO incompatibility
B) Anaphylactic transfusion reaction
C) Fluid overload
D) Hemolytic transfusion reaction
E) Transfusion-related acute lung injury
Correct Answer: E.

Transfusion-related acute lung injury (TRALI) is a rare complication of blood product transfusion but a common cause of death related to transfusion. It commonly presents during or within 6 hours after a
transfusion, with symptoms including fever, cyanosis, hypotension, and hypoxemia. Physical examination may disclose crackles on auscultation of the lungs. Chest x-rays will show new bilateral pulmonary
infiltrates unrelated to cardiac dysfunction. The pathology is thought to be caused by activation of neutrophils in the pulmonary vasculature by antibodies in the transfused product, which leads lo the
release of cytokines and inflammatory mediators. Risk factors for its development include patient risk factors, such as tobacco use, fluid overload, shock, and alcohol use disorder. Platelets, plasma, and
whole blood carry the greatest risk for causing TRALI. Treatment includes discontinuation of the transfusion, oxygen supplementation in the form of noninvasive positive pressure ventilation or intubation,
and hemodynamic support.

Incorrect Answers: A, 8, C, and D.

ABO incompatibility (Choice A) occurs when a patient receives an inappropriately typed and crossed transfusion product. This leads to acute hemolysis as the patient's antibodies attack the transfused red
blood cells. It presents with fever, rigors, flank pain, hematuria, and hypotension. It does not cause hypoxemia or bilateral pulmonary infiltrates as in this patient.

Anaphylactic transfusion reaction (Choice 8) is a rare lgE-mediated transfusion reaction most seen in patients with lgA deficiency or in patients who are allergic to substances used to store blood products.
Symptoms include bronchospasm with associated dyspnea or wheezing, urticaria, flushing, nausea and emesis, edematous lips and tongue, and hypotension with tachycardia. This patient has primarily
respiratory symptoms, including hypoxia and bilateral pulmonary infiltrates, making TRALI a more likely cause of symptoms.

Fluid overload (Choice C) presents with peripheral and pulmonary edema, often after large volume fluid resuscitation. Pulmonary edema can cause dyspnea, hypoxemia, and bilateral pulmonary infiltrates
on chest x-rays, as seen in this patient. However, it is less likely to cause fever and hypotension and would be unlikely to occur after only a single blood product transfusion.

Hemolytic transfusion reaction (Choice D) commonly occurs as a result of ABO incompatibility. It presents with fever, rigors, flank pain, hematuria, and hypotension. Laboratory evaluation may disclose
acute kidney injury or signs of disseminated intravascular coagulation. It is less likely than TRALI to cause the hypoxemia or bilateral pulmonary infiltrates on chest x-ray, as seen in this patient.

Educational Objective: Transfusion-related acute lung injury is a rare complication of blood product transfusion that presents with fever, cyanosis, hypotension, hypoxemia, and new bilateral pulmonary
infiltrates on chest x-rays. It is caused by activation of neutrophils in the pulmonary vasculature by antibodies in the transfused product, which leads to the release of inflammatory mediators. Treatment
includes discontinuation of the transfusion, oxygen supplementation in the form of noninvasive positive pressure ventilation or intubation, and hemodynamic support.

https://t.me/USMLENBME2CK ti
Previous Next Score Report Lab Values Calculator Help pause
Exam Section 3: Item 40 of 50 National Board of Medical Examiners
Comprehensive Clinical Science Self-Assessment

"I 40. A 22-year-old man comes to the physician because of a severe nosebleed and a generalized rash for the past 24 hours. He has a 1-month history of fatigue and generalized weakness. His
temperature is 36.7C (98F), pulse is 100/min, respirations are 18/min, and blood pressure is 142/72 mm Hg. Examination of the skin shows diffuse petechiae over the entire body. There is diffuse
lymphadenopathy and mild hepatosplenomegaly. The nares are packed with gauze, and there is no active bleeding. Laboratory studies show:
Hematocrit 28%
Leukocyte count 36,000/mm3
Segmented neutrophils 10%
Lymphocytes 25%
Lymphoblasts and other immature forms 65%
Platelet count 15,000/mm3
Serum
Urea nitrogen 20 mg/dL
Creatinine 1.2 mg/dL
Uric acid 15.2 mg/dL

A biopsy specimen of bone marrow shows acute lymphoblastic leukemia. In addition to vigorous hydration and alkalinization of the urine, which of the following is most appropriate to administer to
prevent acute renal failure during chemotherapy in this patient?

A) Acetylcysteine
B ) Allopurinol
C) Lisinopril
D) Low-dose dopamine
E) Mesna
Correct Answer: B.

Tumor lysis syndrome (TLS) commonly occurs following the initiation of chemotherapy for leukemia or lymphoma. The abrupt destruction of a large number of tumor cells results in interstitial and serum
release of intracellular contents, manifesting as hyperphosphatemia, hypocalcemia, hyperkalemia, and hyperuricemia. Patients with TLS can present with fever, nausea, vomiting, diarrhea, lethargy, cardiac
dysrhythmias, seizures, tetany, muscle cramps, or syncope, and are at risk for sudden death secondary to associated electrolyte and metabolic abnormalities. Purine nucleic acids are metabolized to
hypoxanthine and xanthine, which are converted to uric acid by xanthine oxidase, resulting in hyperuricemia. Uric acid can crystallize in the renal tubules leading to nephropathy and acute kidney injury, or it
can deposit into the joints, leading to arthralgias (eg, gout). Patients undergoing induction chemotherapy should be monitored for TLS and treated prophylactically with aggressive intravenous hydration along
with allopurinol, a xanthine oxidase inhibitor, or rasburicase, a recombinant uric oxidase. Treatment of TLS involves supportive care and correction of electrolyte and metabolic abnormalities. Rasburicase is
helpful in the treatment of TLS. In patients with severe kidney injury and refractory electrolyte derangements, renal replacement therapy with dialysis may be indicated.

Incorrect Answers: A, C, D, and E.

Acetylcysteine (Choice A) is useful for replenishing hepatic stores of the antioxidant glutathione and is commonly used in the setting of acetaminophen overdose. It is not helpful in TLS, which does not result
e elute=lie eleeletie

https://t.me/USMLENBME2CK ts e t
Previous Next Score Report Lab Values Calculator Help pause
Exam Section 3: Item 40 of 50 National Board of Medical Examiners
Comprehensive Clinical Science Self-Assessment

Uric acid 15.2 mg/dL

A biopsy specimen of bone marrow shows acute lymphoblastic leukemia. In addition to vigorous hydration and alkalinization of the urine, which of the following is most appropriate to administer to
prevent acute renal failure during chemotherapy in this patient?

A) Acetylcysteine
B ) Allopurinol
C) Lisinopril
D) Low-dose dopamine
E) Mesna
Correct Answer: B.

Tumor lysis syndrome (TLS) commonly occurs following the initiation of chemotherapy for leukemia or lymphoma. The abrupt destruction of a large number of tumor cells results in interstitial and serum
release of intracellular contents, manifesting as hyperphosphatemia, hypocalcemia, hyperkalemia, and hyperuricemia. Patients with TLS can present with fever, nausea, vomiting, diarrhea, lethargy, cardiac
dysrhythmias, seizures, tetany, muscle cramps, or syncope, and are at risk for sudden death secondary to associated electrolyte and metabolic abnormalities. Purine nucleic acids are metabolized to
hypoxanthine and xanthine, which are converted to uric acid by xanthine oxidase, resulting in hyperuricemia. Uric acid can crystallize in the renal tubules leading to nephropathy and acute kidney injury, or it
can deposit into the joints, leading to arthralgias (eg, gout). Patients undergoing induction chemotherapy should be monitored for TLS and treated prophylactically with aggressive intravenous hydration along
with allopurinol, a xanthine oxidase inhibitor, or rasburicase, a recombinant uric oxidase. Treatment of TLS involves supportive care and correction of electrolyte and metabolic abnormalities. Rasburicase is
helpful in the treatment of TLS. In patients with severe kidney injury and refractory electrolyte derangements, renal replacement therapy with dialysis may be indicated.

Incorrect Answers: A, C, D, and E.

Acetylcysteine (Choice A) is useful for replenishing hepatic stores of the antioxidant glutathione and is commonly used in the setting of acetaminophen overdose. It is not helpful in TLS, which does not result
in glutathione depletion.

Lisinopril (Choice C) is an angiotensin-converting enzyme inhibitor and is useful for the treatment of hypertension. While hypertension may occur in the setting of acute kidney injury secondary to TLS,
lisinopril plays no role in the routine prophylaxis of TLS.

Low-dose dopamine (Choice 0) has been used historically for the prevention of acute kidney injury. Its use is not supported by high-quality evidence, and there is evidence that ii may worsen renal perfusion.

Mesna (Choice E) is utilized to reduce the risk for hemorrhagic cystitis in patients taking cyclophosphamide. It plays no role in the prophylaxis of TLS.

Educational Objective: Patients undergoing initiation of chemotherapy for leukemia or lymphoma should be monitored for tumor lysis syndrome. Prophylaxis includes aggressive intravenous hydration and
treatment with allopurinol or rasburicase.

https://t.me/USMLENBME2CK ts e t
Previous Next Score Report Lab Values Calculator Help pause
Exam Section 3: Item 41 of 50 National Board of Medical Examiners
Comprehensive Clinical Science Self-Assessment

"I 41. A 25-year-old man is brought to the emergency department by ambulance 25 minutes after his wife found him unconscious on the floor at home this morning. He was discharged from the military
3 months ago after he sustained a traumatic brain injury and underwent right below-the-knee amputation. Since his return, his family says he has had mood swings, aggressive behavior, and anxiety,
which have prevented him from interacting with people. He has major depressive disorder and severe limb pain. His medications are amitriptyline, sertraline, oxycodone, and acetaminophen. Two days
ago, he began a short-term course of alprazolam for persistent anxiety. His family says he generally has been adherent to his medication and cognitive behavioral therapy regimens. He does not
smoke cigarettes. He drank four 12-oz beers weekly as a teenager; now he drinks eight 12-oz beers weekly. On arrival, he is somnolent and responds to painful stimuli only. Temperature is 36.7°C
(98.0°F), pulse is 110/min, respirations are 10/min and shallow, and blood pressure is 85/60 mm Hg. Pulse oximetry on 4 Umin of oxygen by nasal cannula shows an oxygen saturation of 92%.
Examination shows warm, dry, flushed skin. The pupils are mydriatic with nystagmus. Air movement is decreased in the lungs. Cardiac examination shows no abnormalities. The abdomen is soft.
Bowel sounds are decreased. Patellar deep tendon reflexes are 3+ bilaterally. An overdose of which of the following is the most likely explanation for these findings?

A) Acetaminophen
8) Alcohol
C) Alprazolam
D) Amitriptyline
E ) Oxycodone
Correct Answer: D.

This patient has likely attempted suicide by overdose with amitriptyline, a tricyclic antidepressant (TCA). TCA overdose presents with anticholinergic adverse effects (eg, hyperthermia, flushing, mydriasis,
ileus, and urinary retention) and cardiac arrhythmias, generally wide ORS complex tachycardia. Neurologic manifestations include mental status changes, decreased level of consciousness, delirium, and
seizures. Resuscitation can include intubation for patients not protecting the airway or with respiratory failure, the administration of sodium bicarbonate to treat cardiac toxicity, and the administration of
intravenous fluids to correct hypotension. A dose of activated charcoal may be given to bind any remaining intraluminal drug but should only be used if the airway is protected. The mainstay of treatment for
TCA overdose is sodium bicarbonate. A broad set of laboratory studies should be sent to assess for end-organ dysfunction and coingestion.

Incorrect Answers: A, 8, C, and E.

Acetaminophen poisoning (Choice A) presents with nonspecific symptoms of fatigue, malaise, and abdominal pain followed by signs and symptoms of liver failure, including jaundice, altered mental status,
and coagulopathy.

Alcohol (Choice 8) is a central nervous system (CNS) depressant that, in toxic doses, can cause slurred speech, ataxia, emotional lability, and memory lapses. Similarly, alprazolam (Choice C) is a
benzodiazepine medication, a CNS depressant with a toxidrome of slurred speech, ataxia, and altered mental status. While alcohol and benzodiazepines are CNS depressants causing sedation, they would
not cause an anticholinergic toxidrome as seen in TCA toxicity.

Oxycodone (Choice E) is an opioid. Opioid intoxication causes euphoria, altered mental status, sedation, bradycardia, hypotension, depressed respiratory drive, and constricted pupils. Miosis often
differentiates opioid intoxication from other CNS depressant toxidromes.

Educational Objective: Features of tricyclic antidepressant overdose include confusion, delirium, cardiac arrhythmias, hypotension, and anticholinergic toxicity (eg, ileus, urinary retention). Other neurologic
manifestations can include increased deep tendon reflexes and seizures.

https://t.me/USMLENBME2CK ts e t
Previous Next Score Report Lab Values Calculator Help pause
Exam Section 3: Item 42 of 50 National Board of Medical Examiners
Comprehensive Clinical Science Self-Assessment

✓ 42. A 29-year-old woman comes to the physician for a routine health maintenance examination. She has no history of serious illness and takes no medications. Menses occur at regular 28-day intervals,
last 5 days, and are accompanied by mild cramps. Her last menstrual period was 3 weeks ago. She is sexually active and monogamous with one male partner. She has a copper IUD in place. Her
blood pressure is 118/72 mm Hg. Physical examination shows no abnormalities. Pelvic examination shows an IUD string at the cervical os and a mildly tender right adnexal mass. Transvaginal
ultrasonography shows a normal uterus and left adnexa and a 5-cm hemorrhagic cyst in the right adnexa. Which of the following is the most appropriate next step in management?

A) Ceftriaxone and doxycycline therapy


B) Oral contraceptive therapy
C) Ovarian cystectomy
D) Removal of the IUD
E) Repeat transvaginal ultrasonography in 6 weeks
Correct Answer: E.

The corpus luteum develops from the mature follicle after the egg has ovulated during the menstrual cycle. In some instances, the corpus luteum can form a cyst, which can remain asymptomatic or
potentially rupture. While a cyst may remain asymptomatic and be found incidentally on physical examination, a ruptured corpus luteum cyst classically presents with unilateral pelvic or lower abdominal
pain that is severe and sudden in onset, often after sexual intercourse or physical activity. Pelvic ultrasonography will show a complex cystic mass (if unruptured) or free fluid in the pelvis, which may
include hemorrhagic components (hemoperitoneum). Corpus luteum cysts are generally managed with supportive treatment and surveillance. As this patient has an unruptured, stable cyst, the most
appropriate next step in management is repeat transvaginal ultrasonography in 6 weeks to ensure its resolution.

Incorrect Answers: A, B, C, and D.

Ceftriaxone and doxycycline therapy (Choice A) may be used in the treatment of sexually transmitted infections or tuba-ovarian abscess, the latter of which commonly presents with fever, mucopurulent
cervical discharge, and cervical or adnexal tenderness. It may result in tachycardia and hypotension if severe. However, ultrasonography would more commonly show a heterogeneous, multilocular mass
rather than a hemorrhagic cyst.

Oral contraceptive therapy (Choice B) is not necessary in this patient. While oral contraceptives may decrease the risk for ovarian cyst development as they inhibit ovulatory cycles, this patient is mildly
symptomatic and has had no previous corpus luteum cysts. As such, ii is more appropriate to ensure resolution of the cyst in 6 weeks with ultrasonography.

Ovarian cystectomy (Choice C), or surgical removal of the cyst, can be performed for persistent or symptomatic cysts. However, proceeding with an invasive surgical procedure at this time would be
premature, as this patient's cyst may resolve on its own.

Removal of the IUD (Choice D) is not necessary. Copper-containing IUDs are not associated with an increased risk for corpus luteum cysts.

Educational Objective: Corpus luteum cysts can develop from the mature follicle. While they commonly remain asymptomatic, they can rupture and cause unilateral pelvic or lower abdominal pain that is
severe and sudden in onset. Transvaginal ultrasonography is often used to confirm the diagnosis. Asymptomatic, nonruptured cysts can undergo surveillance with follow-up imaging to ensure resolution.

https://t.me/USMLENBME2CK ti e
Previous Next Score Report Lab Values Calculator Help pause
Exam Section 3: ltem 43 of 50 National Board of Medical Examiners
Comprehensive Clinical Science Self-Assessment

"I 43. A 20-year-old primigravid woman at 28 weeks' gestation is brought to the emergency department after she was involved in a high-speed motor vehicle collision. She was the restrained front seat
passenger. She has no history of serious illness. Her pregnancy had been uncomplicated. She has moderate neck and back pain and abdominal tenderness. She appears confused. Her temperature
is 37C (98.6°F), pulse is 120/min, respirations are 24/min, and blood pressure is 100/58 mm Hg. The fetal heart rate is 140--150/min. Examination shows multiple facial contusions. Laboratory studies
show a hematocrit of 32%, leukocyte count of 9000/mm3, and platelet count of 210,000/mm3. In addition to continuous fetal monitoring, which of the following is the most appropriate next step in
management?

A) CT scan of the head and neck


B) Administration of Rho (D) immune globulin
C) Terbutaline tocolysis
D) Cesarean delivery
E) Exploratory laparotomy
Correct Answer: A.

The initial evaluation of a critically ill patient includes assessing the airway, breathing, circulation, central nervous system, and any external threats to morbidity or mortality. After blunt head injury, patients
can present with concussion or traumatic intracranial hemorrhage such as subdural hematoma, subarachnoid hemorrhage, or epidural hematoma. A young patient with altered mental status and signs of
trauma to the head raises concern for traumatic intracranial hemorrhage. CT scan of the head is critical in the diagnosis and management of a patient with head trauma. CT scan of the neck is also indicated
in patients with neck pain as well as altered mental status in a high-speed blunt traumatic injury. Although pregnant, the risk to the mother (and therefore fetus) is high, and benefits of radiation outweigh risks
in such cases.

Incorrect Answers: B, C, D, and E.

Administration of Rho (D) immune globulin (Choice B) would be indicated in cases of any concern for fetomaternal hemorrhage in pregnant women who are Rh-antigen negative. Exposure to the Rh antigen
can result in immunoglobulin formation and present risk for subsequent fetal hemolysis in future pregnancies if the fetus is Rh positive. It can be administered promptly but would not take priority over CT
scan of the head and neck in this case.

Terbutaline tocolysis (Choice C) is indicated to help prevent premature labor. This patient does not have evidence of contractions, so tocolysis is not indicated.

Cesarean delivery (Choice D) would be indicated in pregnant patients with imminent or active cardiac arrest, with the goal to resuscitate the mother and improve fetal survival.

Exploratory laparotomy (Choice E) of the abdomen is indicated for hemodynamically unstable patients presenting after abdominal trauma, as it permits the control of hemorrhage and the repair of identified
injuries. In stable cases, imaging precedes surgical intervention.

Educational Objective: After blunt head injury, patients can present with concussion or traumatic intracranial hemorrhage such as subdural hematoma, subarachnoid hemorrhage, or epidural hematoma. CT
scan of the head and neck is critical in diagnosing traumatic intracranial and cervical spine injuries in patients with altered mental status.

https://t.me/USMLENBME2CK ti
Previous Next Score Report Lab Values Calculator Help pause
Exam Section 3: Item 44 of 50 National Board of Medical Examiners
Comprehensive Clinical Science Self-Assessment

"I 44. A 15-year-old boy is brought to the physician because of fatigue since starting his freshman year of high school 3 months ago. He often falls asleep during class. He urinates four to five times nightly
and often has difficulty falling asleep again. He has no history of serious illness and lakes no medications. He is at the 20th percentile for height and above the 95th percentile for weight and BMI. Vital
signs are within normal limits. Examination shows a velvety, hyperpigmented macular rash over the neck and axillae. The remainder of the examination shows no abnormalities. Laboratory studies
show:
Hematocrit 39%
Leukocyte count 5900/mm3
Platelet count 401,000/mm3
Serum
Na 133 mEq/L
K' 4.3 mEq/l
Cl- 98 mEq/L
HCO 3 22 mEq/L
Urea nitrogen 14 mg/dL
Glucose 148 mg/dL
Creatinine 0.6 mg/dL
Urine
pH 5
Specific gravity 1.035
Glucose 2+
Ketones negative

This patient is at increased risk for which of the following complications of his current condition during the next 15 years?

A) Adenocarcinoma of the bowel


B) Adrenal insufficiency
C) Coronary artery disease
D) Diabetes insipid us
E) Ruptured cervical disc
Correct Answer: C.

Type 2 diabetes mellitus can be asymptomatic or present with polydipsia, polyuria, acanthosis nigricans, and diabetic ketoacidosis or hyperosmolar hyperglycemic state, if severe. While there are several
accepted diagnostic criteria, commonly employed criteria include a hemoglobin A1C greater than or equal to 6.5% or a fasting glucose concentration greater than or equal to 126 mg/dL. The Framingham
Heart Study identified a roughly twofold increase in the rate of coronary artery disease (CAO) in patients with diabetes mellitus. Patients with diabetes mellitus are also at increased risk for extensive,
multivessel CAD. In addition to diabetes mellitus, childhood or adolescent obesity is a risk factor for the premature development of CAD. Initial management for type 2 diabetes mellitus includes weight loss
and lifestyle modifications, with the addition of oral antihyperglycemics if these modifications are unsuccessful. A multifactorial approach is necessary to reduce the risk for developing CAD by controlling
nee4wee eeletlevie teetewedueled eleerh reeleleeve wellneterenew

https://t.me/USMLENBME2CK ti e
Previous Next Score Report Lab Values Calculator Help pause
Exam Section 3: Item 44 of 50 National Board of Medical Examiners
Comprehensive Clinical Science Self-Assessment

Glucose 2+
Ketones negative

This patient is at increased risk for which of the following complications of his current condition during the next 15 years?

A) Adenocarcinoma of the bowel


B) Adrenal insufficiency
C) Coronary artery disease
D) Diabetes insipid us
E) Ruptured cervical disc
Correct Answer: C.

Type 2 diabetes mellitus can be asymptomatic or present with polydipsia, polyuria, acanthosis nigricans, and diabetic ketoacidosis or hyperosmolar hyperglycemic state, if severe. While there are several
accepted diagnostic criteria, commonly employed criteria include a hemoglobin A1C greater than or equal to 6.5% or a fasting glucose concentration greater than or equal to 126 mg/dl. The Framingham
Heart Study identified a roughly twofold increase in the rate of coronary artery disease (CAD) in patients with diabetes mellitus. Patients with diabetes mellitus are also at increased risk for extensive,
multivessel CAD. In addition to diabetes mellitus, childhood or adolescent obesity is a risk factor for the premature development of CAD. Initial management for type 2 diabetes mellitus includes weight loss
and lifestyle modifications, with the addition of oral antihyperglycemics if these modifications are unsuccessful. A multifactorial approach is necessary to reduce the risk for developing CAD by controlling
hyperglycemia and other risk factors, such as obesity, hyperlipidemia, and hypertension.

Incorrect Answers: A, B, D, and E.

Adenocarcinoma of the bowel (Choice A) is incorrect. While diabetes mellitus and obesity are associated with a higher incidence of nonalcoholic fatty liver disease, they are not associated with bowel
adenocarcinoma.

Adrenal insufficiency (Choice B) is incorrect. Type 2 diabetes mellitus occurs as a result of increased peripheral insulin resistance. While diabetes mellitus may result in kidney dysfunction as a result of
chronic microvascular injury, adrenal function is typically normal.

Diabetes insipidus (Choice 0) is incorrect. This adolescent with obesity is more likely lo have type 2 diabetes mellitus. Diabetes mellitus is not related to diabetes insipidus, which occurs as a result of
impaired hypothalamic or pituitary production of vasopressin, or impaired renal responsiveness to vasopressin.

Ruptured cervical disc (Choice E) does not commonly occur in adolescents with diabetes mellitus or obesity. Obesity is associated with a higher incidence of slipped capital femoral epiphysis.

Educational Objective: Diabetes mellitus and childhood or adolescent obesity are significant risk factors for coronary artery disease (CAO). Diabetes mellitus is also a risk factor for multivessel CAD. A
multifactorial approach is necessary to reduce the risk for developing CAD by controlling hyperglycemia and other risk factors, such as obesity, hyperlipidemia, and hypertension.

https://t.me/USMLENBME2CK ti
Previous Next Score Report Lab Values Calculator Help pause
Exam Section 3: Item 45 of 50 National Board of Medical Examiners
Comprehensive Clinical Science Self-Assessment

"I 45. A previously healthy 3-year-old boy who is scheduled for an elective operation begins to cry and cling tightly to his mother on arrival at the hospital. Which of the following is the most likely cause of this
behavior?

A) Fear of mutilation
B) Fear of pain
C) Fear of separation from parent
D) Panic disorder
E ) Stranger anxiety
Correct Answer: C.

Separation from a primary caregiver often leads to fear and distress in children, which is referred to as separation anxiety. Separation anxiety is common in young children, especially those between age 18
months and 3 years. Children around this age have not developed object permanence, or the idea that items out of their sight continue to exist. Separation anxiety can become even more apparent in
unfamiliar situations, such as the healthcare system. This 3-year-old patient is approaching an unfamiliar location and is demonstrating crying and clinging, which are common symptoms of separation anxiety
likely related to a fear of being separated from his mother.

Incorrect Answers: A, B, D, and E.

Fear of mutilation (Choice A) and fear of pain (Choice B) are fears related to receiving bodily harm. This patient, who was previously healthy and has no experience in the healthcare system, is unlikely to
recognize that arriving at the hospital may be associated with pain caused by intravenous line placement or surgery itself. Therefore, fear of separation from his mother is more likely.

Panic disorder (Choice 0) is a type of anxiety disorder occasionally seen in children. The primary presenting feature of panic disorder is the presence of panic attacks, which are episodes of intense anxiety
associated with palpitations, tachypnea, diaphoresis, and an impending sense of doom. These attacks generally lead to avoidance behaviors to prevent further attacks in the future. This patient is experiencing
normal separation anxiety in an unfamiliar situation.

Stranger anxiety (Choice E) is also common in children and manifests as fear and avoidance of strangers. It is more common in children from age 9 months to 2 years. This patient has not met any strangers
yet in this scenario, making fear of separation from his mother the more likely cause of his presentation.

Educational Objective: Separation anxiety is common in children from age 18 months to 3 years. It often manifests with fear and distress, and it is associated with crying and clinging to the primary caregiver
when faced with situations in which the caregiver may leave. This can be exacerbated in unfamiliar situations.

https://t.me/USMLENBME2CK ti
Previous Next Score Report Lab Values Calculator Help pause
Exam Section 3: Item 46 of 50 National Board of Medical Examiners
Comprehensive Clinical Science Self-Assessment

"I 46. A 19-year-old woman is brought to the emergency department by her mother 30 minutes after ingesting an unknown liquid she found in her parents' utility room following an argument with her
boyfriend. Immediately after swallowing the liquid, she telephoned her mother at work, who arrived at home 20 minutes later and found her daughter drooling and reporting severe upper chest pain. On
arrival at the emergency department, the patient is in obvious distress and is drooling and unable to swallow. Temperature is 37.0°C (98.6°F), pulse is 120/min and regular, respirations are 20/min, and
blood pressure is 148/90 mm Hg. Physical examination shows profuse clear drainage from the mouth and erythema and edema of the posterior oropharynx. Cardiopulmonary examination shows no
abnormalities. Two large-bore intravenous catheters are inserted for fluid administration and a dose of intravenous hydromorphone is administered. The patient begins to feel more comfortable.
Results of laboratory studies are within the reference ranges. Chest x-ray shows no abnormalities. Which of the following is the most appropriate management?

A) Administration of activated charcoal


B) Administration of oral bicarbonate
C) Esophagogastroduodenoscopy
D) Induction of vomiting
E) Nasogastric tube placement
Correct Answer: C.

This patient, presenting after an acute ingestion, likely ingested a caustic household chemical. Drain cleaners are a common cause of caustic ingestions. They contain hydroxides, alkali that are proton
acceptors and strong nucleophiles. Alkali ingestion leads to liquefactive necrosis, lactic acidosis, and tissue penetration and ulceration. It can cause chemical burns and ulcerations of the tongue, airway, and
esophagus, resulting in strider, hoarseness, vomiting, and drooling. If severe, esophageal perforation can result, leading to leakage of gastrointestinal contents into the mediastinum. Treatment is initially
focused on supportive measures, including airway management and fluid resuscitation. Visible lesions in the oropharynx increase the likelihood of distal gastrointestinal lesions. Patients should be evaluated
with esophagogastroduodenoscopy (EGO) to assess for visceral injury. For severe injuries, treatment involves corticosteroid administration, antibiotics, and potentially surgical intervention in cases of
perforation.

Incorrect Answers: A, B, D, and E.

Administration of activated charcoal (Choice A) would not be safe for a patient who is drooling, unable to swallow, and has limited ability to protect the airway. Charcoal, if aspirated, can be harmful to the
pulmonary parenchyma and cause aspiration pneumonitis. Activated charcoal can also obscure the results of EGO.

Administration of oral bicarbonate (Choice B) would not be safe for a patient who is unable to swallow and is generally inappropriate management of any acid/base ingestion as it can cause additional
exothermic reactions via acid-base neutralization, which can lead to further injury.

Induction of vomiting (Choice 0) would cause further harm to this patient's esophagus by inducing trauma and increased pressure. It can also cause further exposure of caustic alkali to esophageal mucosa
and worsen damage.

Nasogastric tube placement (Choice E) should also be avoided as it can cause further trauma to any existing esophageal injury. Patients with caustic alkali injury can have esophageal perforation, and the
introduction of a blind nasogastric tube can create a false passage or lumen.

Educational Objective: Caustic household chemicals such as rain cleaners contain caustic alkali that can cause burns and ulcerations of the tongue, airway, and esophagus, resulting in strider, hoarseness,
vomiting, and drooling. Esophagogastroduodenoscopy is most appropriate to evaluate the extent of mucosal injury.

https://t.me/USMLENBME2CK ti e
Previous Next Score Report Lab Values Calculator Help pause
Exam Section 3: Item 47 of 50 National Board of Medical Examiners
Comprehensive Clinical Science Self-Assessment

"I 47. A 23-year-old male college student comes to the clinic for a follow-up visit 14 days after undergoing emergent splenectomy following a motor vehicle collision. The patient has resumed his regular diet
and is ambulating normally. Medical history otherwise is unremarkable, and his only medication is acetaminophen as needed. The patient completed his Haemophi/us influenzae type b vaccinations by
age 18 months and his varicella, polio, and measles-mumps-rubella vaccinations by age 5 years; he received a human papillomavirus vaccination at age 11 years. Temperature is 37.0°C (98.6°F), pulse
is 82/min, and blood pressure is 116/54 mm Hg. Abdominal examination shows a well-healing mid line incision. Which of the following vaccinations is most appropriate to administer to this patient today?

A) Haemophilus influenzae type b


B) Human papillomavirus
C) Inactivated poliovirus
D) Measles, mumps, rubella
E) Varicella
F) No vaccine is indicated at this time
Correct Answer: A.

Asplenia can occur because of congenital anomalies, splenectomy after trauma, or because of chronic vasoocclusion and infarction in conditions such as sickle cell disease. The spleen removes opsonized
pathogens and infected cells by mechanical filtration in the sinusoids and by phagocytosis by splenic macrophages. Additionally, the spleen houses immunoglobulin-producing B lymphocytes. The spleen plays
an important role in the clearance of encapsulated pathogens such as Haemophilus influenzae, Neisseria meningitidis, and Streptococcus pneumoniae, among others. Asplenic patients are at risk for infection
with encapsulated bacteria, making vaccination against encapsulated bacteria, such as Haemophi/us influenzae, the most appropriate for this patient.

Incorrect Answers: B, C, D, E, and F.

Viral infection risk from human papillomavirus (Choice B), inactivated poliovirus (Choice C), measles, mumps, rubella (Choice 0), and varicella (Choice E) is not necessarily increased after splenectomy.

No vaccine (Choice F) would be harmful as patients with asplenia are at increased risk for infection for encapsulated bacteria.

Educational Objective: Splenectomy increases the risk for infection with encapsulated organisms such as Haemophi/us influenzae, Neisseria meningitidis, and Streptococcus pneumoniae. Patients with
impaired splenic function require pneumococcal, Haemophilus influenzae, and meningococcal vaccines.

https://t.me/USMLENBME2CK ts e t
Previous Next Score Report Lab Values Calculator Help pause
Exam Section 3: Item 48 of 50 National Board of Medical Examiners
Comprehensive Clinical Science Self-Assessment

A previously healthy 19-year-old college student comes to the emergency department because of a 3-day history of lower abdominal pain and vaginal spotting. Menses usually occur at regular 28-
day intervals, but she cannot recall the date of her last menstrual period. She takes no medications. She is sexually active and uses condoms inconsistently. She appears pale but is alert and
answers questions appropriately. Her pulse is 110/min, respirations are 22/min, and blood pressure is 80/50 mm Hg. Abdominal examination shows lower quadrant tenderness with marked guarding.
Pelvic examination shows a soft uterus and adnexal tenderness greater on the right than on the left. Her hemoglobin concentration is 6.2 g/dl, hematocrit is 18%, and serum �-hCG concentration is
6645 mlU/mL. Urinalysis shows no abnormalities. Which of the following is the most appropriate next step in management?

A) Observation only
B) CT scan of the abdomen
C) Intravenous antibiotic therapy
D) Culdocentesis
E) Exploratory laparotomy
Correct Answer: E.

Pregnancy is suspected when there is a missed or delayed menstrual period. An ectopic pregnancy is an abnormal pregnancy in which the fertilized ovum implants in the fallopian tube, which is most
common, on the ovary, within the peritoneal cavity, or in any nonendometrial location. It often presents with vaginal bleeding and abdominopelvic pain. 3-hCG concentrations will be detectable, and the
pregnancy may be visible on ultrasonography. Ectopic pregnancies are nonviable. Small, early pregnancies can be managed medically with methotrexate. A rare complication of an unrecognized ectopic
pregnancy is rupture, leading to bleeding and hemoperitoneum. In this instance, physical examination may show abdominal guarding. Severe cases may lead to hemorrhagic shock with tachycardia,
hypotension, and anemia, as seen in this patient. Treatment of a ruptured ectopic pregnancy is emergent exploratory laparolomy along with fluid and blood product resuscitation.

Incorrect Answers: A, B, C, and D.

Observation only (Choice A) would be inappropriate, as this patient is experiencing hemorrhagic shock from intraperitoneal bleeding. Failure to control the bleeding with exploratory laparotomy will result in
ongoing hemorrhage, shock, and eventual death.

CT scan of the abdomen (Choice 8) is not the most appropriate next step in management. While it may show the ruptured ectopic pregnancy, this patient is hemodynamically unstable and requires
emergent surgical evaluation. Obtaining a CT scan of the abdomen would delay the time to the operating room, resulting in an increased risk for morbidity and mortality.

Intravenous antibiotic therapy (Choice C) could be considered if there was concern for a tuba-ovarian abscess, which commonly presents with fever, mucopurulent cervical discharge, and cervical or
adnexal tenderness. It may result in tachycardia and hypotension if severe but would not explain this patient's severe anemia.

Culdocentesis (Choice D) is a rarely performed procedure in which fluid is aspirated from the posterior cul-de-sac, often in the evaluation of lower abdominal pain. In the event of a ruptured ectopic
pregnancy, it would be expected to disclose bloody fluid. However, performing culdocentesis in a patient with a hemodynamically unstable ruptured ectopic pregnancy would be less appropriate than taking
the patient for emergent exploratory laparotomy to control the bleeding.

Educational Objective: An ectopic pregnancy is an abnormal pregnancy in which the fertilized ovum implants in the fallopian tube, which is most common, on the ovary, within the peritoneal cavity, or in any
nonendomelrial location. It often presents with vaginal bleeding and abdominopelvic pain. A rare complication of an unrecognized ectopic pregnancy is rupture, leading to hemoperitoneum, with severe
cases causing hemorrhagic shock. Treatment of a ruptured ectopic pregnancy is emergent exploratory laparotomy.

https://t.me/USMLENBME2CK ti
Previous Next Score Report Lab Values Calculator Help pause
Exam Section 3: Item 49 of 50 National Board of Medical Examiners
Comprehensive Clinical Science Self-Assessment

"I 49. A17-year-old boy is brought to the office by his parents because of changes in his behavior during the past 6 months. They say he has become withdrawn and moody and that his grades have
declined. He has returned home from parties intoxicated three times during the past month. During a wedding reception 3 weeks ago, he was delirious, incoherent, and imbalanced. The patient says
he drinks alcohol but that his drinking is not a problem and he can stop drinking whenever he wants. Vital signs are within normal limits. Physical examination shows no abnormalities. On mental status
examination, he is calm and cooperative. He reports no insomnia, hypersomnia, or suicidal ideation. Which of the following is the most likely diagnosis?

A) Alcohol use disorder


B) Conduct disorder
C) Major depressive disorder
D) Normal adolescent behavior
E) Recreational alcohol use
Correct Answer: A.

Alcohol use disorder refers to an addiction to alcohol that must meet at least two of the following criteria: an inability to cut back on drinking, drinking more than expected, excessive time spent drinking,
alcohol cravings, persistent drinking despite physical or mental problems related to drinking, and persistent drinking despite negative social consequences. Alcohol use disorder is a clinical diagnosis made
by the presence of personal, professional, social, emotional, behavioral, and physical disturbances related to the use of alcohol or attempts to reduce intake. Symptoms may be classified as mild, moderate,
or severe depending on the number of domains affected. Alcohol use disorder can occur in adolescents and is associated with negative social and academic consequences as well as increased risk for high-
risk sexual behavior, accidental death, and suicide.

Incorrect Answers: B, C, D, and E.

Conduct disorder (Choice B) refers to persistent, repetitive, and pervasive patterns of violating major societal norms, rules, and basic rights of others. While the patient's excessive drinking can be viewed as
violating a societal norm, this patient does not display evidence of violating the rights of others, such as violence, dishonesty, destruction of property, or theft.

Major depressive disorder (Choice C) includes 2 or more weeks of five of the following symptoms: depressed mood, anhedonia (eg, decreased interest in socializing), guilt or worthlessness, difficulty
concentrating, psychomotor retardation, suicidal thoughts, and/or neurovegetative symptoms (eg, decreased energy, sleep disturbance, appetite disturbance).

Normal adolescent behavior (Choice D) involves a heightened degree of risk taking. Experimentation with alcohol is common among adolescents. However, this patient displays evidence of a more
pervasive pattern of alcohol use with associated negative social and academic consequences, indicating alcohol use disorder.

Recreational alcohol use (Choice E) is distinguished from alcohol use disorder by moderate use and absence of dependence and negative consequences associated with drinking. Recreational alcohol use
as an adolescent is not legal in the United States.

Educational Objective: Alcohol use disorder is characterized by an inability to cut back on drinking, excessive lime spent drinking, and negative consequences related to drinking. Alcohol use disorder can
occur in adolescents and is associated with negative social and academic consequences as well as increased risk for high-risk sexual behavior, accidental death, and suicide.

https://t.me/USMLENBME2CK ts e t
Previous Next Score Report Lab Values Calculator Help pause
Exam Section 3: Item 50 of 50 National Board of Medical Examiners
Comprehensive Clinical Science Self-Assessment

"I 50. A47-year-old woman comes to the physician because of a 9-month history of hot flashes, night sweats, and pain during sexual intercourse. Her last menstrual period was 1 year ago. Physical
examination shows no abnormalities. Pelvic examination shows vaginal atrophy and no cervical mucus. A Pap smear and mammography show no abnormalities. Which of the following is the most
appropriate next step in management?

A) Pregnancy test
B) Measurement of serum luteinizing hormone concentration
C) Hormone therapy
D) Levothyroxine therapy
E) Vaginal corticosteroid therapy
Correct Answer: C.

Menopause refers to the absence of menstruation for 1 year caused by a diminution in ovarian reserve. On average, ii begins around age 50 years. Symptoms include hot flashes, night sweats, and vaginal
atrophy or dryness. Mood and sleeping patterns may be affected. While ii is commonly a clinical diagnosis, laboratory studies will show increased follicle-stimulating hormone and decreased estradiol
concentrations. As symptoms of menopause are secondary to estrogen deficiency, which can also increase the risk for a variety of other health problems (eg, osteoporosis), hormone therapy with
supplemental estrogen is often considered. While estrogen therapy does relieve symptoms, ii is not without risk. Prolonged exposure to unopposed estrogen can increase the risk for endometrial cancer,
cardiovascular disease, thromboembolism, and breast cancer. The recommended duration of use is limited to 3 to 5 years for these reasons.

Incorrect Answers: A, B, D, and E.

Pregnancy test (Choice A) is not necessary. This patient has not had a menstrual period in more than 1 year, making pregnancy unlikely, and is experiencing symptoms indicative of menopause.

Measurement of serum luteinizing hormone concentration (Choice B) can be used if the diagnosis of menopause is unclear, as it and follicle-stimulating hormone will be increased in the absence of negative
feedback from estradiol. However, menopause is primarily a clinical diagnosis.

Levothyroxine therapy (Choice 0) is used in the treatment of hypothyroidism, which commonly presents with weight gain, lethargy, fatigue, constipation, hair loss, and cold intolerance. It may also result in
amenorrhea or oligomenorrhea. This patient's symptoms are limited to amenorrhea, hot flashes, and vaginal atrophy, making menopause a more likely diagnosis.

Vaginal corticosteroid therapy (Choice E) may be used in the treatment of lichen planus or lichen sclerosis. While this patient has symptoms of vaginal atrophy, this is most likely caused by estrogen
deficiency. Her symptoms would therefore be better treated with topical or systemic estrogen supplementation.

Educational Objective: Menopause commonly occurs at approximately age 50 years with hot flashes, night sweats, and vaginal atrophy or dryness. As symptoms of menopause are secondary to estrogen
deficiency, hormone therapy with supplemental estrogen is often considered. Estrogen therapy can increase the risk for endometrial cancer, cardiovascular disease, thromboembolism, and breast cancer.
Thus, its use should be limited to patients at low risk for these diseases. Additionally, its use is only recommended for 3 to 5 years.

https://t.me/USMLENBME2CK ts e t
Previous Next Score Report Lab Values Calculator Help pause
Exam Section 4: ltem 1 of 50 National Board of Medical Examiners
Comprehensive Clinical Science Self-Assessment

✓ 1. A 67-year-old woman comes to the physician because of a 2-month history of perirectal pain that worsens with bowel movements and a thick, yellow rectal discharge. She has hypertension and
psoriasis. Her medications are metoprolol and topical fluocinolone cream. There is no family history of serious illness. She does not smoke cigarettes, drink alcohol, or use illicit drugs. Her temperature
is 37°C (98.6°F), pulse is 80/min, respirations are 14/min, and blood pressure is 136/70 mm Hg. The abdomen is soft, nontender, and nondistended. Rectal examination shows brown stool. The skin of
the posterior perinea! wall is erythematous with a draining sinus tract. Test of the stool for occult blood is positive. Which of the following is the most likely diagnosis?

A) C/ostridium difficile colitis


B) Colon cancer
C) Crohn disease
D) Diverticular abscess
E) Ulcerative colitis
Correct Answer: C.

Crohn disease is characterized by transmural inflammation of the gastrointestinal tract, most frequently involving the terminal ileum. It can arise anywhere in the gastrointestinal tract and is a subclass of
inflammatory bowel disease. Most patients experience chronic abdominal cramping, bloody diarrhea, and weight loss. Examination often discloses generalized abdominal tenderness, sometimes worse in
the right lower quadrant, and, depending on the severity and extent, may disclose complications such as enterocutaneous fistulas and extraintestinal manifestations such as uveitis. Severe Crohn disease
can result in intestinal obstruction from stricture or inflammation, bowel perforation, intra-abdominal abscesses, and the formation of fistulas. Diagnosis occurs through imaging and biopsy. Laboratory
studies often show increased erythrocyte sedimentation rate and c-reactive protein. On fluoroscopic small bowel follow-through, narrowing of the intestinal lumen at the terminal ileum is commonly seen
along with other sites of strictures or fistulous formation. Treatment includes anti-inflammatory medications, immunomodulators, and glucocorticoids. Complicated, severe cases may require surgery,
including small bowel resection or colectomy.

Incorrect Answers: A, B, D, and E.

Clostridium difficile colitis (Choice A) can present as pseudomembranous colitis. On colonoscopy, yellow pseudomembranes are seen amid inflamed, ulcerated colonic mucosa. However, formation of
draining sinus tracts or fistulas does not occur.

Colon cancer (Choice 8) may present with painless gastrointestinal bleeding but does not commonly form enterocutaneous fistulas.

Diverticular abscess (Choice D) presents with fever, left lower quadrant abdominal pain, diarrhea, tenderness to palpation, and leukocytosis. Abscesses can exist along with other complications, such as
perforation, peritonitis, and sepsis. Draining cutaneous tracts are not common sequelae.

Ulcerative colitis (Choice E) is a type of inflammatory bowel disease that results in mucosal and submucosal ulceration and inflammation that progresses retrograde along the colon from the rectum. This
typically presents with chronic bloody diarrhea, tenesmus, and abdominal pain. In contrast to Crohn disease, transmural inflammation and associated sequelae, including bowel perforation, intra-abdominal
abscesses, and fistulas, are less common.

Educational Objective: Crohn disease is characterized by transmural inflammation of the gastrointestinal tract and presents with chronic abdominal cramping, bloody diarrhea, and weight loss. Severe
Crohn disease can result in intestinal obstruction from stricture or inflammation, bowel perforation, intra-abdominal abscesses, and the formation of fistulas.

https://t.me/USMLENBME2CK ts e t
Previous Next Score Report Lab Values Calculator Help pause
Exam Section 4: Item 2 of 50 National Board of Medical Examiners
Comprehensive Clinical Science Self-Assessment

Twelve hours after discharge from the hospital, a2-day-old male newborn is brought to the emergency department after having a seizure. His parents report that he stiffened, and his eyes rolled back
for 3 or 4 minutes; then he went limp. He was delivered vaginally at term to a 20-year-old woman, gravida 1, para 1, following an uncomplicated pregnancy; his weight on delivery was 3062 g
(6 lb 12 oz). The patient's mother used to drink two glasses of wine each evening with dinner but stopped drinking alcohol when she learned she was pregnant at 4 weeks' gestation. She has never
smoked cigarettes or used illicit drugs. On arrival, the infant is awake but sluggish. Examination shows hypertelorism and low-set ears. The lungs are clear to auscultation. There are no murmurs. A
chest x-ray shows a narrow mediastinum and a right-sided aortic arch. Which of the following is the most likely diagnosis?

A) Down syndrome
B) Severe combined immunodeficiency
C) Thymic-parathyroid dysplasia
0) Trisomy 13 syndrome
E) Wiskott-Aldrich syndrome
Correct Answer: C.

Thymic-parathyroid dysplasia (Di George syndrome) is a disorder caused by a microdeletion in the 22q11.2 chromosome, which results in abnormal development of the third and fourth pharyngeal pouches.
It presents early in childhood with thymic hypoplasia and impaired immunity, conotruncal cardiac defects (eg, truncus arteriosus), hypocalcemia caused by parathyroid hypoplasia, and craniofacial
abnormalities, such as low-set ears, hypertelorism, micrognathia, and cleft lip and palate. It is associated with developmental delay. Severe hypocalcemia can lead to seizures, as in this patient, as well as
laryngospasm, bronchospasm, and emesis. Laboratory evaluation will show leukopenia and low calcium concentrations, while a chest x-ray may show the absence of a thymic shadow, as well as a narrow
mediastinum and right-sided aortic arch, as in this patient. Further evaluation typically includes an echocardiography and renal ultrasonography. Treatment is supportive and calcium repletion is critical,
especially in cases of seizures.

Incorrect Answers: A, B, D and E.

Down syndrome (Choice A) results from trisomy of chromosome 21 and presents with characteristic physical features including broad, flat, facial features with prominent epicanthal folds, and a single
palmar crease. Hypocalcemia and seizure are not common features of Down syndrome.

Severe combined immunodeficiency (Choice B) results from a variety of different mutations and commonly presents with recurrent bacterial, viral, and fungal infections caused by severe impairment in both
B- and T-lymphocyte function in infancy. It is not associated with parathyroid dysplasia or hypocalcemia.

Trisomy 13 syndrome (Choice 0) presents with characteristic features including intellectual disability, cleft lip and palate, holoprosencephaly, microphthalmia, cutis aplasia, feet with a prominent calcaneus
and convexly rounded soles, and polydactyly.

Wiskott-Aldrich syndrome (Choice E) is caused by a mutation in the WAS gene on the X chromosome that encodes a protein essential for actin cytoskeleton rearrangement that occurs during interactions
between T lymphocytes, antigen-presenting cells, and 8 lymphocytes, leading to an impaired innate and adaptive immune system. Patients present with eczema, thrombocytopenia, and infections with
encapsulated bacteria and opportunistic pathogens. Hypocalcemia and seizure are not common features of this syndrome.

Educational Objective: Thymic-parathyroid dysplasia (DiGeorge syndrome), can lead to acute hypocalcemia and can present with seizures. Characteristic findings include signs of improper development of
the third and fourth pharyngeal pouches, including low-set ears, hypertelorism, micrognathia, cleft lip and palate, thymic hypoplasia with impaired immunity, and parathyroid hypoplasia with associated
hvpocalcemia.

https://t.me/USMLENBME2CK ts e t
Previous Next Score Report Lab Values Calculator Help pause
Exam Section 4: ltem 3 of 50 National Board of Medical Examiners
Comprehensive Clinical Science Self-Assessment

✓ 3. A 2-year-old girl is brought to the physician for a well-child examination. She has had two urinary tract infections since birth. She has not had pain with urination or urinary frequency or urgency.
Examination shows no abnormalities. Renal ultrasonography shows no obstruction. Voiding cystourethrography shows grade II vesicoureteral reflux. Which of the following is the most appropriate initial
step to prevent future urinary tract infections in this patient?

A) Monitor temperature curves daily


B) Urine cultures weekly
C) Oral antibiotic prophylaxis
D) 2-Week course of intravenous antibiotic therapy
E) Vesicoureteral reimplantation
Correct Answer: C.

Recurrent urinary tract infections (UTls) in children increases the risk for renal scarring, hydronephrosis, and chronic kidney disease. Vesicoureteral reflux (VUR) is characterized by incomplete closure of the
vesicoureteral Junction during voiding with retrograde flow of urine from the bladder to the ureters. VUR allows for retrograde transport of bacteria to the upper urinary tract and is a significant risk factor for
pyelonephritis. The diagnosis is established by voiding cystourethrography (VCUG), which involves instillation of contrast dye into the bladder via catheterization and allows for visualization of reflux during
voiding. Oral antibiotic prophylaxis is indicated for children with VUR who present with more than one UTI. Surgical repair of VUR may be warranted for children with recurrent UT ls despite oral antibiotic
prophylaxis.

Incorrect Answers: A, B, D, and E.

Monitoring temperature curves daily (Choice A) and urine cultures weekly (Choice B) may allow for the early detection of recurrent UT ls but are not helpful for preventing recurrent infection.

2-Week course of intravenous antibiotic therapy (Choice D) is not able to provide sufficient long-term antimicrobial suppression. Long-term oral antibiotic prophylaxis is more effective in preventing recurrent
uTI

Vesicoureteral reimplantation (Choice E) may provide definitive repair of VUR but is typically performed after failure of oral antibiotic prophylaxis.

Educational Objective: Oral antibiotic prophylaxis is indicated for children with vesicoureteral reflux (VUR) who present with more than one urinary tract infection (UTI). Surgical repair of VUR may be
warranted for children with recurrent UTls despite oral antibiotic prophylaxis.

https://t.me/USMLENBME2CK ts e t
Previous Next Score Report Lab Values Calculator Help pause
Exam Section 4: Item 4 of 50 National Board of Medical Examiners
Comprehensive Clinical Science Self-Assessment

"I 4. A4-year-old boy is brought to the physician by his mother because she is concerned that he is having difficulty making friends. The mother reports that he attends preschool but does not socialize with
the other children and spends most of the time playing by himself. Al school, he is only interested in blocks, reports how tall he can build towers, and resists other activities. At home, he spends hours
building towers out of blocks in his room and becomes very upset if he is interrupted. He does not talk about any of his classmates; when his mother invites another child to the house, he has little
interest in socializing. His mother reports that he understands directions and communicates without difficulty but has never been very affectionate. He is generally well behaved but will have a temper
tantrum if a daily routine is altered or if furniture is rearranged. Pregnancy, labor, and delivery were uncomplicated, and he has reached all appropriate motor milestones for his age. His mother does not
recall if language emerged somewhat later in him than ii did for her daughter but notes that he makes his needs and wants known without any problem. He is neatly dressed. Physical examination
shows no abnormalities. On mental status examination, he makes no eye contact with the physician and perseverates about the towers that he built in the waiting room. Which of the following is the
most likely diagnosis?

A) Attention-deficit/hyperactivity disorder
B) Expressive language disorder
C) Obsessive-compulsive disorder
D) Pervasive developmental disorder
E) Social anxiety disorder (social phobia)
Correct Answer: D.

Autism spectrum disorder, also known as pervasive developmental disorder, is a neurodevelopmental disorder characterized by a broad range of signs and symptoms involving restricted and repetitive
patterns of behavior or activities, difficulties with social interaction, and difficulties with communication. Children with autism often display a lack of interest in socializing, failure in empathy, and challenging
behavior. Children may also display a restricted range of interests, excessive concern with routine, and resistance to change. The clinical manifestations of autism spectrum disorder typically present by age
2 years with delay or regression of communication and/or social skills. Impaired social cognition includes impaired nonverbal communication, such as eye contact and facial expressions. Management of
autism spectrum disorder is individualized depending on the clinical manifestations and degree of impairment of the patient. Therapy involves behavioral, developmental, social, and educational interventions
to improve social functioning, communication skills, and academic functioning.

Incorrect Answers: A, B, C, and E.

Attention-deficit/hyperactivity disorder (Choice A) is defined as a pattern of inattention and/or hyperactivity that persists in at least two settings. Patients may show symptoms of inattention (such as difficulty
following instructions or completing tasks) or a poor attention span in school. Interrupting others, fidgeting, and not waiting one's tum may be symptoms of hyperactivity.

Expressive language disorder (Choice B) is characterized by impairment in spoken and/or written expressive language capabilities without an impairment in language comprehension. lack of interest in
others and excessive concerns with a routine are not features of expressive language disorder.

Obsessive-compulsive disorder (Choice C) manifests with recurrent, intrusive urges or thoughts (obsessions) that the patient tries to relieve or neutralize by repetitive rituals (compulsions). Common
symptom complexes revolve around cleanliness, symmetry, harm to self or others, taboo thoughts or ideas, and completeness. Symptoms are lime-consuming, cause significant distress, and markedly
impair functioning.

Social anxiety disorder (social phobia) (Choice E) refers to an excessive fear of scrutiny, embarrassment, or rejection in social and/or performance situations leading to significant distress and/or impaired
L--••--•

https://t.me/USMLENBME2CK ts e t
Previous Next Score Report Lab Values Calculator Help pause
Exam Section 4: Item 4 of 50 National Board of Medical Examiners
Comprehensive Clinical Science Self-Assessment

recall if language emerged somewhat later in him than ii did for her daughter but notes that he makes his needs and wants known without any problem. He is neatly dressed. Physical examination
shows no abnormalities. On mental status examination, he makes no eye contact with the physician and perseverates about the towers that he built in the waiting room. Which of the following is the
most likely diagnosis?

A) Attention-deficit/hyperactivity disorder
B) Expressive language disorder
C) Obsessive-compulsive disorder
D) Pervasive developmental disorder
E) Social anxiety disorder (social phobia)
Correct Answer: D.

Autism spectrum disorder, also known as pervasive developmental disorder, is a neurodevelopmental disorder characterized by a broad range of signs and symptoms involving restricted and repetitive
patterns of behavior or activities, difficulties with social interaction, and difficulties with communication. Children with autism often display a lack of interest in socializing, failure in empathy, and challenging
behavior. Children may also display a restricted range of interests, excessive concern with routine, and resistance to change. The clinical manifestations of autism spectrum disorder typically present by age
2 years with delay or regression of communication and/or social skills. Impaired social cognition includes impaired nonverbal communication, such as eye contact and facial expressions. Management of
autism spectrum disorder is individualized depending on the clinical manifestations and degree of impairment of the patient. Therapy involves behavioral, developmental, social, and educational interventions
to improve social functioning, communication skills, and academic functioning.

Incorrect Answers: A, B, C, and E.

Attention-deficit/hyperactivity disorder (Choice A) is defined as a pattern of inattention and/or hyperactivity that persists in at least two settings. Patients may show symptoms of inattention (such as difficulty
following instructions or completing tasks) or a poor attention span in school. Interrupting others, fidgeting, and not waiting one's turn may be symptoms of hyperactivity.

Expressive language disorder (Choice B) is characterized by impairment in spoken and/or written expressive language capabilities without an impairment in language comprehension. lack of interest in
others and excessive concerns with a routine are not features of expressive language disorder.

Obsessive-compulsive disorder (Choice C) manifests with recurrent, intrusive urges or thoughts (obsessions) that the patient tries to relieve or neutralize by repetitive rituals (compulsions). Common
symptom complexes revolve around cleanliness, symmetry, harm to self or others, taboo thoughts or ideas, and completeness. Symptoms are lime-consuming, cause significant distress, and markedly
impair functioning.

Social anxiety disorder (social phobia) (Choice E) refers to an excessive fear of scrutiny, embarrassment, or rejection in social and/or performance situations leading to significant distress and/or impaired
functioning.

Educational Objective: Autism spectrum disorder, also known as pervasive developmental disorder, presents with delayed or regressing communication and/or social skills, lack of interest in socializing, a
restricted range of interests, and excessive concern with routine.

https://t.me/USMLENBME2CK ts e t
Previous Next Score Report Lab Values Calculator Help pause
Exam Section 4: Item 5 of 50 National Board of Medical Examiners
Comprehensive Clinical Science Self-Assessment

"I 5. Two days after undergoing urgent cesarean delivery because of a nonreassuring fetal heart tracing, a 37-year-old woman, gravida 1, para 1, has the onset of dysphoria, insomnia, and easy
distractibility. Pregnancy was uncomplicated. She has no history of serious illness, and her only medication is a prenatal vitamin. Her paternal cousin had major depressive disorder, postpartum onset.
The patient's vital signs are within normal limits. Pulse oximetry on room air shows an oxygen saturation of 97%. Physical examination shows no abnormalities. On mental status examination, she has
an irritable mood and tired, concerned affect. Without treatment, which of the following is most likely to develop in this patient?

A) Alopecia
B) Dissociative amnesia with dissociative fugue
C) Loss of maternal bond with newborn
D) Resolution of symptoms within 2 weeks
E) Suicidality
Correct Answer: D.

Postpartum blues are common after delivery and often represent hormonal changes as the patient returns to her prepregnancy state. It typically presents in the first few days after delivery with depressive
symptoms such as dysphoria, anhedonia, irritability, insomnia, and decreased concentration that do not interfere with the patient's ability to care for the newborn or herself. These symptoms often self-resolve
within 2 weeks of onset, which is what would be expected in this patient. Risk factors for development include a history depression, cesarean delivery, and lack of breast-feeding. Postpartum blues differ from
major depressive disorder with peripartum onset in severity of symptoms, duration of symptoms, and effect on day-to-day functioning. Major depressive disorder with peripartum onset is associated with
suicidality and should be treated with psychotherapy and antidepressant medications, whereas postpartum blues generally self-resolve within a few weeks and do not significantly impair functioning.

Incorrect Answers: A, B, C, and E.

Alopecia (Choice A) presents with hair loss that can be localized or diffuse, depending on the underlying cause. Postpartum alopecia may occur after delivery in some patients, but more commonly presents
2 to 4 months after delivery. Additionally, it is not associated with the depressive symptoms.

Dissociative amnesia with dissociative fugue (Choice B) refers to a complete loss of memory associated with wandering behavior. This pattern of behavior may be seen in dissociative identity disorder or may
be caused by a recent traumatic exposure. Postpartum blues do not generally lead to dissociative amnesia or dissociative fugues.

Loss of maternal bond with newborn (Choice C) is unlikely with postpartum blues. A hallmark of postpartum blues, in contrast to major depressive disorder with peripartum onset, is mild depressive
symptoms that do not affect the patient's ability to care for the newborn or herself. Postpartum blues also last for less than 2 weeks, making it unlikely that her symptoms will affect her bond with the
newborn.

Suicidality (Choice E) is not present with postpartum blues. If suicidality does occur, further evaluation for major depressive disorder with peripartum onset or other psychiatric illnesses should occur. Patients
with suicidality should undergo emergent evaluation and treatment.

Educational Objective: Postpartum blues are common after delivery, typically presenting in the first few days with depressive symptoms such as dysphoria, anhedonia, irritability, insomnia, and decreased
concentration. These symptoms do not interfere with the patient's ability to care for the newborn or herself. These symptoms often self-resolve within 2 weeks of onset, in contrast to major depressive
disorder with peripartum onset, in which symptoms are more severe and long-lasting.

https://t.me/USMLENBME2CK ts e t
Previous Next Score Report Lab Values Calculator Help pause
Exam Section 4: Item 6 of 50 National Board of Medical Examiners
Comprehensive Clinical Science Self-Assessment

"I 6. A 32-year-old woman with asthma comes lo the physician because of a 3-month history of progressive wheezing and a cough that is occasionally productive of plugs of brownish sputum. She received
the diagnosis of asthma 15 years ago. She has been taking an inhaled long-acting p-adrenergic agonist and inhaled glucocorticoid daily for 8 years and had good control of her symptoms until 3 months
ago. She has no pets. She has no recent history of travel abroad or new occupational exposure. She appears well and is not in respiratory distress. Vital signs are within normal limits. Pulse oximelry on
room air shows an oxygen saturation of 96%. Examination shows no cyanosis. Scattered inspiratory and expiratory wheezes are heard. There is no clubbing or peripheral edema. Her leukocyte count is
20,000/mm3 (45% segmented neutrophils, 40% eosinophils, 12% lymphocytes, and 3% monocytes). Her serum lgE concentration is 2800 IU/ml. A chest x-ray shows alveolar infiltrates in the upper
lungs bilaterally. Which of the following is the most likely diagnosis?

A) Allergic bronchopulmonary aspergillosis


B) Eosinophilic leukemia
C) Hypereosinophilic syndrome
D) Polyarteritis nodosa
E) Strongy/oides stercora/is infection
Correct Answer: A.

Patients with asthma or cystic fibrosis are at increased risk for developing allergic bronchopulmonary aspergillosis, which is a hypersensitivity reaction caused by the presence of Aspergil/us fumigatus
organisms in the airways. The clinical syndrome is associated with transitory pulmonary infiltrates, peripheral eosinophilia, and bronchiectasis. Patients typically present with constitutional symptoms,
including generalized fatigue and mild fever, progressive cough, and asthma-like symptoms that are refractory to treatment. Brown or black sputum and hemoptysis may be noted. Laboratory studies are
notable for eosinophilia and increased serum lgE concentrations. CT scan of the chest is the gold standard for diagnosis and is characterized by central bronchiectasis and signs of mucoid impaction.
Treatment consists of systemic corticosteroids, antifungal agents, and anti-lgE immunologies depending on the severity and chronicity of the syndrome.

Incorrect Answers: B, C, D, and E.

Hypereosinophilic syndrome (HES) (Choice C) is a heterogenous group of disorders characterized by peripheral eosinophilia (eosinophil count greater than 500/mm3). Primary HES is caused by neoplastic
clonal proliferation as in eosinophilic leukemia (Choice B), while secondary HES is reactive to another process. Parasitic and fungal infections are common causes of secondary HES, as is likely in this case.

Polyarteritis nodosa (Choice 0) is a necrotizing vasculitis involving medium-sized arteries that generally affects the kidneys and visceral organs, sparing the lungs. It typically presents with fever, headache,
malaise, weight loss, and abdominal pain.

Strongy/oides stercoralis infection (Choice E) is caused by an intestinal roundworm and is a common cause of peripheral eosinophilia in patients from endemic regions. It has characteristic intestinal and
pulmonary phases; the pulmonary phase is associated with transient infiltrates on chest imaging. A patient with a history of asthma living in a nonendemic region is more likely to have allergic
bronchopulmonary aspergillosis.

Educational Objective: Allergic bronchopulmonary aspergillosis is a hypersensitivity reaction caused by Aspergil/us fumigatus, and patients with asthma or cystic fibrosis are al increased risk. Patients
present with chronic cough that may be productive of dark-colored sputum and hemoptysis. Characteristic findings include transitory pulmonary infiltrates, peripheral eosinophilia, and bronchiectasis.

https://t.me/USMLENBME2CK ti
Previous Next Score Report Lab Values Calculator Help pause
Exam Section 4: Item 7 of 50 National Board of Medical Examiners
Comprehensive Clinical Science Self-Assessment

"I 7. A 24-year-old woman comes to the office for an annual employment physical examination. She feels well. Medical history is unremarkable and she takes no medications. Vital signs are within normal
limits. Physical examination discloses no abnormalities. As the physician is preparing to leave the examination room the patient asks, "Do you think having a romantic relationship with my gynecologist
is all right?" Which of the following is the most appropriate response?

A) "I am under no obligation to report any romantic relationship to a disciplinary board."


B) "If you switch gynecologists it would be acceptable for your former gynecologist to begin a romantic relationship with you immediately."
C) "A mutually consensual romantic relationship between patient and physician can sometimes be justified."
0) "A romantic relationship in this setting is always unethical."
E) "Would you mind telling me who your gynecologist is?"
Correct Answer: D.

The relationship between physician and patient is asymmetrical in terms of the authority and trust invested in the physician, and a romantic relationship within this context exploits the vulnerability of the
patient. Such relationships are particularly inappropriate when the nature of the physician-patient interaction is intimate, such as in obstetrics and gynecology or psychiatry. The American College of
Obstetricians and Gynecologists (ACOG) guidelines state that sexual or romantic relationships between a gynecologist and a current patient are always unethical and may be grounds for investigation or
criminal prosecution. The American Medical Association's Code of Ethics suggests that physicians completely terminate the physician-patient relationship before pursuing a romantic relationship. However,
romantic relationships subsequent to the physician-patient relationship may be influenced by the prior relationship, creating further ethical dilemmas. This patient who is inquiring about dating her
gynecologist raises a concern for possible sexual misconduct. The patient should be informed that such a relationship is inappropriate. A discussion inquiring about what the patient has experienced with the
gynecologist should follow. Any sexual misconduct or suspected misconduct should be reported to appropriate authorities, such as medical licensing boards, hospital disciplinary boards, or other appropriate
institutional officials.

Incorrect Answers: A, B, C, and E.

"I am under no obligation to report any romantic relationship to a disciplinary board" (Choice A), is incorrect. ACOG guidelines state that medical professionals are obligated to report sexual misconduct or
suspected misconduct to appropriate authorities, such as medical licensing boards, hospital disciplinary boards, or other appropriate institutional officials.

"If you switch gynecologists it would be acceptable for your former gynecologist to begin a romantic relationship with you immediately" (Choice B), is incorrect. Romantic relationships subsequent to the
physician-patient relationship may be influenced by the prior relationship, creating further ethical dilemmas.

"A mutually consensual romantic relationship between patient and physician can sometimes be justified" (Choice C), is incorrect. Given the asymmetric nature of the physician-patient relationship, patients
are not capable of providing consent within this context.

"Would you mind telling me who your gynecologist is" (Choice E) is an appropriate question but may cause alarm and impair further discussion. The patient should first be informed that such a relationship is
inappropriate. A more thorough discussion about what the patient has experienced should be pursued before attempting to obtain information that will allow reporting to appropriate authorities.

Educational Objective: The American College of Obstetricians and Gynecologists guidelines state that sexual or romantic relationships between a gynecologist and a current patient are always unethical and
may be grounds for investigation or criminal prosecution. Pursuing a romantic relationship concurrently with the physician-patient relationship may be viewed as sexual misconduct by medical regulatory
authorities.

https://t.me/USMLENBME2CK ts e t
Previous Next Score Report Lab Values Calculator Help pause
Exam Section 4: Item 8 of 50 National Board of Medical Examiners
Comprehensive Clinical Science Self-Assessment

"I 8. A healthy 52-year-old man comes to the physician for a routine total colonoscopy. He has no history of blood in his stool or rectal bleeding. There is no family history of colorectal polyps or cancer. He
takes no medications and has never smoked cigarettes. Physical examination shows no abnormalities. Colonoscopy shows a 3-mm pedunculated polyp in the cecum. No other abnormalities are noted.
The polyp is completely excised. Histopathologic evaluation shows dysplastic-appearing columnar epithelial cells confined by the basement membrane, lining a fibrovascular stalk. Which of the following
is the most appropriate recommendation for future screening for colon cancer in this patient?

A) Colonoscopy in 1 year
B) Colonoscopy in 3 years
C) Colonoscopy in 10 years
D) Flexible sigmoidoscopy in 1 year
E) No further screening is indicated unless annual test of the stool for occult blood is positive
Correct Answer: B.

The United States Preventive Services Task Force recommends routine colorectal cancer screening for all patients beginning at age 45 years and continuing until age 75 years. Colorectal cancer is common,
and early detection and treatment have been shown to reduce morbidity and mortality. Colonoscopy remains the gold standard screening test as it allows direct visualization and excision of polyps
throughout the length of the colon. Other direct visualization tests such as flexible sigmoidoscopy may be acceptable for some patients. Other testing strategies include stool-based tests, such as fecal occult
blood and fecal immunochemical (FIT) tests, and CT colonography. A variety of screening intervals are acceptable and recommended by national governing bodies, with many patients undergoing either
colonoscopy every 10 years, annual FIT screening, screening every 10 years with flexible sigmoidoscopy and FIT, or screening every 5 years with CT colonography. In the event that adenomatous polyps or
early stage colon cancer is identified, closer follow up may be necessary. Careful risk stratification should be performed based on colonoscopy results to determine an appropriate follow-up interval. This
patient's colonoscopy shows a polyp with dysplasia, which places him at increased risk for colon cancer. Colonoscopy should be repeated in 3 years for patients with 5 to 10 tubular adenomas or sessile
polyps less than 10 mm in diameter, one tubular adenoma or sessile polyp greater than 10 mm in diameter, multiple serrated sessile polyps, or adenomas or sessile polyps with dysplasia or with villous or
tubulovillous histology that are completely removed.

Incorrect Answers: A, C, D, and E.

Repeat colonoscopy in 1 year (Choice A) is indicated for patients with more than 10 adenomas.

Colonoscopy in 10 years (Choice C) is appropriate for patients with normal colonoscopy findings. Patients with abnormal colonoscopy results require closer follow up.

Flexible sigmoidoscopy in 1 year (Choice 0) is not recommended for patients with a previous abnormal colonoscopy, as this evaluation would insufficiently assess the entire colon.

No further screening unless annual test of the stool for occult blood is positive (Choice E) is incorrect. Appropriate risk stratification of colonoscopy results reduces the risk for colon cancer. Occult blood
testing may indicate colon cancer but is neither sensitive nor specific.

Educational Objective: The United States Preventive Services Task Force recommends routine colorectal cancer screening for all patients beginning at age 45 years and continuing until age 75 years. In the
event that adenomatous polyps or early stage colon cancer is identified, closer follow up may be necessary. For patients with adenomas or sessile polyps with dysplasia or with villous or tubulovillous
histology that are completely removed, colonoscopy should be repeated in 3 years.

https://t.me/USMLENBME2CK ti e
Previous Next Score Report Lab Values Calculator Help pause
Exam Section 4: Item 9 of 50 National Board of Medical Examiners
Comprehensive Clinical Science Self-Assessment

"I 9. A healthy 18-month-old boy is brought for a well-child examination. He was born at 37 weeks' gestation, needed no resuscitation, and had no neonatal problems. He was fed formula for the first 2 months
of life, and then his mother substituted cow's milk because of the expense of the formula. He has received no supplements or medications. Examination shows no abnormalities. This child is at greatest
risk for which of the following?

A) Iron deficiency anemia


B) Pellagra
C) Protein-calorie malnutrition
D) Vitamin B, (cobalamin) deficiency
E) Vitamin D deficiency
Correct Answer: A.

Neonates and infants fed with only whole cow's milk are at risk for developing iron deficiency anemia. Both breast milk and cow's milk contain low concentrations of iron. The iron in breast milk has a high
bioavailability while bovine milk proteins impair iron absorption in human infants. In addition to poor absorption, cow's milk is associated with increased occult enteric bleeding that contributes to iron loss during
infancy. Most infants eventually tolerate cow's milk later in life. Iron deficiency anemia early in life can lead to significant neurocognitive and psychomotor impairments that persist even after treatment.

Incorrect Answers: B, C, D, and E.

Pellagra (Choice B) is caused by an acquired deficiency of niacin (nicotinic acid). It is characterized by diarrhea, dementia, and dermatitis. Signs of protein-calorie malnutrition are not present. Pellagra-like
symptoms can also be seen in Hartnup disorder, an autosomal recessive disorder caused by decreased gastrointestinal absorption of tryptophan, the precursor to nicotinic acid.

Protein-calorie malnutrition (Choice C) encompasses a broad range of clinical presentations caused by severe deficiency in total body protein and/or caloric intake. Infants and children may present with failure
to thrive, muscle atrophy, and stunted growth.

Vitamin B12 (cobalamin) deficiency (Choice 0) results in megaloblastic anemia, which presents with macrocytosis and hypersegmented neutrophils. It is more common in patients who chronically abuse alcohol
or in patients with a Roux-en-Y gastric bypass, Crohn disease, or malabsorptive disorders. It is not associated with substituted cow's milk feedings.

Vitamin D deficiency (Choice E) in childhood, or rickets, commonly results from low exposure to ultraviolet radiation and low dietary vitamin D intake (particularly from low vitamin Din breast milk). Signs of
protein-calorie malnutrition are not present.

Educational Objective: Neonates and infants fed with only whole cow's milk are at risk for developing iron deficiency anemia. Breast milk or iron-fortified formula is recommended for the first year of life.

https://t.me/USMLENBME2CK ti e
Previous Next Score Report Lab Values Calculator Help pause
Exam Section 4: ltem 10 of 50 National Board of Medical Examiners
Comprehensive Clinical Science Self-Assessment

10. A5-year-old girl is brought to the physician as a new patient for an examination prior to starting kindergarten. She was noted to have high blood pressure by a previous physician 3 years ago. Her
blood pressure 2 years ago was within normal limits. She has not been seen by a physician since that time because her mother did not have adequate health insurance. The patient is at the 50th
percentile for height and 15th percentile for weight. Her temperature is 36.7°C (98°F), pulse is 80/min, respirations are 25/min, and blood pressure is 140/90 mm Hg. A bruit is heard over the right
aspect of the midabdomen. No other abnormalities are noted. Renal arteriography shows stenosis of the right renal artery. Which of the following is the most appropriate next step in management?

A) Weight loss program


B) Warfarin therapy
C) Angioplasty
D) Renal transplant
E) No treatment is indicated
Correct Answer: C.

Secondary hypertension (hypertension with an identifiable medical cause), in contrast to primary or essential hypertension, accounts for a greater proportion of hypertension in children than in adults. This
commonly occurs as a result of kidney or renovascular disease, including renal artery stenosis. Reduction in renal perfusion leads to activation of the renin-angiotensin cascade, with a subsequent increase
in renal volume retention and blood pressure. Renal artery stenosis in children may be an isolated finding, or it may be associated with fibromuscular dysplasia or neurofibromatosis type1. Treatment
options for secondary hypertension associated with renal artery stenosis in children include calcium channel blockers (CCBs) or angioplasty. CCBs are generally preferable to other agents, such as thiazide
diuretics or angiotensin receptor blockers, both of which can result in reduced glomerular filtration and a greater risk for hyperkalemia. Angioplasty is often reserved for patients who fail oral antihypertensive
treatment.

Incorrect Answers: A, B, D, and E.

Weight loss program (Choice A) is useful for the management of primary or essential hypertension in children and adolescents. Weight loss is not helpful for patients with hypertension secondary to renal
artery stenosis.

Warfarin therapy (Choice B) is not useful for the treatment of hypertension secondary to renal artery stenosis and carries an increased risk for bleeding.

Renal transplant (Choice D) may be necessary for children with hypertension secondary to end-stage kidney disease, which may occur in a variety of conditions, such as polycystic kidney disease or
recurrent pyelonephritis. Renal transplant is not necessary for patients with renovascular disease, as the underlying pathology is located within the renal artery.

No treatment is indicated (Choice E) is incorrect. Untreated hypertension in children is associated with increased morbidity and mortality during adult life caused by cardiovascular disease and
atherosclerosis.

Educational Objective: Secondary hypertension may occur in children, often caused by kidney or renovascular disease, including renal artery stenosis. Treatment options for secondary hypertension
associated with renal artery stenosis in children include calcium channel blockers or angioplasty.

https://t.me/USMLENBME2CK ti e
Previous Next Score Report Lab Values Calculator Help pause
Exam Section 4: Item 11 of 50 National Board of Medical Examiners
Comprehensive Clinical Science Self-Assessment

"I 11. A42-year-old woman, gravida 1, para 1, comes to the office for a routine examination. She is concerned about her risk for coronary artery disease. She has irritable bowel syndrome and migraines.
Medications are sumatriptan and ibuprofen as needed. Her father has hypertension and hyperlipidemia. Her paternal uncle was diagnosed with coronary artery disease at the age of 50 years. The
patient smoked one pack of cigarettes daily for 10 years but quit 7 years ago. She drinks two glasses of wine weekly. She does not use illicit drugs. She runs 3 miles one to three times weekly. She is
168 cm (5 ft 6 in) tall and weighs 77 kg (170 lb); BMI is 27 kg/m2. Blood pressure is 118/70 mm Hg. Examination shows no other abnormalities. Results of fasting serum studies are shown:
Glucose 82 mg/dl
Cholesterol, total 255 mg/dl
HDL-cholesterol 85 mg/dl
LDL-cholesterol 155 mg/dl
Triglycerides 70 mg/dl

The risk assessment score for her 10-year probability of coronary artery disease is 0.4%. The patient is counseled about maintaining a diet low in saturated fat and continuing her exercise regimen.
Which of the following is the most appropriate next step in management?

A) Cardiac stress scintigraphy


B) Daily aspirin therapy
C) Echocardiography
D) HMG-CoA reductase inhibitor therapy
E) No intervention is indicated at this time
Correct Answer: E.

Cardiovascular disease, including coronary artery disease, is a major cause of morbidity and mortality in the United States. Risk factors for its development include obesity, tobacco use, diabetes mellitus,
hypertension, hyperlipidemia, poor diet, and lack of exercise. Modifying these risk factors with lifestyle changes or medications can decrease the risk for development of coronary artery disease. However,
medications can be associated with adverse effects, and their initiation should be guided by the evaluation of risks and benefits in each individual patient. This patient has evidence of hyperlipidemia with
increased total cholesterol and LDL-cholesterol concentrations. In general, if a patient's LDL-cholesterol concentration is below 190 mg/dl, the decision to start a statin, or HMG-CoA reductase inhibitor, is
based on the 10-year cardiovascular disease risk. In a patient with a cardiovascular risk of less than 5%, the risks of a statin medication often outweigh the benefits. In contrast, if a patient's cardiovascular
risk is greater than 10%, starting a statin medication is indicated. Initiation of aspirin follows similar rationale, with the risks outweighing the benefit in those with a low cardiovascular risk. Therefore, while this
patient has hyperlipidemia, no further intervention is indicated at this time.

Incorrect Answers: A, B, C, and D.

Cardiac stress scintigraphy (Choice A), or a nuclear stress test, is designed lo evaluate for ischemic disease of the heart. Perfusion of the heart is evaluated before and after stress, either by exercise or by a
pharmacologic agent, thereby indicating areas of ischemia. It is often used in the evaluation of angina or in patients with a high pretest probability of coronary artery disease. This patient is asymptomatic and
has low risk for cardiovascular disease, making this test inappropriate.

Daily aspirin therapy (Choice B) is indicated as primary prevention in those patients who have a 10-year cardiovascular risk of greater than 20%. It can also be considered in those al intermediate risk after
le elieeueeer efeee4e+eel iele ieelseliewe le iweweeeel iel fee lleeelie"Lie vetiet ie et le iel fee0eeliewee4le eliepee vweliuweeleil eii tlerwwweweeeu

https://t.me/USMLENBME2CK ts e t
Previous Next Score Report Lab Values Calculator Help pause
Exam Section 4: Item 11 of 50 National Board of Medical Examiners
Comprehensive Clinical Science Self-Assessment

A) Cardiac stress scintigraphy


B) Daily aspirin therapy
C) Echocardiography
D) HMG-CoA reductase inhibitor therapy
E) No intervention is indicated at this time
Correct Answer: E.

Cardiovascular disease, including coronary artery disease, is a major cause of morbidity and mortality in the United States. Risk factors for its development include obesity, tobacco use, diabetes mellitus,
hypertension, hyperlipidemia, poor diet, and lack of exercise. Modifying these risk factors with lifestyle changes or medications can decrease the risk for development of coronary artery disease. However,
medications can be associated with adverse effects, and their initiation should be guided by the evaluation of risks and benefits in each individual patient. This patient has evidence of hyper1ipidemia with
increased total cholesterol and LDL-cholesterol concentrations. In general, if a patient's LDL-cholesterol concentration is below 190 mg/dl, the decision to start a statin, or HMG-CoA reductase inhibitor, is
based on the 10-year cardiovascular disease risk. In a patient with a cardiovascular risk of less than 5%, the risks of a statin medication often outweigh the benefits. In contrast, if a patient's cardiovascular
risk is greater than 1 0%, starting a statin medication is indicated. Initiation of aspirin follows similar rationale, with the risks outweighing the benefit in those with a low cardiovascular risk. Therefore, while this
patient has hyper1ipidemia, no further intervention is indicated at this lime.

Incorrect Answers: A, B, C, and D.

Cardiac stress scintigraphy (Choice A), or a nuclear stress test, is designed to evaluate for ischemic disease of the heart. Perfusion of the heart is evaluated before and after stress, either by exercise or by a
pharmacologic agent, thereby indicating areas of ischemia. It is often used in the evaluation of angina or in patients with a high pretest probability of coronary artery disease. This patient is asymptomatic and
has low risk for cardiovascular disease, making this test inappropriate.

Daily aspirin therapy (Choice B) is indicated as primary prevention in those patients who have a 10-year cardiovascular risk of greater than 20%. It can also be considered in those al intermediate risk after
the discussion of benefits and risks, including the increased risk for bleeding. This patient is at low risk for cardiovascular disease, making daily aspirin therapy unnecessary.

Echocardiography (Choice C) is ultrasonography of the heart used to identify structural abnormalities. This patient has no concerning cardiac symptoms or signs of cardiac dysfunction on physical
examination. Echocardiography is not warranted.

HMG-CoA reductase inhibitor therapy (Choice D), or statin therapy, is indicated in the treatment of hyperlipidemia if LDL-cholesterol exceeds 190 mg/dL or if the patient has an increased risk for
cardiovascular disease. This patient's risk is less than 1%. Thus, statin therapy is not indicated.

Educational Objective: The risk for cardiovascular disease can be decreased by lifestyle modifications and treatment of risk factors, such as obesity, tobacco use, diabetes mellitus, hypertension, and
hyperlipidemia. While medical management of hyperlipidemia can decrease the risk for coronary artery disease, ii can also cause adverse effects. Thus, the decision lo start a stalin should be based on
consideration of an individual patient's risks and benefits. Those with LDL-cholesterol concentrations greater than 190 mg/dL or those with a greater than 10% 10-year cardiovascular risk should be
prescribed a statin, while those with a less than 5% risk should undergo routine screening and follow-up evaluation.

https://t.me/USMLENBME2CK ts e t
Previous Next Score Report Lab Values Calculator Help pause
Exam Section 4: ltem 12 of 50 National Board of Medical Examiners
Comprehensive Clinical Science Self-Assessment

"I 12. An investigator would like to determine if a new medication for chronic obstructive pulmonary disease (COPD) decreases hospitalization rates in adults who come to the emergency department (ED)
because of an exacerbation. All patients with COPD who come to the ED because of an exacerbation will be invited to participate in the study. Patients will be randomly assigned to receive the new
medication plus standard therapy or standard therapy alone during the hours that a study nurse is present (7 AM to 7 PM daily). Patients who come to the ED outside of these hours will not be enrolled
in the study. Physicians treating the patients will be blinded to the group assignments. Outcomes will be assessed by a physician not involved in the care of the patients. Which of the following is the
greatest limitation of this study design?

A) Generalizability
B) Inadequate blinding of outcome assessors
C) Lack of a placebo group
D) Lead-time bias
Correct Answer: A.

Generalizability refers to the ability to apply the results of a study conducted with a sample population to the population at large and reflects the external validity of the study. Generalizability or external
validity can be affected by various factors, such as sampling bias (in which the sample population does not reflect the population at large), or the Hawthorne effect (in which the sample population alters its
behavior in response to testing or awareness of being under study). In contrast, internal validity describes the ability of a study to attribute its findings to the cause-effect relationship under study, while also
excluding alternative explanations for the observations. While rigorous experimental design and standardized procedures are important for establishing internal validity, choices that affect experimental
design may result in poor generalizability or external validity. In this scenario, choosing to sample only patients who present to the emergency department while a study nurse is present during the day
ensures that there is standardization of care but creates sampling bias that limits generalizability, since patients in the population at large also present at night.

Incorrect Answers: B, C, and D.

Inadequate blinding of outcome assessors (Choice B) is incorrect. Blinding is a technique for masking the status of the treatment (experimental, standard of care, or placebo) from either the experimenter,
physician administering the treatment, or the patient. Blinding is helpful for reducing bias that may result from awareness of the treatment being administered. The experimental design in this scenario is most
limited by sampling bias, rather than by an issue with blinding.

Lack of a placebo group (Choice C) is incorrect. The use of a placebo is inappropriate in situations where an effective treatment is known to exist. A placebo should not be used in situations where its use
could cause harm. In such scenarios, it is appropriate to compare the experimental treatment against the standard of care.

Lead-time bias (Choice D) is incorrect. Lead-time bias is a form of bias that describes earlier screening for a disease leading to earlier detection of the disease, with a resultant, incorrect observation of
increased survival time.

Educational Objective: Generalizability refers to the ability to apply the results of a study conducted with a sample population to the population at large and reflects the external validity of the study.
Generalizability or external validity can be affected by various factors. In this scenario, sampling only patients who present during the day creates sampling bias that limits the applicability of the study to the
population at large.

https://t.me/USMLENBME2CK ti
Previous Next Score Report Lab Values Calculator Help pause
Exam Section 4: ltem 13 of 50 National Board of Medical Examiners
Comprehensive Clinical Science Self-Assessment

"I 13. A57-year-old man is brought to the emergency department by his neighbors, who found him lying unconscious in a puddle of vomit in his apartment. He has a 20-year history of alcohol use disorder.
On arrival, he is lethargic and reports shortness of breath. He is unable to provide a medical history. His temperature is 39°C (102.2F), pulse is 98/min, respirations are 28/min, and blood pressure is
132/76 mm Hg. Diffuse crackles are heard bilaterally. Arterial blood gas analysis on room air shows:
pH 7.44
Pco, 30 mm Hg
Po, 48 mm Hg

An x-ray of the chest shows pulmonary infiltrates bilaterally. Which of the following is the most appropriate next step in management?

A) Hyperbaric oxygen therapy


B) Intravenous administration of dobutamine
C) Intravenous administration of methylprednisolone
D) Bronchoscopy with lavage
E) Intubation and mechanical ventilation
Correct Answer: E.

This patient is likely suffering from acute respiratory distress syndrome (ARDS). ARDS is characterized by acute onset; diffuse, inflammatory lung injury; and bilateral pulmonary infiltrates on chest imaging
leading to hypoxic respiratory failure. There are numerous causes, including sepsis (especially in the setting of gram-negative bacteria caused by macrophage overactivation by lipopolysaccharide),
pneumonia, chemical pneumonitis (eg, aspiration, likely in this case with the possibility of bacterial aspiration pneumonia), pulmonary contusion, acute pancreatitis, trauma, transfusions, medications, and
amniotic fluid embolism. Alveolar damage and inflammation lead to increased permeability of the alveolar-capillary interface. Urgent endotracheal intubation should occur in patients who are apneic, hypoxic,
unable to protect their airway (eg, lethargic, no gag or cough reflex), or hypercapnic and not responding to less invasive means. Intubation and mechanical ventilation is the most appropriate next step in
management of this condition to secure the airway and ensure adequate oxygenation and ventilation.

Incorrect Answers: A, B, C, and D.

Hyperbaric oxygen therapy (Choice A) is a treatment indicated for carbon monoxide (CO) poisoning. Symptoms of CO poisoning include headache, lightheadedness, nausea, and altered mental status.
Acute management involves removing the patient from the exposure and treatment with 100% supplemental oxygen to displace CO from hemoglobin. Severe cases and high-risk patients demonstrating any
degree of neurologic deficit or significant end-organ dysfunction should be treated with hyperbaric oxygen therapy.

Intravenous administration of dobutamine (Choice B) would be indicated in the treatment of cardiogenic shock. This patient has a fever and no history of heart failure, making cardiogenic shock less likely
and septic shock more likely. At present, his heart rate and blood pressure are within normal limits, indicating that no presser or inotropic support is needed at this time.

Intravenous administration of methylprednisolone (Choice C) is used in the treatment of allergic, autoimmune, and inflammatory conditions such as asthma, Crohn disease, and gout. It is not clear whether
steroids are helpful in ARDS; they would not be the next step in management.

https://t.me/USMLENBME2CK ti
Previous Next Score Report Lab Values Calculator Help pause
Exam Section 4: ltem 13 of 50 National Board of Medical Examiners
Comprehensive Clinical Science Self-Assessment

An x-ray of the chest shows pulmonary infiltrates bilaterally. Which of the following is the most appropriate next step in management?

A) Hyperbaric oxygen therapy


B) Intravenous administration of dobutamine
C) Intravenous administration of methylprednisolone
D) Bronchoscopy with lavage
E) Intubation and mechanical ventilation
Correct Answer: E.

This patient is likely suffering from acute respiratory distress syndrome (ARDS). ARDS is characterized by acute onset; diffuse, inflammatory lung injury; and bilateral pulmonary infiltrates on chest imaging
leading to hypoxic respiratory failure. There are numerous causes, including sepsis (especially in the setting of gram-negative bacteria caused by macrophage overactivation by lipopolysaccharide),
pneumonia, chemical pneumonitis (eg, aspiration, likely in this case with the possibility of bacterial aspiration pneumonia), pulmonary contusion, acute pancreatitis, trauma, transfusions, medications, and
amniotic fluid embolism. Alveolar damage and inflammation lead to increased permeability of the alveolar-capillary interface. Urgent endotracheal intubation should occur in patients who are apneic, hypoxic,
unable to protect their airway (eg, lethargic, no gag or cough reflex), or hypercapnic and not responding to less invasive means. Intubation and mechanical ventilation is the most appropriate next step in
management of this condition to secure the airway and ensure adequate oxygenation and ventilation.

Incorrect Answers: A, B, C, and D.

Hyperbaric oxygen therapy (Choice A) is a treatment indicated for carbon monoxide (CO) poisoning. Symptoms of CO poisoning include headache, lightheadedness, nausea, and altered mental status.
Acute management involves removing the patient from the exposure and treatment with 100% supplemental oxygen to displace CO from hemoglobin. Severe cases and high-risk patients demonstrating any
degree of neurologic deficit or significant end-organ dysfunction should be treated with hyperbaric oxygen therapy.

Intravenous administration of dobutamine (Choice B) would be indicated in the treatment of cardiogenic shock. This patient has a fever and no history of heart failure, making cardiogenic shock less likely
and septic shock more likely. At present, his heart rate and blood pressure are within normal limits, indicating that no presser or inotropic support is needed at this time.

Intravenous administration of methylprednisolone (Choice C) is used in the treatment of allergic, autoimmune, and inflammatory conditions such as asthma, Crohn disease, and gout. It is not clear whether
steroids are helpful in ARDS; they would not be the next step in management.

Bronchoscopy with lavage (Choice D) can be indicated in diagnosis of some infections such as tuberculosis and Pneumocystis jirovecii pneumonia. This patient is without known immunocompromise,
making these infections less likely.

Educational Objective: Acute respiratory distress syndrome results in diffuse inflammatory lung injury and hypoxic respiratory failure. Patients experiencing hypoxic respiratory failure with limited ability to
protect the airway require treatment involving intubation and mechanical ventilation.

https://t.me/USMLENBME2CK ti
Previous Next Score Report Lab Values Calculator Help pause
Exam Section 4: Item 14 of 50 National Board of Medical Examiners
Comprehensive Clinical Science Self-Assessment

✓ 14. A 37-year-old woman is scheduled for arthroscopic repair of a right medial meniscus tear. Her procedure is the surgeon's last scheduled operation of the day. All previous procedures this day involved
the left knee. In the operating room, the resident prepares the left knee for the operation. The operation is discontinued after the surgeon discovers no meniscus tear, and the patient is informed about
the error. Which of the following is the best approach to prevent future errors of this type?

A) Asking the surgeon to resign from the medical staff


B) Having the surgeon and resident present the case at the next morbidity and mortality conference
C) Having the surgeon mark the surgical site with input from the patient prior to induction of anesthesia
D) Having the surgeon review the chart just prior to the procedure
E) Placing an incident report in the files of the resident and surgeon
Correct Answer: C.

"Wrong site" surgery is a rare but potentially catastrophic form of surgical error that is likely to result in malpractice litigation. "Wrong site" surgery can result in potentially serious harm, such as inappropriate
amputation or unnecessary surgery. Procedures for preoperative verification and surgical marking are essential in preventing "wrong site" errors. Physicians should be familiar with The Joint Commission
Universal Protocol for Preventing Wrong Site, Wrong Procedure, Wrong Person Surgery. Steps include preoperative verification of the patient's identity, the procedure to be performed, and the surgical site.
The patient should be involved in the verification process whenever possible, and any discrepancies or questions should be addressed. The use of a checklist can assist in ensuring a standardized
verification process. The surgical site should be marked by the surgeon who will perform the procedure, and the surgical mark should be both unambiguous and positioned such that it will be visible after
the surgical drape is placed. Additionally, a final verification "time our should be performed prior to induction of anesthesia to verify the patient's identity, the procedure, and the surgical site.

Incorrect Answers: A, B, D, and E.

Asking the surgeon to resign from the medical staff (Choice A) or placing an incident report in the files of the resident and surgeon (Choice E) are unnecessary and unlikely to contribute significantly to
preventing similar errors in the future. "Wrong site" errors are often multifactorial and typically result in a significant, negative impact on the surgeon involved in addition to the patient. The occurrence of a
single error does not necessarily imply that the surgeon is unfit to continue practicing medicine. The surgical team should be involved in a root cause analysis with hospital staff and risk management to
reduce the risk for similar errors in the future.

Having the surgeon and resident present the case at the next morbidity and mortality conference (Choice B) may be a useful adjunctive approach for promoting awareness of the possibility of "wrong site"
errors and of emphasizing the importance of following the Universal Protocol. However, this does not supersede the importance of appropriate surgical marking.

Having the surgeon review the chart just prior to the procedure (Choice D) is appropriate but is not a substitute for preoperative verification. Additionally, involving the patient directly in the verification
process can greatly reduce the likelihood of "wrong site" errors.

Educational Objective: "Wrong site" surgery is a rare but potentially catastrophic form of medical error that may result in serious patient harm, and that is likely to result in malpractice litigation. Physicians
should be familiar with The Joint Commission Universal Protocol for Preventing Wrong Site, Wrong Procedure, Wrong Person Surgery. A thorough, standardized process should be followed to verify the
patient's identity, procedure to be performed, and surgical site prior to beginning surgery.

https://t.me/USMLENBME2CK ti
Previous Next Score Report Lab Values Calculator Help pause
Exam Section 4: ltem 15 of 50 National Board of Medical Examiners
Comprehensive Clinical Science Self-Assessment

"I 15. A 52-year-old woman comes to the office because of a 6-week history of moderate epigastric pain and an intermittent mild, midsternal burning sensation when she eats. She has had a 4.5-kg (10-lb)
weight loss during the past 6 months despite no change in appetite. During this time, she also occasionally has had dark stools. She has type 2 diabetes mellitus, hypertension, and osteoarthritis. Her
medications are metformin, hydrochlorothiazide, and ibuprofen. She has smoked one pack of cigarettes daily for 36 years. She appears malnourished. She is 163 0m (5 ft 4 in) tall and weighs 54 kg
(120 lb); BMI is 21 kg/m. Her pulse is 86/min, and blood pressure is 110/65 mm Hg. Examination shows mild epigastric tenderness. There is no hepatosplenomegaly. The remainder of the
examination shows no abnormalities. Her hematocrit is 30%, leukocyte count is 6200/mm3, and platelet count is 530,000/mm3. Serum electrolyte concentrations are within the reference ranges. In
addition to discontinuing ibuprofen therapy, which of the following is the most appropriate next step in management?

A) Celecoxib therapy
B ) Esophagogastroduodenoscopy
C) Follow-up examination in 6 weeks
D) Pantoprazole therapy and follow-up examination in 6 weeks
E) Serologic testing for Helicobacter pylori
Correct Answer: B.

Esophagogastroduodenoscopy is used to evaluate the esophagus, stomach, and the duodenum, and can diagnose conditions such as peptic ulcer disease, esophagitis, or esophageal cancer among others.
This patient has features suggestive of an esophageal or gastric cancer given unintentional weight loss and upper gastrointestinal bleeding leading to occasional dark stools. Ibuprofen and other nonsteroidal
anti-inflammatory drugs (NSAIDs) can cause gastric mucosal irritation leading to upper gastrointestinal bleeding and should be stopped in the setting of epigastric pain and dark stools. Upper gastrointestinal
bleeding in the setting of malnourishment and/or unintentional weight loss should be evaluated directly with esophagogastroduodenoscopy. Gastric cancer classically presents with weight loss, early satiety,
and melena in a patient with a history of smoking, peptic ulcer disease, chronic gastritis, or frequent consumption of nitrate-containing foods.

Incorrect Answers: A, C, D, and E.

Celecoxib therapy (Choice A) would be contraindicated, as celecoxib is an NSAID that can cause or worsen gastric mucosa I irritation. Patients with evidence of upper gastrointestinal bleeding should avoid
NSAIDs.

Follow-up examination in 6 weeks (Choice C) would not be appropriate as the upper gastrointestinal bleeding and weight loss should be addressed and investigated.

Pantoprazole therapy and follow-up examination in 6 weeks (Choice D) would be appropriate for peptic ulcer disease. Peptic ulcer disease classically presents with abdominal pain related to the
consumption (gastric) or lack of consumption (duodenal) of food. Peptic ulcer disease is strongly associated with infection from the bacterium Heficobacter pylori. If a patient has any risk factors for H. pylori
infection, serologic testing should be performed (Choice E). Eradication therapy using a triple or quadruple combined therapy for H. pylori infection can eliminate the bacterium and permit healing of the
peptic ulcer, generally with few complications, if managed promptly.

Educational Objective: Upper gastrointestinal bleeding in the setting of malnourishment and unintentional weight loss raises suspicion for gastric cancer and should be evaluated directly with
esophagogastroduodenoscopy.

https://t.me/USMLENBME2CK ts e t
Previous Next Score Report Lab Values Calculator Help pause
Exam Section 4: ltem 16 of 50 National Board of Medical Examiners
Comprehensive Clinical Science Self-Assessment

"I 16. A67-year-old man comes to the physician because of a 6-month history of increasing difficulty walking. He has not had pain or numbness. He has no history of serious illness or recent infection.
Examination of the lower extremities shows atrophy; muscle strength is 3/5 bilaterally. Deep tendon reflexes are increased in the lower extremities. Sensation is intact. Which of the following is the
most likely diagnosis?

A) Amyotrophic lateral sclerosis


B) Multiple sclerosis
C) Myasthenia gravis
D) Polymyositis
E) Transverse myelitis
Correct Answer: A.

Amyotrophic lateral sclerosis is a progressive neurodegenerative disorder typically beginning in midlife that affects both lower motor neurons arising from the anterior horns of the spinal cord and upper motor
neurons of the lateral corticospinal tract within the brain and spinal cord. Neurons degenerate as a result of an unknown cause that may involve RNA processing and/or excitotoxicity, and gliosis results.
Patient presentations are variable though typically feature a combination of upper motor neuron (eg, increased muscle tone, hyperreflexia, Babinski sign) and lower motor neuron findings (eg, decreased
muscle tone, hyporeflexia, early muscle atrophy) in an asymmetric distribution that may include the bulbar muscles. Some patients may experience cognitive or behavioral changes, though other patients are
cognitively normal. The diagnosis is primarily clinical but may be supported by electromyographic findings of acute and chronic denervation and reinnervation, along with nerve conduction studies showing
reduced compound muscle action potentials caused by denervated and atrophied muscles. MRI of the brain and spine should be performed to rule out other diseases. Treatment should include riluzole,
which may modestly improve survival but does not cure the disease.

Incorrect Answers: B, C, D, and E.

Multiple sclerosis (Choice B) is an autoimmune demyelinating disease that typically presents in white women in their fourth decade of life. It is characterized by central nervous system demyelination of white
matter in both the brain and the spinal cord. This leads to weakness, hyperreflexia, and spasticity. Optic neuritis is a classic presenting diagnosis. Transverse myelitis (Choice E) is another subtype of
demyelinaling disease, often idiopathic in cause, that produces bilateral sensory, motor, and/or autonomic dysfunction, often with a clearly identifiable spinal level below which normal function is reduced or
absent.

Myasthenia gravis (Choice C) is an autoimmune disorder of neuromuscular transmission that has a bimodal age of onset, peaking in both young adults (teens to 20s) and older adults (50s to 60s). Patients
typically present with progressively worsening muscle weakness, usually involving the ocular, bulbar, and neck muscles.

Polymyositis (Choice D) is an autoimmune inflammatory myopathy that presents with symmetrical proximal muscle weakness, most commonly in women. Patients typically present with difficulty performing
overhead tasks, rising from a seated position, and climbing stairs.

Educational Objective: Amyotrophic lateral sclerosis is a progressive neurodegenerative disorder that affects the anterior horns of the spinal cord and the lateral corticospinal tract. Subsequently, patients
demonstrate a combination of upper and lower motor neuron findings in an asymmetric distribution.

https://t.me/USMLENBME2CK ts e t
Previous Next Score Report Lab Values Calculator Help pause
Exam Section 4: ltem 17 of 50 National Board of Medical Examiners
Comprehensive Clinical Science Self-Assessment

✓ 17. A 14-year-old girl is brought to the physician because of an 18-hour history of persistent right-sided lower abdominal pain. She has not had fever, vomiting, or diarrhea. She has no history of serious
illness or operative procedures. Menarche was at the age of 12 years, and menses occur at regular 28-day intervals. Her last menstrual period was 2 weeks ago. She is not sexually active. Her
temperature is 37°C (98.6°F), pulse is 72/min, respirations are 14/min, and blood pressure is 120/80 mm Hg. Abdominal examination shows mild tenderness with guarding over the right lower
quadrant. There is no distention, and bowel sounds are normal. The remainder of the examination shows no abnormalities. Her hemoglobin concentration is 13.9 g/dl, and leukocyte count is
8000/mm? with a normal differential. Urinalysis shows no abnormalities. Abdominal ultrasonography shows a small amount of fluid in the pelvis. Which of the following is the most likely diagnosis?

A) Appendicitis
B) Ectopic pregnancy
C) Mittelschmerz
D) Ovarian torsion
E) Pelvic inflammatory disease
Correct Answer: C.

Mittelschmerz is a common cause of abdominopelvic pain in females. It is caused by peritoneal irritation in the setting of an enlarging ovarian follicle or its rupture during egg release. It commonly presents
with mid-cycle unilateral abdominopelvic pain, associated with mild abdominal tenderness, in patients with regular menstrual cycles. As the pain is secondary to normal physiological hormonal changes and
not pathologic, vital signs and results of laboratory studies will be within normal limits. While ultrasonography is not necessary for diagnosis, ii is often ordered as mittelschmerz can mimic appendicitis. If
completed, ii would be expected to show a small amount of free fluid in the pelvis secondary to follicular rupture, as seen in this patient.

Incorrect Answers: A, B, D, and E.

Appendicitis (Choice A) classically presents with right lower quadrant abdominal pain that is often periumbilical at onset, nausea, anorexia, fever, leukocytosis, and tenderness to palpation in the right lower
quadrant. This patient has mild abdominal pain but no nausea or leukocytosis. Additionally, appendicitis or secondary signs related to it would likely be seen on an abdominal ultrasonography.

Ectopic pregnancy (Choice B) is an abnormal extrauterine pregnancy (eg, in the fallopian tube, cervix, ovary, or peritoneum). It commonly presents with vaginal bleeding, abdominopelvic pain, and
increased �-hCG. This patient is not sexually active, which makes an ectopic pregnancy unlikely.

Ovarian torsion (Choice D) is caused by twisting of the ovary about its ligamentous attachment with resultant compromise of its blood supply leading to ovarian ischemia. Patients present with sharp,
severe, unilateral lower abdominal or pelvic pain, often accompanied by nausea and vomiting. The absence of blood flow to the ovary would be expected on ultrasonography.

Pelvic inflammatory disease (Choice E) is inflammation of the uterus, fallopian tubes, or ovaries, and usually occurs secondary to infection with the sexually transmitted organisms Chlamydia trachomatis or
Neisseria gonorrhoeae. It may also occur because of introduced gastrointestinal flora. It presents with abnormal cervical discharge, cervicitis, and cervical motion tenderness. Peritonitis may ensue in
advanced cases. This patient is not sexually active, which makes pelvic inflammatory disease unlikely.

Educational Objective: Mittelschmerz describes mild mid-cycle, often unilateral, pain caused by peritoneal irritation in the setting of follicular development and rupture. It is a benign process, and is not
associated with any significant hemodynamic, laboratory, or radiologic abnormalities. It can mimic appendicitis; ultrasonography may be necessary to determine the diagnosis.

https://t.me/USMLENBME2CK ts e t
Previous Next Score Report Lab Values Calculator Help pause
Exam Section 4: ltem 18 of 50 National Board of Medical Examiners
Comprehensive Clinical Science Self-Assessment

A 62-year-old man, who underwent right below-the-knee amputation 36 years ago, comes to the emergency department because of a 1-month history of an ulcer on the end of the residual limb. His
amputation was the result of injuries he sustained while serving in the military. He says his prosthetic socket has "felt loose and has been rubbing recently." During the past 6 months, he has had
progressive difficulty walking because of sharp pain that radiates up his leg. The pain has been occurring earlier during his walks; ii slowly improves with rest. He previously walked 1 mile daily for
exercise and could walk several miles without stopping. He has not had vomiting, fever, chills, or malaise. He has type 2 diabetes mellitus treated with diet. One month ago, his hemoglobin A1c was
6%. Vital signs are within normal limits. Examination of the right lower extremity shows mild erythema and a 1 x 1-cm ulcer, with a depth of 3 mm, over the anteromedial aspect of the limb. There is
viable soft tissue, and margins are clear. The limb is warm. The popliteal pulse is 1 +. In addition lo a recommendation for non-weight bearing and avoidance of direct pressure to the site, which of the
following is the most appropriate next step in management?

A) MRI of the right lower extremity


B) Recommendation for refitting of the prosthetic socket
C) Surgical revision of the amputation site
D) Wound culture and antibiotic therapy
E) X-ray of the right lower extremity
Correct Answer: B.

Pressure injury, also known as a decubitus ulcer, is a common complication in patients who utilize a limb prosthesis. In most cases, the injury can be managed conservatively by avoiding aggravating factors
and direct pressure to the site. For this patient, a recommendation for refitting of the prosthetic socket is appropriate given his report of the prosthesis feeling loose and rubbing recently. Several factors can
contribute to an ill-fitting prosthesis, including changes in weight and body habitus, variations in activity levels and mobility, and general wear and tear on the unit. The region of skin interacting with the
prosthesis is subject to high amounts of moisture and shear forces that result in pressure injury. Additional complications include infection, of which this patient is not displaying concerning signs or
symptoms. Close follow-up is recommended to ensure appropriate healing of the wound and monitor for developing infection.

Incorrect Answers: A, C, D, and E.

MRI of the right lower extremity (Choice A) is not necessary for routine assessment of pressure injury. Imaging can be performed if the patient has high-risk features such as indwelling medical devices or if
there are systemic signs of infection. MRI is the preferred imaging modality to assess for underlying abscess and/or osteomyelitis.

Surgical revision of the amputation site (Choice C) is an invasive management strategy that is not warranted at this time. Pressure injury complicated by infection or a significant burden of necrotic tissue
may benefit from surgical debridement.

Wound culture and antibiotic therapy (Choice D) are not indicated. Soft tissue infection generally presents with warmth, erythema, pain, and purulent discharge at the ulcer site as well as systemic signs of
infection, including fever and malaise.

X-ray of the right lower extremity (Choice E) is not recommended in the evaluation of pressure ulcer complications, as it is not sensitive nor specific enough lo rule out infection. MRI is the preferred imaging
modality.

Educational Objective: Decubitus ulcers, or pressure injuries, are a common complication encountered in clinical practice, most commonly occurring at sites of repetitive friction or where a bony prominence
contacts an external surface. Initial management consists of local wound care, assessing for skin, soft tissue, and bone infection, and addressing the source of the injury.

https://t.me/USMLENBME2CK ts e t
Previous Next Score Report Lab Values Calculator Help pause
Exam Section 4: ltem 19 of 50 National Board of Medical Examiners
Comprehensive Clinical Science Self-Assessment

"I 19. A 40-year-old man comes to the physician 6 days after noticing a non painful lump in his neck. He also has a 2-week history of palpitations. He is otherwise asymptomatic. He has no history of serious
illness and takes no medications. His temperature is 37.2°C (99°F), pulse is 110/min, respirations are 16/min, and blood pressure is 134/60 mm Hg. Examination shows a 1.4-cm, firm thyroid nodule.
His serum thyroid-stimulating hormone concentration is less than 0.05 mlU/L, and serum free thyroxine concentration is 3.5 ng/dl (N=0.9-1.7). Which of the following is the most appropriate next step
in management?

A) Fine-needle aspiration biopsy of the nodule


B) Measurement of serum calcitonin concentration
C) Operative resection of the nodule
D) Radioactive iodine uptake scan
E) Ultrasonography of the thyroid gland
Correct Answer: D.

Discovery of a palpable thyroid mass by a patient at home or during a routine clinical visit is a common occurrence, and the diagnostic workup focuses on ruling out malignancy. Risk factors for thyroid
malignancy include young age (the prevalence of benign thyroid nodules increases with age), radiation exposure to the head or neck, and a family history of thyroid cancer or associated congenital
neoplastic syndrome (eg, multiple endocrine neoplasia, type 2). All patients with a new nodule should be evaluated with serum TSH measurement and, if the TSH concentration is low, a radioactive iodine
uptake scan (thyroid scintigraphy) should be ordered. A low TSH concentration indicates a hyperfunctioning nodule. These are rarely malignant and do not require fine-needle aspiration biopsy. Thyroid
ultrasonography should also be performed, and, if the uptake scan was performed initially, fine-needle aspiration biopsy can be avoided. Subsequent workup and management are dependent upon the
results of the TSH measurement, ultrasound characteristics of the nodule, and results of the radioactive iodine uptake scan.

Incorrect Answers: A, B, C, and E.

Fine-needle aspiration biopsy of the nodule (Choice A) can be avoided in patients who have an area of focal uptake on a radioactive iodine uptake scan and/or benign appearance on thyroid
ultrasonography. This patient has a low TSH concentration, and obtaining thyroid scintigraphy prior to ultrasonography is likely to spare him the biopsy.

Measurement of serum calcilonin concentration (Choice 8) is an area of active study regarding thyroid nodules and malignancy. The American Thyroid Association currently does not recommend for or
against testing levels. The diagnostic value of TSH measurement, thyroid scintigraphy, and thyroid ultrasonography is well established.

Operative resection of the nodule (Choice C) is not necessary prior to identification of the mass. Surgical management may be required for hyperfunctioning nodules refractory to treatment or malignancy.

Ultrasonography of the thyroid gland (Choice E) should be performed in the diagnostic workup but can be done after thyroid scintigraphy as data from the uptake scan will inform whether an ultrasound-
guided fine-needle aspiration biopsy is indicated at the time of the study.

Educational Objective: Palpable thyroid masses are a common incidental discovery on physical examination and imaging. Diagnostic workup is necessary to exclude malignancy, and all patients should have
their TSH concentration checked and ultrasonography of the thyroid gland performed. A radioactive iodine uptake scan is recommended if the TSH concentration is below the reference range.

https://t.me/USMLENBME2CK ts e t
Previous Next Score Report Lab Values Calculator Help pause
Exam Section 4: Item 20 of 50 National Board of Medical Examiners°
Comprehensive Clinical Science Self-Assessment

'.IN

V 27I '
4
I\ r'

20. A previously healthy 24-year-old man comes to the emergency department because of a 12-hour history of increasing pain to the left of the center of his chest. Initially, the pain was mild and occurred
only when he breathed deeply. Now, the pain is more severe and constant. He rates the pain as an 8 on a 10-point scale. The pain increases when he lies on his back, decreases when he leans
forward, and does not radiate. He has not had palpitations, shortness of breath, nausea, vomiting, or sweating. He says that he is "recovering from a head cold," which he has had for 14 days. He
appears moderately uncomfortable but is in no respiratory distress. His temperature is 37.6°C (99.6F), pulse is 66/min and regular, respirations are 20/min, and blood pressure is 152/88 mm Hg.
Examination, including cardiovascular examination, shows no abnormalities. His serum creatine kinase activity is 420 mg/dl with an MB fraction of 12% (N<5%) An ECG is shown. Which of the
following is the most appropriate next step in diagnosis?

A) Echoardiography
B) CT scan of the chest
C) Cardiac MRI
D) Coronary angiography
E) Pericardiocentesis
Correct Answer: A.

Echocardiography is indicated for the assessment of a pericardia! effusion in the setting of potential myopericarditis. Pericarditis classically presents with substernal chest pain, which is often pleuritic,
worsens when lying down, and improves with leaning forward. It can occur in patients with inflammatory, infectious, or malignant conditions, such as viral infection, systemic lupus erythematosus,
tuberculosis, or lymphoma, and can present as a complication of myocardial infarction or cardiac surgery. On examination, pericarditis is disclosed by a diastolic friction rub, described as a harsh sound
heared in elistole The cone\ition ig einnnear] clinicallu and gnnnortar] hu inoraasae] inflammatory markore gnioh e orthrwto garlimantatinn rate an4 (_ratio nratoin fie oharactoriae\ hu eliffee gT

« https://t.me/USMLENBME2CK ts e t
Previous Next Score Report Lab Values Calculator Help pause
Exam Section 4: Item 20 of 50 National Board of Medical Examiners
Comprehensive Clinical Science Self-Assessment

appears moderately uncomfortable but is in no respiratory distress. His temperature is 37.6°C (99.6F), pulse is 66/min and regular, respirations are 20/min, and blood pressure is 152/88 mm Hg.
Examination, including cardiovascular examination, shows no abnormalities. His serum creatine kinase activity is 420 mg/dl with an MB fraction of 12% (N<5%) An ECG is shown. Which of the
following is the most appropriate next step in diagnosis?

A) Echocardiography
B) CT scan of the chest
C) Cardiac MRI
D) Coronary angiography
E) Pericardiocentesis
Correct Answer: A.

Echocardiography is indicated for the assessment of a pericardia! effusion in the setting of potential myopericarditis. Pericarditis classically presents with substernal chest pain, which is often pleuritic,
worsens when lying down, and improves with leaning forward. II can occur in patients with inflammatory, infectious, or malignant conditions, such as viral infection, systemic lupus erythematosus,
tuberculosis, or lymphoma, and can present as a complication of myocardial infarction or cardiac surgery. On examination, pericarditis is disclosed by a diastolic friction rub, described as a harsh sound
heard in diastole. The condition is diagnosed clinically and supported by increased inflammatory markers such as erythrocyte sedimentation rate and C-reactive protein. ECG is characterized by diffuse ST
segment elevation and PR segment depression. If not treated, pericarditis can lead to the development of pericardia! effusions, which, if large and quickly accumulating, can result in cardiac tamponade.
Echocardiography should be performed to assess for the presence of a pericardia! effusion, which may require pericardiocentesis for drainage, if large. Overall cardiac function (eg, ejection fraction) should
also be assessed if cardiac enzyme biomarkers are increased and if there is concern for concomitant myocarditis or heart failure. Treatment includes anti-inflammatory medications, such as indomethacin,
glucocorticoids, and/or colchicine. Conditions with a similar presentation should also be ruled out, such as acute myocardial infarction, pulmonary embolism, and aortic dissection.

Incorrect Answers: B, C, D, and E.

CT scan of the chest (Choice B) can provide structural but only limited functional information about the heart, whereas echocardiography permits analysis of structural defects, pressure, velocity, and flow of
blood, which is important in understanding the hemodynamics associated with structural heart disease, cardiac inflammation, or pericardia! effusion/tamponade.

Cardiac MRI (Choice C) is the gold standard for diagnosis of conditions such as left ventricular hypertrophy and cardiac amyloidosis, though availability, cost, and logistics limit its use as a first-line screening
tool.

Coronary angiography (Choice 0) is an invasive test performed via percutaneous catheter to assess the anatomy and patency of the coronary arteries. It is indicated for the diagnostic and therapeutic
management of acute coronary syndrome (eg, ST-segment elevation myocardial infarction) and for the diagnosis and management of coronary artery disease following an abnormal stress test.

Pericardiocentesis (Choice E) is appropriate for pericardia! tamponade or in the workup of a pericardia! effusion; however, these diagnoses should first be established with imaging or physical examination.

Educational Objective: Pericarditis classically presents with substernal chest pain, which is often pleuritic, worsens when lying down, and improves with leaning forward. Echocardiography is indicated for the
assessment of a pericardia! effusion in the setting of pericarditis.

https://t.me/USMLENBME2CK ts e t
Previous Next Score Report Lab Values Calculator Help pause
Exam Section 4: Item 21 of 50 National Board of Medical Examiners
Comprehensive Clinical Science Self-Assessment

"I 21. A 21-year-old man is brought to the emergency department by paramedics because of a 10-hour history of altered mental status. The mother says the patient was "fine" last night. This morning, he did
not wake up at his usual time; when she went to check on him, she could not get him to wake up. He has no history of serious medical or psychiatric illness and takes no medications. His mother says
he does not smoke cigarettes, drink alcohol, or use illicit drugs. On arrival, he is lethargic but not in respiratory distress. He is responsive to painful stimuli. His temperature is 36.7°C (98.0°F), pulse is
120/min, respirations are 26/min, and blood pressure is 135/85 mm Hg. Pulse oximetry on room air shows an oxygen saturation of 100%. Physical examination shows no other abnormalities.
Laboratory studies are shown:
Serum
Na' 143 mEq/L
K" 3.8 mEq/L
Cl 110 mEq/L
HCO 3 13 mEq/l
Urea nitrogen 14 mg/dl
Glucose 120 mg/dl
Osmolality 290 mOsmol/kg HO
Urine
Glucose none
Ketones 3+

Arterial blood gas analysis is shown:


pH 7.43
Pco, 22 mm Hg
Po, 96 mm Hg

Which of the following is the most likely diagnosis?

A) Alcoholic ketoacidosis
B) Diabetic ketoacidosis
C) Ethylene glycol toxicity
0) Methanol toxicity
E) Salicylate toxicity
Correct Answer: E.

Salicylate poisoning in large doses results in a respiratory alkalosis from the stimulation of respiratory centers causing hyperventilation. Subsequently, it results in the development of a delayed anion gap
metabolic acidosis secondary to its decoupling effects on the electron transport chain, resulting in increased anaerobic metabolism and the production of lactate and ketoacids beyond the acidic properties of
salicylic acid itself. It does not typically affect sodium and potassium serum concentrations. Patients with salicylate toxicity will present with hyperventilation, gastrointestinal upset, hyperthermia, tinnitus,
a0itatinn delirium ane\ hallinatione AMnanomant f galielato toxicity innlae gvetomio alkalinizatinn with p eolinn hieprhante inf cir pationte ma naar] dileie in the gottin ef gouors altorer] mental

https://t.me/USMLENBME2CK ts e t
Previous Next Score Report Lab Values Calculator Help pause
Exam Section 4: Item 21 of 50 National Board of Medical Examiners
Comprehensive Clinical Science Self-Assessment

Arterial blood gas analysis is shown:


pH 7.43
Pco, 22 mm Hg
Po, 96 mm Hg

Which of the following is the most likely diagnosis?

A) Alcoholic ketoacidosis
B) Diabetic ketoacidosis
C) Ethylene glycol toxicity
D) Methanol toxicity
E) Salicylate toxicity
Correct Answer: E.

Salicylate poisoning in large doses results in a respiratory alkalosis from the stimulation of respiratory centers causing hyperventilation. Subsequently, it results in the development of a delayed anion gap
metabolic acidosis secondary to its decoupling effects on the electron transport chain, resulting in increased anaerobic metabolism and the production of lactate and ketoacids beyond the acidic properties of
salicylic acid itself. It does not typically affect sodium and potassium serum concentrations. Patients with salicylate toxicity will present with hyperventilation, gastrointestinal upset, hyperthermia, tinnitus,
agitation, delirium, and hallucinations. Management of salicylate toxicity involves systemic alkalinization with a sodium bicarbonate infusion. Patients may need dialysis in the setting of severe altered mental
status, end-stage kidney disease, pulmonary edema, or clinical deterioration.

Incorrect Answers: A, B, C, and D.

Alcoholic ketoacidosis (Choice A) is induced by excessive alcohol intake that results in increased ketone production and a high anion gap metabolic acidosis without hyperglycemia. It would not typically
result in a respiratory alkalosis such as with salicylate poisoning.

Diabetic ketoacidosis (Choice B) occurs as a result of the accumulation of serum ketones such as beta-hydroxybutyric acid, acetoacetic acid, and acetone in the setting of deficient insulin signaling.
Laboratory studies typically show increased glucose, ketones, and a high anion gap caused by the accumulation of unmeasured anions in serum.

Ethylene glycol toxicity (Choice C) results from metabolism of ethylene glycol by alcohol dehydrogenase to toxic metabolites including glycolate, glyoxylate, and oxalate. Patients present with acute kidney
injury, metabolic acidosis, and crystals on urinalysis. Methanol toxicity (Choice D) results similarly from toxic metabolites including formic acid that can cause visual disturbances, retinal edema, optic disc
hyperemia, gastrointestinal distress, putaminal hemorrhage, and neurologic depression. Ethylene glycol and methanol toxicity can cause an increased serum osmolality and an osmolar gap.

Educational Objective: Salicylate toxicity can present with hyperventilation, gastrointestinal upset, hyperthermia, tinnitus, agitation, delirium, and hallucinations. Salicylates cause early onset respiratory
alkalosis from stimulation of respiratory centers causing hyperventilation, followed by a delayed anion gap metabolic acidosis caused by decoupling of the electron transport chain.

https://t.me/USMLENBME2CK ts e t
Previous Next Score Report Lab Values Calculator Help pause
Exam Section 4: Item 22 of 50 National Board of Medical Examiners
Comprehensive Clinical Science Self-Assessment

A 25-year-old man is brought to the emergency department 1 hour after sustaining a gunshot wound to the abdomen. He has pain at the bullet entry site. He is awake, alert, and appropriately
conversant. His temperature is 38.8°C (101.8F), pulse is 85/min, respirations are 18/min, and blood pressure is 128/70 mm Hg. Abdominal examination shows a bullet wound 5 cm to the right of the
umbilicus. There is moderate tenderness over the entry site. The remainder of the examination shows no abnormalities. During operative exploration, 250 ml of succus entericus and eight small bowel
holes are found in the abdomen; the bowel holes are treated by resection of 12 cm of proximal ileum. The intestine is reconstructed with a stapled anastomosis. The midline abdominal fascia is sutured
closed. Which of the following methods of skin and soft tissue wound management at this point is most likely to decrease the risk for postoperative wound infection?

A) Delayed primary closure


B) Hand-sewn suture closure of the skin
C) Use of skin staples
D) Application of cyanoacrylate adhesive to the wound
E) Split-thickness skin graft
Correct Answer: A.

Surgical site infection is a relatively common complication of abdominal surgeries associated with increased morbidity and mortality. It often presents with fever, erythema, induration, and pain al the surgical
site. Physical examination may show purulent drainage upon palpation of the wound. Deeper abscesses are possible and may be less evident on physical examination. Patient risk factors for the
development of surgical site infection include diabetes mellitus, immunosuppression, tobacco use, and increased age. Additionally, traumatic wounds and infected intra-abdominal contents increase the risk
for development of infection despite aggressive irrigation. In these patients, delayed primary closure involves leaving the incision open, often with negative pressure wound therapy, to allow any underlying
infected tissue to drain. This also enables surgeons to do further irrigation and debridement, if necessary, without reopening the abdomen. This patient presented with a traumatic abdominal wound
associated with a perforated viscus and succus entericus, in the setting of a high-grade fever, making the likelihood of a blossoming infection high. Therefore, the most appropriate wound management
technique for this patient is delayed primary closure lo decrease the risk for postoperative infection.

Incorrect Answers: B, C, D, and E.

Hand-sewn suture closure of the skin (Choice B) is commonly used lo close abdominal incisions. Suture closure allows reapproximation of the tissues, which allows healing by primary intention. However,
hand-sewn closure of the skin would prevent the surgical tissues from draining in this patient at high risk for intra-abdominal infection, which may lead to intra-abdominal abscess or wound site infection.

Use of skin staples (Choice C) to close abdominal incisions is a common method. There is evidence that staples have a decreased risk for surgical site infection in comparison to hand-sewn closures, as the
wound is still able to drain between the staples. However, the use of staples is still inferior to delayed primary closure in preventing surgical site infections.

Application of cyanoacrylate adhesive to the wound (Choice 0) is a common technique used on small wounds. Tissue adhesives can allow the approximation of tissue without suture or can strengthen
wound approximation in sutured or stapled wounds. However, its use is contraindicated in infected tissues.

Split-thickness skin graft (Choice E) is not necessary. Split-thickness skin grafts are commonly used in burn patients to replace debrided skin. They may be used for traumatic wounds to replace tissue lost
during the injury. This patient has no evidence of significant tissue loss. Skin grafts are contraindicated in infected tissue.

Educational Objective: Surgical site infection is a relatively common complication of abdominal surgeries associated with increased morbidity and mortality. Delayed primary closure is a wound management
technique used for infected wound sites, such as those as a result of a traumatic injury or those with evidence of contamination. II involves leaving the wound open, often with negative wound pressure
therapy, to allow underlying tissues to drain, which decreases the risk for surgical site infection.

https://t.me/USMLENBME2CK ti
Previous Next Score Report Lab Values Calculator Help pause
Exam Section 4: Item 23 0f 50 National Board of Medical Examiners°
Comprehensive Clinical Science Self-Assessment

✓ 23. A32-year-old woman comes to the physician because of a 2-week history of fatigue and increased urination. She has had a 2.3-kg (5-lb) weight loss despite eating more during this time. She has
had two urinary tract infections in the past. She takes no medications. She does not smoke cigarettes, drink alcohol, or use illicit drugs. The patient is alert and fully oriented. She is 165 cm (5 fl 5 in)
tall and weighs 52 kg (115 lb); BMI is 19 kg/m2. Her temperature is 36.5°C (97.7°F), pulse is 100/min, respirations are 20/min, and blood pressure is 90/60 mm Hg. Pulse oximetry on room air shows
an oxygen saturation of 100%. The oropharynx is dry. The remainder of the examination shows no abnormalities. Laboratory studies show:
Serum
Na 142 mEq/L
K' 5.4 mEq/L
Cl 104 mEq/L
HCO 16 mEq/L
Glucose 800 mg/dL
Creatinine 1.3 mg/dL
Urine
Specific gravity 1.030
Bilirubin none
Protein none
RSC 0--5/hpf
WBC 25--50/hpf
Nitrites positive
Leukocyte esterase positive
Glucose >1000 mg/24 h

Which of the following serum hormonal profiles is most consistent with this patient's presentation?

Insulin Glucagon Catecholamines


A) Decreased decreased decreased
8) Decreased increased decreased
C) Decreased decreased increased
D) Decreased increased increased
E) Increased increased decreased
F) Increased decreased increased
G) Increased decreased decreased
H) Increased increased increased
Correct Answer: D.

« https://t.me/USMLENBME2CK ts e t
Previous Next Score Report Lab Values Calculator Help pause
Exam Section 4: Item 23 0f 50 National Board of Medical Examiners°
Comprehensive Clinical Science Self-Assessment

Insulin Glucagon Catecholamines •


A) Decreased decreased decreased
B) Decreased increased decreased
C) Decreased decreased increased
D) Decreased increased increased
E) Increased increased decreased
F) Increased decreased increased
G) Increased decreased decreased
H) Increased increased increased
Correct Answer: D.

Diabetic ketoacidosis (OKA) presents with fatigue, polyuria, polydipsia, shortness of breath or tachypnea, and diffuse abdominal discomfort, commonly in patients with type 1 diabetes mellitus. It occurs as
a result of the accumulation of serum ketones such as r..-hydroxybutyric acid, acetoacetic acid, and acetone in the setting of insulin deficiency. Dysregulated fatty acid degradation and ketone synthesis
occurs as a result of the relative state of starvation created by absent insulin signaling. Vital signs may show tachycardia, tachypnea, and hypotension caused by severe dehydration. Laboratory studies
typically show metabolic acidosis, increased serum glucose, ketones, and catecholamines, and a high anion gap caused by the accumulation of unmeasured anions in the serum. Metabolic acidosis is
accompanied by a decreased bicarbonate concentration in the serum, as bicarbonate is consumed in the neutralization of increased acidic molecules (hydrogen molecules) in the blood. Serum glucagon
and catecholamine concentrations are typically increased, as both are lipogenic, thereby liberating lipids for metabolism to ketones and increasing response to acute physiologic stress. Treatment includes
replacement of insulin, volume repletion, correction of electrolyte abnormalities, and treatment of any precipitating conditions.

Incorrect Answers: A, B, C, E, F, G, and H.

Choices A and C are characterized by absent insulin, but also by decreased glucagon. Glucagon is a catabolic hormone that results in lipolysis. Absence of insulin in OKA leads to a perceived energy-
deficient state. This perceived starvation state stimulates the production of glucagon in order to liberate lipids that can then be metabolized to ketones.

Choice B is characterized by decreased insulin and increased glucagon, but also by decreased catecholamines. OKA is characterized by a marked increase in serum catecholamines, likely as a response
to acute physiologic stress, but also to stimulate the catabolism of lipids.

Choices E, F, G, and H are all characterized by increased serum insulin concentrations, which is incorrect. This young, thin patient is more likely lo have type 1 diabetes mellitus, which is characterized by
autoimmune destruction of pancreatic islet cells and insulin deficiency. Increased insulin concentration with peripheral insulin resistance is more characteristic of type 2 diabetes mellitus.

Educational Objective: Diabetic ketoacidosis presents with fatigue, polyuria, polydipsia, shortness of breath, and diffuse abdominal pain caused by a deficiency of insulin signaling, commonly in type 1
diabetes mellitus. [t results in a high anion gap metabolic acidosis, which is associated with a decreased bicarbonate concentration. Serum findings also include decreased insulin and increased
concentrations of glucose, glucagon, and catecholamines.

« https://t.me/USMLENBME2CK ts e t
Previous Next Score Report Lab Values Calculator Help pause
Exam Section 4: Item 24 0f 50 National Board of Medical Examiners
Comprehensive Clinical Science Self-Assessment

✓ 24. A previously healthy 37-year-old woman comes lo the physician because of a2-year history of her hands and fingers turning a bluish color with exposure to cold. The discoloration has been occurring
with increasing frequency and now occurs with exposure to air conditioning and cold water. Examination of the hands shows no abnormalities. The brachia I and radial pulses are present bilaterally.
When the hands are rinsed in cold water, the fingers become bluish then bright red with rewarming. Which of the following is the most appropriate next step in management?

A) 13-Adrenergic blocking agent therapy


B) Angiotensin-converting enzyme (ACE) inhibitor therapy
C) Calcium-channel blocking agent therapy
D) Corticosteroid therapy
E) Dorsal sympathectomy
Correct Answer: C.

Vasospasm and the intermittent loss of blood flow to the distal extremities is implicated in causing Raynaud phenomenon, resulting in sequential white, blue, and red discoloration of the affected areas of
skin. In severe cases, ischemic injury can result in digital ulcerations. Raynaud phenomenon may be idiopathic or associated with mixed connective tissue disease, systemic lupus erythematosus, or
CREST syndrome. In addition to counseling the patient to recognize and avoid triggers (eg, cold temperature, significant emotional stress), initial pharmacologic management should be with calcium-
channel blocking (CCB) agent therapy. Long-acting dihydropyridine calcium-channel blockers, such as amlodipine or nifedipine, are recommended because of their efficacy and low risk for adverse effects.
Patients should also be advised lo avoid vasoconstricting medications and cigarette smoking.

Incorrect Answers: A, B, D, and E.

13-Adrenergic blocking agent therapy (Choice A) should be avoided in treatment of Raynaud phenomenon as the negative inotropic and chronolropic cardiac effects can further limit perfusion of the digits
and exacerbate symptoms.

Angiotensin-converting enzyme (ACE) inhibitor therapy (Choice B) has had mixed results in treatment of Raynaud phenomenon. It is generally not recommended because of uncertain benefit. However,
angiotensin II receptor blockers have shown some effectiveness and may be considered as a second-line option for patients unable to tolerate CCB agent therapies.

Corticosteroid therapy (Choice D) also has limited effectiveness in treatment of primary Raynaud disease or CREST syndrome, though ii may be indicated for other mixed connective tissue diseases
associated with the phenomenon. This patient should initially be started on CCB agent therapy.

Dorsal sympathectomy (Choice E) is a minimally invasive management option that is rarely necessary, as most patients with Raynaud phenomenon respond to medical therapy. However, it may be
considered for patients who have refractory symptoms.

Educational Objective: Raynaud phenomenon refers to discoloration of the distal extremities associated with an abnormal vascular response to cold temperatures and emotional stress. Calcium-channel
blocking agents are first-line therapy as they are effective at reducing peripheral vasospasm and are generally well-tolerated by patients.

https://t.me/USMLENBME2CK ti
Previous Next Score Report Lab Values Calculator Help pause
Exam Section 4: Item 25 of 50 National Board of Medical Examiners
Comprehensive Clinical Science Self-Assessment

✓ 25. A 60-year-old man comes to the office for a routine examination. Medical history is remarkable for hypertension and alcohol use disorder. Medications are amlodipine and omeprazole. He has smoked
one pack of cigarettes daily for 40 years, and he drinks approximately twelve beers daily. Vital signs are within normal limits. Physical examination discloses no abnormalities. Results of serum studies
are most likely to show which of the following?

A) Hyperkalemia
B) Hypernatremia
C) Hypomagnesemia
D) Hypophosphatemia
Correct Answer: C.

Hypomagnesemia is a common electrolyte disturbance in the general population, particularly among individuals with alcohol use disorder, occurring in up to one third of this population. Hypomagnesemia is
frequently asymptomatic but may present with psychomotor signs and symptoms including depression, lethargy, weakness, tremor, ataxia, or seizures. Severe hypomagnesemia may also present with
cardiovascular findings such as supraventricular tachycardia, ventricular arrhythmias, long-QT interval, and torsades de pointes. Hypomagnesemia may induce secondary electrolyte abnormalities, including
hypokalemia, hypophosphatemia, and hypocalcemia. Hypomagnesemia occurs in alcohol use disorder as a result of poor dietary intake of magnesium, as well as increased kidney excretion of magnesium.
Proton pump inhibitors may cause hypomagnesemia as a result of an unknown mechanism. Other potential causes of hypomagnesemia include medications, such as loop and thiazide diuretics or
aminoglycosides, and gastrointestinal malabsorption, which may occur in the setting of chronic diarrhea, inflammatory bowel disease, or celiac disease. Treatment is with oral supplementation of magnesium,
although severe hypomagnesemia may require intravenous therapy.

Incorrect Answers: A, B, and D.

Hyperkalemia (Choice A) is incorrect. Hypokalemia is a frequent finding in patients with alcohol use disorder, where it commonly occurs secondary to hypomagnesemia.

Hypematremia (Choice B) is incorrect. Hyponatremia is frequent in patients with alcohol use disorder, particularly in those who drink large volumes of beer.

Hypophosphatemia (Choice D) may occur secondary to hypomagnesemia in patients with alcohol use disorder, but it is less frequent than hypomagnesemia.

Educational Objective: Hypomagnesemia is frequently asymptomatic, but may present with psychomotor findings, arrhythmias, or with secondary electrolyte disturbances. Particularly common risk factors for
hypomagnesemia include alcohol use disorder, use of proton pump inhibitors, diuretics, or aminoglycosides, and gastrointestinal malabsorption. Treatment is with magnesium supplementation.

https://t.me/USMLENBME2CK ts e t
Previous Next Score Report Lab Values Calculator Help pause
Exam Section 4: Item 26 of 50 National Board of Medical Examiners
Comprehensive Clinical Science Self-Assessment

"I 26. An 87-year-old woman comes to the physician because of a 3-week history of nausea and general malaise. She has hypertension. One month ago, her blood pressure was 160/95 mm Hg and
hydrochlorothiazide therapy was begun. Her only other medication is 81-mg aspirin. She is 157 cm (5 fl 2 in) tall and weighs 50 kg (110 lb); BMI is 20 kg/m2. Today, her pulse is 78/min; her blood
pressure is 155/92 mm Hg in the right upper extremity and 153/90 mm Hg in the left upper extremity. The remainder of the examination shows no abnormalities. Serum studies show:
Na' 125 mEq/L
K 3.2mEq/L
Cl- 87 mEq/L
HCO, 25 mEq/L
Ca?+ 9.6mg/dL
Urea nitrogen 14 mg/dl
Glucose 107 mg/dl
Creatinine 0.6 mg/dl

Which of the following is the most appropriate next step in pharmacotherapy?

A) Add lisinopril to the regimen


B) Add losartan to the regimen
C) Add triamterene to the regimen
D) Begin potassium supplementation
E) Discontinue hydrochlorothiazide
Correct Answer: E.

Hyponatremia, or a serum sodium concentration less than 135 mEq/L, may be acute or chronic, with symptoms related to the severity of depletion and the abruptness of onset. Acute decreases in sodium
concentration may lead to neurologic symptoms, such as seizure and altered mental status caused by cerebral edema. Chronic hyponatremia, in contrast, leads to less severe symptoms, such as nausea,
vomiting, lightheadedness, fatigue, and confusion. One well-known cause of significant hyponatremia is the initiation of diuretics, with other causes including kidney dysfunction, adrenal insufficiency, volume
overload, and the syndrome of inappropriate antidiuretic hormone secretion. This patient was recently started on hydrochlorothiazide, shortly after which her symptoms began, making her hyponatremia most
likely secondary to thiazide diuretic use. Therefore, the most appropriate next step in pharmacotherapy is discontinuation of hydrochlorothiazide.

Incorrect Answers: A, B, C, and D.

Adding lisinopril to the regimen (Choice A) or adding losartan to the regimen (Choice B) may be reasonable alternatives to hydrochlorothiazide, as this patient will continue to need a medication to treat her
hypertension. However, she has symptomatic hyponatremia, and its management should take priority in her care. Thus, discontinuing hydrochlorothiazide is the most appropriate next step.

Adding triamterene to the regimen (Choice C) would be harmful to this patient. While it is a potassium-sparing diuretic that may increase this patient's relatively low potassium concentration, it is known to
cause hyponatremia. Since this patient has symptomatic hyponatremia, further decreases in her sodium concentrations could be catastrophic.

https://t.me/USMLENBME2CK ti e
Previous Next Score Report Lab Values Calculator Help pause
Exam Section 4: Item 26 of 50 National Board of Medical Examiners
Comprehensive Clinical Science Self-Assessment
3
Ca?+ 9.6 mg/dl
Urea nitrogen 14 mg/dl
Glucose 107 mg/dl
Creatinine 0.6 mg/dl

Which of the following is the most appropriate next step in pharmacotherapy?

A) Add lisinopril to the regimen


B) Add losartan lo the regimen
C) Add triamterene to the regimen
D) Begin potassium supplementation
E) Discontinue hydrochlorothiazide
Correct Answer: E.

Hyponatremia, or a serum sodium concentration less than 135 mEq/L, may be acute or chronic, with symptoms related to the severity of depletion and the abruptness of onset. Acute decreases in sodium
concentration may lead lo neurologic symptoms, such as seizure and altered mental status caused by cerebral edema. Chronic hyponatremia, in contrast, leads to less severe symptoms, such as nausea,
vomiting, lightheadedness, fatigue, and confusion. One well-known cause of significant hyponatremia is the initiation of diuretics, with other causes including kidney dysfunction, adrenal insufficiency, volume
overload, and the syndrome of inappropriate antidiuretic hormone secretion. This patient was recently started on hydrochlorothiazide, shortly after which her symptoms began, making her hyponatremia most
likely secondary to thiazide diuretic use. Therefore, the most appropriate next step in pharmacotherapy is discontinuation of hydrochlorothiazide.

Incorrect Answers: A, B, C, and D.

Adding lisinopril to the regimen (Choice A) or adding losartan to the regimen (Choice B) may be reasonable alternatives to hydrochlorothiazide, as this patient will continue to need a medication to treat her
hypertension. However, she has symptomatic hyponatremia, and its management should take priority in her care. Thus, discontinuing hydrochlorothiazide is the most appropriate next step.

Adding triamterene to the regimen (Choice C) would be harmful to this patient. While it is a potassium-sparing diuretic that may increase this patient's relatively low potassium concentration, it is known to
cause hyponatremia. Since this patient has symptomatic hyponatremia, further decreases in her sodium concentrations could be catastrophic.

Beginning potassium supplementation (Choice 0) may be useful for this patient, as she has slightly low concentrations of potassium. This decreased concentration of potassium is most likely secondary to
her diuretic use. Thus, discontinuing hydrochlorothiazide, the use of which is likely leading to her symptomatic hyponatremia and asymptomatic hypokalemia, may improve both concentrations.

Educational Objective: Symptoms of hyponatremia are dependent on the severity of depletion and the acuity of onset. Acute onset leads to more severe symptoms, such as seizure and coma, while chronic
onset generally has milder symptoms, such as nausea, vomiting, fatigue, and confusion. Diuretics are a common cause of symptomatic hyponatremia, and, thus, patients should have sodium concentrations
monitored after initiation. If hyponatremia develops, the diuretic should be discontinued.

https://t.me/USMLENBME2CK ti
Previous Next Score Report Lab Values Calculator Help pause
Exam Section 4: Item 27 of 50 National Board of Medical Examiners
Comprehensive Clinical Science Self-Assessment

✓ 27. A 15-year-old girl is brought to the office by her mother because of a 3-day history of pain with urination and an increasingly frequent and urgent need to urinate. She has had three episodes of urinary
tract infection during the past year. She has no other history of serious illness and currently takes no medications. The patient appears subdued and avoids eye contact. The physician asks the mother
to leave the office for the remainder of the interview, and she initially is reluctant. When she leaves the room, the physician asks the patient about her sexual history. The patient appears anxious and
asks, "If I answer these questions, will you promise not to tell my mom?" Which of the following is the most appropriate physician response?

A) Ask the patient why she is concerned about confidentiality


B) Empathize with the patient's desire to keep her sexual history private
C) Inform the patient that her information will be shared only if her safety or that of others is at risk
D) Tell the patient that it is worth risking her mother's disapproval to ensure proper treatment of her condition
E) Reassure the patient that the need to break confidentiality about her health is unlikely
Correct Answer: C.

The physician is obligated to protect the patient's confidentiality, even if the patient is an adolescent. Exceptions to adolescent confidentiality are permitted in situations of child abuse, suicidality, homicidal
ideation, certain cases related to sexually transmitted infections, mental health care, and, in some states, when a patient consents to an abortion. Each state has varying statutes regarding confidentiality;
physicians should be familiar with the statutes in their area. This patient reports symptoms of frequent urinary tract infections, which may indicate consensual sexual activity or sexual abuse. The physician
has an obligation to promote the patient's best interests by inquiring about sexual activity or potential abuse while also maintaining honesty about the limits of confidentiality.

Incorrect Answers: A, B, D, and E.

Asking the patient why she is concerned about confidentiality (Choice A) is likely to make the patient defensive and will reduce the likelihood of obtaining truthful information.

Empathizing with the patient's desire to keep her sexual history private (Choice B) is an important component of fostering a therapeutic relationship but does not directly address the patient's question about
confidentiality.

Telling the patient that it is worth risking her mother's disapproval to ensure proper treatment of her condition (Choice D) assumes that the patient's hesitance to disclose her sexual history is because of her
mother's potential disapproval, which may not be the case.

Reassuring the patient that the need to break confidentiality about her health is unlikely (Choice E) is inappropriate, as the physician does not yet possess adequate information to make this determination. In
the setting of sexually transmitted infections or child abuse it may be necessary to break the patient's confidentiality, and the patient must be informed of this possibility.

Educational Objective: Confidentiality must be maintained, even for a minor, unless it applies to child abuse, suicidality, homicidal ideation, some cases of sexually transmitted infections, mental health care,
and, in some states, when a patient consents to an abortion. Patients who inquire about confidentiality should be informed about these limitations.

https://t.me/USMLENBME2CK ti
Previous Next Score Report Lab Values Calculator Help pause
Exam Section 4: Item 28 of 50 National Board of Medical Examiners
Comprehensive Clinical Science Self-Assessment

"I 28. A 37-year-old man with a2-year history of intermittent right knee pain comes to the office because of a 2-month history of increased knee pain that began after he collided with another person while
skiing. The pain has particularly worsened during the past 2 days and has made bearing weight on his right leg difficult. Immediately following the collision, he felt a sharp pain in the middle of his
kneecap and sustained several scrapes and cuts on his kneecap, which stopped bleeding within minutes but have continued to slowly ooze a clear discharge thereafter. Medical history otherwise is
noncontributory. His only medications are a daily multivitamin and occasional acetaminophen for pain. His temperature is 38°C (100.4F), pulse is 76/min, and blood pressure is 110/60 mm Hg.
Examination shows an erythematous right knee that is warm to touch. The patella is ballotable, and there is moderate Joint effusion. Bending the right knee 10 to 15 degrees causes pain; the patient
cannot tolerate bending ii any further. Which of the following is most likely diagnosis?

A) Anterior cruciate ligament tear


B) Meniscal tear
C) Patellar tendonitis
D) Pyogenic arthritis
E) Synovial chondromatosis
Correct Answer: D.

Pyogenic, or septic, arthritis classically presents with a painful, erythematous, swollen Joint along with systemic signs and symptoms of illness, such as fever, chills, myalgia, arthralgia, and nausea. Septic
arthritis results from a bacterial infection of the synovial Joint space, most commonly by Staphylococcus aureus. This patient's septic arthritis was likely caused by traumatic arthrotomy that permitted
introduction of bacteria and explains the persistent fluid leakage (likely synovial fluid). On physical examination, the affected joint shows swelling, erythema, and pain with passive range of motion.
Arthrocentesis is indicated for the evaluation of monoarticular erythema and swelling to evaluate for septic arthritis, which characteristically shows purulent synovial fluid with greater than 50,000
leukocytes/mm. Serum studies may also show leukocytosis and increased inflammatory markers, such as C-reactive protein and erythrocyte sedimentation rate. Treatment involves urgent intravenous
antibiotics and source control, with patients often requiring irrigation of the joint in the operating room to treat infection and limit further acute and chronic complications.

Incorrect Answers: A, B, C, and E.

Anterior cruciate ligament tear (Choice A) typically presents with acute knee pain and swelling and inability to bear weight immediately after the inciting injury. This patient's knee pain, erythema, and fever is
more likely secondary to septic arthritis. Anterior cruciate ligament tears are suspected with anterior knee laxity (determined using the anterior drawer test) and confirmed on MRI.

Meniscal tear (Choice B) can occur with rotation or twisting of the knee while changing direction. Patients typically present with subacute or chronic knee pain and reports of a popping, locking, or catching
sensation in the knee. On physical examination, patients may have joint line tenderness, abnormal knee motion, or inability to fully extend the knee. The McMurray test, when positive, supports the
diagnosis. MRI is confirmatory.

Patellar tendonitis (Choice C), or inflammation of the patellar tendon, can occur as an overuse injury and presents with pain inferior to the patella during physical activity, climbing stairs, or rising from a chair.
It would not likely present with a joint effusion or fever.

Synovial chondromatosis (Choice E) is a rare disease in which the synovium produces nodules of cartilage. This can cause early osteoarthritis and Joint pain, but a patient presenting with fever and joint
effusion is more likely experiencing septic arthritis.

https://t.me/USMLENBME2CK ts e t
Previous Next Score Report Lab Values Calculator Help pause
Exam Section 4: Item 28 of 50 National Board of Medical Examiners
Comprehensive Clinical Science Self-Assessment
- I • I • - • I • I • • - • 4[4\6 '
4 %

.
Examination shows an erythematous right knee that is warm lo touch. The patella is ballotable, and there is moderate Joint effusion. Bending the right knee 10 to 15 degrees causes pain; the patient
cannot tolerate bending it any further. Which of the following is most likely diagnosis?

A) Anterior cruciate ligament tear


B) Meniscal tear
C) Patellar tendonitis
D) Pyogenic arthritis
E) Synovial chondromatosis
Correct Answer: D.

Pyogenic, or septic, arthritis classically presents with a painful, erythematous, swollen Joint along with systemic signs and symptoms of illness, such as fever, chills, myalgia, arthralgia, and nausea. Septic
arthritis results from a bacterial infection of the synovial Joint space, most commonly by Staphylococcus aureus. This patient's septic arthritis was likely caused by traumatic arthrotomy that permitted
introduction of bacteria and explains the persistent fluid leakage (likely synovial fluid). On physical examination, the affected joint shows swelling, erythema, and pain with passive range of motion.
Arthrocentesis is indicated for the evaluation of monoarticular erythema and swelling to evaluate for septic arthritis, which characteristically shows purulent synovial fluid with greater than 50,000
leukocytes/mm. Serum studies may also show leukocytosis and increased inflammatory markers, such as C-reactive protein and erythrocyte sedimentation rate. Treatment involves urgent intravenous
antibiotics and source control, with patients often requiring irrigation of the joint in the operating room to treat infection and limit further acute and chronic complications.

Incorrect Answers: A, B, C, and E.

Anterior cruciate ligament tear (Choice A) typically presents with acute knee pain and swelling and inability to bear weight immediately after the inciting injury. This patient's knee pain, erythema, and fever is
more likely secondary to septic arthritis. Anterior cruciate ligament tears are suspected with anterior knee laxity (determined using the anterior drawer test) and confirmed on MRI.

Meniscal tear (Choice B) can occur with rotation or twisting of the knee while changing direction. Patients typically present with subacute or chronic knee pain and reports of a popping, locking, or catching
sensation in the knee. On physical examination, patients may have joint line tenderness, abnormal knee motion, or inability to fully extend the knee. The McMurray test, when positive, supports the
diagnosis. MRI is confirmatory.

Patellar tendonitis (Choice C), or inflammation of the patellar tendon, can occur as an overuse injury and presents with pain inferior to the patella during physical activity, climbing stairs, or rising from a chair.
It would not likely present with a joint effusion or fever.

Synovial chondromatosis (Choice E) is a rare disease in which the synovium produces nodules of cartilage. This can cause early osteoarthritis and Joint pain, but a patient presenting with fever and joint
effusion is more likely experiencing septic arthritis.

Educational Objective: Pyogenic, or septic, arthritis classically presents with a painful, erythematous, swollen Joint with purulent joint fluid. Arthrocentesis is confirmatory, and treatment involves antibiotics
and operative source control.

https://t.me/USMLENBME2CK ts e t
Previous Next Score Report Lab Values Calculator Help pause
Exam Section 4: Item 29 of 50 National Board of Medical Examiners°
Comprehensive Clinical Science Self-Assessment

29. A 15-year-old girl is brought to the office because of a 2-week history of a facial rash and fatigue. She has a 9-day history of stiffness and swelling of her fingers on awakening in the morning. She has
no history of serious illness and takes no medications. She appears tired and has mild chest pain with deep inspiration. Temperature is 38.0°C (100.4F), pulse is 94/min and regular, respirations are
24/min, and blood pressure is 112/76 mm Hg. A photograph of the patient's face is shown. There are three nontender ulcers over the soft palate. Diffuse 1- to 2-cm lymph nodes are palpated in the
anterior and posterior cervical chains. The lungs are clear to auscultation. Heart sounds are normal. Examination of the extremities shows mild erythema, tenderness, and swelling of the proximal
interphalangeal and metacarpophalangeal joints, knees, and metatarsophalangeal joints. Urinalysis shows 2+ protein and 1+ blood. Which of the following is most appropriate in determining
management for this patient?

A) Cerebral angiography
B) Echocardiography
C) MRI of the brain
D) Renal biopsy
E) X-rays of the knees
+

« https://t.me/USMLENBME2CK ts e t
Previous Next Score Report Lab Values Calculator Help pause
Exam Section 4: Item 29 of 50 National Board of Medical Examiners
Comprehensive Clinical Science Self-Assessment

A 15-year-old girl is brought to the office because of a 2-week history of a facial rash and fatigue. She has a 9-day history of stiffness and swelling of her fingers on awakening in the morning. She has
no history of serious illness and takes no medications. She appears tired and has mild chest pain with deep inspiration. Temperature is 38.0°C (100.4F), pulse is 94/min and regular, respirations are
24/min, and blood pressure is 112/76 mm Hg. A photograph of the patient's face is shown. There are three nontender ulcers over the soft palate. Diffuse 1- to 2-cm lymph nodes are palpated in the
anterior and posterior cervical chains. The lungs are clear to auscultation. Heart sounds are normal. Examination of the extremities shows mild erythema, tenderness, and swelling of the proximal
interphalangeal and metacarpophalangeal joints, knees, and metatarsophalangeal joints. Urinalysis shows 2+ protein and 1+ blood. Which of the following is most appropriate in determining
management for this patient?

A) Cerebral angiography
B) Echocardiography
C) MRI of the brain
D) Renal biopsy
E) X-rays of the knees
Correct Answer: D.

Systemic lupus erythematosus (SLE) is an autoimmune disorder with a wide range of signs and symptoms, including malar rash, photosensitivity, discoid rash, painless oral ulcers, arthritis, serositis
(including pleuritis or pericarditis), kidney impairment, hematologic abnormalities (eg, hemolytic anemia, leukopenia, lymphopenia, thrombocytopenia), an abnormal antinuclear antibody titer, or lupus-specific
autoantibodies (anti-DNA, anti-Smith, antiphospholipid antibody). This patient has numerous of these criteria, suggesting a diagnosis of SLE. lupus nephritis is a complication of SLE and is common in
younger women; it presents with hematuria and increasing creatinine in the setting of SLE. Evaluation of lupus nephritis requires renal biopsy in order to accurately determine the type and extent of renal
involvement. This information is vital for optimizing the treatment of nephritis. For example, immunosuppressive therapy is warranted for active diffuse or focal proliferative lupus nephritis but not mild
mesangial or mesangial proliferative lupus nephritis.

Incorrect Answers: A, B, C, and E.

Cerebral angiography (Choice A) and MRI of the brain (Choice C) are both imaging modalities that may be used to evaluate a patient with neuropsychiatric symptoms caused by SLE. There is no evidence
that this patient has neuropsychiatric symptoms, and this imaging is not currently warranted.

Echocardiography (Choice B) is useful in evaluating for pericardia! effusions. This patient does have pleuritic chest pain suggestive of potential pleuritis but has no abnormalities on cardiac examination. The
more pressing complication of SLE in this case, which requires further diagnostic effort, is lupus nephritis.

X-rays of the knees (Choice E) are a common modality for the evaluation of knee pain. However, this patient has numerous other symptoms suggestive of a diagnosis of SLE. The arthritis of SLE is
nonerosive and nondeforming. II is typically treated with oral nonsteroidal anti-inflammatory drugs, antimalarial medications, or glucocorticoids. Because of this, not only will x-rays of the knee likely be
normal, but they will not change the management of this patient's arthritis.

Educational Objective: Systemic lupus erythematosus is an autoimmune connective tissue disease, one manifestation of which is lupus nephritis. Hematuria and proteinuria with an increasing creatinine are
characteristic of lupus nephritis. Renal biopsy is necessary to accurately determine the type and extent of renal involvement to optimize treatment.

https://t.me/USMLENBME2CK ts e t
Previous Next Score Report Lab Values Calculator Help pause
Exam Section 4: Item 30 of 50 National Board of Medical Examiners°
Comprehensive Clinical Science Self-Assessment

"I 30. A67-year-old woman comes to the physician because of a 6-month history of mild pain in her legs with walking and at rest. Her pain is somewhat relieved when she
elevates her legs. She has a3-year history of intermittent swelling of her legs. She has no history of serious illness and takes no medications. She has smoked
one-half pack of cigarettes daily for 18 years. She has been able to perform her normal daily activities. Cardiopulmonary examination shows no abnormalities. There
is non pitting edema of the lower extremities. A photograph of the right lower extremity is shown. Which of the following is the most appropriate next step in diagnosis
of the lower extremity findings?

A) Echocardiography
B) Lymphangiography
C) Skin biopsy
D) Angiography of the lower extremities
E) No further evaluation is indicated

Correct Answer: E.

Chronic venous insufficiency frequently occurs with age as a result of a combination of venous reflux, incompetent valves, venous hypertension, and decreased muscle function. Risk factors for development
include elderly age, prolonged standing, obesity, tobacco use, and a family history of venous disease. Patients may be asymptomatic but when present, symptoms include leg heaviness, pain, and cramping.
Physical examination may show varicose, or dilated, veins, nonpitting edema, and lower extremity discoloration, as well as ulceration if the insufficiency is severe. Diagnosis is clinical, although venous
duplex ultrasonography can be used if the diagnosis is unclear. Treatment is initially conservative and includes exercise, compression stockings, and leg elevation. Symptomatic patients can undergo venous
ablation, sclerotherapy, or other surgical interventions. This patient has chronic leg pain associated with varicose veins on evaluation, making venous insufficiency the most likely cause. No further evaluation
is indicated.

Incorrect Answers: A, 8, C, and D.

« https://t.me/USMLENBME2CK ts e t
Previous Next Score Report Lab Values Calculator Help pause
Exam Section 4: Item 30 of 50 National Board of Medical Examiners"
Comprehensive Clinical Science Self-Assessment

Correct Answer: E.

Chronic venous insufficiency frequently occurs with age as a result of a combination of venous reflux, incompetent valves, venous hypertension, and decreased muscle function. Risk factors for development
include elderly age, prolonged standing, obesity, tobacco use, and a family history of venous disease. Patients may be asymptomatic but when present, symptoms include leg heaviness, pain, and cramping.
Physical examination may show varicose, or dilated, veins, nonpitting edema, and lower extremity discoloration, as well as ulceration if the insufficiency is severe. Diagnosis is clinical, although venous
duplex ultrasonography can be used if the diagnosis is unclear. Treatment is initially conservative and includes exercise, compression stockings, and leg elevation. Symptomatic patients can undergo venous
ablation, sclerotherapy, or other surgical interventions. This patient has chronic leg pain associated with varicose veins on evaluation, making venous insufficiency the most likely cause. No further evaluation
is indicated.

Incorrect Answers: A, B, C, and D.

Echocardiography (Choice A) is ultrasonography of the heart used to identify structural abnormalities. This patient has no concerning cardiac symptoms or signs of cardiac dysfunction on physical
examination. Echocardiography is not used in the diagnosis of venous insufficiency and is therefore not warranted unless concomitant heart failure is suspected.

Lymphangiography (Choice B) is used to evaluate the lymphatic system. It can be used to determine whether cancer has metastasized to nearby lymph nodes or in the diagnosis of lymphedema. This
patient has signs of venous insufficiency rather than lymphedema, making lymphangiography an inappropriate next step in diagnosis.

Skin biopsy (Choice C) is most frequently used in the diagnosis of skin cancer. Most commonly, skin cancer presents with abnormal skin changes such as discoloration and bleeding over a small area with
previous sun damage. Skin biopsy can also be used in the diagnosis of vasculitides, although this is less common. This patient has dilated, varicose veins, indicative of venous insufficiency rather than skin
cancer or vasculitis.

Angiography of the lower extremities (Choice D) is not necessary for the diagnosis of venous insufficiency. It is more commonly used to evaluate the arterial system, especially in the diagnosis of peripheral
artery disease or lower extremity aneurysms. This patient has no signs of arterial disease, making angiography unwarranted.

Educational Objective: Chronic venous insufficiency is common in older patients and presents with lower extremity heaviness, pain, and cramping. Physical examination may show varicose veins, nonpitting
edema, and lower extremity discoloration. Risk factors include elderly age, prolonged standing, obesity, tobacco use, and a family history of venous disease. It is a clinical diagnosis, and the treatment is
supportive.

« https://t.me/USMLENBME2CK ts e t
Previous Next Score Report Lab Values Calculator Help pause
Exam Section 4: ltem 31 of 50 National Board of Medical Examiners
Comprehensive Clinical Science Self-Assessment

✓ 31. Ten days after undergoing left above-the-knee amputation for severe diabetic soft-tissue infection, a78-year-old man in the intensive care unit has noisy, labored breathing. His postoperative course
has been complicated by sepsis, delirium, and respiratory insufficiency. His temperature is 37.5°C (99.5°F), pulse is 72/min, respirations are 9/min, and blood pressure is 100/50 mm Hg. Pulse
oximetry on 3 l of oxygen by nasal cannula shows an oxygen saturation of 92%. He is not oriented to person, place, or time. Breath sounds are coarse and distant bilaterally. The extremities are cool
to the touch and mottled. After discussions with the patient's family, and consistent with the patient's wishes, the family has chosen to provide comfort care. The family remains concerned about the
patient's current loud breathing and secretions. Which of the following is the most appropriate next step in management?

A) Administration of humidified oxygen


B) Administration of morphine
C) Administration of scopolamine
D) Intubation and mechanical ventilation
E) Placement of a nasogastric tube
Correct Answer: C.

Patients in the process of dying frequently experience difficulty with managing airway secretions, which is distressing both for the patient and for family members. Pharmacologic agents may be useful in
reducing the production of airway secretions. Commonly used agents include glycopyrrolate and scopolamine, both of which are anticholinergic agents. Glycopyrrolate may be preferable because of its
inability to cross the blood-brain barrier, limiting adverse central nervous system side effects, such as agitation, delirium, and seizures. Scopolamine is available as a patch, which is advantageous for
patients with compromised swallowing or excess secretions. Nonpharmacologic methods of managing airway secretions during the dying process include suctioning, positioning the patient on their side,
and reducing unnecessary intravenous fluid administration.

Incorrect Answers: A, B, D, and E.

Administration of humidified oxygen (Choice A) is unlikely to be helpful. Excessive secretions are more effectively managed by dehydrating the airway, a strategy that underlies the use of anlicholinergic
agents.

Administration of morphine (Choice B) is helpful for managing pain in dying patients but will not directly address the family's concern regarding the patient's loud breathing and secretions.

Intubation and mechanical ventilation (Choice D) are invasive measures that are inappropriate given the family's wish to provide comfort care.

Placement of a nasogastric tube (Choice E) is invasive and is unlikely to be helpful. Excessive airway secretions occur during the dying process as a result of the patient's loss of consciousness and
reduced drive to cough or clear the throat, rather than gastric reflux.

Educational Objective: Patients in the process of dying frequently experience difficulty with managing airway secretions, which is distressing both for the patient and for family members. Pharmacologic
agents may be useful in reducing the production of airway secretions. Commonly used agents include the anticholinergic agents glycopyrrolate and scopolamine.

https://t.me/USMLENBME2CK ti e
Previous Next Score Report Lab Values Calculator Help pause
Exam Section 4: Item 32 0f 50 National Board of Medical Examiners°
Comprehensive Clinical Science Self-Assessment

✓ 32. A previously healthy 4-year-old boy is brought to the emergency department by his mother on a winter evening because of the sudden
onset of coughing and mild difficulty breathing. His symptoms began late this afternoon when he was playing with other children at a party.
His brother has asthma. The patient's temperature is 37.2°C (99°F), pulse is 92/min, and respirations are 36/min. Physical examination
shows intense intermittent coughing and mild inlercostal retractions. Wheezes are heard bilaterally but are greater on the right. An x-ray of
the chest is shown. Which of the following is the most likely diagnosis?

A) Asthma
B) Congenital lobar emphysema
C) Foreign body aspiration
D) laryngotracheitis
E) Pneumonia

Correct Answer: C.

This patient is presenting with the acute onset of coughing, dyspnea, tachypnea, and wheezing, making aspiration of a foreign body the most likely diagnosis. Foreign body aspiration is common in children,
especially those younger than 2 years old, and may be witnessed or unwitnessed. The most common location of the aspirated object is in the right lung, as the left main bronchus leaves the trachea at
sharper angle. These objects often lead to the symptoms seen in this patient, whereas objects lodged in the trachea or larynx often lead to strider, rapid hypoxia, cyanosis, and even respiratory arrest and
death. Physical examination in patients with a foreign body aspiration may disclose strider and wheezing with decreased air movement. Evaluation often includes a chest x-ray, which will frequently show
hyperinflation on the side of the foreign body with a mediastinal shift towards the contralateral side, as seen in this patient. However, most commonly, the aspirated object is not identified on chest x-ray as ii
is frequently a radiolucent object. Thus, a high index of suspicion is needed to make the diagnosis. Treatment includes removal of the foreign body with bronchoscopy.

Incorrect Answers: A, B, D, and E.

Asthma (Choice A) commonly presents with chronic cough and wheezing on physical examination caused by reactive airways. Acute exacerbations can cause significant respiratory distress leading to
hypoxia and hospitalization. However, asthma is generally associated with hyperinflated lungs bilaterally, rather than the unilateral hyperinflation seen in this patient.

Congenital lobar emphysema (Choice B) generally presents by age 6 months with respiratory distress, tachypnea, and cyanosis, as well as wheezing and decreased breath sounds on physical
examination. It causes hyperinflation of a lobe of the lung, leading to mediaslinal shift and the symptoms seen. While the chest x-ray findings may be similar to those seen in this patient, congenital lobar
emphysema would be unlikely to present acutely at age 4 years.

laryngotracheitis (Choice D), or croup, presents with rhinorrhea, nasal congestion, and fever that progresses to strider and a barking cough, most commonly as a result of a viral respiratory illness.
ontewe are caowwelarrte narreuire pf#le euhelettie air uhiel ova he caanew ohaetv_roe leneuun ae the "teaenle cir rs,le natiant elee¢ net late a harline oeweh cteieler ee the "etaanle eieu" pr

« https://t.me/USMLENBME2CK ts e t
Previous Next Score Report Lab Values Calculator Help pause
Exam Section 4: Item 32 of 50 National Board of Medical Examiners"
Comprehensive Clinical Science Self-Assessment

D) laryngotracheitis
E) Pneumonia

Correct Answer: C.

This patient is presenting with the acute onset of coughing, dyspnea, tachypnea, and wheezing, making aspiration of a foreign body the most likely diagnosis. Foreign body aspiration is common in children,
especially those younger than 2 years old, and may be witnessed or unwitnessed. The most common location of the aspirated object is in the right lung, as the left main bronchus leaves the trachea at
sharper angle. These objects often lead to the symptoms seen in this patient, whereas objects lodged in the trachea or larynx often lead to strider, rapid hypoxia, cyanosis, and even respiratory arrest and
death. Physical examination in patients with a foreign body aspiration may disclose strider and wheezing with decreased air movement. Evaluation often includes a chest x-ray, which will frequently show
hyperinflation on the side of the foreign body with a mediastinal shift towards the contralateral side, as seen in this patient. However, most commonly, the aspirated object is not identified on chest x-ray as ii
is frequently a radiolucent object. Thus, a high index of suspicion is needed to make the diagnosis. Treatment includes removal of the foreign body with bronchoscopy.

Incorrect Answers: A, B, D, and E.

Asthma (Choice A) commonly presents with chronic cough and wheezing on physical examination caused by reactive airways. Acute exacerbations can cause significant respiratory distress leading to
hypoxia and hospitalization. However, asthma is generally associated with hyperinflated lungs bilaterally, rather than the unilateral hyperinflation seen in this patient.

Congenital lobar emphysema (Choice B) generally presents by age 6 months with respiratory distress, tachypnea, and cyanosis, as well as wheezing and decreased breath sounds on physical
examination. It causes hyperinflation of a lobe of the lung, leading to mediaslinal shift and the symptoms seen. While the chest x-ray findings may be similar to those seen in this patient, congenital lobar
emphysema would be unlikely to present acutely at age 4 years.

laryngotracheitis (Choice D), or croup, presents with rhinorrhea, nasal congestion, and fever that progresses to strider and a barking cough, most commonly as a result of a viral respiratory illness.
Symptoms are secondary to narrowing of the subglottic airway, which can be seen on chest x-rays, known as the "steeple sign." This patient does not have a barking cough, strider, or the "steeple sign" on
chest x-ray, making this diagnosis unlikely.

Pneumonia (Choice E) commonly presents with fever, dyspnea, and a cough productive of purulent sputum. Chest x-rays will show lobar consolidation or infiltrates depending on the cause of the
pneumonia. This patient has no fever or signs of pneumonia on chest x-ray, making this a less likely diagnosis than the aspiration of a foreign body.

Educational Objective: Foreign body aspiration is common in children and presents with the acute onset of coughing, dyspnea, and tachypnea. Physical examination frequently discloses strider, wheezing,
and decreased breath sounds. Chest x-rays will show hyperinflation on the side of the foreign body with a mediastinal shift toward the contralateral side. Treatment includes removal of the foreign body with
bronchoscopy.

« https://t.me/USMLENBME2CK ts e t
Previous Next Score Report Lab Values Calculator Help pause
Exam Section 4: ltem 33 of 50 National Board of Medical Examiners
Comprehensive Clinical Science Self-Assessment

"I 33. A 62-year-old man comes to the office with his wife for a follow-up examination 4 weeks after he was hospitalized for management of an acute myocardial infarction that required cardiac
catheterization. He has had a 1.4-kg (3-lb) weight gain since discharge. He has not had shortness of breath or chest pain on exertion. He has been sleeping well. His medications are losartan and
atorvastatin; he is intermittently adherent to his regimen. He has had irregular attendance at an outpatient cardiac rehabilitation program; he says he sometimes skips sessions because he is tired. His
wife says he is irritable and often talks of his father who died of a heart attack at the age of 65 years. The couple has not resumed sexual activity because of the patient's fear of having another heart
attack. The patient enjoys seeing his grandchildren but is no longer interested in playing poker with his friends. He is 168 cm (5 ft 6 in) tall and weighs 85 kg (187 lb); BMI is 30 kg/m?. His pulse is
90/min, respirations are 16/min, and blood pressure is 130/84 mm Hg. Physical examination shows no pedal edema. On mental status examination, he describes his mood as "blah" and has a
constricted affect. He reports no suicidal ideation or suicide plan. Which of the following is the most likely diagnosis?

A) Adjustment disorder
B) Generalized anxiety disorder
C) Hypoactive sexual desire disorder
D) Major depressive disorder
E) Post-traumatic stress disorder
Correct Answer: A.

Adjustment disorder refers to the onset of an emotional disturbance within 3 months of an identifiable stressor. The emotional disturbance manifests as marked distress out of proportion to the stressor and a
significant impairment in daily school, work, or social functioning. Patients may internalize or externalize their distress, resulting in symptoms of depression and/or anxiety symptoms, or conduct disturbances
(eg, acting out), respectively. Patients with adjustment disorder do not meet criteria for anxiety, major depressive, or conduct disorders. This patient demonstrates some anxiety and/or depression symptoms
(eg, irritability, loss of interest) and conduct disturbances (eg, skipping cardiac rehabilitation sessions) with a prior identifiable stressor and without meeting the criteria for anxiety, major depressive, or
conduct disorders. Since adjustment disorder is typically self-limited, appropriate treatment includes brief supportive psychotherapy.

Incorrect Answers: B, C, D, and E.

Generalized anxiety disorder (Choice B) is an anxiety disorder that features excessive worrying about a range of topics with associated functional impairment. This patient displays some symptoms of anxiety
about having another heart attack but does not display generalized anxiety over a range of topics.

Hypoactive sexual desire disorder (Choice C) is characterized by deficient or absent sexual thoughts or fantasies, or desire for sexual activity resulting in significant distress, persisting for greater than 6
months, and is not attributable to another medical disorder. This patient does display reduced interest in sexual activity, but this lack of interest is attributable to anxiety about having another heart attack.
Further, he does not meet the time criteria for hypoactive sexual desire disorder.

Major depressive disorder (Choice 0) includes 2 or more weeks of five of the following symptoms: depressed mood, anhedonia (eg, decreased interest in socializing), guilt or worthlessness, difficulty
concentrating, psychomotor retardation, suicidal thoughts, and/or neurovegetative symptoms (eg, decreased energy, sleep disturbance, appetite disturbance). This patient has not met the symptom criteria
for major depressive disorder.

Post-traumatic stress disorder (Choice E) refers to greater than 1 month of symptoms of intrusive thoughts and/or memories of trauma, hyperarousal, avoidance of trauma reminders, and/or negative mood
or thoughts resulting from a traumatic event, such as a life-threatening motor vehicle collision. This patient has undergone a life-threatening event but is not experiencing intrusive thoughts or hyperarousal.

https://t.me/USMLENBME2CK ts e t
Previous Next Score Report Lab Values Calculator Help pause
Exam Section 4: ltem 33 of 50 National Board of Medical Examiners
Comprehensive Clinical Science Self-Assessment

. - - -. - -- .- - -. - . -- - .- -.-- ------- . ' .


90/min, respirations are 16/min, and blood pressure is 130/84 mm Hg. Physical examination shows no pedal edema. On mental status examination, he describes his mood as "blah" and has a
constricted affect. He reports no suicidal ideation or suicide plan. Which of the following is the most likely diagnosis?

A) Adjustment disorder
B) Generalized anxiety disorder
C) Hypoactive sexual desire disorder
D) Major depressive disorder
E) Post-traumatic stress disorder
Correct Answer: A.

Adjustment disorder refers to the onset of an emotional disturbance within 3 months of an identifiable stressor. The emotional disturbance manifests as marked distress out of proportion to the stressor and a
significant impairment in daily school, work, or social functioning. Patients may internalize or externalize their distress, resulting in symptoms of depression and/or anxiety symptoms, or conduct disturbances
(eg, acting out), respectively. Patients with adjustment disorder do not meet criteria for anxiety, major depressive, or conduct disorders. This patient demonstrates some anxiety and/or depression symptoms
(eg, irritability, loss of interest) and conduct disturbances (eg, skipping cardiac rehabilitation sessions) with a prior identifiable stressor and without meeting the criteria for anxiety, major depressive, or
conduct disorders. Since adjustment disorder is typically self-limited, appropriate treatment includes brief supportive psychotherapy.

Incorrect Answers: B, C, D, and E.

Generalized anxiety disorder (Choice B) is an anxiety disorder that features excessive worrying about a range of topics with associated functional impairment. This patient displays some symptoms of anxiety
about having another heart attack but does not display generalized anxiety over a range of topics.

Hypoactive sexual desire disorder (Choice C) is characterized by deficient or absent sexual thoughts or fantasies, or desire for sexual activity resulting in significant distress, persisting for greater than 6
months, and is not attributable to another medical disorder. This patient does display reduced interest in sexual activity, but this lack of interest is attributable to anxiety about having another heart attack.
Further, he does not meet the time criteria for hypoactive sexual desire disorder.

Major depressive disorder (Choice 0) includes 2 or more weeks of five of the following symptoms: depressed mood, anhedonia (eg, decreased interest in socializing), guilt or worthlessness, difficulty
concentrating, psychomotor retardation, suicidal thoughts, and/or neurovegetative symptoms (eg, decreased energy, sleep disturbance, appetite disturbance). This patient has not met the symptom criteria
for major depressive disorder.

Post-traumatic stress disorder (Choice E) refers to greater than 1 month of symptoms of intrusive thoughts and/or memories of trauma, hyperarousal, avoidance of trauma reminders, and/or negative mood
or thoughts resulting from a traumatic event, such as a life-threatening motor vehicle collision. This patient has undergone a life-threatening event but is not experiencing intrusive thoughts or hyperarousal.

Educational Objective: Adjustment disorder refers to the onset of disproportionate distress within 3 months of an identifiable stressor and may be associated with anxiety, major depressive, and/or conduct
disturbances, without meeting the full diagnostic criteria for these disorders.

https://t.me/USMLENBME2CK ts e t
Previous Next Score Report Lab Values Calculator Help pause
Exam Section 4: Item 34 of 50 National Board of Medical Examiners
Comprehensive Clinical Science Self-Assessment

✓ 34. A 17-year-old primigravid patient at 22 weeks' gestation comes to the physician because of a 12-hour history of fever, nausea, and vomiting. She has abdominal tenderness on the right side when
she coughs. Her temperature is 38.3C (101 °F). Pelvic and rectal examinations show right-sided tenderness. Fetal heart tones are normal. Her leukocyte count is 18,000/mm3 with a shift to the left.
Her serum amylase activity is within the reference range. During the next 8 hours, vomiting, fever, and abdominal pain continue. Ultrasonography of the gallbladder shows no abnormalities. Which of
the following is the most appropriate next step in diagnosis?

A) Barium enema
B) Gallium scan
C) Laparoscopy
D) Peritoneal lavage
E) Amniocentesis
Correct Answer: C.

Appendicitis presents classically with right lower quadrant abdominal pain that is often periumbilical at onset, nausea, anorexia, fever, leukocytosis, and tenderness to palpation in the right lower quadrant
on examination. McBurney point, a sensitive location for tenderness on palpation, is located at two-thirds the distance between the umbilicus and the inguinal ligament, because this is the common position
of the appendix. In pregnancy, the location of the enlarging uterus may cause the appendix to be displaced superiorly as compared to that of a nongravid patient. This displacement limits the classic history
and physical examination, which rely on a right lower quadrant position of the appendix. When displaced cephalad, parietal peritoneal inflammation may no longer occur at the McBurney point, even
migrating as cephalad as the right upper quadrant, mimicking diagnoses such as cholecystitis. Diagnosis is facilitated by CT scan (though less ideal in pregnancy due to the risk for teratogenicity to the
fetus), MRI, or ultrasonography, which may show appendiceal enlargement, the presence of an appendicolith, appendiceal wall thickening, or periappendiceal fluid or fat stranding. Fetal heart tracings are
usually reassuring unless maternal hemodynamics have been compromised, and fetal tachycardia may result from response to maternal fever. Treatment is with urgent laparoscopic appendectomy lo
prevent complications such as appendiceal rupture, sepsis, preterm labor, and death.

Incorrect Answers: A, B, D, and E.

Barium enema (Choice A) is often used to evaluate abnormalities of the rectum and colon, such as those seen in inflammatory bowel disease or colon cancer. The barium used allows the soft tissues of the
gastrointestinal tract to be more clearly defined on x-rays. Generally, x-rays are avoided in pregnant patients given the risk for radiation exposure to the fetus.

Gallium scan (Choice B) can be used lo evaluate infection or inflammation throughout the body. It does utilize radiation, which should be avoided in pregnant patients, and is not specific for appendicitis.

Peritoneal lavage (Choice D) is an uncommon technique used to determine if there is a source of intra-abdominal bleeding, especially in trauma patients. If done in pregnant patients, it is often through an
open supraumbilical incision. In the setting of infection, such as in this patient, it is nonspecific and unlikely to reveal the diagnosis.

Amniocentesis (Choice E) is most often used in the evaluation of genetic abnormalities of the fetus. It is not warranted at this time, as this patient has no signs of fetal distress or any symptoms of obstetric
or gynecologic abnormalities on history or physical examination.

Educational Objective: Appendicitis presents classically with right lower quadrant abdominal pain that is often periumbilical at onset, nausea, anorexia, fever, leukocytosis, and tenderness lo palpation in the
right lower quadrant on examination. The appendix may be displaced superiorly in pregnancy as a result of the gravid uterus, and appendicitis may mimic other disorders such as cholecystitis. The best
option for diagnosis is ultrasonography or MRI, as these limit the radiation exposure to the fetus. Treatment with urgent laparoscopic appendectomy is required.

https://t.me/USMLENBME2CK ts e t
Previous Next Score Report Lab Values Calculator Help pause
Exam Section 4: ltem 35 of 50 National Board of Medical Examiners
Comprehensive Clinical Science Self-Assessment

"I 35. An 18-year-old man comes to the clinic because of a 3-hour history of right eye pain with mild light sensitivity and blurry vision that have been present since he awakened this morning. He has had no
symptoms in his left eye. He reports being struck in the face by a soccer ball last night but having no symptoms at that time. Medical history is remarkable for admission to the hospital 2 years ago for
acute chest syndrome secondary to sickle cell disease; he has taken hydroxyurea since that time. He takes no other medications and does not wear corrective lenses. Family history is remarkable for
glaucoma in his maternal grandfather. The patient's vital signs are within normal limits. Examination of the right eye shows blood filling the lower 20% of the anterior chamber. Pupils are equal and
reactive to light and accommodation bilaterally, and visual fields are intact. Visual acuity is 20/60 in the right eye and 20/20 in the left eye. lntraocular pressure is 27 mm Hg in the right eye and 18 mm
Hg in the left eye (N=12--22). Which of the following factors in this patient most strongly increases his risk for complications of his current condition?

A) Age
B) Family history
C) Hydroxyurea use
D) Sickle cell disease
Correct Answer: D.

Hyphema, or the accumulation of blood in the anterior chamber (AC), may occur following blunt ocular trauma, surgery, or spontaneously in the selling of van Willebrand disease, hemophilia, leukemia, or
anticoagulant usage. In the setting of trauma, hyphema occurs as a consequence of damage to the AC angle, iris root, or ciliary body, which are richly vascularized. Immediate and subacute complications of
hyphema include ocular hypertension, which may lead to glaucomatous optic nerve damage, rebleeding, and corneal blood staining. Corneal blood staining may significantly impair visual acuity and may
necessitate corneal transplantation if the central cornea is involved. Chronic complications of hyphema are primarily related to glaucomatous damage to the optic nerve. Hyphema is particularly dangerous in
patients with sickle cell disease or trait because of the complications of red blood cell (RSC) sickling. Sickled RBCs in the AC are less likely to be filtered through the trabecular meshwork and are likely to
result in ocular hypertension. Ocular hypertension may then induce sickling of RBCs in retinal vasculature, leading to irreversible vision loss through induction of retinal arterial or venous occlusions. Most
hyphemas may be managed medically through topical drops to lower ocular pressure. In patients with sickle cell disease or trait, surgical AC washout is indicated if intraocular pressure is not controlled
within 24 hours. In contrast, patients without sickle cell disease or trait may tolerate high intraocular pressure for up to 5 days before surgical washout may be considered.

Incorrect Answers: A, B, and C.

Age (Choice A) may play a role in the incidence of hyphema as a result of the increased rate of ocular trauma in young patients. However, age does not affect the rate of complications once hyphema has
occurred.

Family history (Choice B) may be indicative of hereditary disorders, such as sickle cell disease or van Willebrand disease. A positive family history of bleeding disorders is not as important for ocular
complications as having sickle cell disease.

Hydroxyurea use (Choice C) is useful for the treatment of sickle cell disease by increasing the amount of circulating fetal hemoglobin. Its use does not lead to a significantly increased rate of intraocular
bleeding.

Educational Objective: Hyphema, or the accumulation of blood in the anterior chamber, is common following blunt ocular trauma and may lead to significant vision loss as a result of ocular hypertension and
glaucoma, corneal blood staining, and angle recession. While most patients may tolerate high intraocular pressure secondary to hyphema for up to 5 days before surgical washout becomes necessary,
patients with sickle cell disease or trait are at much higher risk for anterior chamber and retinal vascular red blood cell sickling, which may lead to irreversible vision loss caused by retinal vascular occlusion.
Surgical washout should be considered urgently for these patients if intraocular pressure is not controlled within 24 hours.

https://t.me/USMLENBME2CK ti e
Previous Next Score Report Lab Values Calculator Help pause
Exam Section 4: Item 36 of 50 National Board of Medical Examiners
Comprehensive Clinical Science Self-Assessment

A previously healthy 16-year-old boy comes to the physician because of constant right-sided scrotal pain for 36 hours. He had sexual intercourse with a new partner 1 week ago. His temperature is
38°C (100.4°F), Abdominal examination shows no abnormalities. There is a scant penile discharge. The right scrotum is swollen and tender, especially posteriorly. The pain is less severe when the
scrotum is elevated. Urinalysis and Gram stain of the urethral discharge show leukocytes. Which of the following is the most appropriate next step in management?

A) Semen analysis
B) Transillumination of the scrotum
C) Doppler ultrasonography
D) CT scan
E) Antibiotic therapy
F) Operative procedure
Correct Answer: E.

Epididymitis presents with fever, dysuria, urethral discharge, and tenderness, erythema, and fullness of the scrotum. Men between the second to fourth decade of life are most commonly affected. The most
common causative organism is Chlamydia trachomatis, followed by Neisseria gonorrhoeae. Characteristic clinical features of epididymitis include preservation of the cremasteric reflex (in contrast to its
absence in the setting of testicular torsion); a positive Prehn sign, in which testicular pain is alleviated by elevation of the testis; and normal Doppler ultrasonography findings of the testis (though
hypervascularity may be noted in cases of epididymo-orchilis). Doppler ultrasonography is most useful when testicular torsion is suspected; however, this patient's presentation is consistent with epididymitis,
so the most appropriate next step is to initiate antibiotic therapy to prevent potential spread of sexually transmitted infections and to ease symptoms. Most patients experience resolution of symptoms with
antibiotics, including azithromycin and ceflriaxone, and symptomatic relief with nonsteroidal anti-inflammatory drugs, rest, scrotal support, and ice. Follow-up nucleic acid amplification testing is needed to
identify causative organisms, such as C. trachomatis or N. gonorrhoeae, and screening for concomitant sexually transmitted infections, such as syphilis or HIV. Men over age 35 years or those who have
unprotected anal insertional intercourse are more commonly infected by Escherichia coli as opposed to C. trachomatis or N. gonorrhoeae.

Incorrect Answers: A, B, C, D, and F.

Semen analysis (Choice A) is an appropriate first step in the evaluation of infertility for a male patient. It does not play a role in the evaluation of epididymitis and acute scrotal pain.

Transillumination of the scrotum (Choice B) is useful for the diagnosis of a scrotal hydrocele, an accumulation of serous fluid within the tunica vaginalis, which typically presents with painless scrotal
enlargement.

Doppler ultrasonography (Choice C) is useful for the diagnosis of testicular torsion and may show a lack of testicular perfusion but may not be useful for cases of suspected epididymitis and would not alter
the course of treatment. An operative procedure (Choice F), typically with orchiopexy, is necessary for the treatment of testicular torsion in order to prevent testicular infarction. This patient's history of a
recent, new sexual partner and penile discharge are suggestive of epididymitis rather than torsion.

CT scan (Choice D) is useful for the diagnosis of various pelvic processes, although is inadequate for the evaluation of acute scrotal pathology.

Educational Objective: Epididymitis presents with fever, dysuria, urethral discharge, and tenderness, erythema, and fullness of the scrotum, most frequently caused by infection with Chlamydia trachomatis or
Neisseria gonorrhoeae. Characteristic clinical features of epididymitis include preservation of the cremasteric reflex, a positive Prehn sign, and normal Doppler ultrasonography findings of the testis.
Immediate treatment with antibiotics, including azilhromycin and ceflriaxone, is most appropriate in cases of suspected epididymitis.

https://t.me/USMLENBME2CK ts e t
Previous Next Score Report Lab Values Calculator Help pause
Exam Section 4: Item 37 of 50 National Board of Medical Examiners°
Comprehensive Clinical Science Self-Assessment

"I 37. A 52-year-old woman comes to the physician because of a 1-week history of increasingly severe pain and swelling of her right knee. She
now is unable to walk because of the symptoms. Five years ago, she fell and hit the knee on the bathroom floor. She has hypertension
treated with lisinopril. Her temperature is 36.9"C (98.4F), pulse is 84/min, respirations are 20/min, and blood pressure is 132/76 mm Hg. The
right knee is erythematous, swollen, and warm. There is diffuse tenderness to light touch; passive and active range of motion is limited by
pain. There is no crepitus, but ballottement shows moderate knee effusion. Arthrocentesis is performed, and joint fluid analysis shows a
leukocyte count of 50,000/mm3. A Gram stain is negative; rhomboid-shaped crystals are present. An x-ray of the knee is shown. Which of the
following is the most appropriate pharmacotherapy?

A) Febuxostat
B) Ibuprofen
C) lnfliximab
D) Methotrexate
E) Penicillamine

Correct Answer: B.

Pseudogout results from calcium pyrophosphate crystalline deposition within the joint space and may manifest acutely following trauma, surgery, or exertion. Crystalline deposition provokes an inflammatory
response leading to the classic presentation of joint pain, erythema, effusion, calor, and reduced range of motion. Pseudogout most commonly affects the knee. Arthrocentesis is diagnostic, as it allows for
the sampling of synovial fluid, which shows the presence of calcium pyrophosphate crystals and an increased concentration of leukocytes. Additionally, arthrocentesis is necessary to rule out the presence of
septic arthritis or hemarthrosis given this patient's presentation of monoarticular erythema and swelling. These competing diagnoses cannot be excluded on clinical features alone. Plain film x-rays may show
chondrocalcinosis and degenerative joint changes. First-line medical management includes nonsteroidal anti-inflammatory drugs, such as ibuprofen, and colchicine.

Incorrect Answers: A, C, D, and E.

Febuxostat (Choice A) is used to treat hyperuricemia, such as in gout. Xanthine oxidase inhibitors, such as allopurinol or febuxostat, reduce synthesis of uric acid.

lnfliximab (Choice C) is a monoclonal antibody used to treat autoimmune diseases, such as rheumatoid arthritis. Methotrexate (Choice 0) is an immunosuppressive medication used in the treatment of
various disorders, most notably rheumatoid arthritis, and lymphoma, that competitively inhibits dihydrofolate reductase. Neither are indicated in the treatment of pseudogout.

Penicillamine (Choice E) is used for treatment of rheumatoid arthritis, although it is primarily used to treat hepatolenticular degeneration (Wilson disease).

Educational Objective: Pseudogout results from calcium pyrophosphate crystalline deposition within the Joint space and may manifest acutely following trauma, surgery, or exertion. The resulting
inflammatory response leads to the classic presentation of joint pain, erythema, effusion, calor, and reduced range of motion. Treatment involves nonsteroidal anti-inflammatory drugs.

« https://t.me/USMLENBME2CK ts e t
Previous Next Score Report Lab Values Calculator Help pause
Exam Section 4: ltem 38 of 50 National Board of Medical Examiners
Comprehensive Clinical Science Self-Assessment

"I 38. A 28-year-old woman comes to the office for a follow-up examination 2 weeks after undergoing a total thyroidectomy because of a thyroid nodule. Immediately after the operation, the patient had the
onset of marked hypocalcemia and was admitted to the intensive care unit. During her hospital stay, intravenous calcium and calcitriol were administered. Today, she has had recurrent numbness
around her mouth and muscle cramping. Current medications are calcium carbonate three times daily with meals and calcitriol twice daily. Vital signs are within normal limits. Physical examination
shows spasm of the ipsilateral facial muscles when tapping 2 cm anterior to the ears on the zygomatic arch bilaterally. The remainder of the examination shows no abnormalities. laboratory studies
are most likely to show an increase in which of the following in this patient?

A) Serum calcium concentration


B) Serum magnesium concentration
C) Serum parathyroid hormone concentration
D) Serum parathyroid hormone-related protein concentration
E) Urinary 24-hour calcium excretion
F) Urine potassium excretion
Correct Answer: E.

Hypocalcemia is a common complication following thyroidectomy as a result of resection of the parathyroid glands. Parathyroid hormone (PTH), along with calcitriol (1,25-hydroxyvitamin-D), regulates
calcium homeostasis. PTH is secreted in response to low serum calcium concentrations and acts to raise the concentration by increasing bone resorption, promoting production of calcitriol to increase
intestinal absorption of calcium, and increasing calcium resorption in the renal tubules. Patients with hypoparathyroidism will have an increase in urinary 24-hour calcium excretion as a result of the loss of
resorption stimulation in the kidneys, requiring daily calcium supplementation. Symptoms of hypocalcemia include perioral paresthesia, carpopedal spasm, laryngospasm, and prolonged QT interval. The
Chvostek sign, in which tapping of the facial nerve leads to spasm of the facial muscles, and Trousseau phenomenon, in which an inflated blood pressure cuff occluding the brachia! artery causes carpopedal
spasm, are two physical examination findings that may indicate hypocalcemia.

Incorrect Answers: A, B, C, D, and F.

Serum calcium concentration (Choice A) is likely to be low in this patient, as she is presenting with symptoms of hypocalcemia at her office visit.

Serum magnesium concentration (Choice B) is likely to be normal in this patient. Hypomagnesemia can be a cause of functional hypoparathyroidism in patients as the gland requires magnesium to produce
pTH

Serum parathyroid hormone concentration (Choice C) is likely to be decreased in this patient and the primary cause of her hypocalcemia.

Serum parathyroid hormone-related protein concentration (Choice D) can be a cause of hypercalcemia associated with malignancy. Patients with hypercalcemia and a low or normal PTH should have their
parathyroid hormone-related protein concentration checked as part of the diagnostic workup.

Increases in urine potassium excretion (Choice F) are often caused by diuretic use, mineralocorticoid excess (as in primary hyperaldosteronism), or loss of gastric secretions, which leads to increased
bicarbonate exchange in the renal tubules.

https://t.me/USMLENBME2CK ti
Previous Next Score Report Lab Values Calculator Help pause
Exam Section 4: ltem 38 of 50 National Board of Medical Examiners
Comprehensive Clinical Science Self-Assessment

A) Serum calcium concentration


B) Serum magnesium concentration
C) Serum parathyroid hormone concentration
D) Serum parathyroid hormone-related protein concentration
E) Urinary 24-hour calcium excretion
F) Urine potassium excretion
Correct Answer: E.

Hypocalcemia is a common complication following thyroidectomy as a result of resection of the parathyroid glands. Parathyroid hormone (PTH), along with calcitriol (1,25-hydroxyvitamin-D), regulates
calcium homeostasis. PTH is secreted in response to low serum calcium concentrations and acts to raise the concentration by increasing bone resorption, promoting production of calcitriol to increase
intestinal absorption of calcium, and increasing calcium resorption in the renal tubules. Patients with hypoparathyroidism will have an increase in urinary 24-hour calcium excretion as a result of the loss of
resorption stimulation in the kidneys, requiring daily calcium supplementation. Symptoms of hypocalcemia include perioral paresthesia, carpopedal spasm, laryngospasm, and prolonged QT interval. The
Chvostek sign, in which tapping of the facial nerve leads to spasm of the facial muscles, and Trousseau phenomenon, in which an inflated blood pressure cuff occluding the brachia! artery causes carpopedal
spasm, are two physical examination findings that may indicate hypocalcemia.

Incorrect Answers: A, B, C, D, and F.

Serum calcium concentration (Choice A) is likely to be low in this patient, as she is presenting with symptoms of hypocalcemia at her office visit.

Serum magnesium concentration (Choice B) is likely to be normal in this patient. Hypomagnesemia can be a cause of functional hypoparathyroidism in patients as the gland requires magnesium to produce
pTH

Serum parathyroid hormone concentration (Choice C) is likely lo be decreased in this patient and the primary cause of her hypocalcemia.

Serum parathyroid hormone-related protein concentration (Choice D) can be a cause of hypercalcemia associated with malignancy. Patients with hypercalcemia and a low or normal PTH should have their
parathyroid hormone-related protein concentration checked as part of the diagnostic workup.

Increases in urine potassium excretion (Choice F) are often caused by diuretic use, mineralocorticoid excess (as in primary hyperaldosteronism), or loss of gastric secretions, which leads to increased
bicarbonate exchange in the renal tubules.

Educational Objective: Parathyroid hormone (PTH) is decreased in hypoparathyroidism, which may be caused by the inadvertent removal of one or more parathyroid glands during thyroidectomy. Symptoms
of hypocalcemia, including perioral paresthesia, carpopedal spasm, laryngospasm, prolonged QT interval, Chvostek sign, and Trousseau phenomenon, develop postoperatively. A low concentration of PTH
prevents renal reabsorption of calcium, leading to increased urinary calcium excretion.

https://t.me/USMLENBME2CK ti
Previous Next Score Report Lab Values Calculator Help pause
Exam Section 4: ltem 39 of 50 National Board of Medical Examiners
Comprehensive Clinical Science Self-Assessment

"I 39. A previously healthy 27-year-old woman comes to the physician because of a rash over her chest and back for 5 days. She has a 10-day history of pain with urination, urinary frequency, and nocturia
two to three limes nightly. She took her father's trimethoprim-sulfamethoxazole for 7 days, and her urinary symptoms improved. She currently takes no medications. She does not smoke cigarettes.
Her temperature is 37 .5°C (99.5°F), pulse is 110/min, respirations are 16/min, and blood pressure is 110/70 mm Hg. Examination of the skin shows an erythematous, maculopapular rash over the
trunk. There are no oral lesions. There is no splenomegaly. Laboratory studies show:
Hemoglobin 12.8 g/dL
Leukocyte count 3000/mm3
Segmented neutrophils 15%
Eosinophils 5%
Basophils 1%
Lymphocytes 54%
Monocytes 25%
Platelet count 200,000/mm3

Which of the following is the most appropriate next step in management?

A) Repeat examination in 1 week


B) Granulocyte colony-stimulating factor therapy
C) Prednisone therapy
D) Vitamin B 12 (cyanocobalamin) supplementation
E ) Skin biopsy
Correct Answer: A.

Drug-induced neutropenia is one of the most common idiosyncratic drug reactions reported and a large number of medications are associated with this adverse effect. Neutrophils normally account for
approximately 40% to 60% of the total white blood cell count on a complete blood count. This patient has an abnormally low count in the setting of recent trimethoprim-sulfamethoxazole use for treatment of
a urinary tract infection. She does not have other apparent risk factors or a personal or family history of hematologic disease. The most appropriate next step in management is to repeat examination in 1
week with a repeat complete blood count. Neutropenia in adults is often secondary to drug reactions, infections, malignancy, or direct toxins (including chemotherapy). Antibiotics are commonly implicated in
drug-induced neutropenia. Most cases will resolve within days to weeks after discontinuation of the offending agent.

Incorrect Answers: B, C, D, and E.

Granulocyte colony-stimulating factor therapy (Choice B) is not indicated at this time, as drug-induced neutropenia is expected to resolve with discontinuation of the medication. In unstable or hospitalized
patients, granulocyte colony-stimulating factor therapy is associated with quicker recovery times and shorter duration of hospitalization.

Prednisone therapy (Choice C) is not necessary at this time. Withdrawal of the drug will provide the most benefit. Topical corticosteroids and antihistamines can be used to treat the skin eruption if
bothersome lo the patient.

https://t.me/USMLENBME2CK ti e
Previous Next Score Report Lab Values Calculator Help pause
Exam Section 4: ltem 39 of 50 National Board of Medical Examiners
Comprehensive Clinical Science Self-Assessment

Monocytes 25%
Platelet count 200,000/mm3

Which of the following is the most appropriate next step in management?

A) Repeat examination in 1 week


B) Granulocyte colony-stimulating factor therapy
C) Prednisone therapy
D) Vitamin B 12 (cyanocobalamin) supplementation
E ) Skin biopsy
Correct Answer: A.

Drug-induced neutropenia is one of the most common idiosyncratic drug reactions reported and a large number of medications are associated with this adverse effect. Neutrophils normally account for
approximately 40% to 60% of the total white blood cell count on a complete blood count. This patient has an abnormally low count in the setting of recent trimethoprim-sulfamethoxazole use for treatment of
a urinary tract infection. She does not have other apparent risk factors or a personal or family history of hematologic disease. The most appropriate next step in management is to repeat examination in 1
week with a repeat complete blood count. Neutropenia in adults is often secondary to drug reactions, infections, malignancy, or direct toxins (including chemotherapy). Antibiotics are commonly implicated in
drug-induced neutropenia. Most cases will resolve within days to weeks after discontinuation of the offending agent.

Incorrect Answers: B, C, D, and E.

Granulocyte colony-stimulating factor therapy (Choice B) is not indicated at this time, as drug-induced neutropenia is expected to resolve with discontinuation of the medication. In unstable or hospitalized
patients, granulocyte colony-stimulating factor therapy is associated with quicker recovery times and shorter duration of hospitalization.

Prednisone therapy (Choice C) is not necessary at this time. Withdrawal of the drug will provide the most benefit. Topical corticosteroids and antihistamines can be used to treat the skin eruption if
bothersome to the patient.

Vitamin B4 (cyanocobalamin) supplementation (Choice D) is not likely to benefit the patient at this time. Vitamin B, deficiency presents with megaloblastic anemia and may also manifest neurologic
symptoms. Glossitis is sometimes seen. It is not associated with rash formation.

Skin biopsy (Choice E) is a mildly invasive procedure reserved for persistent symptoms with unexplained cause. In a stable patient with a likely explanation for her symptoms, observation and repeat
laboratory testing is more appropriate.

Educational Objective: Drug-induced neutropenia is a common idiosyncratic drug reaction that can put patients at risk for life-threatening infections. Withdrawal of the agent implicated in the drug
hypersensitivity reaction usually resolves the neutropenia.

https://t.me/USMLENBME2CK ti e
Previous Next Score Report Lab Values Calculator Help pause
Exam Section 4: Item 40 of 50 National Board of Medical Examiners
Comprehensive Clinical Science Self-Assessment

"I 40. A57-year-old woman who was admitted lo the hospital because of pneumonia has had an increase in her serum crealinine concentration from 1 mg/dL lo 2.4 mg/dl during the past 3 days. On
admission 7 days ago, she had severe shortness of breath and hypotension; she was intubated and mechanically ventilated for 2 days. Six days ago, a CT scan of the chest with contrast showed
pneumonia at both lung bases. She also has type 2 diabetes mellitus. Current medications are ceftriaxone, azithromycin, albuterol, insulin, and lisinopril. She is now breathing 3 L/min of oxygen via
nasal cannula and has mild shortness of breath. Her blood pressure has been stable for 6 days. Her urine output has been 1 L/24 h during the past 2 days. Her pulse is 70/min, and blood pressure is
150/90 mm Hg. Pulse oximetry on 3 Umin of oxygen via nasal cannula shows an oxygen saturation of 94%. Crackles are heard at both lung bases. Urinalysis shows:
Protein 1+
RBC 10/hpf
WBC 9/hpf
Casts none

A Wright stain of urine shows eosinophils. Which of the following is the most appropriate next step in management?

A) Add acetylcysteine to the regimen


B) Add nifedipine to the regimen
C) Add prednisone to the regimen
D) Administer a 1-L bolus of 0.9% saline
E) Discontinue ceftriaxone
F ) No intervention is indicated
Correct Answer: E.

Acute interstitial nephritis (AIN) is caused by a hypersensitivity reaction to medications (eg, nonsteroidal anti-inflammatory drugs, diuretics, sulfonamides, rifampin, proton pump inhibitors, antibiotics),
infection, or autoimmune disorders such as sarcoidosis and systemic lupus erythematosus. Patients may be asymptomatic, but common signs, symptoms, and laboratory findings include rash, azotemia,
sterile pyuria, hematuria, and eosinophilia. Patients may have proteinuria, but typically not lo the extent of nephrotic syndrome. Eosinophils are often detected on urinalysis, as shown in this case. AIN can be
complicated by acute kidney injury and a decline in kidney function. Treatment of AIN includes supportive care and discontinuing the causal medication when there is one, in this case, ceftriaxone.

Incorrect Answers: A, B, C, D, and F.

Add acetylcysteine to the regimen (Choice A) is indicated for the treatment of acetaminophen overdose and, in some cases, acute liver injury. There are no features of this patient's history that suggest
current symptoms are induced by acetaminophen or liver injury.

Add nifedipine lo the regimen (Choice B) is not the best next step. Nifedipine is a dihydropyridine calcium-channel blocking medication that is a potent arteriolar vasodilator. It does not have a role in
management of interstitial nephritis.

Add prednisone to the regimen (Choice C) may be beneficial, as steroids have been shown to improve recovery of kidney function in patients with AIN. However, discontinuing the causal medication is the
first step in management.

https://t.me/USMLENBME2CK ts e t
Previous Next Score Report Lab Values Calculator Help pause
Exam Section 4: Item 40 of 50 National Board of Medical Examiners
Comprehensive Clinical Science Self-Assessment

A Wright stain of urine shows eosinophils. Which of the following is the most appropriate next step in management?

A) Add acetylcysteine to the regimen


B) Add nifedipine to the regimen
C) Add prednisone lo the regimen
D) Administer a 1-L bolus of 0.9% saline
E) Discontinue ceftriaxone
F ) No intervention is indicated
Correct Answer: E.

Acute interstitial nephritis (AIN) is caused by a hypersensitivity reaction to medications (eg, nonsteroidal anti-inflammatory drugs, diuretics, sulfonamides, rifampin, proton pump inhibitors, antibiotics),
infection, or autoimmune disorders such as sarcoidosis and systemic lupus erythematosus. Patients may be asymptomatic, but common signs, symptoms, and laboratory findings include rash, azotemia,
sterile pyuria, hematuria, and eosinophilia. Patients may have proteinuria, but typically not to the extent of nephrotic syndrome. Eosinophils are often detected on urinalysis, as shown in this case. AIN can be
complicated by acute kidney injury and a decline in kidney function. Treatment of AIN includes supportive care and discontinuing the causal medication when there is one, in this case, ceftriaxone.

Incorrect Answers: A, B, C, D, and F.

Add acetylcysteine to the regimen (Choice A) is indicated for the treatment of acetaminophen overdose and, in some cases, acute liver injury. There are no features of this patient's history that suggest
current symptoms are induced by acetaminophen or liver injury.

Add nifedipine to the regimen (Choice 8) is not the best next step. Nifedipine is a dihydropyridine calcium-channel blocking medication that is a potent arteriolar vasodilator. It does not have a role in
management of interstitial nephritis.

Add prednisone to the regimen (Choice C) may be beneficial, as steroids have been shown to improve recovery of kidney function in patients with AIN. However, discontinuing the causal medication is the
first step in management.

Administer a 1-L bolus of 0.9% saline (Choice 0) can be helpful if decrease in urine output or acute kidney injury is secondary to dehydration. This patient's kidney injury is likely secondary to interstitial
nephritis and fluids would play a less significant role than discontinuation of the causal medication.

No intervention is indicated (Choice F) is incorrect, as AIN, if left untreated, can lead to acute kidney injury and a decline in kidney function. Discontinuation of any causal medication and supportive care is
necessary.

Educational Objective: Acute interstitial nephritis (AIN) is caused by a hypersensitivity reaction to medications (eg, nonsteroidal anti-inflammatory drugs, diuretics, sulfonamides, rifampin, proton pump
inhibitors, antibiotics), infection, or autoimmune disorders. Treatment of AIN includes supportive care and discontinuing the causal medication.

https://t.me/USMLENBME2CK ti e
Previous Next Score Report Lab Values Calculator Help pause
Exam Section 4: Item 41 of 50 National Board of Medical Examiners
Comprehensive Clinical Science Self-Assessment

"I 41. A40-year-old man comes to the emergency department because of a 1-day history of nausea and vomiting. He has not had fever, chills, abdominal pain, or diarrhea. He says taking a hot shower
alleviates his symptoms. He has no history of serious illness and takes no medications. He says he occasionally uses illicit drugs but refuses to provide any additional information. His temperature is
36.7°C (98°F), pulse is 100/min, respirations are 18/min, and blood pressure is 130/90 mm Hg. Examination shows no other abnormalities. Intoxication from which of the following is the most likely
cause of this patient's current condition?

A) Cannabis
B) Cocaine
C) Heroin
0) LSD
E) Methamphetamine
F) PCP (phencyclidine)
Correct Answer: A.

Cannabis is a hallucinogen that causes euphoria, paranoia, hallucinations, increased appetite, conjunctival injection, and cognitive slowing. Chronic cannabis use can lead to a disorder called cannabis
hyperemesis syndrome, which is characterized by episodic vomiting associated with prolonged, frequent, and high-dose recreational cannabis use. Patients often find relief with hot-water bathing. It typically
resolves after cannabis cessation.

Incorrect Answers: B, C, D, E, and F.

Chronic cocaine abuse (Choice B) may lead to otolaryngologic symptoms such as nasal septal perforation and nosebleeds as well as neuropsychiatric symptoms such as anosmia, cognitive impairment, and
psychotic symptoms (eg, delusions, hallucinations). It is also associated with increased risk for acute coronary syndromes, aortic dissection, and placental abruption.

Heroin (Choice C) and other opiate intoxication causes euphoria, altered mental status, sedation (or stupor, if severe), bradycardia, hypotension, depressed respiratory drive, and constricted pupils.

LSD (Choice 0) is a potent hallucinogen known to cause auditory and visual hallucinations, depersonalization, flashbacks, and paranoia.

Chronic methamphetamine abuse (Choice E) can lead to methamphetamine-induced psychotic disorder, which features chronic delusions, paranoia, and hallucinations. Chronic methamphetamine use may
also lead to cognitive impairment.

PCP (phencyclidine) (Choice F) intoxication is associated with aggression, rotatory nystagmus, and ataxia. Chronic PCP abuse can lead to depression, psychotic symptoms, memory loss, and dysarthria.

Educational Objective: Cannabis hyperemesis syndrome is characterized by episodic vomiting associated with prolonged, frequent, and high-dose recreational cannabis use. Patients often find relief with
hot-water bathing.

https://t.me/USMLENBME2CK ts e t
Previous Next Score Report Lab Values Calculator Help pause
Exam Section 4: Item 42 of 50 National Board of Medical Examiners
Comprehensive Clinical Science Self-Assessment

"I 42. A 62-year-old woman comes to the physician because of a 4-month history of a painless mass in her left breast; the mass has not changed in size. Ten years ago, she had a total abdominal
hysterectomy and bilateral salpingo-oophorectomy. She has hypertension treated with hydrochlorothiazide. Her maternal aunt was diagnosed with breast cancer at the age of 55 years. The patient's
vital signs are within normal limits. Examination shows a 2.5-cm hard, immobile mass in the upper outer quadrant of the left breast. Mammography and fine-needle aspiration biopsy specimen show no
abnormalities. Which of the following is the most appropriate next step in management?

A) Follow-up examination in 3 months


B) Repeat mammography in 6 months
C) MRI of the left breast
D) Tamoxifen therapy
E) Excisional biopsy
Correct Answer: E.

Breast cancer, the most common nondermatologic malignancy in women, typically consists of two main histologic subtypes, ductal or lobular adenocarcinoma, and can present with a breast mass or axillary
lymphadenopathy. Risk factors for development include exogenous estrogen use, family history of breast cancer, genetic mutation (eg, BRCA 1 or 2), early menarche, late menopause, nulliparity, obesity, and
African American ethnicity. In patients over age 30 years with a palpable breast mass or axillary lymphadenopathy, mammography should be the first step in diagnosis. Other radiologic evaluation may
include ultrasonography to identify a lesion suitable for biopsy. However, regardless of imaging findings, biopsy should occur if the physical examination is concerning for breast cancer, as ii is in this patient.
Fine-needle aspiration only has a sensitivity of approximately 74%, as not all samples obtained are adequate for diagnosis. Thus, this patient, with a high-risk lesion on physical examination, should undergo
a more sensitive diagnostic evaluation with excisional biopsy, which would remove the whole mass for analysis, and may be curative if disease remains confined to the lesion.

Incorrect Answers: A, B, C, and D.

Follow-up examination in 3 months (Choice A) and repeat mammography in 6 months (Choice B) are inappropriate options. This patient has a highly suspicious lesion on physical examination, and despite
having normal mammography and fine-needle aspiration results, should undergo a more thorough evaluation with excisional biopsy. Failure to diagnose this patient's likely breast cancer allows time for the
cancer to advance and metastasize, which is associated with a worse prognosis.

MRI of the left breast (Choice C) is not typically used in the diagnosis of breast cancer. More commonly, it is used to evaluate the extent of disease in newly diagnosed breast cancer, or as a screening
modality for those at increased risk for developing breast cancer. It is not the most appropriate next step in management for this patient, as it would not provide a tissue diagnosis. It is used more frequently
for cancer screening in younger women with dense breast tissue, for whom mammography may be less sensitive.

Tamoxifen therapy (Choice 0) is a mixed estrogen receptor agonist and antagonist used in the treatment of breast cancer. This patient should not be treated for breast cancer without a formal diagnosis.
Given the results of her previous diagnostic studies, the most appropriate way to confirm the diagnosis is with an excisional biopsy.

Educational Objective: Breast cancer, the most common nondermatologic malignancy in women, may present with a palpable breast mass. In patients over age 30 years, mammography should be the first
step in diagnosis. Regardless of imaging findings, biopsy should occur if the physical examination is highly concerning for breast cancer. Fine-needle aspiration is an appropriate initial biopsy technique, but it
is associated with a low sensitivity. Those patients with suspicious masses but normal fine-needle aspiration results should therefore undergo excisional biopsy to confirm the diagnosis.

https://t.me/USMLENBME2CK ts e t
Previous Next Score Report Lab Values Calculator Help pause
Exam Section 4: Item 43 of 50 National Board of Medical Examiners
Comprehensive Clinical Science Self-Assessment

"I 43. A 2-year-old girl is brought lo the office by her mother because of bloody drainage from her right ear that the mother first noticed last night when the patient woke up screaming and holding her right
ear. The pain improved after the mother cleaned the ear with a cotton swab and gave the patient ibuprofen, but the mother noted dried, crusted blood on the patient's right ear lobe and pillow when she
awoke this morning. The patient also has a 3-day history of cough, congestion, and runny nose. She has no history of serious illness and receives no routine medications. She is at the 25th percentile
for height and weight and 50th percentile for BMI. Vital signs are within normal limits. The right tympanic membrane cannot be visualized because of seropurulent fluid in the right external auditory
canal. The remainder of the examination, including examination of the left tympanic membrane, discloses no abnormalities. Which of the following is the most likely cause of this patient's ear drainage?

A) Basilar skull fracture


B) Mastoiditis
C) Otitis externa
D) Traumatic injury to the external auditory canal
E) Tympanic membrane perforation
Correct Answer: E.

Acute otitis media is an acute infection of the middle ear and usually spreads to this space via the eustachian (auditory) tubes from the nasopharynx. Acute otitis media classically occurs in children, often
from viral or bacterial pathogens, commonly Streptococcus pneumoniae or Haemophi/us influenzae. It presents with ear pain (often with fever) and an erythematous tympanic membrane with retrotympanic
pus or a middle ear effusion. Recurrent or persistent episodes of acute otitis media may lead lo inflammation of the adjacent mastoid bone, a condition known as mastoiditis. Spontaneous perforation of the
tympanic membrane may occur, which typically results in acute conductive hearing loss, bloody or purulent otorrhea, and, often, dramatic alleviation of pain. Tympanic membrane perforation is an indication
for treatment with antibiotics, typically with a fluoroquinolone or with amoxicillin and clavulanate. Most tympanic membrane perforations heal spontaneously without surgical intervention.

Incorrect Answers: A, B, C, and D.

Basilar skull fracture (Choice A) occurs following significant cranial trauma and presents with hemotympanum and Battle sign (retroauricular ecchymosis).

Mastoiditis (Choice B) describes bacterial infection of the mastoid air cells. It typically results from untreated or recurrent otitis media and presents with postauricular pain, swelling, and erythema.

Otitis externa (Choice C) is commonly known as "swimmer's ear," and presents with mild external ear pain, watery discharge, swelling, and erythema of the external acoustic meatus. Treatment includes
topical fluoroquinolone (ciprofloxacin or ofloxacin) ear drops.

Traumatic injury to the external auditory canal (Choice 0) may occur following manipulation of the ear canal with a finger or cotton swab. This patient's history of pain and bloody drainage from the ear
preceded manipulation with a cotton swab, suggesting that spontaneous perforation of the tympanic membrane secondary to otitis media is more likely.

Educational Objective: Acute otitis media presents with ear pain (often with fever) and an erythematous tympanic membrane with retrotympanic pus or a middle ear effusion. Spontaneous perforation of the
tympanic membrane may occur, which typically results in acute conductive hearing loss, bloody or purulent otorrhea, and, often, dramatic alleviation of pain. Treatment includes antibiotics. Most tympanic
membrane perforations heal spontaneously without surgical intervention.

https://t.me/USMLENBME2CK ts e t
Previous Next Score Report Lab Values Calculator Help pause
Exam Section 4: Item 44 of 50 National Board of Medical Examiners
Comprehensive Clinical Science Self-Assessment

"I 44. A 20-year-old man comes to the student health services on his college campus because of painful urination and scant urethral discharge for the past week. He says that he has had no fever, chills, or
back pain. For the past 6 months, he has engaged in unprotected sexual activity with one partner. He has no history of sexually transmitted diseases or other disorders and takes no medications. He
appears well. His temperature is 37"C (98.6°F), pulse is 64/min, respirations are 12/min, and blood pressure is 115/70 mm Hg. Examination shows no costovertebral angle tenderness, penile lesions, or
expressible discharge. Urinalysis shows no RBCs, 2-5 WBC/hpf, and no casts; throat cultures are pending. Which of the following is the most appropriate management at this lime?

A) Amoxicillin therapy
B) Ceftriaxone and azithromycin therapy
C) Metronidazole therapy
D) Trimethoprim-sulfamethoxazole therapy
E) No treatment until culture and sensitivity test results are known
Correct Answer: B.

Chlamydia trachomatis and Neisseria gonorrhoeae can cause sexually transmitted urethritis in men. Risk factors for sexually transmitted infections (STls) include adolescence, multiple sexual partners, and not
using barrier contraception such as condoms. Urethritis typically presents with watery urethral discharge and dysuria. Infection with C. trachomatis is frequently asymptomatic. The diagnosis is confirmed with
urine nucleic acid amplification. Prompt identification and treatment of STls can prevent spread to or reinfection of sexual partners and prevent complications such as pelvic inflammatory disease (in women)
and sterility. Empiric treatment is appropriate for a patient presenting with symptoms suspicious for an STI. Combination therapy with intramuscular ceflriaxone and oral azithromycin is the most commonly used
regimen and provides good coverage against N. gonorrhoeae and C. trachomatis, respectively.

Incorrect Answers: A, C, D, and E.

Amoxicillin therapy (Choice A) is an alternative therapy for the treatment of an STI with C. trachomatis but is less effective than azithromycin.

Metronidazole therapy (Choice C) is effective for the treatment of trichomoniasis and bacterial vaginosis. However, it does not provide coverage against C. trachomatis or N. gonorrhoeae, which are the most
common causes of urethritis in men.

Trimethoprim-sulfamethoxazole therapy (Choice 0) is useful for the treatment of nonsexually transmitted urethritis or cystitis. It is not effective against C. trachomatis or N. gonorrhoeae.

No treatment until culture and sensitivity test results are known (Choice E) is incorrect. In cases of a suspected STI, antibiotic treatment should be initiated as soon as possible following culture. Single-dose, in-
office treatment improves compliance and reduces the risk for transmission to partners.

Educational Objective: Urethritis in men is commonly caused by sexually transmitted infection by Chlamydia trachomatis and/or Neissena gonorrhoeae. Empiric treatment is appropriate for a patient presenting
with symptoms highly suspicious for a sexually transmitted infection. Combination therapy with intramuscular ceftriaxone and oral azithromycin is a commonly used regimen.

https://t.me/USMLENBME2CK ts e t
Previous Next Score Report Lab Values Calculator Help pause
Exam Section 4: Item 45 of 50 National Board of Medical Examiners
Comprehensive Clinical Science Self-Assessment

"I 45. A 62-year-old woman comes to the physician for PPD skin testing prior to beginning volunteer work in a school. She has hypertension treated with hydrochlorothiazide and osteoarthritis treated with
acetaminophen. She was born in the USA. She has had no sick contacts. She does not smoke cigarettes, drink alcohol, or use illicit drugs. Physical examination shows no abnormalities. A PPD skin
test produces 6 mm of induration at 48 hours. Results of liver function tests are within the reference ranges. A chest x-ray shows no abnormalities. Which of the following is the most appropriate next
step in management?

A) Provide clearance to begin volunteer work


B) Repeat PPD skin testing to assess for booster phenomenon
C) Obtain culture of three induced sputum samples for acid-fast bacilli
D) Begin isoniazid and pyridoxine therapy
E) Begin isoniazid, rifampin, pyrazinamide, and ethambutol therapy
F) Begin isoniazid therapy only
Correct Answer: A.

Mycobacterium tuberculosis is a slow-growing mycobacterial organism transmitted via respiratory secretions. It presents with primary, latent, and reactivation patterns. In primary disease, patients present
with subacute fevers, weight loss, night sweats, cough, and malaise. latent tuberculosis can occur as the organism proliferates within macrophages and is maintained within caseating granulomas. PPD skin
testing stimulates a T-cell driven type IV (delayed) hypersensitivity reaction against subdermally injected tuberculin antigen. A common cause of false positive PPD testing includes prior vaccination with BCG
vaccine. Per Centers for Disease Control and Prevention guidelines, induration exceeding 15 mm is considered a positive result for a PPD skin test in patients with no risk factors for tuberculosis. lnduration
exceeding 10 mm is considered a positive test result for high-risk patients, such as recent immigrants from countries where tuberculosis is common, intravenous drug users, residents of and workers in high-
risk congregate settings (such as prisons, homeless shelters, or nursing care facilities), or those with particular medical conditions, such as silicosis, diabetes mellitus, or chronic renal failure, among others.
lnduration greater than 5 mm is considered positive in patients with HIV, with recent contact with a tuberculosis-positive patient, or with organ transplantation or immunosuppression. This patient's test result
is negative for her demographic, and she should be provided clearance to begin volunteer work.

Incorrect Answers: B, C, D, E, and F.

Repeat PPD skin testing to assess for booster phenomenon (Choice B) is recommended for initial testing in patients who will be periodically retested, such as healthcare workers. In patients with a history of
remote tuberculosis exposure with faded immune memory, an initial PPD test may provide a false negative result. However, exposure to tuberculin will boost immune memory and stimulate a positive result
on the second PPD test. Two-step PPD testing is not necessary for routine PPD testing of non-high-risk patients.

Obtaining cultures of three induced sputum samples for acid-fast bacilli (Choice C) is useful for the diagnosis of tuberculosis in a patient with suspected primary or latent tuberculosis. This patient's PPD
result is negative and should not prompt further diagnostic evaluation.

Beginning isoniazid and pyridoxine therapy (Choice D) or beginning isoniazid therapy only (Choice F) would be appropriate during a 9-month course for the treatment of latent tuberculosis. Patients at risk for
pyridoxine deficiency or peripheral neuropathy may benefit from therapy with isoniazid plus pyridoxine to reduce the risk for peripheral neuropathy related to vitamin Be (pyridoxine) deficiency.

Beginning isoniazid, rifampin, pyrazinamide, and ethambutol therapy (Choice E) is appropriate for the treatment of active primary or recurrent tuberculosis.

https://t.me/USMLENBME2CK ti
Previous Next Score Report Lab Values Calculator Help pause
Exam Section 4: Item 45 of 50 National Board of Medical Examiners
Comprehensive Clinical Science Self-Assessment

A) Provide clearance to begin volunteer work


B) Repeat PPD skin testing to assess for booster phenomenon
C) Obtain culture of three induced sputum samples for acid-fast bacilli
D) Begin isoniazid and pyridoxine therapy
E) Begin isoniazid, rifampin, pyrazinamide, and ethambutol therapy
F) Begin isoniazid therapy only
Correct Answer: A.

Mycobacterium tuberculosis is a slow-growing mycobacterial organism transmitted via respiratory secretions. It presents with primary, latent, and reactivation patterns. In primary disease, patients present
with subacute fevers, weight loss, night sweats, cough, and malaise. latent tuberculosis can occur as the organism proliferates within macrophages and is maintained within caseating granulomas. PPD skin
testing stimulates a T-cell driven type IV (delayed) hypersensitivity reaction against subdermally injected tuberculin antigen. A common cause of false positive PPD testing includes prior vaccination with BCG
vaccine. Per Centers for Disease Control and Prevention guidelines, induration exceeding 15 mm is considered a positive result for a PPD skin test in patients with no risk factors for tuberculosis. lnduration
exceeding 10 mm is considered a positive test result for high-risk patients, such as recent immigrants from countries where tuberculosis is common, intravenous drug users, residents of and workers in high-
risk congregate settings (such as prisons, homeless shelters, or nursing care facilities), or those with particular medical conditions, such as silicosis, diabetes mellitus, or chronic renal failure, among others.
lnduration greater than 5 mm is considered positive in patients with HIV, with recent contact with a tuberculosis-positive patient, or with organ transplantation or immunosuppression. This patient's test result
is negative for her demographic, and she should be provided clearance to begin volunteer work.

Incorrect Answers: B, C, D, E, and F.

Repeat PPD skin testing to assess for booster phenomenon (Choice 8) is recommended for initial testing in patients who will be periodically retested, such as healthcare workers. In patients with a history of
remote tuberculosis exposure with faded immune memory, an initial PPD test may provide a false negative result. However, exposure to tuberculin will boost immune memory and stimulate a positive result
on the second PPD test. Two-step PPD testing is not necessary for routine PPD testing of non-high-risk patients.

Obtaining cultures of three induced sputum samples for acid-fast bacilli (Choice C) is useful for the diagnosis of tuberculosis in a patient with suspected primary or latent tuberculosis. This patient's PPD
result is negative and should not prompt further diagnostic evaluation.

Beginning isoniazid and pyridoxine therapy (Choice D) or beginning isoniazid therapy only (Choice F) would be appropriate during a 9-month course for the treatment of latent tuberculosis. Patients at risk for
pyridoxine deficiency or peripheral neuropathy may benefit from therapy with isoniazid plus pyridoxine to reduce the risk for peripheral neuropathy related to vitamin Be (pyridoxine) deficiency.

Beginning isoniazid, rifampin, pyrazinamide, and ethambutol therapy (Choice E) is appropriate for the treatment of active primary or recurrent tuberculosis.

Educational Objective: PPD skin testing stimulates a T-cell driven type IV (delayed) hypersensitivity reaction against subdermal tuberculin antigen and is useful for the detection of latent tuberculosis
infection. lnduration exceeding 15 mm is considered a positive result for a PPD skin test in patients with no risk factors for tuberculosis. lnduration exceeding 10 mm is considered a positive result for high-
risk patients. lnduration exceeding 5 mm is considered positive for immunocompromised patients.

https://t.me/USMLENBME2CK ti
Previous Next Score Report Lab Values Calculator Help pause
Exam Section 4: Item 46 of 50 National Board of Medical Examiners
Comprehensive Clinical Science Self-Assessment

Y 2s. A 42-year-old man comes to the emergency department because of a 2-day history of nausea, vomiting, and malaise. He has bipolar disorder, herpes simplex virus 2, and chronic shoulder pain
secondary to a torn rotator cuff injury sustained 7 years ago. His medications are valproic acid, sertraline, acyclovir, hydrocodone-acetaminophen, and docusate. He does not drink alcohol or use illicit
drugs. His temperature is 37.2°C (99°F), pulse is 102/min, and blood pressure is 122/84 mm Hg. Physical examination shows dry mucous membranes. Abdominal examination shows tenderness to
palpation. On mental status examination, he has a dysphoric mood and congruent affect. laboratory studies show:
leukocyte count 8000/mm3
Serum
AST 38 U/L
ALT 52 U/L
Amylase 465 U/L

Which of the following medications is the most likely cause of this patient's current symptoms?

A) Acyclovir
B) Docusate
C) Hydrocodone-acetaminophen
D) Sertraline
E) Valproic acid
Correct Answer: E.

Valproic acid is an antiepileptic drug and mood stabilizer that is used to treat patients with schizoaffective or bipolar disorder. Drug-induced acute pancreatitis is a rare adverse effect of valproic acid that is
idiosyncratic, or not related to drug dosage or duration of treatment. The mechanism of this adverse reaction is unknown but possibly related to reactive oxygen species-induced endothelial damage within
the pancreas. For patients in whom valproic acid-induced pancreatitis is suspected, other possible causes of acute pancreatitis should be excluded. In the majority of cases, pancreatitis is self-limited and
resolves with discontinuation of the drug. Other idiosyncratic valproic acid reactions include hepatotoxicity, bone marrow toxicity, and alopecia.

Incorrect Answers: A, B, C, and D.

Acyclovir (Choice A) is not associated with drug-induced pancreatitis. Common adverse effects of acyclovir include nausea, vomiting, and diarrhea. Nephrotoxicity may occur with high-dose intravenous
administration.

Docusate (Choice B) is a stool softener. It is generally well-tolerated and is not associated with drug-induced pancreatitis. Common adverse effects include diarrhea and gastrointestinal discomfort.

Hydrocodone-acetaminophen (Choice C) is not commonly associated with drug-induced pancreatitis. Common adverse effects include hepatotoxicity, reduced gastrointestinal motility, respiratory depression,
and sedation.

Sertraline (Choice D) is a selective serotonin reuptake inhibitor that is not frequently associated with drug-induced pancreatitis. It is generally well-tolerated, though in high doses or in the setting of drug
neteeteeeit vetoed cowetewe dueeled

https://t.me/USMLENBME2CK ti e
Previous Next Score Report Lab Values Calculator Help pause
Exam Section 4: Item 46 of 50 National Board of Medical Examiners
Comprehensive Clinical Science Self-Assessment
••••
Serum
AST 38 U/L
ALT 52 U/L
Amylase 465 U/L

Which of the following medications is the most likely cause of this patient's current symptoms?

A) Acyclovir
B) Docusate
C) Hydrocodone-acetaminophen
D) Sertraline
E) Valproic acid
Correct Answer: E.

Valproic acid is an antiepileptic drug and mood stabilizer that is used to treat patients with schizoaffective or bipolar disorder. Drug-induced acute pancreatitis is a rare adverse effect of valproic acid that is
idiosyncratic, or not related to drug dosage or duration of treatment. The mechanism of this adverse reaction is unknown but possibly related to reactive oxygen species-induced endothelial damage within
the pancreas. For patients in whom valproic acid-induced pancreatitis is suspected, other possible causes of acute pancreatitis should be excluded. In the majority of cases, pancreatitis is self-limited and
resolves with discontinuation of the drug. Other idiosyncratic valproic acid reactions include hepatotoxicity, bone marrow toxicity, and alopecia.

Incorrect Answers: A, B, C, and D.

Acyclovir (Choice A) is not associated with drug-induced pancreatitis. Common adverse effects of acyclovir include nausea, vomiting, and diarrhea. Nephrotoxicity may occur with high-dose intravenous
administration.

Docusate (Choice B) is a stool softener. It is generally well-tolerated and is not associated with drug-induced pancreatitis. Common adverse effects include diarrhea and gastrointestinal discomfort.

Hydrocodone-acetaminophen (Choice C) is not commonly associated with drug-induced pancreatitis. Common adverse effects include hepatotoxicity, reduced gastrointestinal motility, respiratory depression,
and sedation.

Sertraline (Choice D) is a selective serotonin reuptake inhibitor that is not frequently associated with drug-induced pancreatitis. It is generally well-tolerated, though in high doses or in the setting of drug
interactions ii may cause serotonin syndrome.

Educational Objective: Drug-induced acute pancreatitis is a rare adverse effect of valproic acid that is idiosyncratic, or not related to drug dosage or duration of treatment. The mechanism is unknown. In the
majority of cases, the pancreatitis is self-limited and resolves with discontinuation of the drug.

https://t.me/USMLENBME2CK ti e
Previous Next Score Report Lab Values Calculator Help pause
Exam Section 4: Item 47 of 50 National Board of Medical Examiners
Comprehensive Clinical Science Self-Assessment

"I 47. A 72-year-old woman comes to the office with her son for a follow-up examination. She was diagnosed with Parkinson disease 8 years ago. She says her health has declined substantially during the
past 6 months. She feels weaker and has had increasing difficulty walking. She has stopped attending weekly luncheons with her friends. The patient's son says his mother sleeps poorly at night and
naps frequently during the day. He has to encourage her to eat and take her medications. The patient's medications are carbidopa-levodopa and donepezil. She appears sad and is tearful as she
discusses her difficulties. Examination shows a mild resting tremor of the hands and mild bradykinesia. When asked to stand and walk from a seated position, she moves slowly and pushes on the
arms of the chair with her hands; she takes steps with no hesitation but walks slowly. Which of the following is the most appropriate next step in pharmacotherapy?

A) Add mirtazapine to the medication regimen


B) Add rasagiline to the medication regimen
C) Add zolpidem to the medication regimen
D) Increase the dosage of carbidopa-levodopa
E) Increase the dosage of donepezil
Correct Answer: A.

Parkinson disease is a progressive neurodegenerative disorder caused by degradation of dopaminergic neurons in the substantia nigra. Symptoms are associated with an imbalance of dopamine and
acetylcholine, resulting in the characteristic findings of bradykinesia, resting tremor, muscle rigidity, and postural instability. Psychiatric disturbances, sleep disorders, and dementia are common complications
in addition to prominent motor symptoms. Depression is common in elderly patients with Parkinson disease and other types of dementia, such as Alzheimer dementia. It presents with 2 or more weeks of at
least five of the following symptoms: depressed mood, anhedonia (eg, decreased interest in socializing), guilt or worthlessness, difficulty concentrating, suicidal thoughts, and/or neurovegetative symptoms
(eg, decreased energy, sleep disturbance, appetite disturbance). In addition, patients with Parkinson disease may present with worsening motor function while experiencing depression and are at particularly
increased risk for suicide. Treatment of depression consists of pharmacotherapy with a selective serotonin reuptake inhibitor (SSRI) or atypical antidepressant, such as mirtazapine. Mirtazapine may
be particularly useful in patients with insomnia because of its sedating side effects and in those with appetite loss. Cognitive behavioral therapy is also useful for managing depression.

Incorrect answers: B, C, D, and E.

Adding rasagiline to the medication regimen (Choice B) is incorrect. Rasagiline is a monoamine oxidase inhibitor that is useful for treating symptoms of early Parkinson disease. It is less useful for the
treatment of depression associated with Parkinson disease and should be used with caution if combined with particular antidepressants, such as SSRls, because of the risk for serotonin syndrome.

Adding zolpidem to the medication regimen (Choice C) is incorrect. Zolpidem is a nonbenzodiazepine sedative-hypnotic medication. It is utilized to treat insomnia but is not helpful in the treatment of
depression. It is also relatively contraindicated in elderly populations and in those with dementia.

Increasing the dosage of carbidopa-levodopa (Choice 0) is incorrect. Carbidopa-levodopa nonselectively increases dopamine concentrations in the basal ganglia and limbic system, leading to improvement
of Parkinson disease symptoms. While raising the dose of carbidopa-levodopa is useful for treating residual symptoms of Parkinson disease, it does not treat depression, and higher doses are associated
with an increased risk for psychotic symptoms.

Increasing the dosage of donepezil (Choice E) is incorrect. Oonepezil is an anticholinesterase inhibitor that indirectly increases the concentration of the neurotransmitter acetylcholine. It is useful for treating
Alzheimer dementia, but also improves cognition in Parkinson disease dementia. It is not useful for treating depression.
4de.o" lieu4

https://t.me/USMLENBME2CK ti
Previous Next Score Report Lab Values Calculator Help pause
Exam Section 4: Item 47 of 50 National Board of Medical Examiners
Comprehensive Clinical Science Self-Assessment
- .. - e

arms of the chair with her hands; she takes steps with no hesitation but walks slowly. Which of the following is the most appropriate next step in pharmacotherapy?

A) Add mirtazapine to the medication regimen


B) Add rasagiline to the medication regimen
C) Add zolpidem to the medication regimen
D) Increase the dosage of carbidopa-levodopa
E) Increase the dosage of donepezil
Correct Answer: A.

Parkinson disease is a progressive neurodegenerative disorder caused by degradation of dopaminergic neurons in the substantia nigra. Symptoms are associated with an imbalance of dopamine and
acetylcholine, resulting in the characteristic findings of bradykinesia, resting tremor, muscle rigidity, and postural instability. Psychiatric disturbances, sleep disorders, and dementia are common complications
in addition to prominent motor symptoms. Depression is common in elderly patients with Parkinson disease and other types of dementia, such as Alzheimer dementia. It presents with 2 or more weeks of at
least five of the following symptoms: depressed mood, anhedonia (eg, decreased interest in socializing), guilt or worthlessness, difficulty concentrating, suicidal thoughts, and/or neurovegetative symptoms
(eg, decreased energy, sleep disturbance, appetite disturbance). In addition, patients with Parkinson disease may present with worsening motor function while experiencing depression and are at particularly
increased risk for suicide. Treatment of depression consists of pharmacotherapy with a selective serotonin reuptake inhibitor (SSRI) or atypical antidepressant, such as mirtazapine. Mirtazapine may
be particularly useful in patients with insomnia because of its sedating side effects and in those with appetite loss. Cognitive behavioral therapy is also useful for managing depression.

Incorrect answers: B, C, D, and E.

Adding rasagiline to the medication regimen (Choice B) is incorrect. Rasagiline is a monoamine oxidase inhibitor that is useful for treating symptoms of early Parkinson disease. It is less useful for the
treatment of depression associated with Parkinson disease and should be used with caution if combined with particular antidepressants, such as SSRls, because of the risk for serotonin syndrome.

Adding zolpidem to the medication regimen (Choice C) is incorrect. Zolpidem is a nonbenzodiazepine sedative-hypnotic medication. It is utilized to treat insomnia but is not helpful in the treatment of
depression. It is also relatively contraindicated in elderly populations and in those with dementia.

Increasing the dosage of carbidopa-levodopa (Choice D) is incorrect. Carbidopa-levodopa nonselectively increases dopamine concentrations in the basal ganglia and limbic system, leading to improvement
of Parkinson disease symptoms. While raising the dose of carbidopa-levodopa is useful for treating residual symptoms of Parkinson disease, it does not treat depression, and higher doses are associated
with an increased risk for psychotic symptoms.

Increasing the dosage of donepezil (Choice E) is incorrect. Donepezil is an anticholinesterase inhibitor that indirectly increases the concentration of the neurotransmitter acetylcholine. It is useful for treating
Alzheimer dementia, but also improves cognition in Parkinson disease dementia. It is not useful for treating depression.

Educational Objective: Parkinson disease is associated with depression, which may lead to worsening of motor function and increased risk for suicide. Treatment is with selective serotonin reuptake
inhibitors or atypical antidepressants, as well as cognitive behavioral therapy. The atypical antidepressant mirtazapine is particularly useful for patients with concomitant depression and insomnia.

https://t.me/USMLENBME2CK ti
Previous Next Score Report Lab Values Calculator Help pause
Exam Section 4: Item 48 of 50 National Board of Medical Examiners
Comprehensive Clinical Science Self-Assessment

"I 48. A 32-year-old primigravid woman at 10 weeks gestation comes lo the physician for her first prenatal visit. She conceived after in vitro fertilization. Three years ago, a bicornuate uterus was noted on
diagnostic laparoscopy for primary infertility. She is otherwise healthy, and her only medication is a prenatal vitamin. She is 163 cm (5 ft 4 in) tall. She weighed 50 kg (110 1b) prior to her pregnancy;
BMI was 19 kg/m?. She has had a 1-kg (2-lb) weight gain during her pregnancy. Her blood pressure is 110/70 mm Hg. Examination shows a uterus consistent in size with a 10-week gestation. This
patient is at greatest risk for which of the following pregnancy complications?

A) Abruptio placentae
8 ) Placenta accreta
C) Placenta previa
D) Preterm delivery
E) Uterine rupture
Correct Answer: D.

Infertility can have a multitude of causes, including hormonal disorders, inflammatory diseases, structural abnormalities, autoimmune diseases, clotting disorders, and lifestyle factors. Infertility can be
primary in patients who have never conceived, or secondary in patients who had previous conceptions. Regardless, it is defined as the inability of a couple to conceive after 12 months of regular intercourse,
or after 6 months of intercourse in women over age 35 years. The evaluation of infertility often includes a history and physical examination, as well as laboratory evaluation, imaging studies, and, potentially,
laparoscopy. Common causes of infertility include ovulatory dysfunction and endometriosis. Congenital structural uterine abnormalities, as seen in this patient, can lead to infertility. A bicornuate uterus is
caused by failure of the paramesonephric (m~llerian) ducts to fully fuse, resulting in two completely or partially separate uterine cavities. Because of this, there is a diminished area for egg implantation, as
well as placental and fetal development, which predisposes the patient to infertility and recurrent pregnancy loss. A bicornuate uterus also increases the risk for preterm delivery, fetal growth restriction, and
malpresentation. Therefore, this patient is at the highest risk for preterm delivery given her anatomic uterine variant.

Incorrect Answers: A, 8, C, and E.

Abruptio placentae (Choice A) is the premature separation of the placenta from the uterus. It commonly presents with vaginal bleeding, severe uterine pain, and tetanic contractions, typically in the third
trimester. Risk factors include cocaine use, trauma, smoking, and preeclampsia, but not generally congenital uterine anomalies.

Placenta accreta (Choice 8) refers to the abnormal attachment of the placenta to the myometrium/perimetrium and even through to the serosa and, potentially, adjacent organs (termed 'percreta'), rather
than to just the decidualized endometrium. It is commonly diagnosed on prenatal ultrasonography but can present with postpartum hemorrhage caused by retained placental tissue. A bicornuate uterus is not
a risk factor for placenta accreta.

Placenta previa (Choice C) is an abnormal positioning of placental tissue such that the placenta partially or entirely overlies the internal cervical os. It can spontaneously resolve, but if it persists into the third
trimester, it may result in painless vaginal bleeding. Severe bleeding from placenta previa may lead to hemorrhagic shock, and treatment involves delivery with cesarean section. This patient is not at
increased risk for placenta previa.

Uterine rupture (Choice E) is a dangerous obstetric complication where the uterine wall tears, presenting with acute pain and both maternal and fetal hemodynamic instability. The sudden loss of fetal station
is a suggestive finding. Risk factors include prior cesarean delivery or uterine surgery, whereas a bicornuate uterus does not increase the risk.
4do-foul lo ff Al old t

https://t.me/USMLENBME2CK ts e t
Previous Next Score Report Lab Values Calculator Help pause
Exam Section 4: Item 48 of 50 National Board of Medical Examiners
Comprehensive Clinical Science Self-Assessment

A) Abruptio placentae
B ) Placenta accreta
C) Placenta previa
D) Preterm delivery
E) Uterine rupture
Correct Answer: D.

Infertility can have a multitude of causes, including hormonal disorders, inflammatory diseases, structural abnormalities, autoimmune diseases, clotting disorders, and lifestyle factors. Infertility can be
primary in patients who have never conceived, or secondary in patients who had previous conceptions. Regardless, it is defined as the inability of a couple to conceive after 12 months of regular intercourse,
or after 6 months of intercourse in women over age 35 years. The evaluation of infertility often includes a history and physical examination, as well as laboratory evaluation, imaging studies, and, potentially,
laparoscopy. Common causes of infertility include ovulatory dysfunction and endometriosis. Congenital structural uterine abnormalities, as seen in this patient, can lead to infertility. A bicornuate uterus is
caused by failure of the paramesonephric (m~llerian) ducts to fully fuse, resulting in two completely or partially separate uterine cavities. Because of this, there is a diminished area for egg implantation, as
well as placental and fetal development, which predisposes the patient to infertility and recurrent pregnancy loss. A bicornuate uterus also increases the risk for preterm delivery, fetal growth restriction, and
malpresentation. Therefore, this patient is al the highest risk for preterm delivery given her anatomic uterine variant.

Incorrect Answers: A, B, C, and E.

Abruptio placentae (Choice A) is the premature separation of the placenta from the uterus. It commonly presents with vaginal bleeding, severe uterine pain, and tetanic contractions, typically in the third
trimester. Risk factors include cocaine use, trauma, smoking, and preeclampsia, but not generally congenital uterine anomalies.

Placenta accrela (Choice B) refers to the abnormal attachment of the placenta to the myometrium/perimetrium and even through to the serosa and, potentially, adjacent organs (termed 'percreta'), rather
than to just the decidualized endometrium. It is commonly diagnosed on prenatal ultrasonography but can present with postpartum hemorrhage caused by retained placental tissue. A bicornuate uterus is not
a risk factor for placenta accreta.

Placenta previa (Choice C) is an abnormal positioning of placental tissue such that the placenta partially or entirely overlies the internal cervical os. It can spontaneously resolve, but if it persists into the third
trimester, ii may result in painless vaginal bleeding. Severe bleeding from placenta previa may lead to hemorrhagic shock, and treatment involves delivery with cesarean section. This patient is not at
increased risk for placenta previa.

Uterine rupture (Choice E) is a dangerous obstetric complication where the uterine wall tears, presenting with acute pain and both maternal and fetal hemodynamic instability. The sudden loss of fetal station
is a suggestive finding. Risk factors include prior cesarean delivery or uterine surgery, whereas a bicornuate uterus does not increase the risk.

Educational Objective: Congenital uterine anomalies can be the cause of primary infertility as well as recurrent pregnancy loss. For example, a bicornuate uterus with two separate endometrial cavities has
less room for appropriate egg implantation, placental development, and fetal growth. Patients with this abnormality are also at increased risk for preterm delivery, fetal growth restriction, and malpresentation.

https://t.me/USMLENBME2CK ti
Previous Next Score Report Lab Values Calculator Help pause
Exam Section 4: Item 49 of 50 National Board of Medical Examiners
Comprehensive Clinical Science Self-Assessment

"I 49. A 22-year-old man with alcohol use disorder comes to the physician because of a 3-week history of weakness in his right hand. His symptom began after drinking a bottle of whiskey. He has no history
of serious illness and takes no medications. He drinks six cans of beer daily. On examination of the right upper extremity, muscle strength is 2/5 in the supinator, wrist extensors, and finger extensors;
strength is 5/5 in all other muscles. Sensation to pinprick is decreased over the dorsum of the right hand from the wrist to the base of the thumb and index finger. Which of the following is the most
appropriate next step in diagnosis?

A) Determination of erythrocyte sedimentation rate


B) Electromyography and nerve conduction studies
C) MRI of the brachia! plexus
D) MRI of the brain
E) MRI of the cervical spine
Correct Answer: B.

The radial nerve is a nerve of the brachia I plexus and is comprised of afferents and efferents from the C5-T1 nerve roots. It courses through the axilla, down the posterior shaft of the humerus within the
spiral groove, and then along the posterolateral aspect of the forearm. The radial nerve supplies motor fibers to the triceps brachii and the extensor muscles within the posterior compartment of the forearm,
and sensory fibers to the dorsum of the forearm, wrist, and hand. It does not innervate the intrinsic muscles of the hand. It is prone to injury with midshaft humerus fractures, as it runs near the humeral
diaphysis within the spiral groove. Injury to the nerve presents with loss of function of structures distal to the injury. In the case of this patient, the weakness of the wrist and finger extensors suggests injury
within the axilla or arm. In cases of acute intoxication, prolonged compression of the nerve against a hard surface is a common cause during periods of unconsciousness. Electromyography and nerve
conduction studies are most appropriate for determining a diagnosis of radial nerve palsy.

Incorrect Answers: A, C, D, and E.

Determination of erythrocyte sedimentation rate (Choice A) is a nonspecific marker of inflammation, which may be increased in disorders such as myositis, dermatomyositis, and polymyositis.

MRI of the brachial plexus (Choice C) can be helpful in diagnosing brachial plexus injury. Brachial plexus injury, depending on the location within the plexus, would involve motor and sensory loss of multiple
nerves. A mononeuropathy, such as in this patient, makes a brachia! plexus injury less likely.

MRI of the brain (Choice D) would be helpful in diagnosing acute ischemic stroke, whereas this patient is presenting with a peripheral nerve palsy.

MRI of the cervical spine (Choice E) would be helpful in diagnosing central cord syndrome, which presents with decreased sensation and strength of the upper extremities. However, this patient is presenting
with specific peripheral, not central, radial nerve palsy.

Educational Objective: Radial nerve injuries typically present with the inability to extend the elbow, forearm, wrist, or fingers, along with the loss of sensation to the posterior arm and dorsal hand. The
diagnosis of radial nerve palsy is determined with electromyography and nerve conductions studies.

https://t.me/USMLENBME2CK ts e t
Previous Next Score Report Lab Values Calculator Help pause
Exam Section 4: Item 50 of 50 National Board of Medical Examiners
Comprehensive Clinical Science Self-Assessment

"I 50. One day after delivery of a healthy newborn at term, a 22-year-old woman, gravida 2, para 2, has fever. Pregnancy and delivery were uncomplicated. Her temperature is 39.4°C (103°F), pulse is 110/min,
respirations are 22/min, and blood pressure is 125/70 mm Hg. Examination shows normal lochia rubra. The uterus is tender, and the fundus extends to the umbilicus. Her leukocyte count is 17,000/mm3
with a shift to the left. While awaiting results of additional laboratory studies, which of the following is the most likely causal organism?

A) Candida albicans
B) Escherichia coli
C) Mycoplasma pneumoniae
D) Staphylococcus aureus
E ) Staphylococcus epidermidis
Correct Answer: B.

Endometritis is an acute infection of the uterine endometrium that commonly occurs after delivery. Patients typically present with fever, tenderness of the uterine fund us, and mucopurulent vaginal discharge.
Laboratory studies commonly show leukocytosis. Endometritis is caused by ascending bacteria from the vagina, and risk factors for development include cesarean delivery, prolonged membrane rupture,
advanced maternal age, and multiple cervical examinations. Endometritis is commonly polymicrobial, but isolated pathogens often include Escherichia coli, Enterobacter, Enterococcus, Bacteroides, and
C/ostridium. When severe, it can cause bacteremia and sepsis. Treatment with intravenous antibiotics is required.

Incorrect Answers: A, C, D, and E.

Candida albicans (Choice A) can cause vulvovaginitis, which presents with vaginal itching and thick, white discharge. It commonly occurs as a result of the disruption of vaginal flora in the setting of antibiotic
use. It would not explain this patient's fever or uterine tenderness.

Mycoplasma pneumoniae (Choice C) is a cause of upper respiratory infection and atypical pneumonia. These may present with fever, cough, rhinorrhea, and dyspnea. It would not be expected to cause a
postpartum fever associated with uterine tenderness.

Staphylococcus aureus (Choice 0) may be isolated in endometritis, but it is a less common pathogen than E. coli. When present, it can cause toxic shock syndrome as a result of the release of exotoxin with
severe fever, hypotension, and skin changes, such as erythema or desquamation.

Staphylococcus epidermidis (Choice E) is a skin flora that is commonly seen in line- or hardware-associated infections, surgical site infections, and endocarditis. It would not be an expected pathogen in
endometritis.

Educational Objective: Endometritis is an acute infection of the uterine endometrium that commonly occurs after delivery. Patients typically present with fever, tenderness of the uterine fund us, mucopurulent
vaginal discharge, and leukocytosis. Endomelritis is commonly polymicrobial, but isolated pathogens often include Escherichia coli, Enterobacter, Enterococcus, Bacteroides, and Clostridium.

https://t.me/USMLENBME2CK ti
Previous Next Score Report Lab Values Calculator Help pause

You might also like